You are on page 1of 468

Unit 1: Đại cương

1. Glycocalyx là:
A) các chuỗi carbohydrate tích điện âm xuyên vào dịch bào tương, cấu thànhtừ các
glycolipid và glycoprotein thiết yếu
B) lớp màng carbohydrat tích điện âm ở mặt ngoài tế bào
C) lớp các điện tích âm ở Mặt bào tương của màng tế bào
D) kho dữ trữ glycogen lớn ở các cơ “nhanh”
E) một cơ chế liên kết tế bào-tế bào
 B) “Glycocalyx” của tế bào là một lớp áo carbohydrat tích điện âm lỏng lẻo bao
ngoài bề mặt tế bào. Các carbohydrate màng thường kết hợp với protein hoặc lipid
ở dạng glycoprotein hay glycolipid, và phần “glyco” của những phân tử này hầu
như luôn xuyên ra ngoài tế bào.
2. ARN thông tin (mARN)
A) mang các thông tin di truyền tới tế bào chất
B) mang các acid amin được kích hoạt tới ribosom
C) là các phân tử ARN đơn chuỗi gồm 21-23 nucleotid có thể điều hòa phiên mã
D) cấu tạo ribosom
 A) Các phân tử mARN là các chuỗi đơn ARN dài ở tế bào chất, được cấu tạo tử
hàng trăm tới hàng nghìn nucleotid ARN ở các chuỗi chưa ghép đôi. mARN mang
mã di truyền tới tế bào chất để quy định loại protein được hình thành. ARN vận
chuyển (tARN) vận chuyển các acid amin đã hoạt hóa tới ribosom. ARN ribosom,
cùng 75 protein khác nhau, cấu tạo nên ribosom. Tiểu ARN là các phân tử ARN
đơn chuỗi gồm 21-23 nucleotid điều hòa quá trình phiên mã và dịch mã gen.
3. Câu nào sau đây đúng với cả hai quá trình ẩm bào và thực bào?
A) Liên quan tới sự tham gia của các sợi actin
B) Xảy ra tự phát và không chọn lọc
C) Các túi nhập bào kết hợp cùng những ribosom tiết enzym thủy phân vào túi
D) Chỉ quan sát thấy ở đại thực bào và bạch cầu đa nhân trung tính
E) Không cần ATP
 A) Cả ẩm bào và thực bào đều có sự tham gia của biến đổi màng tế bào. Màng
tế bào lõm vào khi ẩm vào và nhô ra trong thực bào. Cả hai hiện tượng này đều cần
có sự tham gia của actin và các thành phần khung tế bào khác. Thực bào không tự
phát và có tính chọn lọc, được kích hoạt bởi các phản ứng receptor-phối thể đặc
hiệu.
4. So sánh hai loại tế bào ở cùng một người, việc mỗi loại tế bào biểu hiện các
protein khác nhau cho thấy
A) khác biệt ADN nhân ở hai loại tế bào
B) khác biệt số lượng gen đặc thù trong bộ gen ở hai loại tế bào
C) Sự biểu hiện đặc trưng cho từng tế bào và sự bất hoạt của các gene cụ thể
D) khác biệt số lượng nhiễm sắc thể trong mỗi tế bào
E) tuổi của các tế bào
 C) Sự đa dạng của các protein biểu hiện bởi mỗi tế bào phản ánh sự biểu hiện
và ức chế của gen đặc hiệu. Mỗi tế bào đều có cùng AND trong nhân và cùng số
lượng gen. Bởi vậy, sự khác biệt không phải kết quả của sự khác nhau về gen mà
bởi sự ức chế và/hoặc kích thích chọn lọc của các yếu tố điều chỉnh khác nhau của
gen.
5. Tiểu ARN (miRNA)
A) được hình thành trong trong tế bào chất và ức chế quá trình dịch mã hay thúc
đẩy sự giáng hóa mARN trước khi được dịch mã
B) được hình thành trong nhân và được xử lý bởi enzym dicer (1 loại enzyme cắt
RNA mạch kép) trong tế bào chất
C) là các mảnh ARN ngắn chuỗi kép (21-23 nucleotid) điều hòa biểu hiện gen
D) ức chế phiên mã gen
 A) Tiểu ARN (miARN) được tạo nên trong tế bào chất từ tiền miARN và biến
đổi bởi enzym dicer tạo thành phức hợp RISC (RNA-induced silencing complex),
từ đó hình thành các miARN.Các miARN điều hòa biểu hiện gen bằng việc gắn
vào vùng liên kết trên ARN và ức chế dịch mã hoặc kích thích giáng hóa mARN
trước khi nó được dịch mã bởi ribosom.
Câu hỏi 6-8
A) Nhân con (hạch nhân)
B) Nhân
C) Lưới nội chất trơn
D) Lưới nội chất có hạt
E) Bộ máy Golgi
F) Thể nội bào (endosome)
G) Peroxisome
H) Tiêu thể (lysosome)
I) Dịch bào tương
J) Khung tế bào
K) Glycocalyx
L) Các vi ống
Lựa chọn các cấu trúc liên quan tới protein thiếu hay đột biến trong mỗi trường
hợp sau đây.
6. Các nghiên cứu trên bệnh nhi nam 5 tuổi cho thấy sự tích tụ các este cholesteryl
và trygicerid trong gan, lách, ruột; cũng như vôi hóa tuyến thượng thận hai bên.
Các nghiên cứu bổ sung chỉ ra rằng nguyên nhân là thiếu hụt hoạt động acid lipase
A.
6. H) Các acid lipase, cùng các acid hydrolase khác, nằm trong tiêu thể. Kết hợp
các túi nhập bào và tự phân với các tiêu thể khởi động quá trình nội bào cho phép
các tế bào tiêu hóa các mảnh tế bào và phần tử được nuốt vào từ ngoại bào, bao
gồm cả vi khuẩn. Trong môi trường acid thông thường của tiêu thể, acid lipase sử
dụng hydro để phân hủy lipid thành acid béo và glycerol. Các acid lipase khác bao
gồm các nuclease, protease, và các enzym thủy phân polysaccharid.
7. Sự phân cắt bất thường của mannose tồn dư sau dịch mã glycoprotein dẫn tới sự
hình thành bệnh tự miễn giống lupus ở chuột. Sự phân cắt bất thường này là do đột
biến enzym α-mannosidase II.
 E) Các protein màng được glycosyl hóa trong quá trình tổng hợp chúng tại
khoang của lưới nội chất có hạt. Tuy vậy, hầu hết các biến đổi sau dịch mã của các
chuỗi oligosaccharid diễn ra trong khi vận chuyển protein qua các lớp khuôn của
bộ máy Golgi, nơi có các enzym như α-mannosidase II.

8. Việc nhận thấy sự phân cắt bất thường mannose tồn dư sau dịch mã glycoprotein
gây ra bệnh tự miễn ở chuột cho thấy vai trò của cấu trúc này trong đáp ứng miễn
dịch sinh lý.
 K) Các chuỗi oligosaccharid – được thêm vào glycoprotein ở mặt màng của lưới
nội chất có hạt, và được biến đổi trong suốt quá trình vận chuyển qua bộ máy
Golgi – gắn vào mặt ngoài màng của tế bào. Lớp carbohydrat tích điện âm này
được gọi chung là glycocalyx. Nó tham gia vào các tương tác tế bào-tế bào, tế bào-
phối tử, và đáp ứng miễn dịch.
Câu hỏi 9-11
A) Nhân con (hạch nhân)
B) Nhân
C) Lưới nội chất trơn
D) Lưới nội chất có hạt
E) Bộ máy Golgi
F) Thể nội bào (endosome)
G) Peroxisome
H) Tiêu thể (lysosome)
I) Dịch bào tương
J) Khung tế bào
K) Glycocalyx
L) Các vi ống
Nối các vị trí trong tế bào nêu trên với mỗi bước của quá trình tổng hợp và đóng
gói protein tương ứng dưới đây.
9. Khởi đầu dịch mã
 I) Khởi đầu dịch mã, cả trong trường hợp tạo protein dịch bào tương, protein
gắn màng, hay protein tiết, đều diễn ra trong dịch bào tương và có sự tham gia một
phần của ribosom. Một protein tiết chỉ được nhận biết khi điểm N-kết thúc của
chuỗi polypeptid xuất hiện. Tại điểm này, ribosom gắn bào bề mặt dịch bào tương
của lưới nội chất có hạt. Dịch mã tiếp tục, và chuỗi polypeptid mới được đẩy vào
khuôn của lưới nội chất.
10. Ngưng tụ và đóng gói protein
 E) Các protein hình thành được ngưng tụ, phân loại và đóng gói vào các túi tiết
trong những phần tiêu hủy của bộ máy Golgi, còn được gọi là mạng Golgi mặt
trans. Tại đây protein tiết ra ngoài sẽ được tách riêng với những protein đến
khoang nội bào hay màng tế bào.
11. Phiên mã gen
 B) Các bước của quá trình phiên mã đều diễn ra trong nhân, không phụ thuộc
điểm đến cuối cùng của sản phẩm protein. Phân tử ARN thông tin hình thành được
vận chuyển qua các lỗ màng nhân và được dịch mã để trở thành thành phần dịch
tương bào hay khoang của lưới nội chất hạt.
12. “Tính dư thừa” hay “tính thoái hóa” của mã di truyền biểu hiện ở bước nào
dưới đây của quá trình tổng hợp protein?
A) Nhân đôi AND
B) Phiên mã
C) Điều chỉnh sau phiên mã
D) Dịch mã
E) Glycosyl hóa protein
 D) Trong suốt quá trình nhân đôi và phiên mã, phân tử acid nucleic mới là một
bản sau chính xác của phân tử ADN mẫu. Đây là kết quả của việc bắt cặp các base
từng chiếc một, đặc hiệu, và đoán trước được. Tuy nhiên, trong quá trình dịch mã,
mỗi acid amin ở chuỗi polypeptid mới lại được mã hóa theo “codon” – tức chuỗi
ba nucleotid liên tiếp. Mỗi bộ ba chỉ mã hóa một acid amin, nhưng phần lớn acid
amin có thể được mã hóa bởi nhiều bộ ba khác nhau. Tính dư thừa là do 60 bộ ba
chỉ mã hóa khoảng 20 acid amin.
13. Enzym nào dưới đây không tham gia trực tiếp quá trình phiên mã?
A) Helicase
B) ARN polymerase
C) Điểm (trình tự nu) kết thúc
D) Các phân tử ARN “hoạt hóa”
E) Điểm (trình tự nu) khởi đầu
 A) Helicase là một trong nhiều protein tham gia vào quá trình nhân đôi AND.
Nó không tham gia phiên mã. ARN polymerase đính vào điểm khởi đầu và giúp
thêm các phân tử ARN “hoạt hóa” vào phân tử ARN đang hình thành dễ dàng hơn
cho tới khi polymerase đi tới điểm kết thúc của phân tử AND đang phiên mã.
14. Protein nào sau đây có khả năng là sản phẩm của gen tiền ung thư nhất?
A) Receptor yếu tố tăng trưởng
B) Protein khung tế bào
C) Kênh Na+
D) Ca++-ATPase
E) Chuỗi nhẹ của myosin
 A) Một gen ung thư không chỉ bị kích hoạt bất thường mà còn đột biến làm cho
sản phẩm của nó khiến tế bào tăng trưởng không kiểm soát. Một gen tiền ung thư
đơn giản là phiên bản “bình thường” của gen ung thư. Theo định nghĩa, các gen
tiền ung thư được chia thành nhiều dòng protein cùng tham gia kiểm soát sự tăng
trưởng của tế bào. Những dòng này bao gồm, nhưng không chỉ giới hạn trong số,
các yếu tố tăng trưởng và các receptor của chúng, các protein kinase, các yếu tố
phiên mã, và các protein điều hòa sự tăng trưởng nhanh.
15. Hiện tượng nào sau đây không xảy ra trong quá trình nguyên phân?
A) Các nhiễm sắc thể kết đặc
B) Nhân đôi bộ gen
C) Phân cắt màng nhân
D) Các chromatid xếp thành một hàng ở mặt phẳng xích đạo
E) Các chromatid tách thành hai bộ 46 nhiễm sắc thể “con”
B) Sự nhân đôi gen xảy ra trong pha S của chu kỳ tế bào và xảy ra trước nguyên
phân. Sự kết đặc nhiễm sắc thể xảy ra ở kỳ đầu nguyên phân. Sự phân cắt màng
nhân xảy ra ở cuối kỳ đầu, đầu kỳ giữa (prometaphase). Sự sắp xếp chromatid trên
mặt phẳng xích đạo và tách hai bộ nhiễm sắc thể con xảy ra ở kỳ cuối.

16. Đặc điểm nào sau đây của màng sinh học bị ảnh hưởng nhiều nhất bởi thành
phần cholesterol?
A) Độ dày
B) Tính “thấm” ion
C) Tính linh động
D) Tính glycosyl hóa
E) Kỵ nước
 C) Thành phần cholesterol của màng quyết định mật độ bao bọc của các
phospholipid. Càng nhiều thành phần cholesterol, màng càng nhiều dịch và các
thành phần màng bao gồm các phân tử protein và bản thân phospholipid càng dễ di
động theo phương nằm ngang. Ở mức độ ít hơn, thành phần cholesterol cũng ảnh
hưởng tới tính thấm của màng với các phân tử tan trong nước.
17. Sự có mặt của yếu tố nào sau đây phân biệt tế bào nhân chuẩn (eukaryotic) với
các đơn bị sống thấp hơn?
A) AND
B) ARN
C) Màng
D) Protein
E) Nhân
 E) Các acid nucleic và protein cùng nhau tạo nên các đơn vị cơ bản của sự sống
có khả năng nhân đôi, ví dụ ở virus. Màng và thậm chí cả các bào quan có thể xuất
hiện ở tế bào nhân sơ nhưng chỉ các tế bào nhân chuẩn mới có nhân.
18. Giả sử truyền một lượng lớn máu cho một bệnh nhân có các receptor nhận cảm
áp lực động mạch mất chức năng và huyết áp tăng từ 100 lên 150 mmHg. Trường
hợp khác, giả sử cùng một lượng máu được truyền vào cùng một bệnh nhân khi các
receptor nhận cảm áp lực hoạt động bình thường và huyết áp tăng từ 100 lên 125
mmHg. Mức điều hòa ngược đạt được gần đúng của các receptor nhận cảm áp lực
trên bệnh nhân này khi chúng hoạt động chức năng bình thường là?
A) −1.0
B) −2.0
C) 0.0
D) +1.0
E) +2.0
 A) Mức điều hòa ngược đạt được của hệ thống kiểm soát được tính bằng giá trị
hiệu chỉnh chia cho số lượng điều chỉnh còn lại của hệ thống. Trong ví dụ này,
huyết áp tăng từ 100 tới 150 mmHg khi các receptor nhận cảm áp lực mất chức
năng. Khi chúng hoạt động bình thường, áp lực chỉ tăng 25 mmHg. Như vậy, hệ
thống phản hồi (feedback) “hiệu chỉnh” -25mmHg, từ 150 về 125mmHg. Việc
huyết áp vẫn tăng +25mmHg được gọi là “lỗi”. Như vậy, ở ví dụ này, giá trị hiệu
chỉnh bằng
-25mmHg và lỗi còn lại là +25mmHg. Vì thế, mức điều hòa ngược dạt được của
các receptor nhận cảm áp lực ở người này là -1, cho thấy đây là hệ thống kiểm soát
phản hồi âm tính.

Unit 2: Màng, Điện thế màng


1. Câu nào sau đây mô tả đúng nhất sự thay đổi thể tích tế bào xảy ra khi các hồng
cầu (Trước đó đạt cân bằng trong môi trường dịch NaCl 280 milli osmol) được đưa
vào môi trường dịch chứa 140 mM NaCl và 20 mM ure, một phân tử lớn nhưng có
thể thấm qua màng?
A) Tế bào ban đầu co lại, sau đó nở to dần và vỡ
B) Tế bào co nhỏ nhanh và dần trở về thể tích ban đầu của chúng
C) Tế bào nở to và vỡ
D) Tế bào nở to nhanh và dần trở về thể tích ban đầu của chúng
E) Không có sự thay đổi thể tích
 B) Một dịch chứa 140 mM NaCl có độ thẩm thấu 280 mOsm, tương đương độ
thẩm thấu nội bào “bình thường”. Nếu hồng cầu được đặt vào dịch chỉ gồm 140
mM NaCl, sẽ không có thay đổi thể tích tế bào vì độ thẩm thấu cân bằng giữa trong
và ngoài tế bào. Tuy nhiên, sự có mặt 20 mM ure sẽ làm tăng độ thẩm thấu và
khiến môi trường ngoại bào ưu trương so với dịch nội bào. Nước ban đầu sẽ đi ra
ngoài tế bào, nhưng vì màng bào tương thấm ure nên ure sẽ đi vào tế bào để cân
bằng nồng độ giữa hai bên màng. Điều này dẫn tới nước quay lại tế vào và tế bào
trở về thể tích ban đầu.
2. Tính độ thẩm thấu của dịch chứa 12 mM NaCl, 4 mM KCl, và 2 mM CaCl2 theo
mOsm/L
A) 16
B) 26
C) 29
D) 32
E) 38
F) 42
 E) Dịch 1 mM sẽ có độ thẩm thấu 1 mOsm khi phân tử chất tan không phân ly.
Tuy nhiên, NaCl và KCl đều phân ly thành hai ion và CaCl2 phân ly thành 3 ion;
nên 12 mM NaCl có độ thẩm thấu 24 mOsm, 4 mM KCl có độ thẩm thấu 8 mOsm,
và 2 mM CaCl2 có độ thẩm thấu 6 mOsm. Cộng lại ta có dung dịch 38 mOsm.
Câu hỏi 3-6

Nội bào (mM) Ngoại bào


(mM)
140 K+ 14 K+
10 Na+ 100 Na+
-
11 Cl 110 Cl-
-4 ++ 2
10 Ca Ca++
Bảng trên cho nồng độ 4 ion qua màng của tế bào mẫu. Dựa vào bảng để trả lời 4
câu hỏi sau đây.

3. Điện thế khuếch tán Cl- qua màng tế bào này?


A) 0 mV
B) 122 mV
C) −122 mV
D) 61 mV
E) −61mV
 E) Điện thế khuếch tán Clo (ECl-), một anion hóa trị 1, được tính dựa trên
phương trình Nernst:
ECl- (mV) – 61 x log (Ci/Co), với Ci là nồng độ nội bào (trong màng) và Co là nồng
độ ngoại bào (ngoài màng).
Trong trường hợp này, ECl- = 61 x log (11/110) = - 61 mV
4. Điện thế khuếch tán K+ qua màng tế bào này?
A) 0 mV
B) 122 mV
C) −122 mV
D) 61 mV
E) −61mV
 E) Điện thế khuếch tán Kali (EK+), một cation hóa trị 1, được tính dựa trên
phương trình Nernst:
EK+ (mV) = – 61 x log (Ci/Co). Ta có EK+ = – 61 x log (140/14) = - 61 mV
5. Nếu điện thế màng của tế bào này bằng 80 mV, lực tác động mạnh nhất lên ion
nào?
A) Ca++
B) Cl-
C) K+
D) Na+
 A) Về mặt định lượng, lực tác động lên ion là sự chênh mV giữa điện thế nghỉ
của màng (Vm) và điện thế khuếch tán ion đó (Eion). Ở tế bào này, EK = -61 mV, ECl
= -61 mV, ENa = +61 mV, và ECa = 525 mV. Vì vậy, Ca++ là ion có độ chênh lệch
giữa điện thế khuếch tán và điện thế nghỉ lớn nhất. Việc này đồng nghĩa Ca++ dễ
qua màng khi mở kênh qua một kênh mở nhất (trong ví dụ cụ thể này).
6. Nếu tế bào chỉ cho K+ đi qua, việc giảm nồng độ K+ ngoài màng từ 14 xuống 1.4
mM sẽ có tác động như thế nào?
A) Khử cực 10 mV
B) Ưu phân cực 10 mV
C) Khử cực 122 mV
D) Ưu phân cực 122 mV
E) Khử cực 61 mV
F) Ưu phân cực 61 mV
 F) Nếu một màng chỉ cho một ion duy nhất đi qua, Vm sẽ bằng điện thế khuếch
tán của ion đó. Ở tế bào này, EK = -61 mV. Nếu nồng độ K+ ngoài màng giảm 10
lần, EK = 61 x log (1.4/140) = -122 mV, tức ưu phân cực 61 mV.
7. Biểu đồ sau biểu thị mối quan hệ giữa độ dài-lực căng của một đơn vị cấu trúc
ngắn (sarcomere). (Dữ liệu của Gordon AM, Huxley AF, Julian FJ: Biểu đồ độ
dài-lực căng của cơ vân loài đơn xương sống. J Physiol 171:28P, 1964.) Vì sao
lực căng đạt cực đại ở khoảng giữa B và C?

Lực
căn
g
tăn
g
(%)

Chiều dài sarcomere (micromet)

A) Các sợi actin xếp chồng lên nhau


B) Các sợi myosin xếp chồng lên nhau
C) Sợi myosin co ngắn cực đại
D) Các đĩa Z của sarcomere tiếp giáp điểm tận của sợi myosin
E) Các sợi actin và myosin chồng tối ưu lên nhau
F) Các sợi actin và myosin chồng tối thiểu lên nhau
 E) Sự tăng lực căng ở một sarcomere đơn lẻ trực tiếp liên quan tới số cầu nối
myosin hoạt động gắn với các sợi actin. Sự chồng nau giữa myosin và actin tối ưu
ở độ dài sarcomere trong khoảng 2.0 tới 2.5 micromet, điều này tạo kết nối tối đa
giữa đầu myosin và sợi actin. Ở độ dài dưới 2.0 micromet, các sợi actin lấn vào dải
H, vị trí không có đầu myosin. Ở độ dài lớn hơn 2.5 micromet, các sợi actin bị kéo
tới phần cuối của sợi myosin, làm giảm số cầu nối có thể tạo ra.
8. Khuếch tán đơn thuần và khuếch tán được thuận hóa có chung đặc trưng nào sau
đây?
A) Bị chặn lại bởi các chất ức chế đặc hiệu
B)Không cần ATP
C) Cần protein mang
D) Là các quá trình động học bão hòa
E) Vận chuyển chất tan ngược chiều gradient nồng độ
 B) Không như các hình thức vận chuyển bậc I và II, khuếch tán đơn thuần và
khuếch tán được thuận hóa không cần thêm năng lượng và, bởi vậy, có thể hoạt
động khi không có ATP. Chỉ khuếch tán được thuận hóa hình thành động học bão
hòa và liên quan tới protein mang. Theo định nghĩa, cả hai hình thức này không thể
đưa phân tử từ nơi có nồng độ thấp sang nơi có nồng độ cao. Chất ức chế đặc hiệu
không ảnh hưởng tới khuếch tán đơn thuần vốn chỉ xảy ra ở lớp màng lipid kép
không có sự trợ giúp của protein
9. Quá trìnhkích thích-co cơ ở cơ vân bao gồm tất cả các hiện tượng sau đây,
NGOẠI TRỪ?
A) Thủy phân ATP
B) Ca++ gắn vào calmodulin
C) Thay đổi cấu trúc receptor dihydropyridine
D) Sự khử cực tại màng của hệ thống ống ngang (hệ thống ống T)
E) Tăng độ dẫn với Na+ của màng sợi cơ
 B) Quá trình kích thích-co cơ ở cơ vân khởi đầu bởi sự khử cực của màng cơ
vân (sarcolemma). Sự khử cực này kích hoạt cơ chế mở tất cả-hoặc-không của các
kênh Na+ nhạy cảm với điện thế và điện thế hoạt động được dẫn truyền vào sâu
trong sợi cơ qua hệ thống ống T. Ở thể ba lưới bào tương-ống T, sự khử cực của
ống T gây ra thay đổi cấu trúc của receptor dihydropyridine và sau đó là của
receptor ryanodine ở lưới bào tương. Quá trình sau dẫn tới việc Ca++ được giải
phóng vào bào tương và gắn vào troponin C (không phải calomodulin) ở lá actin.
10. Một động tác co đơn thuần của cơ vân dễ bị khử bởi hành động nào sau đây
nhất?
A) Đóng thụ thể acetylcholine nicotin sau synap
B) Loại bỏ acetylcholine khỏi liên kết thần kinh-cơ
C) Loại bỏ Ca++ khỏi đầu neuron vận động
D) Loại bỏ Ca++ bào tương
E) Thụ thể dihydropyridine trở về cấu trúc dạng nghỉ của nó
 D) Sự co cơ vân được điều hòa chặt chẽ bởi nồng độ Ca++ trong bào tương.
Chừng nào nồng độ Ca++ bào tương còn cao, không hiện tượng nào trong số: loại
bỏ acetylcholine khỏi liên kết thần kinh cơ, loại bỏ Ca++ khỏi đầu trước synap,
đóng kênh receptor acetylcholine, hay cấu trúc receptor dihydropyridine trở về
trạng thái nghỉ gây ra ảnh hưởng lên trạng thái co của cơ.
11. Câu nào sau đây mô tả chính xác nhất về sự co cơ trơn?
A) Không phụ thuộc vào Ca++
B) Không cần điện thế hoạt động
C) Cần nhiều năng lượng hơn cơ vân
D) Tổng thời gian co cơ ngắn hơn cơ vân
 B) Khác với cơ vân, cơ trơn có thể được kích thích co mà không cần hình thành
điện thế hoạt động. Ví dụ, cơ trơn co đáp ứng với bất kỳ kích thích nào gây tăng
nồng độ Ca++ nội bào. Bao gồm việc mở kênh Ca++, trì hoãn khử cực, và nhiều
yếu tố mô và hormone tuần hoàn nào gây giải phóng nguồn Ca++ nội bào.
Quá trình co cơ trơn sử dụng ít năng lượng hơn và kéo dài hơn co cơ vân. Quá
trình này phụ thuộc rất nhiều vào Ca++.
12. Câu nào sau đây mô tả đúng nhất đặc tính có ở cơ trơn nội tạng nhưng không
thấy ở cơ vân?
A) Co cơ phụ thuộc vào ATP
B) Có động tác co đáp ứng khi căng cơ
C) Không có các sợi actin
D.) Tỷ lệ cầu nối cao
E) Lực co cơ tối đa thấp
 B) Một đặc trưng quan trọng của cơ trơn nội tạng là khả năng co đáp ứng với sự
căng cơ. Căng cơ gây khử cực và hình thành điện thế hoạt động. Những điện thế
hoạt động này, phối hợp với những điện thế sóng chậm thông thường, tạo nên
những đáp ứng co nhịp nhàng. Cũng như cơ vân, sự co cơ trơn phụ thuộc vào cả
actin và ATP. Tuy nhiên, chu kỳ cầu nối ở cơ trơn chậm hơn đáng kể so với ở cơ
vân, điều này cho phép lực co tối đa đạt được cao hơn.
13. Điện thế nghỉ của sợi thần kinh bọc myelin chủ yếu phụ thuộc vào gradient
nồng độ của ion nào sau đây?
A) Ca++
B) Cl−
C) HCO3−
D) K+
E) Na+
 D) Điện thế nghỉ ở bất kỳ tế bào nào cũng phụ thuộc vào gradient nồng độ của
các ion qua màng và các chất đồng vận của chúng (phương trình Goldman). Ở sợi
thần kinh myelin hóa, cũng như hầu hết mọi tế bào, màng nghỉ hầu như luôn thấm
K+. Điện tích âm của màng thấy ở hầu hết mọi tế bào (kể cả các tế bào thần kinh)
là do nồng độ trong màng và độ thấm cao của K+.
14. Calmodulin có cấu trúc và chức năng tương tự nhất với protein nào sau đây?
A) G-actin
B) Myosin chuỗi nhẹ
C) Tropomyosin
D) Troponin C
 D) Ở cơ trơn, việc gắn 4 ion Ca++ với protein calmodulin cho phép tương tác
giữa phức hợp Ca++-calmodulin với kinase myosin chuỗi nhẹ. Phản ứng này kích
hoạt kinase myosin chuỗi nhẹ, và cuối cùng dẫn tới co cơ. Ở cơ vân, dấu hiệu kích
hoạt Ca++ được nhận bởi troponin C. Cũng như calmodulin, mỗi phân tử troponin
C có thể gắn với 4 ion Ca++. Từ đó, cấu trúc protein troponin C thay đổi, dẫn tới
phân tử tropomyosin bị đẩy ra và bộc lộ vùng hoạt động của lá actin.
15. Đâu là kết quả của sự myelin hóa các sợi thần kinh lớn?
A) Giảm tốc độdẫn truyền xung thần kinh
B) Điện thế hoạt động chỉ hình thành ở nút Ranvier
C) Tăng nhu cầu năng lượng để duy trì các gradient nồng độ
D) Tăng điện dung màng (điện dung đại diện cho khả năng dẫn điện của một chất)
E) Tăng khuếch tán không chọn lọc các ion qua màng sợi trục
 B) Sự myelin hóa sợi trục các dây thần kinh lớn cho các kết quả sau: Nó giúp
cách ly màng sợi trục, giảm điện dung màng và vì vậy giảm sự rò rỉ các ion qua
màng. Các điện thế hoạt động ở những sợi trục được myelin hóa chỉ hình thành ở
những khoảng cách đều không bọc myelin được gọi là nút Ranvier. Những kênh
Na+ cổng điện thế tập trung tại những nút này. Sự sắp xếp này vừa làm tăng tốc độ
dẫn truyền xung thần kinh dọc sợi trục và giảm thiểu tối đa số điện tích qua màng
trong một xung, từ đó giảm thiểu lượng năng lượng cần cho bơm Na+, K+ -
ATPase lập lại gradient nồng độ Na+ và K+ tương ứng.
16. Trong một thí nghiệm minh họa cho sinh viên y khoa, một bác sỹ chuyên khoa
thần kinh sử dụng kích thích từ trường vào vỏ nảo để kích thích dây thần kinh trụ
của tình nguyện viên. Với kích thích cường độ thấp, điện thế hoạt động được ghi
nhận ở ngón tay trỏ. Khi tăng cường độ kích thích, điện thế hoạt động được ghi
nhận ở các sợi cơ ở cả ngón trỏ và cơ tam đầu. Nguyên lý cơ bản nào giải thích cho
đáp ứng phụ thuộc cường độ này?
A) Các neuron vận động lớn chi phốinhữngvùng vận động lớnyêu cầu kích thích
khử cực mạnh hơn
B) Triệu hồi nhiều vùng vận động yêu cầu kích thích khử cực mạnh hơn
C) Cơ tam đầu bịchi phối bởi nhiều neuron vận động hơn
D) Các vùng vận động của cơ tam đầu nhỏ hơn các vùng chi phối của cơ ngón tay
E) Các cơ ngón tay chỉ do thần kinh trụ chi phối
 A) Các sợi cơ liên quan tới kiểm soát vận động tinh tế nhìn chung được chi phối
với các neuron vận động nhỏ tương ứng với các vùng vận động nhỏ - bao gồm các
vùng chi phối các sợi cơ đơn. Những neuron này kích hoạt đáp ứng với các kích
thích khử cực nhỏ hơn so với các neuron vận động của cùng vận động lớn hơn. Vì
vậy, trong quá trình co cơ yếu, sự tăng co cơ có thể diễn ra theo từng bước nhỏ,
cho phép kiểm soát vận động tinh tế. Khái niệm này được gọi là nguyên lý kích
thích.
17. Cơ trơn và cơ vân giống nhau ở?
A) Khả năng co cơ khi không có điện thế hoạt động
B) Co cơ phụ thuộc ion Ca++
C) Sự có mặt của hệ thống ống T
D) Vai trò của myosin kinase trong co cơ
E) Sự sắp xếp chồng nhau của các sợi actin và myosin
 B) Điểm chung lớn nhất của quá trình co cơ trơn, cơ vân và cơ tim là vai trò
khởi động co cơ của Ca++. Các cơ tim và cơ vân có một số đặc trưng khác với cơ
trơn. Ví dụ, các protein co cơ của cơ tim và cơ vân nằm trong những đơn vị co cơ
riêng biệt. Cả hai loại cơ này đều có hệ thống ống T và cần điện thế hoạt động để
hình thành co cơ. Trong khi đó, cơ trơn có cấu trúc đơn giản hơn, chỉ hoạt động
dựa trên sự phosphoryl hóa myosin chuỗi nhẹ, và có thể co khi không có điện thế
hoạt động.
18. Ở môt cơ bình thường, khỏe mạnh, điều gì sẽ xảy ra khi lan truyền một điện thế
hoạt động trên màng của một neuron vận động?
A) Mở các kênh Ca++ cổng điện thế ở màng trước synap
B) Kéo theo sự khử cực ở màng của hệ thống ống T
C) Luôn gây ra co cơ
D) Tăng nồng độ Ca++ nội bào ở đầu neuron vận động
E) Tất cả các ý trên đều đúng
 E) Kết nối thần kinh cơ có một thành phần được gọi là yếu tố bảo vệ với chức
năng đảm bảo mọi xung thần kinh đi tới đầu tận của neuron vận động sẽ tạo ra một
điện thế hoạt động ở màng cơ vân. Trên cơ khỏe mạnh, bình thường, điều này dẫn
tới đáp ứng co cơ. Độ nhậy với điện thế của các kênh Ca++ ở màng trước synap và
nồng độ Ca++ ngoài màng cao hình thành một dòng Ca++ đủ để kích thích sự tập
hợp các túi synap vào màng trước synap và giải phóng acetylcholin. Acetylcholine
được giải phóng trở nên dư thừa dẫn đến khử cực ở màng sau synap và tạo nên
điện thế hoạt động.
19. Trong co cơ vân có sự giảm chiều dài của:
A) Băng A của đơn vị co cơ
B) Băng I của đơn vị co cơ
C) Các lá cơ dày
D) Các lá cơ mỏng
E) Các đĩa Z của đơn vị co cơ
B) Độ dài vật lý của các lá actin và myosin không thay đổi trong quá trình co cơ.
Vì vậy, băng A, tạo bởi các lá myosin, cũng không thay đổi. Khoảng cách giữa các
đĩa Z giảm, nhưng bản thân đĩa Z không thay đổi. Chỉ có băng I ngắn lại khi cơ co.
20. Hình cắt ngang của sợi cơ vân qua vùng H đi qua thành phần nào sau đây?
A) Actin và titin
B) Actin, không có myosin
C) Actin, myosin, và titin
D) Myosin và actin
E) Myosin, không có actin
 E) Vùng H là khu vực trung tâm của đơn vị co cơ, bao gồm các băng nhẹ hơn
của cả 2 bên và đường M. Ở vùng này, các lá myosin là trung tâm của đường M, và
không có sự xếp chồng của các lá actin, Vì vậy, lát cắt ngang chỉ cho thấy hình ảnh
của myosin.
21. Cơn co tetani xảy ra là do tăng nồng độ nội bào của chất nào sau đây?
A) ATP
B) Ca++
C) K+
D) Na+
E) Troponin
 B) Co cơ phụ thuộc vào sự tăng nồng độ Ca++ nội bào. Khi tần suất co tăng,
các đợt co tiếp theo thậm chí bắt đầu trước khi đợt co trước kết thúc. Như vậy,
cường độ của các cơn co riêng biệt được cộng dồn. Ở tần số co rất cao sẽ xảy ra
cơn co tetani. Dưới những điều kiện này, Ca++ nội bào tập trung và giữ mức co tối
đa trong giới hạn.
22. Tăng thân nhiệt ác tính là một rối loạn di truyền nguy cơ tử vong đặc trưng bởi
sự đáp ứng quá mức với thuốc mê dạng hít và dẫn tới nhiệt độ cơ thể tăng vọt, co
cứng cơ vân, và toan chuyển hóa lactic. Thay đổi cấp độ phân tử nào sau đây giải
thích cho những biểu hiện lâm sàng này?
A) Giảm độ nhạy hiệu thế của receptor dihydropyridine
B) Tăng hoạt động của Ca++-ATPase lưới nội bào chất tế bào cơ
C) Tăng thời gian mở kênh receptor ryanodine
D) Giảm mật độ các kênh Na+ nhạy cảm điện thế ở màng hệ thống ống T
 C) Chừng nào kênh receptor ryanodine ở lưới nội chất tế bào cơ còn mở, Ca++
sẽ tiếp tục đi vào bào tương cơ và kích thích co cơ. Sự co cơ kéo dài này sinh nhiệt,
gây co cứng cơ, và dẫn tới tình trạng toan hóa lactic. Ngược lại, các yếu tố ức chế
giải phóng Ca++ hay kích thích hấp thụ Ca++ vào lưới nội chất, hoặc ngăn cản sự
khử cực của màng hệ thống ống T hay dẫn truyền khử cực để giải phóng Ca++ sẽ
giúp giãn cơ.
23. Tập thể hình có thể làm tăng đáng kể khối cơ vân chủ yếu do?
A) Tập hợp các sarcomere giữa những tơ cơ liền kề
B) Phì đại các sợi cơ riêng biệt
C) Tăng cấp máu cho cơ vân
D) Tăng số lượng các neuron vận động
E) Tăng số lượng các khớp thần kinh-cơ
 B) Co cơ tối đa lặp lại hoặc kéo dài dẫn tới tăng phối hợp sự tổng hợp các
protein co cơ và khối cơ nói chung. Sự tăng khối cơ, hay phì đại cơ, quan sát thấy
ở mức sợi cơ riêng biệt.
24. Cơ chế vận chuyển nào sau đây không bị giới hạn tỷ lệ bởi Vmax nội màng?
A) Khuếch tán tạo thuận qua protein mang
B) Vận chuyển chủ động sơ cấp qua protein mang
C) Đồng vận chuyển cùng chiều thứ cấp
D) Đồng vận chuyển ngược chiều thứ cấp
E) Khuếch tán đơn giản qua kênh protein
 E) Khuếch tán tạo thuận và vận chuyển chủ động sơ cấp và thứ cấp đều cần
protein mang hoặc protein vận chuyển và cần thay đổi cấu hình. Tỷ lệ khuếch tán
đơn thuần liên quan tuyến tính với nồng độ dung dịch.
25. Giả sử mọi chất tan phân ly hoàn toàn, dung dịch nào sau đây ưu trương so với
1 mmol NaCl?
A) 1 mmol CaCl2
B) 1 mmol glucose
C) 1 mmol KCl
D) 1 mmol sucrose
E) 1.5 mmol glucose
 A) Thuật ngữ ưu trương chỉ một dung dịch có độ thẩm thấu cao hơn dung dịch
so sánh. Độ thẩm thấu của dung dịch 1 mmol NaCl là 2 mOsm/L. Độ thẩm thấu
của dung dịch 1 mmol glucose hay sucrose đều là 1 mOsm/L. Độ thẩm thấu của
dung dịch 1.5 mmol glucose là 1.5 mOsm/L. Các dung dịch này đều nhược trương
so với dung dịch 1 mmol NaCl. Độ thẩm thấu của dung dịch 1 mmol KCl là 2
mOsm/L. Dung dịch này đẳng trương với dung dịch 1 mmol NaCl. Chỉ có dung
dịch 1 mmol CaCl2 với độ thẩm thấu 3 mOsm/L là ưu trương so với dung dịch 1
mmol NaCl.
Câu hỏi 26 và 27
Biểu đồ cho thấy sự thay đổi điện thế màng trong quá trình hình thành điện thế
hoạt động ở sợi trục mực khổng lồ (đường kính khoảng 0.5mm, đóng vai trò trong
hệ thống đẩy nước phản lực ở mực).

Dựa vào đó trả lời hai câu hỏi sau.


26. Sự thay đổi điện thế màng giữa điểm B và D chủ yếu do?
A) Ức chế bơm Na+, K+-ATPase
B) K+ đi vào tế bào
C) K+ đi ra ngoài tế bào
D) Na+ đi vào tế bào
E) Na+ đi ra ngoài tế bào
 D) Ở điểm B ở điện thế hoạt động này, Vm đã đạt tới điện thế ngưỡng, và kích
hoạt mở các kênh Na+ cổng điện thế. Dòng Na+ đi vào dẫn tới pha khử cực nhanh
tự duy trì của điện thế hoạt động.
27. Sự thay đổi điện thế màng giữa điểm D và E chủ yếu do?
A) Ức chế bơm Na+, K+-ATPase
B) K+ đi vào tế bào
C) K+ đi ra ngoài tế bào
D) Na+ đi vào tế bào
E) Na+ đi ra ngoài tế bào
 C) Pha khử cực nhanh kết thúc ở điểm D bởi sự bất hoạt các kênh Na+ cổng
điện thế và mở các kênh K+ cổng điện thế. Sự mở các kênh K+ dẫn tới dòng K+ đi
từ bào tương vào dịch ngoại bào và gây khử cực ở màng tế bào.
28. Việc cơ trơn co muộn, thời gian co kéo dài và cần lực khởi động co lớn hơn so
với cơ vân là do?
A) Cơ trơn có nhiều sợi myosin hơn
B) Cơ trơn có nhu cầu năng lượng cao hơn
C) Sự sắp xếp các lá myosin và actin
D) Tỷ lệ quay vòng các cầu nối myosin ở cơ trơn chậm hơn
E) Sự hấp thu Ca++ sau co cơ chậm hơn
D) Mức quay vòng các cầu nối ở cơ trơn chậm hơn đồng nghĩa với phần trăm
các cầu nối có thể hoạt động ở một thời điểm bất kỳ cao hơn. Càng có nhiều cầu
nối hoạt động, lực cần có để khởi động càng mạnh. Mặc dù tỷ lệ quay vòng chậm
có nghĩa sẽ mất thời gian hơn để đầu myosin gắn với lá actin, song cũng đồng
nghĩa với thời gian gắn dài hơn, và co cơ lâu hơn. Nhờ tỷ lệ quay vòng cầu nối
chậm, cơ trơn thực chất cần ít năng lượng hơn để duy trì co cơ so với cơ vân
29. Một loại thuốc đang được thử nghiệm như một liệu pháp điều trị hứa hẹn cho
bệnh hen. Các nghiên cứu tiền lâm sàng chỉ ra rằng loại thuốc này dẫn tới giãn các
tế bào cơ trơn khí quản lợn trong môi trường nuôi cấy đã qua gây co bởi
acetylcholine. Cơ chế hoạt động nào nhiều khả năng gây ra hiệu ứng này?
A) Giảm ái lực của troponin với Ca++
B) Giảm độ thấm màng với K+
C) Tăng độ thấm màng với Na+
D) Ức chế bơm Ca++-ATPase lưới nội chất tế bào cơ
E) Kích hoạt adenylate cyclase
 E) Sự kích hoạt adenylate hay guanylate cyclase đều gây giãn cơ trơn. Các
nucleotid vòng được sản xuất bởi các enzym này lần lượt kích hoạt các kinase phụ
thuộc cAMP và cGMP. Các kinase này phosphoryl hóa những enzym loại Ca++
khỏi bào tương, và vì vậy ức chế sự co cơ. Ngược lại, việc giảm độ thấm màng với
K+ hay tăng độ thấm với Na+ đều gây khử cực màng và co cơ. Tương tự, ức chế
Ca++-ATPase (một trong các enzym được kích hoạt bởi các kinase phụ thuộc
nucleotid vòng) ở lưới nội chất tế bào cơ cũng gây co cơ. Các cơ trơn không tiết
troponin.
Câu hỏi 30 và 31
Biểu đồ sau mô tả các đặc trưng của quá trình giật cơ đẳng trường của hai cơ vân A
– B đáp ứng với một kích thích khử cực.

Dựa vào đó trả lời hai câu hỏi sau.


30. Câu nào sau đây mô tả đống nhất về cơ B khi đối chiếu với cơ A?
A) Thích hợp với co cơ nhanh
B) Cấu tạo bởi các sợi cơ lớn hơn
C) Ít ty thể hơn
D) Được chi phối bởi các dây thần kinh nhỏ hơn
E) Cần cấp máu ít hơn
 D) Cơ B được đặc trưng bởi cơ giật chậm (loại 1), cấu tạo hầu hết từ các sợi cơ
giật chậm. Những sợi này có kích thước nhỏ hơn và được chi phối bởi các dây thần
kinh nhỏ hơn. Chúng thường nhận lượng máu cấp nhiều hơn, nhiều ty thể hơn và
số lượng lớn myoglobin; từ đó hỗ trợ quá trình phosphoryl oxy hóa mức độ cao.
31. Khoảng nghỉ giữa thời điểm cuối khử cực màng và khởi đầu co cơ ở cả hai cơ
A và B phản ánh thời gian cần thiết cho hiện tượng nào sau đây xảy ra?
A) Giải phóng ADP từ đầu myosin
B) Tổng hợp ATP
C) Tích lũy Ca++ trong chất tế bào cơ
D) Trùng hợp G-actin và F-actin
E) Đầu myosin hoàn thiện một vòng cầu nối
 C) Co cơ được khởi động bởi tăng nồng độ Ca++ trong bào tương cơ. Khoảng
nghỉ giữa nhịp khử cực và khởi đầu co cơ, còn được gọi là “lag”, phản ánh thời
gian cần thiết để khử cực gây tăng nồng độ Ca++ bào tương cơ. Quá trình này liên
quan tới sự thay đổi cấu trúc của receptor nhận cảm điện thế (receptor
dihydropyridine) ở màng ống T; kéo theo thay đổi cấu trúc receptor ryanodine ở
lưới nội chất tế bào cơ, và giải phóng Ca++ từ lưới này.
Câu hỏi 32-34
Một phụ nữ 55 tuổi đi khám vì nhìn đôi, sụp mi, nhai nuốt khó, và yếu chi. Triệu
chứng tăng lên khi tập thể dục và xảy ra nhiều hơn về cuối ngày. Bác sỹ nghĩ nhiều
tới chẩn đoán nhược cơ và chỉ định test Tensilon, cho kết quả dương tính.
32. Tăng sức cơ trong quá trình thực hiện test Tensilon là do tăng?
A) Lượng acetylcholine (ACh) giải phóng từ các sợi thần kinh vận động
B) Nồng độ ACh ở các đĩa tận cơ
C) Số lượng receptor Ach ở các đĩa tận cơ
D) Sự tổng hợp nonepinephrine
 B) Nhược cơ là một bệnh tự miễn mà các kháng thể hủy hoại receptor ACh
nicotinic sau synap. Sự hủy hoạt này ngăn cản khởi động điện thế hoạt động ở
màng sau synap. Tensilon là một chất ức chế acetylcholinesterase có hồi phục làm
tăng nồng độ acetylcholine trong khớp thần kinh-cơ, từ đó tăng sức co cơ.
33. Giải thích nào sau đây có căn cứ nhất cho các triệu chứng được mô tả ở bệnh
nhân này?
A) Phản ứng tự miễn
B) Ngộ độc botulinum
C) Giảm các kênh Ca++ cổng điện thế ở những neuron vận động nhất định
D) Sự phát triển của các đơn vị vận động lớn sau bại liệt
E) Quá gắng sức
 A) Nhược cơ là một bệnh tự miễn do sự xuất hiện của kháng thể với receptor
anti-acetylcholine trong huyết tương. Gắng sức quá mức có thể làm mỏi khớp, và
cả sự giảm mật độ kênh Ca++ nhạy cảm điện thế ở màng trước synap và ngộ độc
botulinum có thể gây yếu cơ. Tuy nhiên, những tác động này xảy ra trước synap và
bởi vậy không liên quan tới ức chế acetylcholinesterase. Mặc dù các đơn vị vận
động lớn hình thành để chi phối và kiểm soát vận động tinh tế của bệnh nhân trong
quá trình hồi phục sau bại liệt, chúng không ảnh hưởng tới sức cơ.
34. Thuốc nào sau đây làm cải thiện triệu chứng của bệnh nhân này?
A) Atropine
B) Kháng huyết thanh Botulinum
C) Cura
D) Halothane
E) Neostigmine
 E) Neostigmine là một chất ức chế acetylcholinesterase. Dùng thuốc này sẽ làm
tăng lượng ACh trong synap và khả năng gây phân cực tại màng sau và kích hoạt
điện thế hoạt động của chất này. Kháng huyết thanh độc tố botulinum chỉ có tác
dụng với độc tố botulinum. Các cura chặn receptor ACh hệ nicotinic và gây yếu
cơ. Atropine là một chất đối kháng ACh hệ muscarinic, và halothane là một khí
gây mê; cả hai chất này không tác dụng trên khớp thần kinh-cơ.
35. Các sơ đồ dưới đây mô tả hai thùng chứa, mỗi thùng gồm hai buồng ngăn A và
B chứa dung dịch Na+ và cách nhau bởi một màng thấm Na+. Sơ đồ bên trái biểu
thị sự phân bố của các ion Na+ ở trạng thái nghỉ khi không có điện thế tác động. Ở
trường hợp này, nồng độ của ion Na+ giữa hai buồng A và B bằng nhau ([Na]A =
[Na]B). Sơ đồ bên phải minh họa tác động của một dòng điện thế +60-millivolt lên
màng (buồng B so với buồng A). Cho nhiệt độ là 37°C, kết quả sự phân bố Na+
giữa hai buồng như thế nào?
A) [Na]A = 10[Na]B
B) [Na]A = 2[Na]B
C) [Na]A = 60[Na]B
D) [Na]B = 10[Na]A
E) [Na]B = 60[Na]A
 D) Khi thêm một điện thế +60mV vào buồng B, các ion Na+ mang điện tích
dương sẽ đi từ buồng B sang buồng A tới khi lực khuếch tán theo gradient nồng độ
đủ để cản lực điện động. Sử dụng phương trình Nernst, một lực điện động 60 mV
sẽ được khử bởi gradient nồng độ Na+ gấp 10 lần. Vì vậy, ở trạng thái cân bằng
mới, nồng độ Na+ ở buồng A sẽ gấp 10 lần ở buồng B.
Câu hỏi 36-38
Sơ đồ sau đây biểu diễn mối quan hệ đẳng trường giữa độ căng cơ-chiều dài ở một
cơ vân điển hình nguyên vẹn.

Trả lời câu hỏi bằng các chữ cái tương ứng trong sơ đồ trên.
36. Sức căng cơ “hoạt động” hay sức căng cơ gây co cơ.
 B) Ở biểu đồ này, sự co phụ thuộc sức căn bằng hiệu của sức căng tổng (A) và
sức căng thụ động tạo bởi các thành phần không gây co cơ (C). Mối quan hệ chiều
dài-sức căng ở cơ nguyên vẹn thể hiện mối quan hệ hai pha thấy ở các đơn vị cơ
(sarcomere) độc lập và phản ánh các tương tác vật lý tương tự giữa các lá actin và
myosin.
37. Chiều dài cơ khi sức căng cơ hoạt động hoạt động đạt tối đa.
 E) Sức căng “hoạt động” đạt cực đại ở chiều dài cơ sinh lý bình thường. Tại
điểm này, có sự xếp chồng lý tưởng giữa các lá actin và myosin để giúp hình thành
số cầu nối tối đa và hình thành sức căng.
38. Vai trò của các thành phần không gây co cơ lên tổng sức căng.
 C) Điểm C biểu thị sức căng thụ động bởi các thành phần không gây co cơ, gồm
màng cơ, gân, và dây chằng. Sức căng thụ động là thành phần tăng của sức căng
tổng khi cơ nguyên vẹn (lành) bị kéo căng quá chiều dài bình thường của nó.
39. Yếu tố gây dừng sự co cơ trơn?
A) Khử phosphoryl hóa myosin kinase
B) Khử phosphoryl hóa chuỗi nhẹ myosin
C) Dòng ion Ca++ qua màng huyết tương
D) Ức chế myosin phosphatase
E) Ca++ đi vào lưới nội chất tế bào cơ
. B) Sự co cơ trơn được điều hòa bở cả Ca++ và sự phosphoryl hóa chuỗi nhẹ
myosin. Khi nồng độ Ca++ trong tế bào chất giảm sau khi khởi động co cơ, myosin
kinase bị bất hoạt. Tuy nhiên, sự hình thành cầu nối vẫn tiếp tục, cả khi không có
Ca++, tới khi các chuỗi nhẹ myosin bị dephosphoryl hóa nhờ hoạt động của
phosphatase chuỗi nhẹ myosin.
Câu hỏi 40-42
Một bệnh nhân nam 56 tuổi khám chuyên khoa thần kinh vì yếu hai chân tăng dần
trong ngày và khi tập luyện. Điện cơ ngoài màng ghi được ở từng sợi cơ vân cho
thấy các điện thế cuối đĩa bình thường. Tuy nhiên kích thích điện tần số thấp
neuron vận động gây ra một khử cực nhỏ bất thường ở các sợi cơ. Cường độ của
sóng khử cực tăng sau khi tập luyện.
40. Dựa vào các kết quả trên, yếu chân trên bệnh nhân này nhiều khả năng do
nguyên nhân nào sau đây?
A) Thiếu hụt acetylcholinesterase
B) Phong bế các receptor acetylcholine sau synap
C) Giảm dòng Ca++ nhạy cảm điện thế trước synap
D) Ức chế sự tái hấp thu Ca++ vào lưới nội chất tế bào cơ
E) Giảm tổng hợp acetylcholine
 C) Các điện thế cuối đĩa nhỏ bình thường cho thấy sự tổng hợp và đóng gói đầy
đủ ACh, cũng như sự có mặt và chức năng bình thường của các kênh receptor
ACh. Giải thích hợp lý nhất cho các triệu chứng ở bệnh nhân này là sự thiếu hụt
trước synap-trong trường hợp này, là việc thiếu kênh Ca++ nhạy cảm điện thế làm
tăng Ca++ bào tương cơ gây kích thích gải phóng ACh vào synap. Tăng khử cực
trước synap thấy sau tập luyện chỉ điểm cho dòng Ca++ tập trung ở màng trước
synap sau khi nhiều điện thế hoạt động liên tiếp đi tới đầu tận thần kinh.
41. Có thể chẩn đoán xác định nếu có yếu tố nào sau đây?
A) Kháng thể kháng receptor acetylcholine
B) Kháng thể kháng kênh Ca++ nhạy cảm điện thế
C) Đột biến gen mã hóa receptor ryanodine
D) Ít túi ở màng trước synap
E) Ít acetylcholine ở khớp thần kinh-cơ
 B) Ức chế các kênh Ca++ nhạy cảm điện thế trước synap nhiều khả năng nhất
do sự có mặt của kháng thể kháng lại kênh này. Các kháng thể kháng receptor
ACh, một đột biến gen tạo receptor ryanodine, và dư thừa ACh ở khớp thần kinh
cơ đều là các lỗi sau synap. Mặc dù là một sai sót trước synap, thiếu túi ACh ít khả
năng xảy ra trong giả thiết này, vì các điện thế nhỏ cuối đĩa bình thường được ghi ở
màng sau synap.
42. Cơ chế phân tử của những triệu chứng trên giống nhất với?
A) Acetylcholine
B) Độc tố Botulinum
C) Curare
D) Neostigmine
E) Tetrodotoxin
 B) Độc tố botulinum ức chế co cơ theo đường trước synap bằng cách giảm
lượng ACh được giải phóng vào khớp thần kinh-cơ. Ngược lại, cura tác động sau
synap, phong bế các receptor ACh hệ nicotinic và ngăn cản kích thích màng tế bào
cơ. Tetrodotoxin phong bế các kênh Na+ nhạy cảm điện thế, tác động lên cả khởi
động và phát động điện thế hoạt động ở neuron vận động. Cả ACh và neostigmin
đều kích thích co cơ.
Câu hỏi 43-45
Nối mỗi mô tả sau với các điểm điện thế hoạt động thần kinh tương ứng trong biểu
đồ sau.

43. Điểm tại đó điện thế màng (Vm) gần nhất với điện thế cân bằng của Na+.
 D) Trong một điện thế hoạt động ở tế bào thần kinh, Vm đạt tới ENa ở pha khử
cực nhanh khi độ thấm của màng với Na+ (PNa) tăng tỷ lệ với độ thấm của nó với
K+ (PK). Ở một tế bào “điển hình”, ENa xấp xỉ 60mV. Vm gần với ENa nhất ở
điểm D của biểu đồ. Tại điểm này, PNa/PK lớn nhất.
44. Điểm tại đó lực tác động lên Na+ lớn nhất.
 F) Lực tác động lên Na+ lớn nhất tại điểm mà Vm xa ENa nhất. Nếu ENa rất
dương (xấp xỉ 60mV), Vm xa ENa nhất ở điểm F, hoặc khi tế bào siêu phân cực
nhất.
45. Điểm tại đó tỷ số giữa độ thấm K+ trên độ thấm Na+ (PK/PNa) lớn nhất.
. F) Nhìn chung, Vm gần nhất với điện thế cân bằng của ion thấm nhất. Ở các tế
bào thần kinh, PK>>PNa ở trạng thái nghỉ. Điều này dẫn tới Vm gần nhất với EK.
Ở pha sau điện thế hay siêu phân cực của điện thế hoạt động, tỷ lệ PK/PNa lớn hơn
khi nghỉ. Điều này do sự mở các kênh K+ nhạy cảm điện thế dư thừa và bất hoạt
các kênh Na+ cổng điện thế. PK/PNa lớn nhất tại điểm F, cũng là điểm Vm cận EK
nhất
46. ATP được sử dụng trực tiếp trong các quá trình sau đây NGOẠI TRỪ?
A) Tích lũy Ca++ ở lưới nội chất tế bào cơ
B) Vận chuyển glucose vào các tế bào cơ
C) Vận chuyển H+ từ các tế bào thành vào lòng dạ dày
D) Vận chuyển K+ từ ngoại bào vào nội bào
E) Vận chuyển Na+ từ ngoại bào vào nội bào
 B) Sự tập trung Ca++ ở lưới nội chất tế bào cơ, sự vận chuyển Na+ vào và K+
ra ngoài tế bào, hay sự vận chuyển K+ từ tế bào thành đều xảy ra chủ yếu nhờ các
cơ chế vận chuyển hoạt động liên quan tới các enzym ATPase. Ở trường hợp này,
chỉ có sự vận chuyển glucose, nhờ khuếch tán được thuận hóa ở cơ, là không trực
tiếp sử dụng ATP.
47. Trong thí nghiệm được minh họa ở sơ đồ A, các thể tích bằng nhau của dung
dịch X, Y, Z được đưa vào các phần của hai bình chữ U như hình vẽ. Hai phần của
mỗi bình được ngăn bởi các màng bán thấm (ví dụ: không cho các ion và phân tử
lớn đi qua). Sơ đồ B minh họa tình trạng các dịch sau khi quá trình cân bằng xảy
ra. Giả sử phân ly hoàn toàn, xác địch các dung dịch X, Y, Z
. B) Sự tái phân bố thể tích dịch ở sơ đồ B phản ánh khuếch tán mạng lưới của
nước, hay sự thẩm thấu, do chênh lệch độ thẩm thấu của các dịch hai bên màng bán
thấm. Quá trình thẩm thấu xảy ra từ dung dịch tập trung nhiều nước sang tập trung
ít nước, hay từ nơi có độ thẩm thấu cao sang nơi có độ thẩm thấu thấp. Ở sơ đồ B,
quá trình thẩm thấu từ X sang Y và từ Y sang Z. Vì vậy, độ thậm thấy của dung
dịch Z cao hơn Y, và Y cao hơn X.
48. Lực tạo ra bởi một sợi cơ vân đơn độc có thể tăng khi?
A) Giảm nồng độ K+ ngoại bào
B) Tăng cường độ kích thích khử cực
C) Tăng tần số kích thích sợi cơ
D) Tăng số kênh Na+ cổng điện thế ở màng bào tương tế bào cơ
E) Tăng độ thấm màng với K+
 C) Tăng nồng độ Ca++ bào tương cơ có thể làm tăng lực tạo ra ở một sợi cơ.
Điều này là do tăng tần số kích thích sợi cơ. Cả việc tăng cường độ khử cực ở
màng sau synap của khớp thần kinh-cơ hay tăng số kênh Na+ cổng điện thế đều
không làm ảnh hưởng tới sự giải phóng Ca++ từ lưới nội chất. Ngược lại, cả việc
giảm nồng độ K+ ngoại bào và tăng tính thấm màng tế bào cơ với K+ đều làm
giảm khả năng bị kích thích của tế bào cơ.
Câu hỏi 49 và 50

49. Đường A mô tả đúng nhất động học của hiện tượng nào sau đây?
A) Sự di chuyển của CO2 qua màng bào tương
B) Sự di chuyển của O2 qua lớp lipid kép
C) dòng Na+ qua kênh receptor acetylcholin hệ nicotinic
D) Vận chuyển K+ vào tế bào cơ
E) Sự di chuyển phụ thuộc điện thế của Ca++ vào đầu tận neuron vận động
 D) Đường A phản ánh động học của quá trình bị giới hạn bởi Vmax nội bào.
Trong những đáp án đã cho, chỉ có sự vận chuyển K+, diễn ra nhờ hoạt động của
bơm Na+-K+-ATPase, là kết quả của vận chuyển chủ động. Các đáp án còn lại đều
là ví dụ minh học của khuếch tán đơn thuần.
50. Đường B mô tả đúng nhất động học của hiện tượng nào sau đây?
A) Sự đồng vận chuyển của glucose theo Na+ ở một tế bào biểu mô
B) Sự vận chuyển Ca++ vào lưới nội chất tế bào cơ của một tế bào cơ trơn
C) Sự vận chuyển K+ vào một tế bào cơ
D) Sự vận chuyển Na+ ra ngoài một tế bào thần kinh
E) Sự vận chuyển O2 qua lớp lipid kép nhân tạo
 E) Đường B biểu diễn một quá trình không bị giới hạn bởi Vmax nội bào. Điều
này loại trừ vận chuyển chủ động và khuếch tán được thuận hóa. Như vậy, trong số
các đáp án đưa ra, chỉ có tỷ lệ của vận chuyển O2 qua lớp lipid kép nhân tạo nhờ
khuếch tán đơn giản có khả năng được phản ánh bởi đường B.
Câu hỏi 51 và 52

51. Đường A biểu diễn một điện thế hoạt động điển hình ghi được dưới những điều
kiện được kiểm soát của một tế bào thần kinh bình thường khi đáp ứng với một
kích thích khử cực. Sự nhiễu loạn nào sau đây giải thích cho quá trình chuyển đổi
từ đáp ứng ở đường A sang điện thế hoạt động ở đường B?
A) Phong bế các kênh Na+ nhạy cảm điện thế
B) Phong bế các kênh K+ nhạy cảm điện thế
C) Phong bế các kênh “rò rỉ” Na-K
D) Thay thế các kênh K+ nhạy cảm điện thế với các kênh Ca++ “chậm”
E) Thay thế các kênh Na+ nhạy cảm điện thế với các kênh Ca++ “chậm”
 E) Các kênh được gọi là kênh Ca++ chậm này có mức bất hoạt chậm hơn, bởi
vậy thời gian mở của chúng cũng được kéo dài. Điều này dẫn tới trì hoãn pha tái
phân cực của điện thế hoạt động, tạo nên một “cao nguyên” trước khi kênh bị bất
hoạt.
52. Nhiễu loạn nào sau đây giải thích cho việc cùng một kích thích không gây ra
điện thế hoạt động tương tự ở đường C?
A) Phong bế các kênh Na+ nhạy cảm điện thế
B) Phong bế các kênh K+ nhạy cảm điện thế
C) Phong bế các kênh “rò rỉ” Na-K
D) Thay thế các kênh K+ nhạy cảm điện thế với các kênh Ca++ “chậm”
E) Thay thế các kênh Na+ nhạy cảm điện thế với các kênh Ca++ “chậm”
. A) Khi không có siêu phân cực, kích thích khởi động điện thế hoạt động không
hình thành nhiều khả năng do kết quả của sự phong bế các kênh cổng điện thế chịu
trách nhiệm kích hoạt sự khử cực tất cả-hoặc-không. Ở các tế bào thần kinh, đó là
các kênh Na+ cổng điện thế.
53. Một cầu thủ bóng đá 17 tuổi bị gãy xương chày trái. Sau khi được nẹp cẳng
chân 8 tuần, cô ấy nhận thấy cơ bụng chân nhỏ đi đáng kể so với trước khi nẹp.
Đâu là giải thích hợp lý nhất cho điều này?
A) Giảm số sợi cơ ở cơ bụng chân trái
B) Giảm tưới máu tới cơ do nẹp bó chặt
C) Giảm tạm thời sự tổng hợp hai protein actin và myosin
D) Tăng hoạt động ly giải đường ở cơ bị tác động
E) Phong bế thần kinh tiến triển
. C) Cơ vân liên tục tái tạo tùy thuộc vào cường độ sử dụng của chúng. Khi một
cơ ở trạng thái không hoạt động trong một thời gian dài, mức tổng hợp các protein
co cơ ở những sợi cơ giảm đi, dẫn tới giảm toàn bộ khối cơ. Sự giảm khối cơ có
hồi phục này được gọi là chứng teo cơ.
54. Cơ trơn co theo nhịp khi không có kích thích bên ngoài là do có?
A) Các kênh Ca++ nhạy cảm điện thế “chậm”
B) Hoạt động của xung tạo nhịp nội bào
C) Nồng độ Ca++ bào tương cơ khi nghỉ cao hơn
D) Điện thế màng siêu phân cực
E) Các điện thế nghỉ với nhiều “cao nguyên”
 B) Để một cơ co tự nhiên và có nhịp điệu, phải có một “máy tạo nhịp” nội bào
có nhịp điệu. Ví dụ, cơ trơn ở ruột biểu hiện điện thế sóng chậm nhịp điệu nhanh
chóng gây khử cực và tái phân cực màng tế bào cơ. Tự sóng chậm này không gây
co cơ, nhưng nếu cường độ đủ mạnh, nó có thể khởi hoạt một hoặc nhiều điện thế
hoạt động hơn dẫn tới dòng Ca++ đi vào và co cơ. Mặc dù chúng là cơ trơn điển
hình, các kênh Ca++ “chậm” nhạy cảm điện thế và các điện thế hoạt động “cao
nguyên” đều không đóng vai trò thiết yếu trong quá trình co cơ theo nhịp. Nồng độ
cao Ca++ bào tương ở trạng thái nghỉ giúp tiếp tục quá trình co cơ, và siêu phân
cực sẽ dừng co cơ.
Câu hỏi 55-59
A) Khuếch tán đơn thuần
B) Khuếch tán tạo thuận
C) Vận chuyển chủ động sơ cấp
D) Đồng vận chuyển cùng chiều
E) Đồng vận chuyển ngược chiều
Nối mỗi quá trình dưới đây với phương thức vận chuyển đúng trên đây (mỗi
phương án có thể được trả lời nhiều hơn một lần).
55. Vận chuyển ion Na+ nhạy cảm với Ouabain từ bào tương cơ ra dịch ngoại bào
 C) Ouabain ức chế Na+, K+-ATPase. Mỗi lần, enzyme phụ thuộc ATP này vận
chuyển ba ion Na+ ra ngoài và hai ion K+ vào tế bào. Đây là ví dụ kinh điển của
phương thức vận chuyển chủ động sơ cấp.
56. Cơ vân tiêu thụ glucose
 B) Glucose được vận chuyển vào các tế bào cơ vân nhờ khuếch tán được thuận
hóa phụ thuộc insulin.
57. Vận chuyển Ca++ phụ thuộc Na+ từ bào tương cơ ra dịch ngoại bào
 E) Hoạt động của Na+, K+-ATPase duy trì nồng độ K+ cao tương đối bên trong
tế bào và Na+ cao tương đối ở dịch ngoại bào. Gradient nồng độ Na+ cao này qua
màng bào tương, cùng với điện tích màng âm trong tế bào, liên tục đẩy ion Na+ từ
dịch ngoại bào vào bào tương cơ. Năng lượng này được sử dụng để vận chuyển các
phân tử khác, như Ca++, ngược chiều gradient nồng độ của chúng. Do ATP là cần
thiết để duy trì gradient Na+ vận hành các quá trình đồng vận chuyển ngược chiều
này, loại hình vận chuyển này được gọi là vận chuyển chủ động thứ cấp.
58. Vận chuyển glucose từ lòng ruột vào tế bào thành ruột
 D) Cũng như đồng vận chuyển thuận chiều Na+-Ca++, hướng vận chuyển qua
màng của Na+ có thể được vận dụng để gắn protein và được sử dụng để đồng vận
chuyển các phân tử ngược gradient nồng độ của chúng vào bào tương. Một ví dụ
của loại hình đồng vận chuyển thứ cấp này là sự vận chuyển glucose vào các tế bào
thành ruột.
59. Sự di chuyển của ion Na+ vào tế bào thần kinh khi điện thế hoạt động đi lên
 A) Trong suốt pha khử cực nhanh của điện thế hoạt động thần kinh, các kênh
Na+ nhạy cảm điện thế mở ra và tạo điều kiện cho dòng Na+ đi vào tế bào chất.
Vận chuyển qua các kênh ở màng là ví dụ của khuếch tán đơn thuần.
60. Các đường A, B, C ở biểu đồ tóm tắt những thay đổi điện thế màng (Vm) và
tính thấm màng (P) ở một tế bào cơ trong suốt một điện thế hoạt động.
Chọn đáp án sắp xếp đúng nhất.
 E) Đường A biểu diễn dạng đặc trưng của một điện thế hoạt động, bao gồm khử
cực nhanh theo sau bởi tái cực nhanh tạm thời đi quá điện thế nghỉ. Đường B mô ta
đúng nhất sự thay đổi PNa trong suốt một điện thế hoạt động. Sự tăng nhanh PNa
đạt tới gần như song song với pha khử cực nhanh của điện thế hoạt động. Đường C
mô tả đúng nhất khởi đầu chậm của sự tăng PK phản ánh sự mở các kênh K+ cổng
điện thế.
61. Nếu nồng độ nội bào của một chất qua được màng tăng gấp đôi từ 10 lên 20
milimol và nồng độ ngoại bào chỉ còn 5 milimol, tỷ lệ khuếch tán của chất đó qua
màng bào tương sẽ tăng bao nhiêu chỉ bởi một yếu tố này?
A) 2
B) 3
C) 4
D) 5
E) 6
 B) Sự khuếch tán lưới của một chất qua màng thấm góp phần (tỉ lệ với) chênh
lệch nồng độ của chất đó hai bên màng. Ban đầu, chênh lệch nồng độ là 5 mmol
(10-5). Khi nồng độ nội bào tăng gấp đôi lên 20 mmol, chênh lệch bấy giờ là 15
mmol (20-5), tức gấp 3 so với lúc trước. Như vậy, tốc độ khuếch tán cũng sẽ tăng
lên 3.
62. Cặp dung dịch (dung môi nước) nào sau đây có áp lực thẩm thấu bằng nhau
qua màng tế bào bình thường khi trạng thái cân bằng được thiết lập?

 E) Glycerol và ure đều là các phân tử thấm, tức có thể khuếch tán qua màng tới
khi nồng độ trong và ngoài tế bào như nhau. Vì vậy, trong trạng thái cân bằng, độ
thẩm thấu trong và ngoài tế bào là 600 mOsm/L (300 từ ure và 300 từ glycerol).
Đáp án A sai vì phân tử albumin nhỏ hơn IgG. Khối lượng phân tử khác nhau có
nghĩa dung dịch albumin 10% sẽ chứa nhiều phân tử trong một đơn vị thể tích hơn
dung dịch IgG 10%, và vì vậy có độ thẩm thấu cao hơn. Đáp án B: Dung dịch A
NaCl 100 mmol/l có độ thẩm thấu 200 mOsm/L vì Na và Cl phân ly. Suy ra, dung
dịch A có độ thẩm thấu gấp hai lần dung dịch B. Đáp án C và D: cả hai dung dịch
đều có độ thẩm thấu tương đương nhau; tuy nhiên cả ure và glycerol đều là các
phân tử thấm (trong khi glucose và NaCl không phải), nên ure và glycerol khuếch
tán vào trong tế bào và tự khử áp lực thẩm thấu lên màng tế bào ban đầu.
63. Một cậu bé 12 tuổi vào viện vì giảm thị lực và nhìn đôi diễn biến 4 tháng nay,
kèm mệt mỏi về cuối ngày. Ngoài ra không có dấu hiệu nào khác. Qua thăm khám
cho thấy bệnh nhân bị sụp mi mắt trái có cải thiện sau một giấc ngủ. Các cơ quan
khác đều bình thường. Các cơ khác không yếu liệt. Xét nghiệm cận lâm sàng cho
thấy có kháng thể chống acetylcholin trong huyết tương, chức năng tuyến giáp
bình thường, chụp cắt lớp vi tính não và ổ mắt bình thường. Chẩn đoán sơ bộ là?
A) U tế bào sao
B) Bệnh Graves
C) Viêm tuyến giáp Hashimoto
D) Nhược cơ thiếu niên
E) Xơ cứng rải rác
 D) Nhược cơ là một bệnh tự miễn mắc phải gây ra mỏi và yếu cơ vân. Bệnh này
gây ra do các kháng thể IgG kháng với receptor acetylcholine tại màng sau synap
của khớp thần kinh-cơ. Triệu chứng chính là yếu cơ, nặng lên khi vận động. Các
bệnh nhân thường cảm thấy khỏe vào buổi sáng, nhưng yếu dần đi trong ngày. Yếu
cơ thường gây ra các triệu chứng nhìn đôi và sụp mi. Sự có mặt của các kháng thể
với acetylcholine trong huyết tương đặc hiệu cho nhược cơ và loại trừ các đáp án
khác. Cùng với đó, CLVT não và ổ mắt bình thường cũng loại trừ đặc hiệu cho u tế
bào sao (đáp án A), một loại u não có thể chèn ép vào các dây thần kinh sọ. Nhìn
đôi thường xảy ra ở những bệnh nhân bệnh Graves (đáp án B), nhưng kết quả kiểm
tra chức năng tuyến giáp bình thường (đồng thời loại trừ viêm tuyến giáp
Hashimoto). Xơ cứng rải rác (đáp án E) cũng thường biểu hiện yếu liệt chân
nhưng, một lần nữa, sự có mặt của kháng thế kháng acetylcholine là đủ đặc hiệu để
chẩn đoán bệnh nhược cơ.

64. Biểu đồ chiều dài-độ căng minh họa


một cơ vân có số sợi đỏ và trắng bằng nhau. Kính thích giật cơ cực đại được sử
dụng để khởi động một co cơ đẳng trường ở mỗi chiều dài cơ được nghiên cứu.
Chiều dài lúc nghỉ là 20 cm. Lực căng hoạt động tối đa mà cơ có thể thực hiện
được là bao nhiêu với tiền tải 100 gr?
A) 145–155 gr
B) 25–35 gr
C) 55–65 gr
D) 95–105 gr
E) Không xác định được
 C) Biểu đồ cho thấy mối quan hệ giữa tiền gánh của (hay) lực căng thụ động
(đường cong Z), lực căng tổng (X), và lực căng chủ động (Y). Lực căng chủ động
không thể đo được trực tiếp: nó là chênh lệch giữa tổng lực căng và lực căng thụ
động. Để trả lời câu hỏi này, trước hết sinh biên cần tìm điểm giao giữa 100 gr và
đường tiền tải (đường lực căng thụ động), và hạ xuống đường lực căng chủ động.
Tương ứng với 100 gr tiền tải là lực căng tổng khoảng hơn 150 gr, và lực căng chủ
động hơn 50 gr. Lưu ý rằng lực căng chủ động bằng lực căng tổng trừ đi lực căng
thụ động như đã nói ở trên. Vẽ ba đường cong này để đảm bảo chính xác về mặt
toán học là rất khó nên biểu đồ này cũng như các biểu đồ khác ở USMLE có thể
không tuyệt đối đúng.
65. Độ nhạy của hệ thống co cơ trơn với calci thường tăng ở trạng thái ổn định
dưới điều kiện bình thường. Sự tăng độ nhạy này với calci góp phần giảm chất nào
sau đây?
A) Actin
B) Adenosine Triphosphate (ATP)
C) Phức hợp calcium-calmodulin
D) Calmodulin
E) Phosphatase chuỗi nhẹ myosin (MLCP)
 E) Cơ trơn là cơ duy nhất có khả năng căng cơ khác nhau với một nồng độ calci
nội bào hằng định. Sự thay đổi độ nhạy với calci này của cơ trơn góp phần gây ra
khác biệt ở hoạt động của phosphatase myosin chuỗi nhẹ (MLCP). Cơ trơn co khi
myosin chuỗi nhẹ được phosphoryl hóa nhờ hoạt động của kinase myosin chuỗi
nhẹ (MLCK). MLCP là một phosphatase có thể khử phosphoryl myosin chuỗi nhẹ,
gây bất hoạt và từ đó giảm co cơ. Đáp án A: cả actin và myosin đều là những thành
phần quan trọng của hệ co cơ trơn cũng như cơ vân và cơ tim, nhưng chúng không
liên quan tới độ nhạy của cơ với calci. Đap án B: ATP cần cho co cơ trơn. Giảm
mức ATP sẽ làm giảm khả năng co cơ trơn cả khi nồng độ calci cao. Đáp án C:
Phức hợp calci-calmodulin gắn với MCLK, dẫn tới phosphoryl hóa myosin chuỗi
nhẹ. Giảm phức hợp này cũng làm giảm co cơ trơn. Đáp án D: một lần nữa, sự gắn
ion calci vào calmodulin là bước khởi đầu để hoạt hóa hệ thống co cơ trơn.
66.
Biểu đồ cho thấy mối quan hệ giữa lực-tốc độ của quá trình co cơ đẳng trương ở
một cơ vân. Ba đường cong khác nhau biểu thị sự khác nhau giữa?
A) Tần số co cơ
B) Phì đại cơ
C) Khối cơ
D) Hoạt động của myosin ATPase
E) Sự chi phối (Sự huy động) của các đơn vị vận động
. D) Biểu đồ cho thấy tốc độ tối đa của thu ngắn (Vmax) xảy ra khi không có hậu
tải tại cơ (lực=0). Tăng hậu tải làm giảm tốc độ thu ngắn cơ cho tới khi chạm điểm
co cơ đằng trường – nơi không có sự thu ngắn sợi cơ và tốc độ co khi này = 0 (các
đường cong giao trục X). Tốc độ thu ngắn tối đa liên quan với hoạt động của
ATPase cơ, tăng cao khi hoạt động ATPase được tăng cường. Đáp án A: tăng tần
số co cơ làm tăng tải lượng cơ nâng được trong giới hạn của cơ, nhưng không ảnh
hưởng tới tốc độ co. Đáp án B, C, E: phì đại cơ, tăng khối cơ, hay thêm đơn vị vận
động chi phối đều làm tăng tải lượng tối đa cơ nâng được, song không ảnh hưởng
tới tốc độ co cơ tối đa.
67. Một bệnh nhân nữ 24 tuổi cấp cứu tại bệnh viện đại học sau tai nạn xe máy (ô
tô) với nhiều vết rách sâu ở cổ tay trái đứt vào một dây chằng cơ lớn. Đầu bị cắt
đứt của dây chằng được chồng 6cm để khâu nối. Cơ hoạt động như thế nào sau 6
tuần so với trước chấn thương? Biết rằng các đơn vị vận động không thể tăng sinh
hồi phục hoàn toàn trong vòng 6 tuần.
Căng cơ thụ Căng cơ chủ
động động tối đa
A) giảm giảm
B) giảm tăng
C) tăng tăng
D) tăng giảm
không thay không thay
E)
đổi đổi
D) Kéo căng cơ để nối dây chằng dẫn tới tăng sức căng thụ động hay tiền tải.
Việc tăng lực căng thụ động này kéo dài cơ quá chiều dài lý tưởng của nó, và vì
vậy dẫn tới giảm lực căng chủ động tối đa cơ thực hiện được. Nguyên nhân lực
căng chủ động tối đa giảm là do sự chồng các lá actin và myosin giảm khi cơ bị
kéo giãn, sự chồng này chỉ đạt hiểu quả cao nhất ở chiều dài lúc nghỉ của cơ.

Unit 3: Tuần Hoàn phần 1


Câu hỏi 1-4
Một bệnh nhân nữ 64 tuổi có nhịp tin lúc nghỉ là 70 chu kỳ/phút, huyết áp động
mạch 130/85 mmHg, và nhiệt độ bình thường.

Trên đây là biểu đồ liên hệ áp lực-thể tích của tâm thất trái.
1. Cung lượng tim của bệnh nhân nữ tính theo mL/min?
A) 2000
B) 3000
C) 4000
D) 6000
E) 7000
 E) Bệnh nhân này có nhịp tim 70 chu kỳ/phút, cung lượng tim tính bằng công
thức: CO (cung lượng tim) = HR (nhịp tim) x Thể tích nhát bóp. Thể tích nhát bóp
có trong biểu đồ là 100ml, thay đổi trong đoạn C-D. Như vậy CLT=7000mL/min.
2. Tiếng T2 tương ứng với điểm nào trên đường biểu diễn áp lực-thể tích thất?
A) Tại điểm D
B) Giữa điểm A và B
C) Giữa điểm B và C
D) Giữa điểm C và D
E) Giữa điểm D và A
 A) Trong pha tống máu, các van động mạch chủ và phổi mở ra để máu đi vào
các động mạch chủ và phổi. Pha tống máu nằm giữa C và D, tương ứng các van
động mạch mở ở C và đóng ở D. Đóng các van này gây ra tiếng T2.
3. Tiếng T3 tương ứng với điểm nào trên đường biểu diễn áp lực-thể tích thất?
A) Tại điểm D
B) Giữa điểm A và B
C) Giữa điểm B và C
D) Giữa điểm C và D
E) Giữa điểm D và A
 B) Giữa điểm A và B là giai đoạn đổ đầy thất. Máu được tống vào tâm thất gây
rung thành tâm thất, dẫn đến tiếng T3.
4) Phân suất tống máu thất của bệnh nhân này là?
A) 33%
B) 50%
C) 60%
D) 67%
E) 80%
 D) Phân suất tống máu là thể tích nhát bóp/thể tích cuối tâm trương. Thể tích
nhát bóp bằng 100mL, thể tích cuối tâm thu ở điểm D là 150mL. Như vậy tính
được phân suất tống máu bằng 0.667 hay 66.7%.
5. Tính thấm với kali cao nhất ở pha nào của điện thế hoạt động cơ thất?
A) 0
B) 1
C) 2
D) 3
E) 4
 D) Trong pha 3 của điện thế hoạt động cơ thất, tính thấm với kali của cơ tâm
thất tăng cao, khiến điện thế màng càng âm hơn.
6. Giá trị bình thường của phân số tống máu tâm thất trên một người trưởng thành
ở trạng thái nghỉ ngơi là bao nhiêu?
A) 20%
B) 30%
C) 40%
D) 60%
E) 80%
 D) Phân số tống máu bình thường khoảng 60%, dưới 60% gợi ý tim hoạt động
yếu.
7. Một bệnh nhân nam 30 tuổi có chỉ số phân số tống máu 0.25 và thể tích cuối tâm
thu 150 ml. Thể tích cuối tâm trương là?
A) 50 ml
B) 100 ml
C) 125 ml
D) 200 ml
E) 250 ml
 D) Thể tích cuối tâm trương luôn lớn hơn thể tích cuối tâm thu. Nhân hệ số tống
máu với thể tích cuối tâm trương sẽ cho thể tích tống máu, ở trường hợp này bằng
50 ml. Như vậy thể tích cuối tâm trương hơn thể tích cuối tâm thu 50 ml, tức bằng
200 ml.
8. Câu nào sau đây đúng nhất khi nói về cơ tim?
A) Hệ thống ống T của cơ tim chứa được ít calci hơn nhiều so với của cơ vân
B) Sức mạnh và độ co của cơ tim phụ thuộc vào lượng calci quanh các tế bào cơ
tim
C) Ở cơ tim, khởi động điện thế hoạt động làm mở các kênh calci chậm
D) Tái khử cực cơ tim do mở các kênh natri
E) Các mucopolysaccharide trong hệ thống ống T giúp gắn các ion clo
 B) Cơ tim chứa nhiều calcium trong hệ thống ống và phụ thuộc nhiều vào calci
ngoại bào hơn cơ vân. Một lượng lớn calci được gắn bởi các mucopolysaccharid
trong hệ thống ống T. Calci này cần thiết cho quá trình co cơ tim, và sức co cơ phụ
thuộc vào nồng độ calci quanh tế bào cơ tim. Khởi đầu điện thế hoạt động, các
kênh natri nhanh mở đầu tiên, sau đó đến các kênh calci chậm.
10. Trường hợp nào sau đây dễ gây co cứng cơ tim nhất?
A) Tăng thân nhiệt
B) Cường giao cảm
C) Giảm ion kali ở dịch ngoại bào
D) Dư thừa ion kali ở dịch ngoại bào
E) Dư thừa ion calci ở dịch ngoại bào
 E) Tim co cứng khi nồng độ calci quanh các sợi cơ tim tăng cao, điều này xảy
ra khi nồng độ ion calci ngoại bào tăng quá cao. Nồng độ kali dịch ngoại bào dư
thừa làm giãn cơ tim vì điện thế màng của sợi cơ tim dương hơn.
11. Hiện tượng xảy ra ở cuối kỳ tống máu thất?
A) Đóng các van nhĩ thất
B) Mở van động mạch chủ
C) Tiếp tục mở van động mạch chủ
D) Mở các van nhĩ thất
E) Đóng van động mạch phổi
 E) Ở cuối thì tống máu thất, cả hai van động mạch cùng đóng, theo sau là thì
giãn đẳng tích.
12. Pha nào trong chu trình tim xảy ra ngay sau khi bắt đầu sóng QRS?
A) Giãn đẳng tích
B) Tống máu thất
C) Nhĩ thu
D) Cuối chu trình tim
E) Co đẳng tích
 E) Ngay sau sóng QRS, hai thất bắt đầu co và pha đầu tiên là co đẳng tích. Pha
này xảy ra trước pha tống máu và làm tăng áp lực tâm thất đủ để mở cơ học hai
van động mạch.
13. Điều gì làm cơ tim giãn và nhão?
A) Dư thừa ion calci trong máu
B) Dư thừa ion kali trong máu
C) Dư thừa ion natri trong máu
D) Tăng kích thích giao cảm
E) Tăng nồng độ norepinephrine trong máu
 B) Thừa kali máu và dịch ngoại bào làm cơ tim giãn và nhão, đồng thời gây
giảm nhịp tim. Tác động này rất đáng kể vì gât ra một điện thế màng dương hơn ở
các sợi cơ tim. Khi điện thế màng trở nên dương hơn, cường độ điện thế hoạt động
sẽ giảm, làm cho tim co yếu đi.
Thừa calci máu, kích thích giao cảm và tăng nồng độ norepinephrin máu đều làm
tăng co cơ tim.
14. Một vận động viên khỏe mạnh 25 tuổi nặng 80 kg có cung lượng tim đạt đỉnh
là bao nhiêu khi kích thích giao cảm tối đa?
A) 3 L/phút
B) 5 L/phút
C) 10 L/phút
D) 13 L/phút
E) 25 L/phút
 E) Lưu lượng tim tối đa bình thường là 13 L/phút. Mức này giảm ở mọi loại suy
tim và tăng rõ rệt khi có kích thích giao cảm.
15. Tiếng T1 gây ra do hiện tượng nào sau đây?
A) Đóng van động mạch chủ
B) Máu đổ vào tâm thất trong thì tâm trương
C) Đầu tâm trương
D) Mở các van nhĩ thất
E) Đóng các van nhĩ thất
 E) Như đã đề cập ở Chương 9, tiếng tim đầu tiên theo định nghĩa xảy ra ngay
sau khi áp lực tâm thất vượt quá áp lực tâm nhĩ. Điều này dẫn tới các van nhĩ thất
đóng lại. Tiếng tim thứ hai xảy ra khi đóng các van động mạch chủ và phổi.
16. Tình trạng nào sau đây ở nút xoang A-V làm giảm nhịp tim?
A) Tăng tính thấm với natri
B) Giảm nồng độ acetylcholine
C) Tăng nồng độ norepinephrine
D) Tăng tính thấm với kali
E) Tăng tính thấm với calci
. D) Tăng tính thấm với kali gây tăng phân cực ở nút A-V, làm giảm nhịp tim.
Tăng tính thấm với natri thực chất khử cực một phần nút A-V còn tăng nồng độ
norepinephrine làm tăng nhịp tim
17. Kích thích giao cảm của tim gây
A) Giải phóng acetylcholine ở các đầu tận giao cảm
B) Giảm mức phát xung của nút xoang
C) Giảm khả năng hưng phấn của tim
D) Giải phóng norepinephrine ở các đầu tận giao cảm
E) Giảm khả năng co bóp của tim
 D) Tăng kích thích giao cảm ở tim làm tăng nhịp tim, sự co tâm nhĩ, sự co tâm
thất và cả lượng norepinephrine giải phóng ra ở các đầu tận thần kinh giao cảm
thất. Nó không làm giải phóng acetylcholine hay tăng tính thấm với natri của nút
A-V (điều sẽ gây tăng mức đi lên của điện thế màng tới ngưỡng tự hưng phấn, từ
đó làm tăng nhịp tim).
18. Tổng thời gian truyền xung điện tim từ nút A-V tới bó His là?
A) 0.22 s
B) 0.18 s
C) 0.16 s
D) 0.13 s
E) 0.09 s
 D) Xung từ nút S-A truyền nhanh qua các đường dẫn tới nút A-V sau 0.03s, tới
bó A-V sau 0.12s và tới các sợi thất sau 0.16s. Tổng thời gian là 0.13s.
19. Câu nào sau đây giải thích đúng nhất về ảnh hưởng của kích thích giao cảm lên
tim?
A) Giảm tính thấm với natri ở nút S-A
B) Giảm tính thấm với natri ở nút A-V
C) Tăng tính thấm với kali ở nút S-A
D) Tăng mức đi lên của điện thế màng (nghỉ) của nút S-A
E) Giảm tính thấm với calci của cơ tim
 D) Trong quá trình kích thích giao cảm, tính thấm của nút SA và AV đều tăng.
Đồng thời tính thấm của cơ tim với calci cũng tăng dẫn tới tăng sức co cơ tim.
Cùng với đố có sự tăng mức điện thế nghỉ ở màng nút SA. Tăng tính thấm với kali
ở nút SA không xảy ra khi có kích thích giao cảm.
20. Cấu trúc nào sau đây có tốc độ dẫn truyền điện thế hoạt động tim chậm nhất?
A) Cơ tâm nhĩ
B) Đường liên nút xoang trước
C) Các sợi bó nhĩ-thất
D) Các sợi Purkinje
E) Cơ tâm thất
 C) Các cơ tâm nhĩ và tâm thất có độ dẫn truyền điện thế hoạt động tim khá
nhanh, tương tự đường liên nút trước. Tuy nhiên, các sợi cơ bó nhĩ thất lại có độ
dẫn truyền chậm vì kích thước các sợi này nhỏ hơn kích thước các cơ nhĩ thất bình
thường. Đồng thời, độ dẫn truyền chậm một phần do số lượng ít các khớp cách
(liên kết khe) giữa các tế bào cơ liên tiếp trên đường dẫn truyền, cản trở việc dẫn
các ion kích thích từ một tế bào sang tế bào tiếp theo.
21. Nếu nút xoang nhĩ phát xung ở giây 0.00, điện thế hoạt động thường tới bề mặt
lá tạng ngoại tâm mạc ở đáy tâm thất trái sau bao lâu?
A) 0.22 s
B) 0.18 s
C) 0.16 s
D) 0.12 s
E) 0.09 s
 A) Sau khi nút xoang nhĩ phát xung, điện thế hoạt động được dẫn qua tâm nhĩ,
tới hệ bó nhĩ thất, cuối cùng tới vách tâm thất và lan ra toàn bộ tâm thất. Nơi cuối
cùng xung lan tới là bề mặt lá tạng ngoại tâm mạc tại đáy của tâm thất trái, tại giây
thứ 0.22.
22. Nếu nút xoang nhĩ phát xung ở giây 0.00, điện thế hoạt động thường tới bó nhĩ-
thất (bó His) sau bao lâu?
A) 0.22 s
B) 0.18 s
C) 0.16 s
D) 0.12 s
E) 0.09 s
 D) Điện thế hoạt động tới bó nhĩ thất ở 0.12s; tới nút nhĩ thất ở 0.03s và đi qua
nút này mất 0.09s nên tới bó His ở 0.12s.
23. Tình trạng nào sau đây của nút xoang nhĩ sẽ làm giảm nhịp tim?
A) Tăng nồng độ norepinephrine
B) Tăng tính thấm với natri
C) Tăng tính thấm với calci
D) Tăng tính thấm với kali
E) Giảm nồng độ acetylcholine
 D) Tăng tính thấm với natri và calci ở nút xoang nhĩ làm tăng nhịp tim. Tăng
tính thấm với kali làm tăng phân cực ở nút xoang nhĩ, từ đó làm giảm nhịp tim.
24. Acetylcholine gây ra hiện tượng nào sau đây?
A) Tăng phân cực ở nút xoang nhĩ
B) Khử cực ở nút nhĩ thất
C) Giảm tính thấm của nút xoang nhĩ với ion kali
D) Tăng nhịp tim
E) Tăng tính thấm của cơ tim với ion calci
 A) Acetylcholine không làm khử cực nút nhĩ thất hay tăng tính thấm của cơ tim
với calci nhưng gây tăng phân cực ở nút xoang nhĩ nhờ tăng tính thấm với ion kali,
vì vậy làm giảm nhịp tim.
25. Điện thế màng (mức ngưỡng) để nút xoang nhĩ phát xung?
A) −40 mV
B) −55 mV
C) −65 mV
D) −85 mV
E) −105 mV
 A) Điện thế màng nghỉ bình thường của nút xoang nhĩ là -55mV. Khi natri đi
(rò rỉ) vào màng, điện thế màng tăng lên tới ngưỡng -40mV. Đây là mức ngưỡng
làm khởi động điện thế hoạt động ở nút xoang nhĩ.
26. Tình trạng nào sau đây của nút nhĩ thất làm giảm nhịp tim?
A) Tăng tính thấm với natri
B) Giảm nồng độ acetylcholine
C) Tăng nồng độ norepinephrine
D) Tăng tính thấm với kali
E) Tăng tính thấm với calci
D) Tăng tính thấm với kali làm giảm điện thế màng của nút nhĩ thất, làm nút này
tăng phân cực mạnh, và vì vậy càng khó đạt ngưỡng dẫn truyền hơn. Điều này làm
giảm nhịp tim. Tăng tính thấm với natri và calci cũng như tăng nồng độ
norepinephrine đều làm tăng điện thế màng, và tăng nhịp tim tương ứng
27. Nếu các sợi Purkinje thất là nơi tạo xung phát nhịp của tim (trở thành chủ
nhịp), nhịp tim khi này sẽ bằng?
A) 30/phút
B) 50/phút
C) 65/phút
D) 75/phút
E) 85/phút
 A) Nếu không có sự dẫn truyền xung từ nút xoang nhĩ tới nút nhĩ thất hoặc nút
xoang nhĩ không phát xung, nút nhĩ thất sẽ thay thế để trở thành nơi tạo nhịp của
tim. Nhịp của nút nhĩ thất là 40-60/phút. Nếu các sợi Purkinje chịu trách nhiệm
phát nhịp, nhịp tim sẽ ở khoảng giữa 15-40/phút.
28. Tổng thời gian để xung nhịp tim đi từ nút nhĩ thất tới hệ thống bó nhĩ thất là?
A) 0.03 s
B) 0.06 s
C) 0.09 s
D) 0.13 s
E) 0.17 s
. D) Xung từ nút xoang nhĩ tới nút nhĩ thất sau 0.03s. Sau đó đi qua nút nhĩ thất
và hệ thống bó mất 0.13s dẫn tới đến được vách liên thất ở 0.16s.
29. Điện thế nghỉ của các sợi nút xoang bằng?
A) −100 mV
B) −90 mV
C) −80 mV
D) −55 mV
E) −20 mV
 D) Điện thế màng nghỉ của các sợi nút xoang là -55mV, trong khi điện thế của
cơ tim là -85 đến -90mV. Một điểm khác biệt lớn khác giữa các sợi nút xoang và
cơ tâm thất là khả năng tự kích thích của các sợi xoang do dòng ion natri đi vào (rò
rỉ vào trong qua màng)
30. Nếu các sợi Purkinje nằm xa đoạn nối nhĩ thất trở thành máy phát xung tạo
nhịp của tim (chủ nhịp), nhịp tim khi này sẽ bằng?
A) 30/phút
B) 50/phút
C) 60/phút
D) 70/phút
E) 80/phút
 A) Nếu các sợi Purkinje đóng vai trò tạo nhịp của tim, nhịp tim sẽ trong khoảng
15-40/phút. Trong khi đó, các sợi nút nhĩ thất phát xung 40-60 lần/phút, còn nút
xoang phát xung 70-80 lần/phút. Nếu nút xoang bị ức chế vì một lý do nào đó, nút
nhĩ thất sẽ trở thành nơi tạo nhịp, còn nếu nút nhĩ bị ức chế, các sợi Purkinje sẽ
đảm nhiệm vai trò này.
31. Kích thích giao cảm tim sẽ gây ra tình trạng nào sau đây?
A) Giải phón acetylcholine ở các đầu tận giao cảm
B) Giảm nhịp tim
C) Giảm dẫn truyền xung tim
D) Giảm lực co tâm nhĩ
E) Tăng lực co tâm thất
 E) Kích thích giao cảm tim thường làm tăng nhịp tim, tăng tốc độ dẫn truyền
xung tim và tăng lực co tâm thất, tâm nhĩ. Tuy nhiên nó không làm giải phóng
acetylcholine ở các đầu tận giao cảm vì tại vị trí này chứa norepinephrine. Kích
thích phó giao cảm làm giải phóng acetylcholine. Kích hoạt hệ thần kinh giao cảm
làm tăng tính thấm của các sợi cơ tim, nút xoang nhĩ và nút nhĩ thất với natri và
calci.
32. Khi ghi chuyển đạo I của điện tâm đồ, tay phải là điện cực âm còn điện cực
dương là
A) Tay trái
B) Chân trái
C) Chân phải
D) Tay trái và chân trái
E) Tay phải và chân trái
 A) Theo quy ước, tay trái là điện cực dương của chuyển đạo lưỡng cực chi I
EKG.
33. Khi ghi chuyển đạo aVL của điện tâm đồ, điện cực dương là
A) Tay trái
B) Chân trái
C) Chân phải
D) Tay trái và chân trái
E) Tay phải và chân trái
 A) 32. A) Theo quy ước, tay trái là điện cực dương của chuyển đạo I (aVL)
EKG.
34. Một bệnh nhân nam 70 tuổi khám sức khỏe định kỳ hàng năm có kết quả điện
tâm đồ như sau. Khoảng Q-T kéo dài bao lâu?

A) 0.12 s
B) 0.16 s
C) 0.22 s
D) 0.30 s
E) 0.40 s
 E) Quá trình tâm thất co kéo dài từ điểm bắt đầu sóng Q tới điểm cuối sóng T.
Đoạn này được gọi là khoảng Q-T và thường kéo dài 0.35s. Ở ví dụ cụ thể này
khoảng Q-T dài hơn một chút và vào khoảng 0.40s.
35. Nhịp tim bệnh nhân này là?
A) 64
B) 70
C) 88
D) 94
E) 104
. B) Nhịp tim có thể tính bằng 60 chia khoảng RR, ở trường hợp này là 0.86s.
Kết quả nhịp tim khoảng 70 chu kỳ/phút.
36. Khoảng QT bình thường là bao nhiêu?
A) 0.03 s
B) 0.13 s
C) 0.16 s
D) 0.20 s
E) 0.35 s
 E) Quá trình tâm thất co kéo dài từ điểm bắt đầu sóng Q tới điểm cuối sóng T.
Đoạn này được gọi là khoảng Q-T và thường kéo dài 0.35s.
37. Điện cực dương khi ghi chuyển đạo II của điện tâm đồ là
A) tay trái
B) chân trái
C) chân phải
D) tay trái + chân trái
E) tay phải + chân trái
 B) Theo quy ước, chân trái là điện cực dương của chuyển đạo II điện tâm đồ.
38. Điện cực âm khi ghi chuyển đạo III của điện tâm đồ là
A) tay trái
B) chân trái
C) chân phải
D) tay trái + chân trái
E) tay phải + chân trái
 A) Theo quy ước, tay trái là điện cực âm của chuyển đạo III.
39. Một bệnh nhân nam 65 tuổi được làm điện tâm đồ tại một phòng cấp cứu địa
phương sau tai nạn xe đạp. Cân nặng 80 kg, huyết áp động mạch chủ 160/90
mmHg. Phức hợp QRS 0.5 mV ở chuyển đạo I, 1.5 mV ở chuyển đạo III và bao
nhiêu ở chuyển đạo II?
A) 0.5 mV
B) 1.0 mV
C) 1.5 mV
D) 2.0 mV
E) 2.5 mV
 Định luật Einthoven phát biểu rằng điện thế ở chuyển đạo I cộng điện thế
chuyển III bằng điện thế chuyển đạo II, ở trường hợp này là 2.0 mV.
40. Một sóng khử cực thất −90° ở mặt phằng trán sẽ gây âm nhiều ở chuyển đạo
nào sau đây?
A) aVR
B) aVL
C) Chuyển đạo II
D) Chuyển đạo III
E) aVF
 E) Ở Hình 12-3 (TMP12), phần dương của chuyển đạo aVF có trục 90o và phần
âm có trục -90o. Chú ý chênh lệch giữa đầu dương và âm của vector này là 180o.
Câu hỏi 41-43
Một bệnh nhân nữ 60 tuổi có kết quả điện tâm đồ ghi ở một phòng cấp cứu địa
phương như sau sau một tai nạn xe máy (ô tô). Cân nặng 70 kg, huyết áp động
mạch chủ 140/80 mm Hg.

41. Trục tính được từ các chuyển đạo chuẩn I, II và III ở điện tâm đồ của bệnh
nhân này bằng?
A) −90°
B) −50°
C) −12°
D) +100°
E) +170°
 B) Trục được xác định bằng cộng điện thế của phức hợp QRS ở các chuyển đạo
I, II và III. Trường hợp này có giá trị −50°.
42. Nhịp tim tính được dựa vào chuyển đạo I bằng?
A) 70
B) 88
C) 100
D) 112
E) 148
 A) Nhịp tim có thể tính bằng 60 chia khoảng RR, ở trường hợp này là 0.68s.
Kết quả nhịp tim khoảng 88 chu kỳ/phút.
43. Chẩn đoán sơ bộ trên bệnh nhân này?
A) Hẹp van hai lá
B) Block nhánh trái
C) Hẹp van động mạch phổi
D) Block nhánh phải
E) Phì đại thất trái
 B) Xem hình 12-14 (TMP12) ở trên, phức hợp QRS có độ rộng hơn 0.12s, gợi ý
có block nhánh bó His. Điện tâm đồ trục trái, cho thấy block nhánh trái.
44. Bệnh nào sau đây thường có điện tâm đồ trục phải?
A) Tăng huyết áp hệ thống
B) Hẹp van động mạch chủ
C) Hở van động mạch chủ
D) Béo bụng mỡ
E) Tăng áp lực động mạch phổi
 E) Tăng huyết áp hệ thống gây trục chuyển trái do tâm thất trái giãn rộng. Hẹp
và hở van động mạch chủ cũng làm to thất trái và trục chuyển trái. Mỡ bụng tạo áp
lực cơ học sẽ làm tim quay trái vì chuyển trái trục tim. Tăng huyết áp mạch phổi
gây to thất phải vì vậy làm trục chuyển phải.
45. Một sóng khử cực thất 60° ở mặt phằng trán sẽ gây dương nhiều ở chuyển đạo
nào sau đây?
A) aVR
B) aVL
C) Chuyển đạo I
D) Chuyển đạo II
E) aVF
 D) Chuyển đạo II có vector dương một góc 60°. Đầu dương của chuyển đạo II
chỉ −120°.
Câu hỏi 46 và 47
Một bệnh nhân nam 62 tuổi tiền sử hút thuốc lâu năm nặng 250 lb có kết quả điện
tâm đồ ghi ở bệnh viện địa phương như sau.

46. Trục điện tâm đồ tính được dựa vào các chuyển đạo chuẩn I, II và III?
A) −110°
B) −20°
C) +90°
D) +105°
E) +180°
 D) Chuyển đạo III có vector lớn nhất nên trục điện sẽ gần với chiều vector
chuyển đạo này hơn các chuyển đạo I và II. Góc chuyển đạo III là 120° và vector
tổng (trục điện trung bình) gần với chuyển đạo đó có giá trị +105°.
47. Chẩn đoán sơ bộ của bệnh nhân này là?
A) Phì đại thất trái
B) Block nhánh trái
C) Hẹp van ba lá
D) Block nhánh phải
E) Phì đại thất phải
 D) Chẩn đoán là block nhánh phải. Xác định được nhờ trục điện chuyển phải và
phức hợp QRS kéo rất dài. Ở phì đại thất phải, phức hợp này chỉ kéo dài vừa phải.
48. Một bệnh nhân nữ 60 tuổi mất khả năng thực hiện một số công việc sinh hoạt
bình thường. Trên điện tâm đồ có phức hợp QRS rộng 0.2s, sóng T cao ở chuyển
đạo I, và sóng R âm ở chuyển đạo III. Chẩn đoán sơ bộ ở bệnh nhân này là gì?
A) Phì đại thất phải
B) Block nhánh trái
C) Hẹp van động mạch phổi
D) Block nhánh phải
E) Phì đại thất trái
 D) Bệnh nhân này có trục trái vì sóng R ở chuyển đạo III âm nhiều. Đồng thời,
sóng T hướng lên ở chuyển đạo I, ngược với hướng của cả phức hợp QRS. Đây là
đặc điểm của block nhánh. Cùng với đó, phức hợp QRS rộng 0.20s – kéo rất dài.
Một phức hợp QRS có độ rộng trên 0.12s thường gây ra bởi block dẫn truyền. Tất
cả yếu tố này cho thấy bệnh nhân có block nhánh trái.
49. Một bệnh nhân nữ 70 tuổi được đưa vào khoa cấp cứu bệnh viện vì đau ngực.
Dựa vào kết quả điện tâm đồ, chẩn đoán bệnh nhân này là gì?
A) Nhồi máu vùng trước cấp tính ở thất trái
B) Nhồi máu vùng trước cấp tính ở thất phải
C) Nhồi máu vùng sau cấp tính ở thất trái
D) Nhồi máu vùng sau cấp tính ở thất phải
E) Phì đại thất phải
A) Bệnh nhân này có nhồi máu trước cấp tính ở thất trái. Điều này có thể xác
định bằng cách vẽ các dòng tổn thương ở các chuyển đạo khác nhau. Các chuyển
đạo chi được sử dụng để xác định nhồi máu có tới từ bên phải hay bên trái và ở đáy
hay mỏm tim. Các chuyển đạo ngực được sử dụng để xác định nhồi máu trước hay
sau. Khi chúng ra phân tích các dòng tổn thương, một điện thế âm gây ra bởi dòng
tổn thương xảy ra ở chuyển đạo I và một điện thế dương bởi một dòng tổn thương
khác xảy ra ở chuyển đạo III. Điều này được xác định bằng cachs tách điểm J khỏi
quãng TP. Đầu âm kết thúc của véc tơ kết quả bắt nguồn từ vùng nhồi máu, ở bên
trái tim. Ở chuyển đạo ngực V2, điện cực nằm trong một vùng có điện thế rất âm,
xảy ra ở những bệnh nhân có tổn thương ở mặt trước
50. Một bệnh nhân nam 30 tuổi được làm điện tâm đồ tại phòng khám bác sỹ tư
của anh ấy nhưng kết quả bị mất. Kỹ thuật viên đo điện tâm đồ nhớ được QRS
chênh nhiều và dương ở chuyển đạo aVF, bằng 0 ở chuyển đạo I. Vậy trục điện
trung bình ở mặt phẳng đứng dọc bằng?
A) 90°
B) 60°
C) 0°
D) −60°
E) −90°
 A) Vì độ chênh điện tâm đồ bằng 0 ở chuyển đạo I, trục tạo với chuyển đạo một
góc 90°. Vì vậy, trục điện trung bình bằng + hoặc - 90°. Vì chuyển đạo aVF chênh
dương, trục bằng +90°.
51. Tình trạng nào sau đây xảy ra ở điểm J của điện tâm đồ trên bệnh nhân bị hủy
hoại cơ tim?
A) Toàn bộ tim khử cực
B) Toàn bộ tim khử cực trừ phần cơ tim tổn thương
C) Khoảng một nửa tim khử cực
D) Toàn bộ tim tái phân cực
E) Toàn bộ tái phân cực tim trừ phần cơ tim tổn thương
 A) Ở điểm J, toàn bộ tim khử cực cả ở bệnh nhân có cơ tim tổn thương hay
lành. Khu bực tim tổn thương sẽ không tái phân cực mà duy trì khử cực liên tục.

52. Một bệnh nhân nam 50 tuổi là nhân viên mới ở công ty phần mềm ABC. Điện
tâm đồ dưới đây được ghi khi khám sức khỏe định kỳ. Chẩn đoán sơ bộ là?

A) Tăng huyết áp hệ thống mạn tính


B) Tăng huyết áp mạch phổi mạn tính
C) Block tim độ 2
D) Nhịp nhanh kịch phát
E) Hẹp van ba lá
 A) Thấy rằng phức hợp QRS dương ở chuyển đạo I và âm ở chuyển đạo III, cho
thấy có chiều trục hướng sang trái. Điều này xảy ra khi có tăng huyết áp hệ thống
mạn tính.
Tăng huyết áp mạch phổi làm tăng khối cơ tâm thất phải, làm trục chuyển phải.
53. Một bệnh nhân nam 55 tuổi được ghi điện tâm đồ trong buổi khám định kỳ
hàng năm, có độ chênh (sóng R trừa sóng Q/S) ở chuyển đạo chi chuẩn I là -
1.2mV, chuyển đạo chi chuẩn II là +1.2mV. Trục điện trung bình của QRS?
A) −30°
B) +30°
C) +60°
D) +120°
E) −120°
 D) Sóng QRS ở chuyển đạo I là -1.2mV còn ở chuyển đạo II là +1.2mV nên giá
trị tuyệt đối của các độ chênh là như nhau. Vì vậy, trục điện trung bình phải nằm
chính giữa hai chuyển đạo này, và vì trục chuyển đạo II bằng 60°, I 180° nên giá trị
kết quả là 120°.
54. Một bệnh nhân 65 tuổi có tiếng thổi ở tim có trục QRS trung bình 120°, và
phức hợp QRS kéo dài 0.18s. Chẩn đoán sơ bộ của bệnh nhân này là?
A) Hẹp van động mạch chủ
B) Hở van động mạch chủ
C) Hẹp van động mạch phổi
D) Block nhánh phải
E) Block nhánh trái
 D) Trục QRS 120° cho thấy chuyển phải. Vì phức hợp QRS kép dài 0.18s nên
cho thấy có block dẫn truyền. Vì vậy, điện tâm đồ này gợi ý có block nhánh phải.
55. Một bệnh nhân nữ 60 tuổi hay mệt mỏi. Điện tâm đồ có phức hợp QRS dương
ở chuyển đạo aVF và âm ở chuyển đạo chi chuẩn I. Chẩn đoán sơ bộ tình trạng này
là?
A) Tăng huyết áp hệ thống mạn tính
B) Tăng huyết áp mạch phổi
C) Hẹp van động mạch chủ
D) Hở van động mạch chủ
 B) Điện tâm đồ bệnh nhân này dương ở aVF và âm ở chuyển đạo chi chuẩn I.
Như vậy, trục điện trung bình nằm trong khoảng 90 và 180°, tức trục phải. Tăng
huyết áp hệ thống, hẹp van động mạch chủ, và hở van động mạch chủ đều gây phì
đại thất trái, từ đó làm trục điện chuyển trái. Tăng huyết áp mạch phổi gây trục
chuyển phải và vì vậy đặc trưng cho điện tâm đồ này.
56. Một bệnh nhân nữ 60 tuổi vào khoa cấp cứu bệnh viện vì đau ngực.

Dựa vào điện tâm đồ trên đây, chẩn đoán sơ bộ có khả năng nhất ở bệnh nhân này
là gì?
A) Nhồi máy trước cấp tính ở đáy tim
B) Nhồi máu trước cấp tính ở mỏm tim
C) Nhồi máu sau cấp tính ở đáy tim
D) Nhồi máu sau cấp tính ở mỏm tim
E) Phì đại thất phải
 D) Hình sau cho thất có dòng tổn thương ở sơ đồ dưới. Đây không phải điện thế
của QRS mà là điện thế của tổn thương. Các điện thế này ở chuyển đạo II và III,
cùng âm, và vector kết quả gần như nằm dọc. Đầu âm của véc tơ chỉ nơi bắt nguồn
của tổn thương, ở đây là mỏm tim. Khoảng TP cao hơn điểm J cho biết tổn thương
nằm ở mặt sau. Như vậy, điệm tâm đồ này cho thấy có một nhồi máu sau cấp tính
ở mỏm tim.
57. Một bệnh nhân nam 50 tuổi có huyết áp 140/85 và nặng 200 lb. Bệnh nhân nói
rằng mình cảm thấy không khỏe, điện tâm đồ không có sóng P, nhịp tim 46, và
phức hợp QRS vẫn xuất hiện đều.
Chẩn đoán sơ bộ là?
A) Block tim độ 1
B) Block tim độ 2
C) Block tim độ 3
D) Block tim xoang nhĩ
E) Nhịp chậm xoang
 D) Khi bệnh nhân không có sóng P và nhịp tim chậm, nhiều khả năng xung do
nút xoang phát ra đã bị chặn hoàn toàn trước khi tới được cơ tâm nhĩ. Đây được
gọi là block nhĩ xoang. Tâm thất nhận nhịp mới thường phát ra từ nút nhĩ thất –
làm nhịp tim trong khoảng 40-60 nhịp/phút. Trong khi đó, với nhịp chậm xoang,
bệnh nhân vẫn sẽ có sóng P đi kèm với phức hợp QRS. Với block tim độ 1, 2, 3,
bệnh nhân có sóng P mặc dù không luôn đi kèm phức hợp QRS.
58. Một bệnh nhân nam 80 tuổi được ghi điện tâm đồ ở phòng khám bác sỹ địa
phương, chẩn đoán rung nhĩ. Câu nào sau đây đúng với bệnh nhân rung nhĩ?
A) Rung thất thường đi kèm với rung nhĩ
B) Các sóng P của điện tâm đồ mạnh
C) Nhịp co tâm thất không đều và nhanh
D) Sóng “a” nhĩ bình thường
E) Tâm nhĩ có thể tích nhỏ hơn bình thường
 C) Rung nhĩ có nhịp tim nhanh và không đều. Các sóng P mất hoặc rất yếu.
Tâm nhĩ sinh các chuyển động quay vòng, và thể tích tâm nhĩ thường tăng, gây ra
rung nhĩ.
59. Chuyển động quay vòng của tâm thất có thể dẫn tới rung thất. Tình trạng nào
sau đây của cơ tâm thất làm tăng xu hướng chuyển động quay vòng?
A) Giảm thời gian trơ
B) Nồng độ kali ngoại bào thấp
C) Tăng thời gian trơ
D) Đường dẫn truyền ngắn hơn (giảm thể tích tâm thất)
E) Tăng các kích thích phó giao cảm tới tim
 A) Các chuyển động quay vòng ở cơ tâm thất nếu tim giãn hoặc giảm tốc độ
dẫn truyền. Nồng độ kali ngoại bào cao và kích thích giao cảm, chứ không phải
phó giao cảm, làm tăng xu hướng chuyển động quay vòng. Giai đoạn trơ dài hơn
cản trở chuyển động quay vòng của tim, vì khi các xung truyền đi ở tim và tới cơ
tâm thất, nơi có thời gian trơ dài hơn, điện thế hoạt động sẽ dừng lại ở vị trí này.
60. Một bệnh nhân nam 75 tuổi ngất khi vào khoa cấp cứu bệnh viện và tỉnh lại sau
5 phút. Trên điện tâm đồ có 75 sóng P/ phút và 35 sóng QRS/ phút với độ rộng
bình thường.
Chẩn đoán sơ bộ bệnh nhân này là gì?
A) Block nhĩ-thất độ 1
B) Hội chứng Stokes-Adams
C) Nhịp nhanh nhĩ kịch phát
D) Các luân phiên điện học
E) Các nhịp co nhĩ sớm
 B) Một khởi đầu đột ngột của block nhĩ thất đến và đi được gọi là hội chứng
Adams-Stokes. Bệnh nhân này có 75 sóng P/phút, có nghĩa tâm nhĩ co bình
thường. Trong khi blck nhĩ thất sẽ chỉ có 35 sóng QRS/phút.
61. Một bệnh nhân nam 60 tuổi nặng 220lb có chuyển đạo II điện tâm đồ như sau.

Chẩn đoán là?


A) Nhịp nút nhĩ thất
B) Block tim nhĩ thất độ 1
C) Block tim nhĩ thất độ 2
D) Block tim nhĩ thất độ 3
E) Cuồng nhĩ
 D) Theo định nghĩa, block nhĩ thất độ 1 xảy ra khi khoảng PR quá 0.20s mà
không mất sóng QRS nào. Ở hình dưới đây, khoảng PR khoảng 0.30s, được nhận
định là kéo dài. Mặt khác, không có mất QRS. Ở block nhĩ thất độ 2 và 3 sẽ có mất
sóng QRS.
62. Một bệnh nhân nữ 35 tuổi cảm giác bất thường ở ngực sau khi hút thuốc lá có
điện tâm đồ như sau.

Chẩn đoán của bệnh nhân này là gì?


A) Ngoại tâm thu bắt nguồn từ tâm nhĩ
B) Ngoại tâm thu Máy tạo nhịp nối nhĩ thất bắt nguồn từ phần cao nút nhĩ thất
C) Ngoại tâm thu bắt nguồn từ phần thấp nút nhĩ thất
D) Ngoại tâm thu bắt nguồn từ mỏm tâm thất
E) Ngoại tâm thu bắt nguồn từ đáy tâm thất
E) Lưu ý rằng các ngoại tâm thu thất (nhịp co thất sớm) (PVC) có sóng QRS ở
điện tâm đồ rộng và cao hơn. Trục điện trung bình của nhịp co sớm được xác định
bằng các phức hợp QRS rộng ở các chuyển đạo chi tiêu chuẩn. PVC bắt nguồn từ
đầu âm của trục điện trung bình thu được ở đáy của tâm thất. Ở điện tâm đồ này,
QRS của PVC rông hơn và cao hơn nhiều sóng QRS bình thường
Câu hỏi 63 và 64
Một bệnh nhân nam 55 tuổi tới khám định kỳ có điện tâm đồ ghi ở phòng khám
bác sỹ tư như sau.

63. Chẩn đoán của bệnh nhân này là gì?


A) Điện tâm đồ bình thường
B) Cuồng nhĩ
C) Chủ nhịp là phần cao của nút nhĩ thất
D) Chủ nhịp là phần giữa của nút nhĩ thất
E) Chủ nhịp là phần dưới của nút nhĩ thất
 B) Bệnh nhân này có cuồng nhĩ đặc trưng bởi nhiều sóng P đi kèm mỗi phức
hợp QRS. Ở điện tâm đồ này, có thể thấy một số vị trí có 2 sóng P kèm mỗi QRS,
và một số vị trí khác có 3 sóng P kèm mỗi QRS. Cũng thấy tim nhịp nhanh và các
khoảng RR không đều đặc trưng cho cuồng nhĩ.
64. Nhịp thất của bệnh nhân này mỗi phút là?
A) 37.5
B) 60
C) 75
D) 100
E) 120
 E) Nhịp thất trung bình 120 chu kỳ/phút ở điện tâm đồ này điển hình cho cuồng
nhĩ. Một lần nữa, lưu ý rằng nhịp tim không đều do xung không thể qua nút nhĩ
thất nhanh vì khoảng trơ.
65. Một bệnh nhân nữ 60 tuổi được chẩn đoán rung nhĩ. Câu nào sau đây đúng nhất
để mô tả tình trạng này?
A) Nhịp thất co bằng 140/phút
B) Các sóng P của điện tâm đồ rõ
C) Thất co cách đều
D) Các sóng QRS rõ hơn bình thường
E) Tâm nhĩ nhỏ hơn bình thường
 A) Rung nhĩ có nhịp tim nhanh không đều. Các sóng P mất hoặc rất yếu/ Tâm
nhĩ có các chuyển động quay vòng và thường giãn rộng, gây rung nhĩ.
66. Đâu là đặc điểm của rung nhĩ?
A) Xảy ra không thường xuyên ở các bệnh nhân phì đại tâm nhĩ
B) Nhịp thất khoảng 40 nhịp/phút
C) Hiệu quả của bơm tâm thất giảm 20-30%
D) Nhịp thất đều
E) Sóng P nhĩ dễ thấy
 C) Rung nhĩ xảy ra thường xuyên ở những bệnh nhân có giãn tâm nhĩ. Điều này
làm tăng xu hướng xảy ra các chuyển động quay vòng. Nhịp thất không đều vì
xung tới nút nhĩ thất sớm nhưng trong số đó, nhiều lần nút nhĩ thất ở giai đoạn trơ.
Vì vậy nút nhĩ thất sẽ chỉ cho xung thứ 2 đi qua sau xung thứ nhất 0.35s. Cũng có
một khoảng giữa khi các xung nhĩ tới nút nhĩ thất, gây nhịp tim không đều và rất
nhanh khoảng 125-150 nhịp/phút.
67. Một bệnh nhân nữ 65 tuổi bị nhồi máu cơ tim cách đây 10 ngày quay lại phòng
khám của bác sỹ gia đình vì nhịp tim có vẻ tăng.

Dựa vào điện tâm đồ trên đây, chẩn đoán


nào có khả năng đúng nhất?
A) Hội chứng Adams-Stokes
B) Rung nhĩ
C) Nhịp nhanh nút nhĩ thất
D) Nhịp nhanh nhĩ kich phát
E) Nhịp nhanh thất kịch phát
 E) Thuật ngữ kịch phát chỉ việc nhịp tim trở nên nhanh bất thường bộc phát, sự
bộc phát này khởi đầu đột ngột và kéo dài vài giây, vài phút, vài giờ hoặc lâu hơn,
rồi kết thúc cũng đột ngột như khi bắt đầu và nút xoang nhĩ lại quay lại với vai trò
tạo nhịp. Cơ chế được cho là gây ra hiện tượng này là theo con đường feedback
chuyển động quay vòng hình thành một vùng khu trú tự kích thích lặp lại. Điện
tâm đồ cho thấy có nhịp nhanh thất kịch phát. Có thể xác định nơi bắt nguồn từ các
thất vì các thay đổi của phức hợp QRS có điện thế cao và nhìn khác các phức hợp
QRS bình thường trước đó. Điều này đặc trưng cho ổ phát xung nằm ở thất.
68. Một bệnh nhân nam 65 tuổi khám định kỳ hàng năm có điện tâm đồ như sau.

Chẩn đoán nào sau đây đúng nhất?


A) Nhịp nhanh nhĩ kịch phát
B) Block nhĩ thất độ 1
C) Block nhĩ thất độ 2
D) Block nhĩ thất độ 3
E) Cuồng nhĩ
. C) Ở điện tâm đồ này ta thấy một sóng P đi trước, theo sau là 4 phức hợp QRS
đầu tiên. Sau đó là một sóng P không có QRS. Điều này đặc trưng cho block nhĩ
thất cấp 2.
69. Điều nào sau đây làm giảm nguy cơ rung thất?
A) Tim giãn to
B) Tăng thời gian trơ của tâm thất
C) Giảm tốc độ dẫn truyền xung điện
D) Tim được tạo nhịp thay thế 60 chu kỳ
E) Truyền epinephrine
. B) Tim giãn làm tăng nguy cơ xảy ra rung thất vì tăng khả năng chuyển động
quay vòng. Đồng thời, nếu tốc độ dẫn truyền giảm, xung truyền ở tim lâu hơn, làm
giảm nguy cơ rung thất. Tim được tạo nhịp thay thế 60 chu kỳ hay truyền
epinephrine đều làm tăng khả năng bị kích thích của tim. Nếu giai đoạn trơ dài, khả
năng tạo các con đường quay vòng giảm, vì khi xung truyền trong tim, tâm thất
vẫn ở trạng thái trơ.
70. Điều nào sau đây xảy ra khi tim được kích thích tạo nhịp thay thế 60 chu kỳ?
A) Tốc độ dẫn truyền qua cơ tim giảm
B) Thời gian trơ của tâm thất kéo dài hơn
C) Giảm xu hướng chuyển động quay vòng
D) Giảm xu hướng rung thất
 A) Nguy cơ xảy ra rung thất tăng ở tim được tạo nhịp thay thế 60 chu kỳ.
Khoảng trơ ngắn lại và giảm dẫn truyền qua cơ tim xảy ra, làm tăng khả năng các
con đường quay vòng. Vì vậy, khi kích thích điện đi qua tim và tới cơ tâm thất sẽ
làm khởi động lại kích thích, nguy cơ rung thất tăng nếu cơ có khoảng trơ ngắn.
71. Câu nào sau đây mô tả đúng nhất một bệnh nhân có ngoại tâm thu nhĩ (nhịp co
nhĩ sớm)?
A) Mạch quay theo ngay sau nhịp co sớm đập yếu
B) Thể tích nhát bóp theo ngay sau nhịp co sớm tăng
C) Không thấy sóng P
D) Khả năng xuất hiện nhịp co sớm giảm ở những người hấp thu lượng lớn caffein
E) Làm kéo dài khoảng QRS
 A) Nhịp tim theo ngay sau nhịp co nhĩ sớm yếu do khoảng tâm trương rất ngắn
ở bệnh này. Vì vậy, thời gian đổ đầy thất rất ngắn, từ đó thể tích nhát bóp giảm.
Sóng P thường thấy được ở rối loạn nhịp tim này trừ khi trùng vào phức hợp QRS.
Khả năng có nhịp co sớm tăng ở những bệnh nhân nhiễm độc tim và có các ổ nhồi
máu.
Câu hỏi 72 và 73
Một bệnh nhân nam hiện 63 tuổi có tiền sử nhồi máu cơ tim lúc 55 tuổi. Chuyển
đạo chi chuẩn I được ghi như sau.

72. Nhịp tim của bệnh nhân bằng?


A) 40 nhịp/phút
B) 50 nhịp/phút
C) 75 nhịp/phút
D) 100 nhịp/phút
E) 150 nhịp/phút
 E) Nhịp tim có thể được tính bằng 60 chia khoảng RR, ở đây có giá trị 150
nhịp/phút. Đây là một trường hợp nhịp nhanh, định nghĩa bằng nhịp tim > 100
nhịp/phút.
73. Chẩn đoán hiện tại của bệnh nhân là?
A) Nhịp nhanh xoang
B) Block tim độ 1
C) Block tim độ 2
D) Giảm khoảng ST
E) Block tim độ 3
 A) Mối quan hệ giữa các sóng P và các phức hợp QRS bình thường và không có
nhịp nào mất. Vì vậy, bệnh nhân này có nhịp xoang, không có block tim. Cũng
không có khoảng ST chênh xuống ở bệnh nhân này. Vì ra có các sóng P, QRS, và
T bình thường, kết luận đây là nhịp nhanh xoang.
74. Một bệnh nhân nam 55 tuổi được chẩn đoán có hội chứng Adams-Stokes. Hai
phút sau khi khởi phát hội chứng gây block xung điện tim hoạt động, đâu là vị trí
phát nhịp của tim?
A) Nút xoang
B) Nút nhĩ thất
C) Bó sợi Purkinje
D) Vách tim
E) Nhĩ trái
 B) Trong cơn hội chứng Adams-Stokes, block nhĩ thất hoàn toàn đột ngột xuất
hiện, và khoảng thời gian block có thể kéo dài vài giây tới thâm chí vài tuần. Nơi
tạo nhịp mới của tim ở xa điểm block nhưng thường ở nút nhĩ thất hoặc bó nhĩ thất.
75. Nếu kích thích gây nhịp nhanh nhĩ kịch phát bắt nguồn gần nút nhĩ thất, câu
nào sau đây đúng để nói về sóng P ở chuyển đạo I?
A) Sóng P bắt nguồn từ nút xoang
B) P thẳng đứng
C) P ngược
D) Mất sóng P
 C) Trong nhịp nhanh nhĩ kịch phát, xung khởi đầu bởi một trọng tâm lạc chỗ
đâu đó ở tâm nhĩ. Nếu điểm khởi phát ở gần nút nhĩ thất, sóng P sẽ quay lại nút
xoang nhĩ và tới tâm thất cùng líc. Vì vậy, sóng P sẽ quay ngược.
76. Một bệnh nhân nam 45 tuổi có điện tâm đồ như dưới đây ghi tại buổi thăm
khám định kỳ hàng năm.

Chẩn đoán nào sau đây là đúng nhất?


A) Nhịp nhanh nhĩ kịch phát
B) Block nhĩ thất độ 1
C) Block nhĩ thất độ 2
D) Nhịp nhanh thất kịch phát
E) Cuồng nhĩ
 A) Điện tâm đồ này đặc trưng cho một nhịp nhanh nhĩ kịch phát. Điều này đồng
nghĩa cơn nhịp nhanh bắt đầu và biến mất ở thời điểm bất kỳ. Hình dạng cơ bản
của phức hợp QRS và độ lớn của nó hầu như không đổi so với QRS bình thường,
loại trừ khả năng có nhịp nhanh thất kịch phát. Điện tâm đồ này không đặc trưng
cho cuồng nhĩ vì chỉ có 1 sóng P đi kèm 1 phức hợp QRS.
77. Một bệnh nhân nữ 60 tuổi tới khám định kỳ. Bác sỹ chỉ định làm điện tâm đồ,
kết quả trình bày dưới đây.

Chẩn đoán nào sau đây là đúng nhất?


A) Block nhĩ thất độ 1
B) Block nhĩ thất độ 2
C) Block nhĩ thất độ 3
D) Nhịp nhanh nhĩ kịch phát
 E) Block tim độ 1, 2, và 3 cũng như nhịp nhanh nhĩ kịch phát đều có sóng P trên
điện tâm đồ. Tuy nhiên, thường không có sóng P thực sự nào ở rung nhĩ, cũng như
nhịp tim sẽ không đều. Vì vậy điện tâm đồ này đặc trưng cho rung nhĩ.
Câu hỏi 78 và 79
Một bệnh nhân nam 80 tuổi tới khám bác sỹ gia đình để kiểm tra sức khỏe định kỳ
hàng năm có điện tâm đồ như sau.

78. Nhịp tim của bệnh nhân là?


A) 105
B) 95
C) 85
D) 75
E) 40
. E) Nhịp tim của bệnh nhân này là 40 nhịp/phít, được tính bằng 60 chia khoảng
RR. Đây là đặc trưng của block nhĩ thất.
79. Chẩn đoán của bệnh nhân này là
A) Block nhánh trái
B) Block nhĩ thất độ 1
C) Block nhĩ thất độ 2
D) Luân phiên điện học
E) Block nhĩ thất hoàn toàn
 E) Điện tâm đồ này đặc trưng cho block nhĩ thất hoàn toàn, còn được gọi là
block nhĩ thất độ 3. Các sóng P hầy như hoàn toàn tách biệt các phức hợp QRS, có
lúc 3 có lúc 2 sóng P giữa các phức hợp QRS. Block nhĩ thất độ 1 có các khoảng
PR kéo dài, còn block nhĩ thất độ 2 có khoảng PR kéo dài kèm các nhịp nhỡ. Tuy
nhiên điều này không thấy ở điện tâm đồ ở đây, vì không có mối liên hệ giữa các
sóng QRS và P.

Unit 4: Tuần hoàn phần 2


1. .Một phụ nữ 28 tuổi, khoẻ mạnh, đứng lên từ tư thế nằm ngửa. Sự thay đổi về
tim mạch sau đây sẽ diễn ra đối với người phụ nữ đó?

 B) Thay đổi từ tư thế nằm ngửa sang tư thế đứng gây ra sự giảm đột ngột của
huyết áp động mạch do được nhận biết bởi thụ thể áp suất nằm tại nơi chia nhánh
của động mạch cảnh chung và ở cung động mạch chủ. Kích hoạt thụ thể áp suất
động mạch gây nên tình trạng tăng cường giao cảm đến tim, mạch ngoại vi và thận
và giảm hệ phó giảm cảm đến tim. Tăng hoạt động của hệ giao cảm ở các mạch
máu ngoại vi dẫn đến sự tăng sức cản ngoại vi. Tăng hoạt động hệ giao cảm và
giảm hoạt động hệ phó giao cảm đến tim dẫn đến làm tăng nhịp tim. Và tăng hoạt
động của hệ giao cảm ở thận dẫn đến giảm lưu lượng máu tới thận.
TMP12 205–207

2. Một nam sinh viên Y 25tuổi, khoẻ mạnh, làm một thử nghiệm luyện tập tại câu
lac bộ sức khoẻ địa phương. Sự thay đổi về sinh lý nào sau đây sẽ diễn ra với hệ cơ
của anh ấy trong quá trình luyện tập?

 G) Sự tăng hoạt động chuyển hoá cục bộ trong khi luyện tập làm cho các tế bào
giải phóng các chất gây gĩan mạch ví dụ như adenosine. Sự tăng nồng độ
adenosine của mô làm giảm sức cản động mạch và làm tăng dẫn truyền trong mạch
máu và lưu lượng máu tới hệ cơ xương.
TMP12 191–195

3. Một phụ nữ 60 tuổi bị chóng mặt sau mỗi lần ra khỏi giường vào buổi sáng và
mỗi khi đứng dậy, tình trạng kéo dài 6 tháng nay. Huyết áp động mạch của cô ấy là
130/90 mmHg khi nằm và 95/60 mmHg khi ngồi. Sự thay đổi về sinh lý nào sau
đây sẽ có thể xảy ra để đáp ứng với thay đổi từ thư thế nằm ngửa sang tư thế đứng
thẳng?
 G) Thay đổi từ tư thế nằm ngửa sang tư thế đứng gây ra sự giảm đột ngột của
huyết áp động mạch do được nhận biết bởi thụ thể áp suất nằm tại các xoang mạch
cảnh và cung động mạch chủ. Kích hoạt thụ thể áp suất gây nên tình trạng giảm
hoạt động của hệ phó giao cảm và tăng hoạt động của hệ giao cảm, dẫn đến sự tăng
hoạt động của renin huyết tương (hay tăng giải phóng renin).
TMP12 205-207
4. Sự thay đổi về sinh lý nào sau đây có thể xảy ra để đáp ứng với việc tăng peptid
natri lợi niệu tâm nhĩ?

 H) Peptid natri lợi niệu tâm nhĩ (ANP) ức chế sự giải phóng renin (và sự hình
thành angiotensin II). ANP cũng ức chế sự sản xuất aldosterone, gây ra sự tăng
cường bài tiết sodium.
TMP12 208
5. Sau đây là liệt kê các áp lực thuỷ tĩnh và áp suất keo trong một giường chất vi
mô:

Áp suất thẩm thấu keo huyết tương = 25 mmHg

Áp lực thuỷ tĩnh mao mạch = 25 mmHg

Áp lực thuỷ tĩnh tĩnh mạch = 5 mmHg

Áp suất động mạch = 80 mmHg

Áp lực thuỷ tĩnh dịch kẽ = -5 mmHg

Áp suất thẩm thấu dịch kẽ = 10 mmHg

Hệ số lọc mao mạch = 10ml/min/mmHg

Tốc độ dẫn truyền dịch qua thành mao mạch là bao nhiêu?

A) 25 ml/phút
B) 50 ml/phút
C) 100 ml/phút
D) 150 ml/phút
E) 200 ml/phút
 D) Tốc độ trao đổi dịch qua thành mao mạch được tính bằng tích số của hệ số
lọc mao mạch với áp suất lọc qua lưới. Áp suất lọc qua lưới = áp lực thuỷ tĩnh mao
mạch – áp suất thẩm thấu keo huyết tương + áp suất thẩm thấu keo dịch kẽ - áp lực
thuỷ tĩnh dịch kẽ. Vì vậy, tốc độ trao đổi dịch qua thành mao mạch là 150 ml/min
- Tốc độ lọc = Hệ số lọc mao (Kf) × Áp suất lọc qua lưới
- Tốc độ lọc = Kf ×[Pc−Πc+Πi−PI]

- Tốc độ lọc = 10 ml/min/mm Hg × [25 − 25 + 10 − (−5)]


- Tốc độ lọc = 10 × 15= 150 ml/min
TMP12 181–182
6. Sau đây là liệt kê về áp lực thuỷ tĩnh và áp suất keo cũng như hệ số lọc qua
thành mao mạch của 1 cơ:
Áp lực thuỷ tĩnh mao mạch (Pc) = 25 mmHg
Áp suất thẩm thấu keo huyết tương (Πp) = 25 mmHg
Áp suất thẩm thấu keo dịch kẽ (ΠI) = 10 mmHg
Áp lực thuỷ tĩnh dịch kẽ = -5 mmHg
Tốc độ lọc mao mạch = 150 ml/phút
Hệ số lọc mao mạch là bao nhiêu?
A) 0
B) 5
C) 10
D) 15
E) 20
 C) Tốc độ lọc (FR) là tích số của hệ số lọc (Kf) và áp suất lọc qua lưới (NP) của
thành mao mạch. Vì vậy, hệ số lọc sẽ bằng tốc độ lọc chia cho áp suất lọc qua lưới.
Áp suất lọc qua lưới cho dịch di chuyển qua thành mao mạch = áp lực thuỷ tĩnh
mao mạch – áp suất thẩm thấu keo huyết tương + áp suất thẩm thấu keo dịch kẽ -
áp lực thuỷ tĩnh dịch kẽ. Áp suất lọc qua lưới trong câu hỏi này được tính là 15
mmHg và tốc độ lọc à 150/ Vì vậy, Kf bằng 150/15 hoặc bằng 10ml/phút/mmHg.
NP=[Pc−Πp+ΠI −PI]

NP = [25 − 25 + 10 − (−5)]
NP = 15
Kf = 150/15 = 10 ml/phút/mm Hg
TMP12 181–182
7. . Một loại thuốc làm giảm đường kính của động mạch ở giường cơ bắp của một
con vật thí nghiệm. Sự thay đổi về sinh lý nào sau đây có thể sẽ xảy ra để đáp ứng
với giảm đường kính?

 E) Một thuốc làm giảm đường kính của tiểu động mạch tại giường mao mạch
cơ sẽ làm tăng sức cản mạch máu. Sựu tăng sức cản của mạch làm giảm độ dẫn
truyền của mạch máu và lưu lượng máu. Sự giảm đường kính của tiểu động mạch
cũng làm giảm áp lực thuỷ tĩnh mao mạch và tốc độ lọc mao mạch.
TMP12 163–164, 181–182
8. Một phụ nữ 35 tuổi đến khám sức khoẻ tại bác sĩ gia đình của cô ấy. Huyết áp
của cô ấy là 160/75 mmHg và nhịp tim là 74 nhịp/phút. Các xét nghiệm sau đó bởi
bác sĩ tim mạch cho thấy bệnh nhân có hở van động mạch chủ vừa. Sự thay đổi nào
sau đây xảy ra ở bệnh nhân này ?
 A) Sự khác biệt giữa huyết áp tâm thu và huyết áp tâm trương là huyết áp hiệu
số. hai yếu tố chính ảnh hưởng đến huyết áp hiệu số là thể tích nhát bóp đầu ra của
tim và sức co giãn của hệ mạch. Ở bệnh nhân có hở van động mạch chủ vừa (do sự
đóng không kín của van động mạch chủ), máu khi được bơm vào động mạch chủ
ngay lập tức sẽ chảy ngược về thất trái. Sự chảy ngược của dòng máu về thất trái
làm tăng thể tích nhát bóp và huyết áp tâm thu. Dòng chảy ngược nhanh cũng làm
giảm huyết áp tâm trương. Vì vậy, những bệnh nhân có hở van động mạch chủ vừa
có huyết áp tâm thu cao, huyết áp tâm trương thấp và huyết áp hiệu số cao.
TMP12 168–169
9. Bệnh nhân nam 65 tuổi với tiền sử suy tim sung huyết 5 năm đang được điều trị
với thuốc ức chế men chuyển angiotensin (ACEI). Sự thay đổi nào sau đây có thể
sẽ xảy ra để đáp ứng với thuốc ACEI này?

 E) Angiotensin II là một chất co mạch mạnh. Angiotensin II được hình thành


bởi 1 enzym (renin) hoạt động trên một chất là angiotensinogen. Angiotensin I
được chuyển thành angiotensin II bởi một enzyme chuyển đổi. Angiotensin II là
chất gây co mạch mạnh và là hormone giữ sodium gây tăng huyết áp động mạch.
Chỉ định một thuốc ức chế men chuyển ACE được kỳ vọng sẽ làm giảm sự hình
thành angiotensin II, sức cản mạch ngoại vị và huyết áp động mạch.
TMP12 220–223
10. Các kích thích có ý thức ví dụ đọc, giải quyết vấn đề và nói chuyện đều đưa
đến kết quả rõ rệt là làm tăng lưu lượng máu não. Sự thay đổi về nồng độ các chất
trong tế bào não nào sau đây có thể giải thích cho việc tăng lưu lượng máu não đó?
 B) Các kích thích có ý thức làm tăng lưu lượng máu não do làm giảm sức cản
mạch não. Đường kính mạch máu não bị giảm do một số các yếu tố chuyển hoá
liên quan đến các kích thích có ý thức. Các yếu tố chuyển hoá làm tằn lưu lượng
máu não bao gồm tăng cacbon dioxid, ion hydrogen (giảm pH), và adenosine.
TMP12 191–194
11. Một nam giới 55 tuổi, tiền sử khoẻ mạnh, đến khám sức khoẻ định kỳ. Kiểm tra
sức khoẻ cho thấy huyết áp của bệnh nhân là 170/98 mmHg. Các xét nghiệm sau
đó cho thấy rằng bệnh nhân có tăng huyết áp do hẹp động máu thận, là kết quả của
chứng hẹp thận trái. Các thông tin nào dưới đây có thể là thông tin về bệnh nhân
với tăng huyết áp do động mạch thận?
.A) Hẹp động mạch thận 1 bên dẫn đến sự giải phóng renin và sự hình thành
angiotensin II từ thận bị ảnh hưởng. Angiotensin II kích thích sự sản xuất
aldosterone và làm tăng sức cản mạch ngoại vi bởi việc co hầu hết các mạch máu
trong cơ thể.
TMP12 222-224
12. Histamin được truyền vào động mạch cánh tay. Sự thay đổi nào sau đây của hệ
vi tuần hoàn có thể xảy ra tại tay được truyền?

 A) Histamin là một chất gây giãn mạch thường được giải phóng bởi dưỡng bào
và bạch cầu ưa baso. Truyền histamine vào động mạch cánh tay sẽ làm giảm sức
cản hệ động mạch và làm tăng tính thấm với nước của thành mao mạch. Sự giảm
sức cản tiểu động mạch cũng làm tăng áp lực thuỷ tĩnh mao mạch. Sự tăng áp lực
thuỷ tĩnh và tính thấm với nước của mao mạch dẫn đến sự tăng mức lộc mao mạch.
TMP12 163–164, 181–182

13. Bradykinin được truyền vào động mạch cánh tay của bệnh nhân nam 22 tuổi.
Sự thay đổi nào sau đây của hệ vi tuần hoàn có thể xảy ra tại tay được truyền?
 A) Brandykinin là một chất gây giãn mạch được tin là đóng một vai trò trong
việc điều hoà lưu lượng máu và sự rò rỉ mao mạch trong các mô bị viêm. Truyền
brandykinin vào động mạch cánh tay sẽ làm tăng đường kính của hệ động mạch và
làm giảm sức cản động mạch. Sự giảm sức cản động mạch cũng sẽ đẫn đến việc
làm tăng áp lực thuỷ tĩnh và mức lọc của mao mạch. Sự tăng mức lọc dẫn đến sự
tăng áp lực thuỷ tĩnh của dịch kẽ và lưu lượng bạch huyết.
TMP12 163–164, 181–182, 187–188
14. Tăng ứng suất cắt (kết quả khi lực tác động vào làm gây biến dạng trượt trên
mặt phẳng song song với hướng tác động của lực. Công thức tính ứng suất cắt
trung bình là:

với
= ứng suất cắt
F = lực tác dụng
A = diện tích mặt cắt ngang)
trong một mạch máu dẫn đến thay đổi nào sau đây
A. Giảm sản xuất endothelin
B. Giảm sản xuất guanosine monophosphate vòng (cGMP)
C. Tặng giải phóng nitric oxide
D. Tăng sản xuất renin
E. Giảm sản xuất prostacyclin
 C) Tăng ứng suất cắt (đã giải thích) trong mạch máu là một trong những yếu tố
kích thích chủ yếu để giải phóng oxide nitric bởi tế bào nội mô. Nitric oxide tăng
dòng máu bằng việc tăng guanosine vòng
TMP12 195-196
15. Một người đàn ông 72 tuổi phải mổ để lấy khối u ở bụng. Mô bệnh học chỉ ra
rằng khối u có chứa một lượng lớn mạch máu. Sự gia tăng của chất nào sau đây có
nhiều khả năng làm phát triên mạch máu trong khối u đặc
A. Hormone tăng trưởng
B. Nồng độ glucose huyết tương
C. Yếu tố phát triển angiostatin
D. Nồng độ oxi mô
E. Yếu tố phát triển nội mô mạch máu
 E) Khối u đặc là khối u tăng về chuyền hóa cần sự tăng lên về lượng với oxi và
các chất khác. Khi chuyển hóa của khổi mô tăng lên trong khoảng thời gian dài, số
lượng mạch máu trong mô cũng sẽ tăng lên.một trong những yếu tố quan trọng
tăng sự phát triển của mạch máu mới là yếu tố phát triển mạch máu của tế bào nội
mô (VEGF). Có thể, sự thiếu oxi mô cũng như các chất dinh dưỡng khác, hoặc cả
2 dẫn đến sự hình thành của VEGF
TMP12 198
16. Đường kính của đoạn động mạch tiền mao mạch tăng lên trong giường động
mạch cơ. Điều nào sau đây của vi tuần hoàn sẽ giảm đi?
A. Lưu lượng lọc mao mạch
B. Độ dẫn truyền mao mạch
C. Dòng máu trong mao mạch
D. Áp lực thủy tĩnh mao mạch
E. Sức trở khác của tiểu động mạch
 E) Sự tăng lên của đường kính đoạn tiểu động mạch tiền mao mạch sẽ giảm sức
cản của tiểu động mạch. Sự giảm sức cản này sẽ dẫn đến tăng tính dẫn của mạch
máu và dòng máu qua mao mạch, áp suất thủy tĩnh và lưu lượng loc.
TMP12 163-164, 181-182
17. Dưới các điều kiện được kiểm soát, dòng máu chảy qua một mạch máu là
100ml/min với gradient áp lực là 50 mm Hg. Dòng chảy qua mạch máu này xấp xỉ
bao nhiêu nếu tăng đường kính lên 50%, cho rằng gradient áp lực được duy trì tại
100 mmHg?
A. 100ml/
B. 150ml/phút
C. 300ml/phút
D. 500ml/phút
E. 700ml/phút
 D) Dòng máu trong một mạch máu tỷ lệ thuận với đường kính mạch máu mũ
bốn, tăng đường kính động mạch lên 50% (1.5 lần) sẽ tăng dòng máu lên 1,5 mũ 4
X dòng máu bình thường. vì thế dòng máu se tăng lên 100ml/min X 5.06, hay xấp
xỉ 500ml/ min.
TMP12 163-164
18.Một phụ nữ 24 tuổi sinh một bé nữ 6lb 8oz (2.94kg). Bé sơ sinh được chẩn đoán
là còn ống động mạch. Sự thay đổi nào sẽ diễn ra trên bé này?

 A) Ở một bệnh nhân còn ống động mạch, một lượng lớn máu bơm vào động
mạch chủ bởi thất trái ngay lập tức chảy về động mạch phổi và sau đó vào phổi
và nhĩ trái. Luồng thông của máu từ động mạch dẫn đến huyết áp tâm trương
thấp, trong khi tăng dòng chảy của máu vào nhĩ trái và thất tăng thể tích nhát
bóp và huyết áp tâm trương. Sự kết hợp của tăng huyết áp tâm thu và giảm
huyết áp tâm trương dãn đến tăng huyết áp hiệu số
TMP12 169
19. Những sự thay đổi nào sau đây diễn ra để gây ra sự tăng lên lớn nhất trong
việc di chuyển của natri qua thành mao mạch cơ?
 A) Sự di chuyển của natri qua thành mao mạch tỷ lệ thuận với tính thấm của nó
với natri, diện tích bề mặt của thành và gradient nồng độ qua thành mao mạch, vì
thế tăng tính thấm với natri, diện tính bề mặt, và gradient nồng độ muối sẽ tăng sự
di chuyển của natri qua thành mao mạch
TMP12 178-180
20. Một người đàn ông 60 tuổi đến khám hằng năm tại bác sĩ gia đình. Bệnh nhân
có huyết áp trung bình động mạch là 130 mmHg và nhịp tim là 78 nhịp/phút. Nồng
độ cholesterol tăng đến bách phân vị thứ 25 và ông ấy được chẩn đoán là bị xơ vữa
động mạch. Sự thay đổi nào sẽ diễn ra trên bệnh nhân này?
 B) Một người với xơ vữa động mạch có thể có giảm độ dãn nở của mạch máu.
Việc giảm độ dãn nở mạch máu này có thể dẫn tới sự tăng huyết áp tâm thu và
huyết áp hiệu số.
TMP12 168-169
21. . Trong khi thực hành tại labo tim mạch, một sinh viên tách động mạch cảnh
của một con vật và thắt một phần với một nút thắt quanh mạch máu. Sự thay đổi
nào sẽ diễn ra để đáp ứng với việc co động mạch cảnh?

 B) Sự co của động mạch cảnh làm giảm huyết áp động mạch tại xoang cảnh,
nơi có các thụ thệ áp suất động mạch. Việc giảm huyết áp động mạch sẽ kích hoạt
các thụ thể áp suất này, từ đó dẫn đến sự tăng hoạt động của hệ giao cảm và giảm
hoạt động của hệ phó giao cảm (hay trương lực phế vị vagaltone). Việc tăng hoạt
động của hệ giao cảm dẫn tới sự co của các mạch máu ngoại vi, trong đó có mạch
máu thận. Việc tăng hoạt động của hệ giao cảm cũng làm tăng sức cản ngoại vi và
giảm lưu lượng máu tới thận. Sự kết hợp của tăng hoạt động hệ giao cảm và giảm
hoạt động hệ phó giao cảm cũng dẫn tới làm tăng nhịp tim.
TMP12 205-207
22. Một bóng catheter được đẩy từ tĩnh mạch chủ trên vào tim và làm phồng ra để
tăng áp lực buồn nhĩ lên 5 mmHg. Điều gì sau đây sẽ tăng lên để đáp ứng với việc
tăng áp lực buồng nhĩ?
A. Peptide lợi niệu tâm nhĩ
B. Angiotensin II
C. Aldosterone
D. Hoạt động của thần kinh giao cảm của thận
 A) Peptid lợi niệu tâm nhĩ được giải phóng từ các myocyte của tâm nhĩ để đáp
ứng với sự tăng của áp lực tâm nhĩ.
TMP12 208
23. Đường kính của đoạn động mạch tiền mao mạch giảm xuống trong giường
động mạch cơ. Điều nào sau đây của vi tuần hoàn sẽ thay đổi
A. Giảm lưu lượng lọc mao mạch
B. Tăng thể tích khoảng kẽ
C. Tăng dòng lymph
D. Tăng áp lực thủy tĩnh mao mạch
E. Giảmsức trở khác của tiểu động mạch
 A) Sự giảm đường kính của tiểu động mạch tiền mao mạch làm tăng sức cản
động mạch trong khi làm giảm độ dẫn truyền mạch và lưu lượng máu mao mạch,
áp lực thuỷ tĩnh, tốc độ lọc, thể tích khoảng kẽ và áp lực thuỷ tĩnh dịch kẽ.
TMP12 163-164, 181-182
24. Một người đàn ông 50 tuổi có tiền sử tăng huyết áp 3 năm. anh ấy phàn nàn về
mệt mỏi và hay bị chuột rút ở cơ. không có tiền sử gia đình về tăng huyết áp. Bệnh
nhân không có bất cứ vấn đề nào về y khoa trong quá khứ. Khám lâm sàng cho
thấy huyết áp là 168/104. Thêm vào đó xét nghiệm lâm sàng chỉ ra rằng bệnh nhân
này có tăng aldosteron nguyên phát. Điều nào sau đây sẽ được tìm thấy trên bệnh
nhân có tăng aldosterol nguyên phát?
. C) Sự giải phóng aldosterone quá mức dẫn đến sự tăng tái hấp thu Na và đào
thải K ở ống thận. Việc tăng tái hấp thu Na và nước dẫn đến tình trạng tăng thể tích
dich ngoại bào, từ đó dẫn ức chế giải phóng renin từ thận. Việc tăng đào thải K+
dẫn tới việc giảm nồng độ K+ huyết tương, hay ha K+ huyết
TMP12 221-222
25. Một ngươi đàn ông 72 tuổi phải mổ để lấy khối u ở bụng. Mô bệnh học chỉ ra
rằng khối u có chứa một lượng lớn mạch máu. Sự giảm của chất nào sau đây có
nhiều khả năng làm phát triên mạch máu trong khối u đặc?
A. Hormone tăng trưởng
B. Nồng độ glucose huyết tương
C. Yếu tố phát triển angiostatin
D. Nồng độ oxi mô
E. Yếu tố phát triển nội mô mạch máu
 E) Việc giảm áp lực oxy mô được cho là một kích thích quan trọng cho yếu tố
phát triển nội mô mạch máu và sự phát triển của mạch máu ở các khối u đặc.
TMP12 198
26. Dưới các điều kiện được kiểm soát, dòng máu chảy qua một mạch máu là
100ml/min với gradient áp lực là 50 mm Hg. Dòng chảy qua mạch máu này xấp xỉ
bao nhiêu nếu tăng đường kính lên 4 lần, cho rằng gradient áp lực được duy trì tại
50 mmHg?
A. 300ml/phút
B. 1600ml/phút
C. 1000ml/phút
D. 16.000ml/phút
E. 25.600ml/phút
 E) Theo định luật Poiseuille, lưu lượng chảy qua một mạch máu tăng tương
xứng với luỹ thừa bốn của bán kính. Một sư gia tăng 4 lần đường kính (hay bán
kính) của mạch máu sẽ làm tăng luỹ thừa bậc 4, hay 256 lần bình thường. Vì vậy,
lưu lượng qua mạch máu sau khi tăng đường kính mạch máu lên 4 lần sẽ làm tăng
lưu lượng từ 100 lên 25.600 ml/phút.
TMP11 163-164
27. Khi tham gia vào phòng thí nghiệm sinh lý tuần hoàn, một sinh viên y khoa đã
phân lập động mạch cảnh của một con vật ở gần chỗ chia nhánh của động mạch
cảnh và co thắt một phần động mạch với các nút thắt xung quanh các mạch máu.
Thay đổi nào trong bộ các thay đổi sau đây có thể sẽ xảy ra để đáp ứng với sự co
thắt của động mạch cảnh?

 H) Sự co thắt củađộng mạch cảnh làm giảm huyết áp tại vị trí nút xoang cảnh.
Việc làm giảm áp suất của nút xoang cảnh dẫn tới giảm các xung động thần kinh
của nút xoang cảnh tới các trung tâm vận mạch , từ đó có thể dẫn tới làm tăng hoạt
động của hệ giao cảm và làm giảm hoạt động của hệ phó giao cảm, Sự tăng hoạt
động của hệ giao cảm gây ra sự co thắt mạch ngoại vi và từ đó làm tăng sức cản
ngoại vi của hệ mạch.
TMP12 205-208
28. Một người đàn ông 22 tuổi vào khoa cấp cứu tại bệnh viện sau khi bị đắt một
động mạch lớn do tai nạn xe máy. Ước tính lượng máu bệnh nhân đã mất là 70ml.
Huyết áp của bệnh nhân là 90/55 mmHg. Thay đổi nào trong bộ các thay đổi sau
đây có thể sẽ xảy ra để đáp ứng với tình trạng mất máu của bệnh nhân này?

 A) Các thụ thể áp suất động mạch được kích hoạt trong đáp ứng với sự giảm
của huyết áp động mạch. Khi có chảy máu, sự giảm huyết áp động mạch tới các
thụ thể áp suát và dẫn tới sự tăng hoạt động của giao cảm từ các trung tâm vận
mạch và làm giảm hoạt động của các thần kinh phó giao cảm. Sự tăng hoạt động
của hệ giao cảm này dẫn tới sự co thắt của các mạch máu ngoại vi, làm tăng tổng
sức cản mạch ngoại vi, và làm huyết áp trở về bình thường. Sự g hoạt động của hệ
phó giao cảm và và giao cảm tăng lên dẫn tới sự tăng nhịp tim.
TMP12 205-208
29. Một người đàn ông 22 tuổi có lưu lượng máu cơ là 250ml/phút và hematocrit là
50. Anh ấy có huyết áp động mạch trung bình là 130mmHg, huyết áp tĩnh mạch cơ
là 5 mmHg, và nhịp tim là 80 nhịp/phút. Sức cản mạch máu trong cơ của người
này là xấp xỉ bao nhiêu?

A) 0,10 mmHg/ml/phút
B) 0,20 mmHg/ml/phút
C) 0,50 mmHg/ml/phút
D) 1,00 mmHg/ml/phút
E) 2,50 mmHg/ml/phút
 C) Sức cản mạch máu = Huyết áp động mạch – Huyết áp tĩnh mạch : lưu lượng
máu. Trong ví dụ, huyết áo động mạch là 130 mmHg, huyết áp tĩnh mạch là 5
mmHg và lưu lượng máu là 250ml/phút. Từ đó, sức cản mạch máu là 125 : 250,
hay = 0,50 mmHg/ml/phút.
TMP12 162-163
30. Một phụ nữ 28 tuổi, khoẻ mạnh, đứng dậy từ tư thế đang nằm ngửa. Thay đổi
tư thế từ nằm ngửa sang đứng gây ra sự giảm thoáng qua của huyết áp động mạch,
điều này sẽ được phát hiện bởi thụ thể áp suất động mạch tại cung động mạch chủ
và xoang động mạch cảnh. Thay đổi nào trong số các thay đổi của hệ tim mạch sau
đây sẽ có thể xảy ra nhất để đáp ứng với sự kích hoạt các thụ thể áp suất?

 A) Sự kích hoạt các thụ thể áp suất dẫn tới sự tăng hoạt động của hệ giao cảm,
từ đó làm tăng nhịp tim, sức co bóp của cơ tim và sự co thắt và các động mạch nhỏ
và tĩnh mạch. Sự tăng co thắt các tĩnh mạch gây ra tăng áp suất làm đầy trung bình
tuần hoàn, sự trở về tĩnh mạch và lượng máu tim bơm ra.
TMP12 205-208
31. Một phụ nữ 35 tuổi đến khám bệnh tại bác sĩ gia đình của cô ấy. Cô ấy có
huyết áp động mạch trung bình lad 105 mmHg và nhịp tim là 74 nhịp/phút. Các sét
nghiệm sau đó của bác sĩ tim mạch cho thấy rằng bệnh nhân có hẹp van động mạch
chủ vừa. Thay đổi nào trong bộ các thay đổi dưới đây có thể xảy ra ở bệnh nhân
này?
 E) Huyết áp hiệu số là sư khác biệt của huyết áp tâm trương và huyết áp tâm
thu. 2 yếu tố chủ yếu làm ảnh hưởng tới huyết áp hiệu số là thể tích nhát bóp của
tim và sự giãn nở (??) của hệ thống động mạch. Sự tăng của thể tích nhát bóp dẫn
tới tăng huyết áp tâm thu và hiệu số, tỏng khi tăng sự giãn nở của hệ thống động
mạch làm giảm huyết áp hiệu số. Hở van động mạch chủ vừa gây ra giảm thể tích
nhát bóp, từ đó gây ra giảm huyết áp tâm thu và huyết áp hiệu số.
TMP12 168-169
32. Một người đàn ống 25 tuổi vào khoa cấp cứu của bệnh nhân sau khi bị đứt một
động mạch lớn do tai nạn tại trang trại. Lượng máu bệnh nhân đã mất xấp xỉ 800
ml. Huyết áp trung bình của bệnh nhân là 65 mmHg, và nhịp tim của anh ta tăng
cao là hậu quả của việc kích hoạt các phản xạ của thụt hể hoá học. Thay đổi nào
trong các thay đổi về nồng độ huyết tương sau có thể sẽ xảy ra ở bệnh nhân này để
dẫn tới sự kích hoạt mạnh mẽ nhất các phản xạ của thụ thể hoá học?
 G) Khi huyết áp xuống dứoi 80 mmHG, các thụ thể hoá hoạc ở cung đọng mạch
chủ và xoang động mạch cảnh được kích hoạt để gây ra phản xạ tự nhiên làm giảm
đến mức tối thiểu sự giảm huyết áp. Các thu thể hoá học là các tế bào nhạy cảm
hoá học, nhạy cảm với sự thiếu oxy, sự dư thừa CO2 hay sự dư thừa của ion H+
(hay giảm pH). Tín hiệu sẽ từ các thụ thể hoá học sẽ được chuyển đến trung tâm
vận mạch để kích thích trung tâm vận mạch từ đó làm tăng huyết áp động mạch.
TMP12 208
33. Sự tăng của thành phần nào sau đây có thể làm tăng lưu lượng bạch huyết?
A)Tính thấm nước của thành mao mạch
B) Áp suát thẩm thấu keo huyết tương
C) Áp suất thuỷ tĩnh mao mạch
D)Sức cản các động mạch nhỏ
E)A và C
 E) 2 yếu tố chính làm tăng lưu lượng bạch huyết là sự tăng tốc độ lọc mao mạch
và sự tăng hoạt động của các bơm bạch huyết. Sự tăng áp suất thẩm thấu keo huyết
tương làm giảm tốc độ lọc mao mạch, thể tích và áp lực thuỷ tĩnh dịch kẽ, và lưu
lượng bạch huyết. Đối lập lại, sự tăng tính thấm nước ủa thành mao mạch và tăng
áp lực thuỷ tĩnh mao mạch làm tăng tốc độ lọc mao mạch, thể tích và áp lực dịch
kẽ, và làm tăng lưu lượng bạch huyết. Sự tăng sức cản của các động mạch nhỏ làm
giảm áp lực thuỷ tĩnh mao mạch, tốc độc lọc mao mạch, thể tích và áp lực dịch kẽ
và lưu lượng bạch huyết.
TMP12 181-187
34. Dưới điều kiện sinh lý bình thường, lưu lượng máu tới hệ cơ xương được quyết
định chủ yếu bởi thành phần nào sau đây?
A)Các dây thần kinh giao cảm
B) Angiotensin II
C) Vasopressin
D)Nhu cầu chuyển hoá
E)Áp suất thẩm thấu mao mạch
 D) Mặc dù các dây thần kinh giao cảm, angiotensin II, vasopressin là các chất
gây co mạch mạnh, lưu lượng máu tới hệ cơ xương dưới điều kiện sinh lý bình
thường chủ yếu được quyết định bởi nhu cầu chuyển hoá cục bộ.
TMP12 194-196
35. Chất nào sau đây trong huyết tương là yếu tố chủ đạo đóng góp vào tạo nên áp
suất thẩm thấu keo huyết tương?
A) NaCl
B) Glucose
C) Albumin
D) Cholesterol
E)K+
 C) Các ion hay phân tử không thể đi qua lỗ của thành mao mạch được sử dụng
làm áp suất thẩm thấu. Thành mao mạch có tính thâm cao với NaCl, glucose,
cholesterol và K+ nhưung hầu như không có tính thấm với albumin. Vì vậy,
albumin trong huyết tương là thành phần đóng góp chủ yếu cho áp suất thẩm thấu
keo huyết tương.
TMP12 184
36. Một nữ sinh viên y khoa 22 tuổi, khoẻ mạnh, được làm thử nghiệm hoạt động
thể lực căng thẳng tại một câu lạc bộ sức khoẻ địa phương. Sự tăng của thành phần
nào sau đây có thể sẽ xảy ra ở hệ cơ xương của người này trong quá trình thực hiện
bài tập?
A) Độ dẫn truyền mạch máu
B) Lưu lượng máu chảy
C) Nồng độ CO2
D) Đường kính các động mạch nhỏ
E)Tất cả các ý trên
 E) Trong khi luyện tập thể lực, nồng độ CO2 và acid lactic trong tế bào tăng
cao. Các chất chuyển hoá này làm giãn các mạch máu, làm giảm sức cản ở động
mạch nhỏ và làm tăng độ dẫn truyền mạch máu và lưu lượng máu chảy.
TMP12 194-195
37. Cho rằng các mạch máu từ A đến D có cùng độ dài, mạch máu nào có lưu
lượng lớn nhất?

 D) Các dòng chảy trong mạch tỷ lệ thuận với chênh lệch áp suất giữa các mạch
máu và luỹ thừa bậc bốn của bán kính mạch máu. Ngược lại, lưu lượng máu tỷ lệ
nghịch với độ nhớt của máu. Bởi vì dòng máu tỉ lệ với lũy thừa bậc bốn của bán
kính mạch máu, các mạch máu có bán kính lớn nhất (mạch máu D) sẽ có dòng
chảy lớn nhất.
TMP12 163
38. Mạch máu nào sau đấy có sức cản mạch lớn nhất?

 A) Sức cản của mạch máu = chênh lệch áp suất : lưu lượng máu trong mạch.
Trong ví dụ, mạch máu A có sức cản mạch lớn nhất (100 mmHg/1000ml/phút, hay
0,1 mmHg/ml/phút)
TMP12 162-163
39. Sự tăng gấp đôi của thành phần nào sau đây có thể dẫn đến sự tăng lớn nhất
của sự vận chuyển oxy qua thành mao mạch?
A) Áp suất thuỷ tĩnh mao mạch
B) Khe gian bào thành mao mạch
C) Chênh lệch nồng độ oxy
D) Áp suất thẩm thấu keo huyết tương
E)Tính thấm nước thành mao mạch
 C) Sự vận chuyển oxy qua thành mao mạch tỷ lệ thuận với diện tích bề mặt mao
mạch, tính thấm của thành mao mạch với oxy và chênh lệch nồng độ oxy qua
thành mao mạch. Vì vậy, sự tăng gấp 2 lần chênh lệch nồng độ oxy dẫn tới sự tăng
mạnh mẽ nhất của vận chuyển oxy qua thành mao mạch. Sự tăng gấp hai của khe
gian bào trên thành mao mạch sẽ không làm thay đổi đáng kể sự vận chuyển oxy,
bởi vì oxy có thể thấm qua màng trong của tế bào thành.
TMP12 179-180
40. Loại mạch nào trong các mạch sau đây có tổng diện tích cắt ngang lớn nhất
trong hệ tuần hoàn?
A) Động mạch chủ
B) Các động mạch nhỏ
C) Mao mạch
D) Các tĩnh mạch nhỏ
E) Tĩnh mạch chủ
 C) Hệ mao mạch là mạch máu có tổng diện tích cắt ngang lớn nhất hệ tuần
hoàn. Các tĩnh mạch nhỏ cũng có tổng diện tích cắt nagng khá lớn, nhưng không
lớn bằng hệ mao mạch, điều đó giải thích vì sao một lượng lớn máu được chứa
trong hệ tĩnh mạch khi so sánh với hệ động mạch.
TMP12 160-161
41. Thành phần nào sau đây của hệ tuần hoàn chứa nhiều nhất (phần trăm lớn nhất)
thể tích máu của toàn cơ thể?
A) Động mạch
B) Mao mạch
C) Tĩnh mạch
D) Hệ tuần hoàn phổi
E) Tim
 C) Phần trăm lượng máu của toàn cơ thể được chứa trong hệ tĩnh mạch là xấp xỉ
64%.
TMP12 157
42. Sự tăng yếu tố nào sau đây có thể dẫn tới làm giảm lưu lượng máu trong một
mạch máu?
A) Chênh lệch áp suất qua mạch máu
B) Bán kính của mạch máu
C) Áp suất thẩm thấu keo huyết tương
D) Độ nhớt của máu
E) Nồng độ Na+ huyết tương
.D) Tốc độ của lưu lượng máu tỷ lệ thuận với luỹ thừa bậc bốn của bán kính
mạch máu và với chênh lệch áp suất qua mao mạch. Ngược lại, tốc độ của máu lại
tỷ lệ nghịch với độ nhớt của máu. Vì vậy, sự tăng độ nhớt của máu sẽ làm giảm tốc
độ máu chảy trong một mạch.
TMP12 163-164
43. Đoạn nào sau đây của hệ tuần hoàn có tốc độ của máu chảy lớn nhất?
A) Động mạch chủ
B) Các động mạch
C) Mao mạch
D) Các tĩnh mạch nhỏ
E) Tĩnh mạch
 A) Tốc độ của lưu lượng máu trong một phần của hệ tuần hoàn tỷ lệ nghịch với
tổng diện tích cắt ngang của phần đó. Vì động mạch chủ có diện tích cắt ngang nhỏ
nhất trong hệ tuần hoàn, vì vậy mà nó sẽ có tốc độ máu chảy lớn nhất.
TMP12 161-162
44. Sự giảm của yếu tố nào sau đây sẽ làm tăng huyết áp hiệu số?
A) Huyết áp tâm thu
B) Thể tích nhát bóp
C) Độ giãn nở của động mạch
D) Độ chun giãn tĩnh mạch
E) Thể tích huyết tương
 C) Chênh lệch giữa huyết áp tâm thu và huyết áp tâm trương được gọi là huyết
áp hiệu số. 2 yếu tốc chính làm ảnh hưởng đến huyết áp hiệu số là thể tích nhát bóp
và sự giản nở của động mạch. Huyết áp hiệu số tỷ lệ thuận với thế tích nhát bóp và
tỷ lệ nghịch với độ giãn nở của động mạch. Vì vậy, sự giảm độ giãn nở của động
mạch sẽ làm tăng huyết áp hiệu số.
TMP11 168-169
45. Sự tăng của yếu tố nào sau đây sẽ làm giảm tốc độ lọc qua mao mạch?
A) Áp suất thuỷ tĩnh mao mạch
B) Áp suất thẩm thấu keo huyết tương
C) Áp suất thẩm thấu keo dịch kẽ
D) Áp suất thuỷ tĩnh tĩnh mạch
E) Đường kính các động mạch
 B) Sự tăng áp suất thẩm thấu keo huyết tương có thể làm giảm áp suất lọc và
tốc độ lọc mao mạch. Sự tăng áp suất thuỷ tĩnh mao mạch và áp suất thẩm thấu keo
dịch kẽ cũng sẽ làm tăng lọc mao mạch. Sự tăng áp suất thuỷ tĩnh tĩnh mạch và bán
kính động mạch sẽ làm tăng áp suất thuỷ tĩnh mao mạch và tốc độ lọc mao mạch.
TMP12 181-185
46. Sự tăng của yếu tố nào sau đấy sẽ làm tăng tốc độ lọc qua mao mạch?
A) Tính dẫn nước của thành mao mạch
B) Sức cản của động mạch
C) Áp suất thẩm thấu keo huyết tương
D) Áp suất thuỷ tĩnh dịch kẽ
E) Nồng độ Na huyết tương
 A) Sự tăng tính thấm với nước của thành mao mạch sẽ làm tăng tốc độ lọc mao
mạch, trong khi đó tăng sức cản động mạch, áp suất thẩm thấu huyết tương và áp
suất thuỷ tĩnh dịch kẽ đều có thể làm giảm tốc độ lọc. Nồng độ Na huyết tương
không có ảnh hưởng gì đến sự lọc.
TMP12 181-186
47. Sự giảm của yếu tố nào sau đây sẽ làm tăng lưu lượng bạch huyết?
A) Áp suất thuỷ tĩnh mao mạch
B) Áp suất thuỷ tĩnh dịch kẽ
C) Áp suất thẩm thấu keo huyết tương
D) Hoạt động của hệ bơm bạch huyết
E) Đường kính các động mạch
 C) Tốc độ lưu lượng bạch huyết tăng tỷ lệ thuận với áp suất thuỷ tĩnh dịch kẽ và
hoạt động của bơm bạch huyết. Sự giảm áp suất thẩm thấu keo huyết tương có thể
làm tăng tốc độ lọc, thể tích dịch kẽ áp suất thuỷ tĩnh dịch kẽ và lưu lượng bạch
huyết. Sự giảm đường kính động mạch có thể làm giảm áp suất thuỷ tĩnh mao
mạch, tốc độ lọc và lưu lượng bạch huyết.
TMP12 181-188
48. Hệ mao mạch nào sau đây có tính thấm thấp nhất với các phân tử của huyết
tương?
A) Cầu thận
B) Gan
C) Cơ
D) Ruột
E) Não
 E) Não bộ có chỗ nối giữa các mao mạch tế bào màng trong rất chặt chẽ, chỉ
cho phép các phân tử rất nhỏ đi qua ví dụ như nước, oxy và CO2 đi vào hoặc ra
khỏi các mô não.
TMP12 178
49. Yếu tố nào sau đây sẽ làm tăng sự dịch chuyển qua glucose qua thành mao
mạch?
A) Tăng nồng độ Na huyết tương
B) Tăng chênh lệch nồng độ glucose qua thành mao mạch
C) Giảm tính thấm thành mạch với glucose
D) Giảm diện tích bề mặt thành mạch nhưng không tăng số lượng các lỗ
E) Giảm nồng độ K huyết tương
 B) Các yếu tố quyết định đến sự di chuyển của glucose qua thành mao mạch
bao gồm tính thấm của thành mao mạch với glucose, chênh lệch nồng độ glucose
qua thành mạch, và diện tích bề mặt thành mao mạch, Vì vậy, sự tăng chênh lệch
nồng độ của glucose qua thành mạch sẽ làm tăng sự di chuyển của glucose.
TMP12 179-180
50. Một người đàn ông 65 tuổicó suy tim sung huyết. Cung lượng tim là 4l/phút,
huyết áp động mạch là 115/85 mmHg và nhịp tim là 90 nhịp/phút. Các kết quả xét
nghiệm sau đó do bác sĩ tim mạch chỉ định cho thấy bệnh nhân có áp suất tâm nhĩ
phải là 10 mmHg. Sự tăng của yếu tố nào sau đây sẽ có thể có ở bệnh nhân này?
A) Áp suất thẩm thấu keo huyết tương
B) Áp suất thẩm thấu keo dịch kẽ
C) Áp suất động mạch
D) Lượng máu tim tống đi
E) Áp suất thuỷ tĩnh tĩnh mạch chủ
 E) Sự tăng huyết áp tâm thu lên 10 mmHg sẽ làm giảm sự trở về tim của máu
tĩnh mạch và làm tăng áp suất thuỷ tĩnh tĩnh mạch chủ. Áp suất thẩm thấu keo
huyết tương, áp suất thẩm thấu keo dịch kẽ, huyết áp động mạch và lượng máu tim
bơm ở bệnh nhân này nói chung đều thấp hơn so với bình thường.
TMP12 172-173
51. Phần nào sau đây của hệ tuần hoàn của độ giãn nở lớn nhất?
A) Mao mạch
B) Động mạch lớn
C) Tĩnh mạch
D) Động mạch chủ
E) Động mạch nhỏ
 C) Độ giãn nở của hệ mạch tỷ lệ thuận với tính căng phồng được của mạch và
thể tích mạch với tất cả các phần của hệ tuần hoàn. Độ giãn nở của một tĩnh mạch
của một hệ thống gấp 24 lần so với động mạch đi cùng nó bởi tĩnh mạch có độ
căng phồng gấp 8 lần và thể tích gấp 3 lần so với động mạch.
TMP12 167
52. Sử dụng những dự kiện sau đây, hãy tính hệ số lọc cho giường mao mạch:
Áp suất thẩm thấu keo huyết tương = 30 mmHg
Áp suất thuỷ tĩnh mao mạch = 40 mmHg
Áp suất thuỷ tĩnh dịch kẽ = 5 mmHg
Áp suất thẩm thấu keo dịch kẽ = 5 mmHg
Tốc độ lọc = 150 ml/phút
Áp suất thuỷ tĩnh tĩnh mạch = 10 mmHg
A) 10 mm/phút/mmHg
B) 15 mm/phút/mmHg
C) 20 mm/phút/mmHg
D) 25 mm/phút/mmHg
E) 30 mm/phút/mmHg
 B) Hệ số lọc (Kf) = tốc độ lọc : áp suất lọc qua lưới. Áp suất lọc qua lưới = áp
suất thuỷ tĩnh mao mạch – áp suất thẩm thấu keo huyết tương + áp suất thẩm thấu
keo dịch kẽ - áp suất thuỹ tĩnh dịch kẽ. Áp suất lọc qua lưới tỏng ví dụ này là 10
mmHg. Vì vậy, Kf = 150 ml/phút : 10 mmHg, và bằng 15 ml/phút/mmHg.
TMP12 181-186
53. Những thay đổi sinh lý nào sau đây sẽ có thể xảy ra với 1 người khi thay đổi tư
thế từ nằm ngửa sang đứng dậy?

 B) Thay đổi tư thế từ nằm ngửa sang đứng sẽ dấn đến ứ trệ máu ở (sự tập hợp
máu vào) chi dưới và giảm huyết áp. Sự tập trung máu xuống chi dưới làm tăng áp
suất thuỷ tĩnh tĩnh mạch. Sự giảm huyết áp động mạch kích hoạt các thu thể áp suất
động mạch, từ đó làm tăng hoạt động của hệ thần kinh giao cảm và giảm hoạt động
của hệ phó giao cảm. Sự tăng hoạt động của hệ giao cảm làm co mạch máu thận và
giảm lưu lượng máu thận. Nhịp tim cũng sẽ tăng lên.
TMP12 205-207
54. Lưu lượng máu đến mô duy trì tương đối ổn định cho dù có sự giảm huyết áp
động mạch (cơ chế tự điều chỉnh). Điều nào sau đây sẽ xảy ra để đáp ứng với sự
giảm huyết áp động mạch?
A. Giảm lưu lượng (độ dẫn truyền)
B. Giảm nồng độ CO2 mô
C. Tăng nồng độ O2 mô
D. Giảm sức cản mạch máu
E. Giảm đường kính tiểu động mạch (bán kính động mạch nhỏ)
 D) Sự giảm áp suất tưới máu (làm đầy) tại mô dẫn tới sự giảm nồng độ O2 mô
và sự tăng nồng độ CO2 mô. Cả 2 hiện tượng này dẫn tới sự tăng đường kính (bán
kính) tiểu động mạch, giảm sức cản mạch máu và làm tăng lưu lượng (độ dẫn
truyền mạch máu)
TMP12 194-196
(Đáp án (phần đáp án này bị sai số câu, có câu 55 đáp án trả lời cho 1 câu
nào đấy mà không có trong đề, còn đáp án đánh số 56 lại trả lời cho câu 55,
cứ thế (đáp án đánh số 57 là trả lời câu 56, 58 trả lời câu 57…)
55. Dòng máu chảy có khuynh hướng xoáy nhất ở đâu?
A. Tiểu động mạch
B. Mao mạch
C. Tiểu động mạch nhỏ
D. Động mạch chủ
 D) Sự phân chia của huyết áp động mạch xảy ra mạnh nhất tại nơi (Phần lớn
nhất của huyết áp động mạch là tại phần) có sức cản mạch máu lớn nhất, đó là chỗ
nối tiểu động mạch và mao mạch.
TMP12 158
56. Cơ chế tự điều chỉnh của lưu lượng máu mô đáp ứng với sự tăng của huyết áp
động mạch diễn ra là kết quả của quá trình nào dưới đây?
A. Giảm sức cản mạch máu
B. Sự giảm ban đầu của sức căng thành mạch
C. Tăng quá mức sự vận chuyển các chất dinh dưỡng, ví dụ như O2, tới mô
D. Giảm chuyển hoá mô
 D) Dòng máu có xu hướng chảy xoáy tại nơi có tốc độ dòng chảy cao (Khuynh
hướng máu chảy xoáy nhất là ở mạch máu nơi có tốc độ của dòng máu chảy lớn).
Động mạch chủ có tốc độ dòng máu chảy lớn nhất.
TMP12 161-162
57. Áp lực nào sau đây thường là âm tính trong một giường mao mạch cơ ở chi
dưới?
A. Áp suất thẩm thấu keo huyết tương
B. Áp suất thuỷ tĩnh mao mạch
C. Áp suất thuỷ tĩnh dịch kẽ
D. Áp suất thẩm thấu keo dịch kẽ
E. Áp suất thuỷ tĩnh tĩnh mạch
 C) Sự tăng áp suất tưới máu tại (làm đầy đến) mô dẫn đến sự vận chuyển quá
mức các chất dinh dưỡng ví dụ như O2 đến mô. Sự tăng nồng độ O2 mô làm co
thắt các tiểu động mạch và đưa (trả) lưu lượng máu và sự chuyển các chất dinh
dưỡng về ngưỡng bình thường.
TMP12 194-195
58. Yếu tố nào sau đây có thể làm giảm áp suất thuỷ tĩnh tĩnh mạch ở chân?
A. Tăng áp suất tâm nhĩ phải
B. Thai nghén
C. Vận động (Sự di chuyển của) các cơ ở chân
D. Xuất hiện dịch cổ trướng (dư thừa trong bụng)
 C) Áp suất thuỷ tĩnh dịch kẽ ở giường mao mạch cơ bình thường âm tính (-3
mmHg). Nguyên nhân cơ bản của áp suất âm này là do vai trò (Sự) bơm máu của
hệ bạch huyết.
TMP12 183-184
59. Sự di chuyển của các chất tan ví dụ như Na+ qua thành mao mạch xảy ra chủ
yếu nhờ quá trình nào sau đây?
A. Lọc
B. Vận chuyển chủ động
C. Vận chuyển qua lỗ
D. Khuếch tán
 C) (Sự di chuyển của) Các cơ ở chi dưới vận động đẩy (làm cho) máu về (chảy
đến) tĩnh mạch chủ, từ đó làm giảm áp suất thuỷ tĩnh tĩnh mạch ngoại vi. Sự tăng
áp suất tâm nhĩ phải sẽ làm gỉam lượng máu tĩnh mạch về tim (sự trở về tĩnh mạch)
và lại làm tăng áp suất thuỷ tĩnh tĩnh mạch ngoại vi. Thai nghén và dịch cổ trướng
(xuất hiện của dịch dư thừa ổ bụng) có thể làm chèn ép tĩnh mạch chủ và làm tăng
áp suất thuỷ tĩnh tĩnh mạch chi dưới
TMP12 172-173
60. Chất nào sau đây có tốc độ di chuyển qua qua thành mao mạch lớn nhất?
A. Na+
B. Albumin
C. Glucose
D. O2
 D) Cơ chế chính (ban đầu) của sự di chuyển các chất tan qua thành mao mạch là
sự khuyếch tán đơn giản.
TMP12 179
61. Sự giảm yếu tố nào sau đây có thể xảy ra để đáp ứng với sự tăng trực tiếp áp
suất động mạch thận?
A. Bài tiết nước
B. Bài tiết Na+
C. Thể tích dịch ngoại bào
D. Mức (Tốc độ) lọc cầu thận
E. Sự tuôn máu vào tâm thất trong giai đoạn đầu hoặc giữa kỳ tâm trương
 D) Bởi vì O2 tan trong lipid và có thể đi qua thành mao mạch dễ dàng nên nó có
tốc độ di chuyển qua thành mao mạch lớn nhất.
TMP12 179
62. Sự dư thừa sản xuất sản phẩm nào dưới đây sẽ dẫn đến tăng huyết áp mạn tính?
A. Peptid lợi niệu tâm nhĩ
B. Prostacyclin
C. Angiotensin II
D. NO
 C) Sự tăng áp suất động mạch thận dẫn đến sự tăng bài tiết natri và lợi niệu. Sự
mất Na+ và nước sẽ làm giảm thể tích dịch ngoại bào. Lưu lượng hoặc Mức (Tốc
độ) lọc cầu thận sẽ bình thường hoặc hơi tăng nhẹ để đáp ứng với sự tăng áp suất
động mạch thận.
TMP12 213-215
63. Sự giảm (tăng) yếu tố nào sau đây sẽ xảy ra để đáp ứng với với sự tăng hấp thu
Na+?
A. Angiotensin II
B. NO (Oxid nitric)
C. Bài tiết Na+
D. Peptid lợi niệu tâm nhĩ
 C) Oxid nitric và prostacyclin là các chất giãn mạch và lợi niệu mạnh. Thêm
nào đó, peptid lợi niệu tâm nhĩ cũng là một chất lợi niệu và làm giảm huyết áp.
Ngược lại, angiotensin II là hormone (một chất) làm co mạch manh, chống bài niệu
(hạn chế lợi niệu qua nước tiểu) và tăng huyết áp.
TMP12 195-196
64. Điều nào sau đấy sẽ xảy ra để đáp ứng với sự cho thắt động mạch thận?
A. Tăng bài tiết Na+
B. Giảm huyết áp động mạch
C. Giảm bài tiết renin
D. Tăng angiotensin II
 A) Sự tăng hấp thu Na+ có thể dẫn đến sự tăng bài tiết Na+ để duy trì cân bằng
Na+. Angiotensin II giảm để đáp ứng với sự tăng hấp thu Na+ mạn tính trong khi
oxid nitric và peptid lợi niệu tâm nhĩ tăng.
TMP12 217-222
65. Sự tăng áp suất tâm nhĩ sẽ dẫn đến điều nào sau đây?
A. Giảm peptid lợi niệu tâm nhĩ trong huyết tương
B. Tăng nồng độ angiotensin II huyết tương
C. Tăng nồng độ aldosterone huyết tương
D. Tăng nhịp tim
 D) Sự co thắt động mạch thận làm tăng bài tiết renin, tăng tạo angiotensin II, và
tăng huyết áp động mạch. Sự bài tiết Na+ giảm, thế nhưng chỉ tạm thời, bởi vì khi
huyết áp động mạch tăng, sự bài tiết Na+ sẽ trở về bình thường thông qua cơ chế
lợi niệu áp lực.
TMP12 223-225
66.D) Sự tăng áp suất tâm nhĩ gây ra sự tăng nhịp tim thông qua một phản xạ thần
kinh có tên là phản xạ Bainbridge. Cơ quan nhận cảm với sự căng giãn ở tâm nhĩ
gây ra phản xạ Bainbridge truyền tín hiệu hướng tâm của nó qua dây phế vị tới
hành não. Các tín hiệu ly tâm sẽ được truyền ngược lại qua thần kinh số X và thần
kinh phó giao cảm để làm tăng nhịp tim. Sự tăng áp suất tâm nhĩ cũng sẽ làm tăng
nồng độ peptid lợi niệu tâm nhĩ trong huyết tương, từ đó làm giảm nồng độ
angiotensin II và aldosterone trong huyết tương.
TMP12 208-209
66. Điều nào sau đây có thể xảy ra trong phản ứng Cushing gây ra bởi thiếu máu
não cục bộ?
A) Tăng hoạt động hệ phó giao cảm
B) Giảm huyết áp động mạch
C) Giảm nhịp tim
D) Tăng hoạt động hệ giao cảm
 D) Phản ứng Cushing là phản ứng đặc trung của sự thiếu máu cục bộ hệ thần
kinh trung ương, là kết quả của sự tăng áp lực dịch não tuỷ xung quanh não bên
trong hộp sọ. Khi áp lực dịch não tuỷ tăng, nó làm giảm lượng máu cung cấp cho
não và đưa đến phản ứng của hệ thần kinh trung ương. Phản ứng với sự thiếu máu
của hệ thần kinh trung ương bao gồm tăng hoạt động hệ giao cảm, giảm hoạt động
hệ phó giao cảm; và tăng nhịp tim, huyết áp động mạch và tổng sức cản ngoại vi.
TMP12 209-210
67. Điều nào sau đây thường xảy ra trong suy tim mất bù?
A) Tăng sự mất Na+ và nước của thận
B) Giảm áp suất trung bình làm đầy hệ thống
C) Tăng norepinephrine trong thần kinh giao cảm của tim
D) Khó thở khi nằm
E) Giảm cân

68. Chát ức chế enzyme chuyển angiotensin được kê cho một bệnh nhân nam 65
tuổi với tiền sử cao huyết áp 20 năm nay. Thuốc đó làm giảm huyết áp động mạch
và tăng nồng độ renin và bradykinin huyết tương. Điều nào sau đây có thể giải
thích sự hoạt hoá của bradykinin huyết tương?
A) Sự ức chế preprobradykinin
B) Giảm sự chuyển hoá từ angiotensin I thành angiotensin II
C) Tăng tạo angiotensin II
D) Tăng tạo killikrein
E) Ức chế kininases
 E) Sự chuyển từ angiotensin I thành angiotensin II được thực hiện nhờ một
enzym chuyển có mặt ở nội mô mao mạch phồi và ở thận. Enzym chuyển cũng có
tác dụng là một kininase làm giáng hoá bradykinin. Vì vậy, chất ức chế enzyme
chuyển không những làm giảm quá trình tạo thành angiotensin II mà còn ức chế
kininase và sự giáng hoá bradykinin.
TMP12 220
69. Một người đàn ông 60 tuổi có huyết áp động mạch trung bình là 130 mmHg,
nhịp tim là 78 lần/phút, áp suất nhĩ phải là 0 mmHg, và cung lượng tim là 3,5
L/phút. Ông ta cũng có huyết áp hiệu số là 35 mmHg và hematocrit là 40. Tổng
sức cản hệ mạch ngoại vi ở người đàn ông này xấp xỉ là bao nhiêu?
A) 17 mm Hg/L/phút
B) 1.3 mm Hg/L/phút
C) 13 mm Hg/L/phút
D) 27 mm Hg/L/phút
E) 37 mm Hg/L/phút
 E) Tổng sức cản hệ mạch ngoại vi = huyết áp động mạch – áp suất nhĩ phải :
cung lượng tim. Trong ví dụ, tổng sức cản mạch ngoại vi = 130 mmHg : 3,5
L/phút, và xấp xỉ bằng 37 mmHg/L/phút.
TMP12 162-163
70. Trong biểu đồ sau, cung lượng tim và lượng máu tĩnh mạch về tim được biểu
diễn bởi đường nét liền màu đỏ (với điểm cân bằng ở A), khẳng định nào sau đây
là đúng?
A) Áp suất làm đầy hệ thống trung bình là 12 mmHg
B) Áp suất tâm nhĩ phải là 2 mmHg
C) Sức cản với máu tĩnh mạch về tim là 1,4 mmHg/L/phút
D) Áp suất lưu lượng phổi là xấp xỉ 7 L/phút
E) Sức cản với máu tĩnh mạch về tim là 0,71 mmHg/L/phút
 C) Công thức tính sức cản với lượng máu về tim là áp suất trung bình làm đầy
hệ thống – áp suát nhĩ phải : cung lượng tim. Trong ví dụ, áp suất trung bình làm
đầy hệ thống là 7 mmHg và áp suất nhĩ phải là 0 mmHg. Cung lượng tim là
5L/phút. Sử dụng các giá trị đó áp dụng vào công thức tính sức cản lượng máu về
tim, ta được kết quả là 1,4 mmHg/L/phút. Lưu ý rằng công thức này chỉ sử dụng
đối với phần tuyến tính trên đường cong lượng máu tĩnh mạch về tim.
TMP12 238-239
71. Một người đàn ông 30 tuổi đang nghỉ ngơi, và xung động hệ giao cảm của anh
ấy đang tăng dần đến mức tối đa. Nhóm những thay đổi nào sau đây có thể xảy ra
để đáp ứng với sự tăng xung động của hệ giao cảm?
 A) Khi xung động hệ giao cảm tăng đến mức tối đa, nhiều thay đổi xảy ra. Đầu
tiên, áp suất trung bình làm đầy hệ thống tăng lên có ý nghĩa (đáng kể) tuy nhiên
đồng thời với nó sức cản cho lượng máu về tim cũng tăng lên, Lượng máu tĩnh
mạch về tim được tinh bằng công thức: áp suất trung bình làm đầy hệ thống – áp
suất nhĩ phải / sức cản lượng máu về tim. Khi xung động hệ giao cảm lên tối đa, sự
tăng áp suất trung bình làm đầy hệ thống lớn hơn sự tăng sức cản lượng máu về
tim. Từ đó, trong công thức trên, mẫu số tăng nhiều hơn so với tử số. Kết quả là
làm tăng lượng máu về tim.
TMP12 238-239
72. Nếu bệnh nhân có lượng tiêu thụ oxy là 240 ml/phút, nồng độ oxy trong máu
tĩnh mạch phổi là 180 ml/L và trong máu động mạch phổi là 160 ml/L, cung lượng
tim ở bệnh nhân này là bao nhiêu lít một phút?
A) 8
B) 10
C) 12
D) 16
E) 20
 C) Vấn đề này liên quan đên định luật Fick để đánh giá cung lượng tim. Công
thức để tính cung lượng tim là lượng oxy được hấp thu trong một phút bởi phổi
chia cho hiệu số nồng độ oxy trong máu động mạch và tĩnh mạch. Trong câu hỏi
này, lượng oxy được tiêu thụ bởi cơ thể là 240 ml/phút, và trong trạng thái bình
thường đây chính bằng lượng oxy được hấp thu bởi phổi. Vì vậy, khi cho những
giá trị trên vào công thức tính, chúng ra có được cung lượng tim bằng 12 L/phút.
TMP12 240
73. Nếu lồng ngực ở một người bình thường được phẫu thuật mở, điều gì sẽ xảy ra
với đường cong cung lượng tim?
A) Lệch về bên trái 2 mmHg
B) Lệch vè bên trái 4 mmHg
C) Lệch về bên phải 2 mmHg
D) Lệch về bên phải 4 mmHg
E) Không thay đổi
 D) Áp suất bình thường trong khoang màng phổi là -4 mmHg. Khi phẫu
thuật mở lồng ngực, tất cả các áp suất trong lồng ngực đều ngay lập tức có
giá trị bằng 0 mmHg, đó là áp suất của không khí. Sự tăng áp suất trong lồng
ngực này dẫn đến sự co rút của tâm nhĩ và làm giảm chênh lệch áp suất giữa
hai tâm nhĩ (chênh áp xuyên thành của mỗi tâm nhĩ) (??). Cụ thể, chênh áp
xuyên thành tâm nhĩ phải giảm 4 mmHg. Vì vậy, đường cong cung lượng
tim sẽ bị lệch về bên phải 4 mmHg.

74. Điều nào sau đây thường làm cho đường cong cung lượng tim lêch về bên trái
theo trục của áp suất tâm nhĩ phải?
A) Phẫu thẩu mở lồng ngực
B) Chèn ép tim nặng
C) Thở kháng lại áp lực âm
D) Thổi kèn trumpet
E) Thở áp lực dương
.C) Nhiều yếu tố có thể làm cho đường cong cung lượng tim lệch về bên phải
hay bên trái. Những yếu tố này bao gồm phẫu thuật mở lồng ngực, điều này làm
cho đường cong cung lượng tim lệch 4 mmHg về bên phải; và chèn ép tim nặng,
điều này làm tăng áp suất trong màng ngoài tim, từ đó làm sự co bóp của tim, đặc
biệt là tâm nhỉ. Thổi kèn trumpet hay thở áp lực dương làm tăng rất lớn áp suất
khoang màng phổi, từ đó làm co rút tâm nhĩ và làm lệch đường cong cung lượng
tim về bên phải. Thở phản lại áp lực âm sẽ làm lệch đường cong cung lượng tim về
bên trái.
TMP12 234
75. Điều nào sau đây sẽ làm tăng đoạn bằng của đường cong cung lượng tim?
A) Phẫu thuật mở khung sườn
B) Đặt bệnh nhân vào thở máy
C) Chèn ép tim
D) Tăng sự kích thích hệ phó giao cảm đến tim
E) Tăng sự kích thích hệ giao cảm đến tim
 E) Phần ổn định (bằng phẳng) của đường cong cung lượng tim, được đo bằng
(là một chỉ số đo lường) sự co bóp của tim, giảm trong nhiều tình trạng khác nhau,
đó có thể là chèn ém tim nặng làm tăng áp suất trong khoang ngoài tim hay tăng
kích thích hệ phó giao cảm của tim. Tăng kích thích hệ giao cảm của tim làm tăng
ngưỡng của đường cong cung lượng tim bằng cách làm tăng nhịp tim và sự co bóp
của cơ tim.
TMP12 231
76. Điều nào sau đây thường làm cho đường cong cung lượng tim lệch về bên phải
theo trục của áp suất tâm nhĩ phải?
A) Tăng áp suất khoang màng phổi lên -6 mmHg
B) Tăng áp suất trung bình làm đầy hệ thống
C) Đưa bệnh nhân ra khỏi trạng thái (Khi dừng) thở máy và bắt đầu hô hấp bình
thường
D) Phẫu thuật mở lồng ngực
E) Thở kháng lại áp lực âm
 D) Nhiều yếu tố có thể làm cho đường cong cung lượng tim lệch về bên phải
hay bên trái. Những yếu tố này bao gồm phẫu thuật mở lồng ngực, điều này làm
cho đường cong cung lượng tim lệch 4 mmHg về bên phải; và chèn ép tim nặng,
điều này làm tăng áp suất trong màng ngoài tim, từ đó làm sự co bóp của tim, đặc
biệt là tâm nhỉ. Thổi kèn trumpet hay thở áp lực dương làm tăng rất lớn áp suất
khoang màng phổi, từ đó làm co rút tâm nhĩ và làm lệch đường cong cung lượng
tim về bên phải.
TMP12 234
77. Tình trạng nào sau đây có thể làm giảm áp suất làm đầy hệ thống?
A) Sử dụng norepinephrine
B) Tăng thể tích máu
C) Giảm (Tăng) kích thích hệ giao cảm
D) Tăng sự giãn nở tĩnh mạch
E) Co cơ xương
 D) Áp suất trung bình làm đầy hệ thống là áp suất của lượng máu làm đầy kín
hệ tuần hoàn. Áp suất trung bình làm đầy hệ thống tăng lên bởi các yếu tố làm tăng
thể tích máu và bởi các yếu tố làm giảm sự giản nỡ (compliance) hệ mạch. Do vậy,
sự giảm giản nỡ (compliance) tĩnh mạch, chứ không phải là tăng compliance, có
thể dẫn đến sự tăng áp suất trung bình làm đầy hệ thống. Sử dụng norepinephrine
và kích thích hệ giao cảm gây ra co tiểu động mạch trong hệ tĩnh mạch, và giảm sự
giãn nỡ (compliance) mạch máu, dẫn đến sự tăng áp suất trung bình làm đầy hệ
thống. Tăng thể tích máu và sự co cơ xương gây ra sự co thắt của hệ mạch, cũng
làm tăng áp suất làm đầy.
TMP12 236
78. Điều nào sau đây thường có liên quan đến sự tăng lượng máu tĩnh mạch trở về
tim?
A) Giảm áp suất trung bình làm đầy hệ thống
B) Giãn các tĩnh mạch lớn cấp tính
C) Giảm trương lực hệ giao cảm
D) Tăng sự giản nỡ (compliance) tĩnh mạch
E) Tăng thể tích máu
 E) Máu tĩnh máu trở về tim cân được tính bắng áp suất trung bình làm đầy hệ
thống trừ đi áp suất nhĩ phải chia cho sức cản lượng máu tĩnh mạch về tim. Vì vậy,
giảm áp suất làm đầy hệ thống sẽ làm giảm lượng máu tĩnh mạch về tim. Những
yếu tố làm giảm áp lực làm đầy hệ thống bao gồm sự giãn to của tĩnh mạch lớn,
giảm trương lực hệ giao cảm, tăng sự giãn nở tĩnh mạch(??) và tăng thể tích máu.
TMP12 236-237
79. Đường cong nào trong biểu đồ sau (được vẽ lại từ Guyton AC, Jones CE,
Coleman TB: Sinh lý tuần hoàn: Cung lượng tim và Sự điều chỉnh, ấn bản thứ 2,
Philadelphia: WB Saunders, 1973) có sức cản lượng máu tĩnh mạch về tim lớn
nhất?

A) Đ ường màu xanh lam với áp suất trung bình hệ thống (Psf) = 10
B) Đường màu xanh lục với Psf = 10,5
C) Đường màu đen với Psf = 2,3
D) Đường màu đỏ với Psf = 7
B) Sức cản lượng máu về tim là sự ngịch đảo của đường dốc trong phần tuyến
tính của đường cong lượng máu tĩnh mạch về tim. Vì vậy, đường cong có độ dốc
nhỏ nhất sẽ có sức cản lượng máu về tim lớn nhất.
TMP12 238
80. Điều này sau đây thường có liên quan với sự tăng cung lượng tim?
A) Tăng độ giãn nở hệ tĩnh mach
B) Chèn ép tim
C) Phẫu thuật mở lồng ngực
D) Thiếu máu vừa
E) Hẹp động mạch chủ nặng
 D) Giảm cung lượng tim có thể là kết quả của tim bị yếu hay sự giảm lượng
máu về tim. Tăng sự giãn nở tĩnh mạch làm giảm lượng máu về tim. Chèn ép tim,
phẫu thuật mở lồng ngực và hẹp van động mạch chủ nặng đều làm yếu tim hiệu
quả và từ đó làm giảm cung lượng tim. Thiếu máu vừa có thể gây ra sự giãn tiểu
động mạch, từ đó làm tăng lượng máu tĩnh mạch về tim và từ đó làm tăng cung
lượng tim.
TMP12 239
81. Tình trạng nào sau đây có thể gây ra giảm cung lượng tim?
A) Cường giáp
B) Bệnh Beri-beri
C) Rò A-V (thông động tĩnh mạch)
D) Thiếu máu
E) Nhồi máu cơ tim cấp
 E) Cung lượng tim tăng trong nhiều tình trạng bởi sự tăng lượng máu về tim.
Cung lượng tim tăng trong cường giáp bởi sự tăng sử dụng oxy của các mô ngoại
vi dẫn đến sự giãn nở hệ động mạch và từ đó tăng lượng máu tĩnh mạch về tim.
Bện beriberi gây ra sự tăng cung lượng tim bởi sự thiếu vitamin thiamin gây ra sự
giãn mạch ngoại vi. Rò A-V cũng làm giảm sức cản lượng máu về tim và vì thế là,
tăng cung lượng tim. Thiếu máu, bởi sự giảm vận chuyển oxy đến các mô, gây ra
sự tăng lượng máu về tim và từ đó cũng làm tăng lưu lượng tim. Cung lượng tim
trong bệnh nhân với nhồi máu cơ tim.
TMP12 232-233
82. Khi bắt đầu hoạt động thể dục, điều nào sau đây sẽ xảy ra?
A) Giảm lưu lượng máu não
B) Tăng sự co thắt tĩnh mạch
C) Giảm lưu lượng máu mạch vành
D) Giảm áp suất trung bình làm đầy hệ thống
E) Tăng xung động hệ phó giao cảm đến tim
 B) Khi hoạt động thể lực, có rất ít thay đổi đến lưu lượng mach não và lưu
lượng mạch vành tăng lên. Bởi sự tăng xung động hệ giao cảm, áp suất trung bình
làm đầy hệ thống tăng và các tĩnh mạch co lại. Khi hoạt động thể lực cũng sẽ có sự
giảm trong xung động hệ phó giao cảm đến tim.
TMP12 238-239
83. Điều nào sau đây thường làm tăng mức bằng phẳng của đường cong cung
lượng tim?
A) Viêm cơ tim
B) Chèn ép tim nặng
C) Giảm kích thích của hệ phó giao cảm của tim
D) Nhồi máu cơ tim
E) Hẹp van hai lá
 C) Độ bằng phẳng của đường cong cung lượng tim, được đo bằng tinh co thắt
của co tim (là một chỉ số đánh giá khả năng co bóp của tim), giảm trong nhiều
trường hợp. Trong số đó bao gồm viêm cơ tim, chèn ép tim nặng làm tăng áp lực
trong khoảng quanh tim, nhồi máu cơ tim và một số bệnh van tim ví dụ như hẹp
van hai lá. Giảm kích thích hệ phó giam cảm của tim thực chất làm tăng vừa độ
bằng phẳng của đường cong cung lượng tim bằng cách làm tăng nhịp tim.
TMP12 231
84. Nếu một người đã hoạt động thể lực được 1 tiếng, cơ quan nào sau đây sẽ có
giảm lưu lượng máu thấp(ít) nhất?
A) Não
B) Ruột
C) Thận
D) Cơ vân không hoạt động thể lực
E) Tuỵ
 A) Khi tăng xung động hệ giao cảm, 2 cơ quan chính vẫn giữ nguyên lưu lượng
máu và não và tim. Khi hoạt động thể lực tỏng 1 giờ, lưu lượng tới ruột giảm đáng
kễ, tương tự với lưu lượng máu tới thận và tuỵ. Lưu lượng máu cơ xương tới các
cơ không hoạt động cũng sẽ giảm khi đó. Vì vậy, lưu lượng máu não duy trì sát
gần với giá trị kiểm soát của nó.
TMP12 244
85. Điều nào sau đây làm tăng nguy cơ của các tình trạng (biến cố) bất lợi cho tim?
A) Giảm nồng độ LDL trong máu
B) Giảm nồng độ HDL trong máu
C) Giới nữ
D) Tăng (Hạ) huyết áp vừa
E) Giảm triglyceride máu
 B) Có nhiều yếu tố làm giảm nguy cơ của các tình trạng (biến cố) bất lợi cho
tim, bao gồm giảm nồng độ LDL trong máu, giới nữ, tăng (hạ) huyết áp mức độ
vừa và giảm triglyceride máu. Giảm nồng độ HDL máu làm tăng các nguy cơ tim
mạch, vì HDl là một cholesterol có tác dụng bảo vệ.
TMP12 248-249
86. Yếu tố gây co mạch (Chất có tác dụng vận mạch) nào sau đây là yếu tố kiểm
soát lưu lượng mạch vành quan trọng nhất?
A) Adenosine
B) Bradykinin
C) Prostaglandin
D) CO2
E) K+
 A) Bradykinin, prostaglandins, CO2 và K+ là các chất giãn mạch cho hệ mạch
vành. Tuy nhiên, yếu tố kiểm soát chính của lưu lượng mạch vành là adenosine.
Adenisin được hình thành từ adenosine monophosphate giáng hoá 1ATP. Sau đó,
phần nhỏ adenosine monophoaphat sẽ tiếp tục bị giáng hoá để giải phóng
adenosine vào dịch mô của cơ tim, và adenosine này gây giãn các động mạch vành.
TMP12 247
87. Khi gắng sức vừa, một người đàn ống 70 tuổi trải qua chứng nhồi máu cơ tim
và chết do rung thất. Ở bệnh nhân này, yếu tố nào có khả năng làm tăng khuynh
hướng rung của tim sau khi nhồi máu cơ tim?
A) Tăng kích thích hệ phó giao cảm của tim
B) Tăng (giảm) đường kính tâm thất
C) Nồng độ K+ trong dịch ngoại bào tim thấp
D) Điện thế màng tâm thất âm hơn
E) Sự hiện hành của vết thương tại nơi bị tổn thương (“Dòng thương tổn” xuất
phát từ vùng bị tổn thương – cái này là cơ sở để giải thích các biến đổi trong
nhồi máu cơ tim cũng như giúp xác định vị trí nhồi máu dựa trên điện tâm
đồ)
 E) Sự thiếu cung cấp máu cấp tính đến các cơ tim gây ra sự mất K+ từ các tế
bào cơ tim. Điều này làm tăng cục bộ nồng độ K+ ngoại bào. Theo đó, nó làm tăng
tính dễ bị kích thích của tế bào cơ tim và do đó làm chúng dễ bị rung hơn. Vì vậy,
sự giảm nồng độ K+ trong dịch ngoại bào tim không dẫn đến sự rung. Hoạt động
mạnh hệ giao cảm và không có phản xạ phó giao cảm cũng làm tăng tính dễ kích
thích của cơ tim và đưa đến sự rung. Điện thế màng càng âm bảo vệ tim khỏi sự
rung, và một vết thương hiện hành làm cho điện tịch di chuyển từ nơi bị thiếu máu
của tim đến nơi bình thường và có thể loại bỏ sự rung (“Dòng thương tổn” tạo ra
một dòng điện đi từ vùng tim nhồi máu tới vùng bình thường và có khả năng loại
trừ hiện tượng rung)
TMP12 250-251
88. Khẳng định nào sau đây về lưu lượng mạch vành là chính xác nhất?
A) Lưu lượng mạch vành khi nghỉ ngơi bình thường là 500 ml/phút
B) Phần lớn dòng chảy diễn ra trong thì tâm thu
C) Trong thì tâm thu, phần trăm giảm trong lưu lượng dưới màng trong tim lớn
hơn phần trăm giảm trong lưu lượng ngoại tâm mạc (giá trị phần trăm giảm
của lưu lượng vùng dưới nội tâm mạc lớn hơn giá trị phần trăm giảm của lưu
lượng vùng thượng tâm mạc)
D) Sự giải phóng adenosine thường làm giảm lưu lượng mạch vành
 C) Lưu lượng mạch vành khi nghỉ ngơi bình thường là 225 ml/phút. Truyền
adenosine hay giải giải phóng adenosine cục bộ thường làm tăng lưu lượng mạch
vành. Sự co thắt của các tế bào cơ tim xung quanh hệ mạch, đặc biệt là các mạch
máu dưới màng trong tim (dưới nội tâm mạc), gây ra sự giảm lưu lượng máu. Vì
thế, trong kì tâm thu của chu kỳ tim, lưu lượng dưới màng trong tim (vùng dưới
nội tâm mạc) giảm rõ rang trong khi sự giảm lưu lượng vùng ngoại tâm mạc chỉ rất
nhỏ.
TMP12 247
89. Tình trạng nào sau đây thường gây ra sự giãn các tiểu động mạch khi hoạt động
thể lực?
A) Giảm nồng đô K+ huyết tương
B) Tăng giải phóng histamine
C) Giảm nồng độ NO2 huyết tương
D) Tăng nồng độ adenosine huyết tương
E) Giảm nồng độ mol huyết tương (giảm áp lực thẩm thấu huyết tương)
 D) Có nhiều yếu tố làm giãn hệ động mạch trong khi hoạt động thể lực bao gồm
sự tăng nồng độ K+, nồng đọ NO2 huyết tương, nồng độ adenosine huyết tương và
nồng độ mol huyết tương (áp lực thẩm thấu huyết tương). Mặc dù histamine gây
giãn các động mạch, nhưng sự giải phóng histamine thường không xảy ra khi hoạt
động thể lực.
TMP12 243
90. Ở một người đã chạy gần được 10km thì giường mạch nào sau đây sẽ có sự co
mạch lớn nhất?
A) Não
B) Vành
C) Cơ hoạt động thể lực
D) Ruột
E) Da
 D) Giường mạch máu không bị co thắt bởi sự tăng kích thích hệ giao cảm trong
khi hoạt động thể lực là hệ mạch não và mạch vành. Trong các cơ hoạt động, đáp
sứng chuyển hoá giãn mạch lớn hơn hệ giao cảm sẽ dẫn đến sự giãn mạch. Trong
hệ mạch da, sự co mạch xảy ra chỉ khi bắt đầu hoạt động, khi cơ thể đã nóng lên,
các mạch máu ở da giẫn da. Hệ mạch ruột co đáng kể trong hoạt động thể lực kèo
dài.
TMP12 244
91. Mạch máu nào sau đây chịu trách nhiệm vận chuyển phần lớn lưu lượng máu
tĩnh mạch rời cơ tâm thất?
A) Tĩnh mạch trước tim (các tĩnh mạch tim trước)
B) Nút xoang vành (Xoang vành)
C) Tĩnh mạch phổi (các tĩnh mạch phế quản)
D) Tĩnh mạch đơn
E) Tĩnh mạch tim nhỏ
 B) Các tĩnh mạch trước tim (các tĩnh mạch tim trước) và các tĩnh mạch tim nhỏ
đều lấy (dẫn lưu) máu tĩnh mạch từ tim. Tuy nhiên, 75% lưu lượng mạch vành lấy
máu từ tim (dẫn lưu máu) bởi các nút xoang cảnh.
TMP12 246
92. Một người đàn ông 70 tuổi có cân nặng 100 kg và huyết áp là 160/90 mmHg đã
được chẩn đoán đau thắt ngực là do thiếu máu cơ tim. Điều trị nào sau đây có lợi
cho bệnh nhân này?
A) Tăng lượng Ca trong chế độ ăn
B) Bài tập co cơ đẳng tích
C) Cho kích kích receptor beta-1
D) Truyền angiotensin II
E) Nitroglycerin
 E) Nhiều thuốc đã được chứng minh là có hiệu quả với bệnh nhân có thiếu máu
cơ tim. Chặn receptor beta (không phải kích thích) ức chế tác dụng của hệ giao
cảm lên tim và rất có hiệu quả. Chất ức chế men chuyển angiotensin ngăn chặn sự
sản xuất angiotensin II và vì vậy giảm ảnh hưởng lên tim. Nitroglycerin gây giải
phóng nitric oxid dẫn đến giãn mạch vành. Các bài tập co cơ đẳng tích làm tăng áp
suất máu đáng kể và có thể gây hại, và tăng lượng Ca trong chế độ ăn không gây
ảnh hưởng đáng kể.
TMP12 252
93. Tình trạng nào sau đây thường xảy ra trong khi tập luyện?
A) Giãn hệ động mạch ở cơ không hoạt động
B) Giảm xung động hệ giao cảm
C) Co hệ tĩnh mạch
D) Giảm tiết epinephrine bởi tuyến thượng thận
E) Giảm tiết norepinephrine bởi tuyến thượng thận
 C) Trong khi tập luyện, hệ giao cảm tăng hoạt động đáng kể gây ra sự co thắt hệ
động mạch nhỏ trong nhiều nơi của cơ thể bao gồm cả các cơ không hoạt động thể
lực. Tăng hoạt động hệ giao cảm cũng gây ra co thắt hệ tĩnh mạch trong toàn cơ
thể. Trong khi luyện tập, cũng có sự tăng giải phóng norepinephrine và epinephrine
bởi các tuyến thượng thận.
TMP12 244-245
94. Điều nào sau đây là nguyên nhân thường gặp nhất của giảm lưu lượng mạch
vành ở bệnh nhân có thiếu máu cơ tim?
A) Tăng tiết adenosine
B) Xơ vữa động mạch
C) Co cứng mạch vành
D) Tăng hoạt động hệ giao cảm ở mạch vành
E) Bít xoang vành
 B) Nhiều yếu tố đóng góp vào việc làm giảm lưu lượng mạch vành ở bệnh nhân
có thiếu máu cơ tim. Một số bệnh nhân sẽ có co thắt mạch vành từ đó làm giảm lưu
lượng mạch vành. Tuy nhiên, nguyên nhân chủ yếu làm giảm lưu lượng mạch vành
là xơ vữa động mạch làm hẹp lòng mạch vành.
TMP12 248
95. Cách nào sau đây là cách điều trị có thể chấp nhận được cho bệnh nhân với
nhồi máu cơ tim cấp?
A) Luyện tập hàng ngàny
B) Kích thích receptor beta
C) Ngừng sử dụng nitroglycerin
D) Ngừng sử dụng aspirin
E) Tạo hình mạch vành
 E) Có nhiều cách điều trị có thể chấp nhận được cho bệnh nhân với thiếu máu
cơ tim. Nhiều bệnh nhân uống 1 liều aspirin hàng ngày để ngăn chăn xơ vữa mạch
vành. Tạo hình mạch vành với đặt stent hoặc phẫu thuật bắc cầu mạch vành đều có
hiệu quả làm tăng lưu lượng mạch vành. Giảm huyết áp, ức chế enzyme chuyển
angiotensin hay chặn receptor beta đều có hiệu quả điều trị. Tuy nhiên, kích thích
receptor beta hay tập luyên đều có hại với bệnh nhân thiếu máu cơ tim.
TMP12 252-253
96. Điều nào sau đây được khuyến cáo cho bệnh nhân mắc thiếu máu cơ tim?
A) Sử dụng kích thích receptor anpha
B) Ngừng sử dụng thuốc tăng huyết áp
C) Giảm lượng cân nặng dư thừa
D) Truyền angiotensin II
E) Bài tập co cơ đẳng tích

97. Điều nào sau đây là một trong những nguyên nhân hàng đầu gây tử vong sau
nhồi máu cơ tim?
A) Tăng cung lượng tim
B) Giảm thể tích khoảng kẽ phổi
C) Sự rung của tim
D) Tăng co thắt cơ tim
98. Khẳng định nào sau đây về kết quả của kích thích hệ giao cảm là chính xác
nhất?
A) Tăng lưu lượng ngoại tâm mạc
B) Giảm sức cản hệ tĩnh mạch
C) Giảm sức cản hệ động mach
D) Giảm nhịp tim
E) Co thắt hệ tĩnh mạch dự trữ
99. Điều nào sau đây thường liên quan đến giai đoạn mạn tính của suy tim còn bù?
Cho rằng bệnh nhân đang ở trạng thái nghỉ ngơi.
A) Khó thở
B) Giảm áp suất tâm nhĩ phải
C) Giảm nhịp tim
D) Ra mồ hôi
E) Tăng áp suất trung bình làm đầy hệ thống
100. Điều nào sau đây thường xảy ra trong suy tim phải?
A) Tăng áp suất động mạch phổi
B) Tăng áp suất tâm nhĩ trái
C) Tăng áp suất tâm nhĩ phải
D) Phù phổi
E) Tăng áp suất trung bình làm đầy hệ thống
101. Điều nào sau đây thường gây ra sự giữ lại cả muối và nước ở thân trong suy
tim còn bù?
A) Giảm sự hình thành angiotensin II
B) Giảm sự hình thành aldosterone
C) Giãn tiểu động mạch đến do giao cảm
D) Tăng tốc độ lọc cầu thận
E) Tăng sự hình thành ADH
102. Điều nào sau đây sẽ có lợi cho bệnh nhân có phù phổi cấp?
A) Truyền thuốc gây co mạch
B) Truyền dung dịch cân bằng điện giải
C) Cho furosemide
D) Cho thuốc co thắt phế quản
E) Truyền máu toàn phần
103. Một người đàn ông có cơn đau tim 2 ngày trước, và huyết áp của ông ấy tiếp
tục giảm xuống. Ông ấy đang trong tình trạng sốc tim. Điều trị nào sau đây sẽ có
lợi nhất?
A) Garo tứ chi
B) Cho thuốc ức chế hệ giao cảm
C) Cho furosemide
D) Cho chất làm tăng thể tich máu
E) Tăng lượng Na+ trong chế độ ăn uống
104. Nếu một bênh nhân nam 21 tuổi có dữ trự tim là 300% và cung lượng tim lớn
nhất là 16 l/phút, cung lượng tim lúc nghỉ của bệnh nhân là bao nhiêu?
A) 3 L/phút
B) 4 L/phút
C) 5.33 L/phút
D) 6 L/phút
E) 8 L/phút
105. Điều nào sau đây xảy ra trong suy tim và làm tăng bài tiết muối ở thận?
A) Tăng giải phóng aldosterone
B) Tăng giải phóng yếu tố lợi niệu tâm nhĩ
C) Giảm tốc độ lọc cầu thận
D) Tăng giải phóng angiotensin II
E) Giảm huyết áp động mạch trung bình
106. Điều trị nào sau đây thích hợp cho bệnh nhân trong tình trạng sốc tim?
A) Garo cầm máu cả 4 chi
B) Lấy máu bệnh nhân vừa phải
C) Sử dụng furosemide
D) Truyền thuốc gây co mạch
107. Tình trạng nào sau đây thường đi kèm với suy tim phải cấp tính?
A) Tăng áp lực tâm nhĩ phải
B) Tăng áp lực tâm nhĩ trái
C) Tăng lượng nước tiểu
D) Tăng cung lượng tim
E) Tăng huyết áp động mạch
108. Điều nào sau đây thường xảy ra trong suy tim mất bù?
A) Phì đại cơ tim
B) Tăng norepinephrine trong thần kinh giao cảm tim
C) Tăng lượng Ca trong màng lưới cơ tương của các tế bào cơ tim
D) Giảm áp lực nhĩ phải
E) Tăng huyết áp động mạch
109. Điều nào sau đây thường có lợi cho những bệnh nhân có phù phổi cấp?
A) Garo cầm máu ở cả 4 chi
B) Truyền plasma
C) Truyền dextran
D) Truyền norepinephrine
E) Truyền angiotensin II
110. Điều nào sau đây thường đi kèm với suy tim còn bù?
A) Tăng cung lượng tim
B) Tăng thể tích máu
C) Giảm áp lực trung bình làm đầy hệ thống
D) Áp lực nhĩ phải bình thường
111. Điều nào sau đây thường có liên quan đến việc tăng áp lực trung bình làm đầy
hệ thống?
A) Giảm thể tích máu
B) Suy tim sung huyết
C) Ức chế hệ giao cảm
D) Giãn hệ tĩnh mạch
112. Điều nào sau đây thường xảy ra trong giai đoạn đầu của suy tim còn bù?
A) Tăng áp lực nhĩ phải
B) Nhịp tim bình thường
C) Giảm bài tiết angiotensin II
D) Giảm bài tiết aldosterone
E) Tăng lượng muối và nước trong nước tiểu thải ra
113. Điều nào sau đây thường xảy ra trong suy tim mất bù?
A) Tăng huyết áp
B) Tăng áp lực trung bình làm đầy mạch phổi
C) Giảm áp lực hệ mao mạch phổi
D) Tăng cung lượng tim
E) Tăng norepinephrine ở cuối các dây thần kinh giao cảm tim
114. Điều nào sau đây thường xảy ra trong suy tim mất bù?
A) Tăng mất muối và nước ở thận
B) Giảm áp lực trung bình làm đầy hệ thống
C) Tăng norepinephrine trong thần kinh giao cảm tim
D) Khó thở khi nghỉ ngơi
E) Giảm cân
115. Một bênh nhân nam, 80 tuổi, tại phòng cấp cứu được phát hiện tiếng thổi ở
tim. Hình ảnh XQ ngực cho thấy bóng tim to nhưng không có phù dịch trong phổi.
Trục QRS trong kết quả điện tâm đồ là 170 độ. Áp lực phổi bít bình thường. Chẩn
đoán bệnh nhân này là gì?
A) Hẹp van hai lá
B) Hẹp van động mạch chủ
C) Hẹp van động mạch phổi
D) Hẹp van ba lá
E) Hở van hai
116. Điều nào sau đây có liên quan với tiếng tim thứ hai T2?
A) Máu bị đẩy nhanh vào tâm thất bởi sự co thất của tâm nhĩ
B) Đóng các van nhĩ thất
C) Đóng van động mạch phổi
D) Mở các van nhĩ thất
E) Máu bị đẩy nhanh vào tâm thất trong giai đoạn sớm tới giữa kỳ tâm trương
117. Một phụ nữ 40 tuổi được phát hiện tiếng thổi ở tim. Tiếng thôi có âm sắc cao
được nghe thấy rõ nhất ở tâm thất trái. Hình ảnh XQ ngực cho thấy bóng tim to.
Huyết áp động mạch tại động mạch chủ là 140/40 mmHg. Chẩn đoán ở bệnh nhân
này là gì?
A)Hẹp van động mạch chủ
B) Hở van động mạch chủ
C) Hẹp van động mạch phổi
D)Hẹp van hai lá
E) Hở van ba lá
118. Trong rối loạn nào sau đây sẽ thường xuất hiện tâm thất trái phì đại?
A) Hở van động mạch phổi
B) Hở van ba lá
C) Hẹp van hai lá
D) Hẹp van ba lá
E) Hẹp van động mạch chủ
 E) Phì đại tâm thất trái xảy ra khi tâm thất trái hoặc phải gây ra một áp lực lớn
hoặc khi nó phải bơm thêm 1 lượng máu trong mỗi nhát bóp. Trong hở van động
mạch chủ, máu sẽ bị trôi ngược trở lại buồng tâm thất trong kỳ tâm trương. Lượng
máu thêm vào này phải được đưa ra trong nhịp tim tiếp theo. Trong hở van hai lá,
một lượng máu được bơm vào trong động mạch chủ, cùng lúc đó một lượng máu
tương tự trôi ngược trở lại tâm nhĩ trái. Vì vậy, tâm thất trái phải bơm thêm một
lượng máu trong mỗi nhịp tim. Trong hẹp van động mạch chủ tâm thất trái phải co
bóp rất mạch để tạo ra một sức căng lớn để tăng áp lực động mạch chủ đến một giá
trị cao cần thiết để đưa máu vào động mạch chủ. Trong hẹp van hai lá tâm thất
bình thường, bởi vì tâm nhĩ đã sản xuất thêm một áp lực để đưa máu qua van hai lá
bị hẹp.
TMP12 267-268
119. Tiếng thổi ở tim nào sau đây được nghe thấy trong kỳ tâm thu?
A) Hở van động mạch chủ
B) Hở van động mạch phổi
C) Hẹp van ba lá
D) Hẹp van hai lá
E) Còn ống động mạch
 E) Có một số tiếng thổi tâm trương mạnh có thể nghe thấy dễ dàng bằng ống
nghe. Trong kỳ tâm trương, hở van động mạch chủ và động mạch phổi thông qua
van bị hở sẽ gây ra tiếng thôi tại thời điểm đó. Hẹp van hai lá và ba lá là tiếng thổi
tâm trương bởi dòng máu chảy qua van bị hẹp trong kỳ tâm trương. Tiếng thổi
trong còn ống động mạch được nghe thấy ở cả thì tâm thu và tâm trương.
TMP12 265-266
120. Sự tăng áp lực tâm nhĩ trái thường sẽ xảy ra trong tiếng thổi ở tim nào sau
đây?
A) Hẹp van ba lá
B) Hở van động mạch phổi
C) Hở van hai lá
D) Hở van ba lá
 C) Hở van hai lá gây ra sự tăng mạnh trong áp lực tâm nhĩ trái. Tuy nhiên, hẹp
và hở van ba lá và hở van động mạch phổi chỉ gây tăng áp lực tâm nhĩ phải và
không có ảnh hưởng gì lên áp lực tâm nhĩ trái.
TMP12 267-268
121. Một bệnh nhân nữ, 50 tuổi tại phòng cấp cứu được chẩn đoán có tiếng thổi ở
tim. Tiếng thổi có âm sắc thấp được nghe thấy rõ nhất ở khoang liên sườn thứ 2
phía bên phải xương ức. Hình ảnh XQ ngực cho thấy bóng tim to. Trục QRS trung
bình trên điện tâm đồ là -45 độ. Chẩn đoán ở bệnh nhân này là?
A) Hẹp van hai lá
B) Hẹp van động mạch chủ
C) Hẹp van động mạch phổi
D) Hẹp van ba lá
E) Hở van ba lá

122. Một bệnh nhân nam, 29 tuổi, được chẩn đoán có tiếng thổi ở tim. Trục QRS
trung bình trên điện tâm đồ là 165 độ. Nồng độ oxy máu động mạch bình thường.
Chẩn đoán nào sau đây là có thể nhất?
A) Hẹp van động mạch chủ
B) Hở van động mạch chủ
C) Hẹp van động mạch phổi
D) Hẹp van hai lá
E) Tứ chứng Fallot
123. Trong bệnh nào sau đây thường hay xảy ra tâm thất phải phì đại?
A) Tứ chứng Fallot
B) Hẹp van động mạch chủ vừa
C) Hở van động mạch chủ vừa
D) Hẹp van hai lá
E) Hẹp van ba lá
124. Tiếng thổi ở tim nào sau đây chỉ nghe thấy được trong thì tâm thu?
A) Còn ống động mạch
B) Hẹp van hail á
C) Hẹp van ba lá
D) Thông liên thất
E) Hở van động mạch chủ
125. Bệnh nào sau đây có thể có nồng độ oxy máu động mạch thấp?
A) Tứ chứng Fallot
B) Hẹp van động mạch phổi
C) Hở van ba lá
D) Còn ống động mạch
E) Hẹp van ba lá
126. Hiện tượng nào sau đây có liên quan đến tiếng tim đầu tiên?
A) Máu tống vào buồng tâm thất do sự co thắt của động mạch
B) Đóng các van nhĩ-thất
C) Đóng van động mạch phổi
D) Mở các van nhĩ-thất
E) Máu tống vào buồng tâm thất vào giai đoạn đầu đến giữa của kỳ tâm trương
127. Điện tâm đồ được thực hiện trên một bệnh nhân nữ 50 tuổi. Kết quả cho thấy
có sự dày lên của tâm thất phải. Các dữ liệu khác cho thấy bệnh nhân có sự giảm
mạnh của nồng độ oxy máu động mạch và sự cân bằng áp lực tâm thu ở cả hai
buồng tâm thất. Tình trạng hiện tại của bệnh nhân là?
A) Thông liên thất
B) Tứ chứng Fallot
C) Hẹp van động mạch phổi
D) Hở van động mạch phổi
E) Còn ống động mạch
128. Tiếng thổi nào sau đây chỉ nghe thấy trong kỳ tâm trương?
A) Còn ống động mạch
B) Hở van hai lá
C) Hẹp van ba lá
D) Thông liên thất
E) Hẹp van động mạch chủ
129. Hiện tượng nào sau đây có liên quan đến tiếng tim thứ ba?
A) Máu được tống vào buồng tâm thất do sự co thắt của động mach
B) Đóng các van nhĩ-thất
C) Đóng van động mạch phổi
D) Mở các van nhĩ-thất
E) Máu được tong vào buồng tâm thất trong giai đoạn đầu đến giữa kỳ tâm
trương
130. Tình trạng nào sau đây thường xảy ra trong sốc mất máu tiến triển?
A) Tăng tính thấm mao mạch
B) Sự hồi phục stress của các tĩnh mạch
C) Nhiễm kiềm mô
D) Tăng lượng nước tiểu
E) Tăng áp suất trung bình làm đầy hệ thống
131. Trong tình trạng nào sau đây việc sử dụng thuốc kích thích thần kinh giao
cảm sẽ là liệu pháp được lựa chọn để chống sốc?
A) Chấn thương cột sống
B) Sốc do nôn quá nhiều
C) Sốc mất máu
D) Sốc do lợi tiểu quá mức
 A) Thuốc kích thích giao cảm được kê để chống lại tình trạng hạ huyết áp trong
nhiều trường hợp khác nhau, bao gồm chấn thương cột sống trong đó đường ra của
thần kinh giao cảm bị gián đoạn. Thuốc kích thích giao cảm còn được dung trong
gây mê sâu làm giảm xung động giao cảm, và trong sốc do quá mẫn là kết quả của
giải phóng histamine và sự giãn mạch tương ứng. Thuốc kích thích giao cảm, ví dụ
norepinephrine, tăng áp lực máu bởi gây ra sự co mạch. Sốc gây ra bởi nôn quá
mức, mất máu hay sử dụng thuốc lợi tiểu quá mức dẫn đến giảm thể tích giảm dẫn
tới giảm thể tích máu và giảm áp lực trung bình làm đầy hệ thống. Sử dụng dung
dịch cân bằng điện giải sẽ giải quyết tốt nhất trường hợp này.
TMP12 281
132. Một người phụ nữ 30 tuổi đến phòng cấp cứu do nôn dữ dội. Cô ấy có dạ tái
nhợt, tim đập nhanh, huyết áp động mạch là 80/50 và có vấn đề về đi lại. Liệu pháp
nào bạn sẽ tiến hành để phòng chống sốc?
A) Truyền hồng cầu khối
B) Sử dụng kháng histamine
C) Truyền dung dịch cân bằng điện giải
D) Truyền thuốc kích thích thần kinh giao cảm
E) Sử dụng glucocorticoid
 C) Nôn có thể gây ra sự mất lượng lớn điện giải, dấn tới sự giảm thật sự của thể
tích huyết tương. Vì vậy, thể tích máu giảm, và huyết áp động mạch chạm đến mức
vô cùng thấp. Liệu pháp thích hợp là thay thế phần thể tích đã bị mất đi bởi việc
truyển dung dịch cân bằng điện giải.
TMP12 280-281
133. Một người đàn ông 65 tuổi đến phòng cấp cứu tại địa phương một vài phút
sau khi nhận mũi tiêm chủng influenza. Ông ấy có vẻ ngoài xanh xao, tim đập
nhanh, huyết áp động mạch là 80/50 và có vấn đề về đi lại. Liệu pháp nào bạn sẽ
tiến hành để phòng chống sốc?
A) Truyền máu
B) Sử dụng thuốc kháng histamine
C) Truyền dung dịch cân bằng điện giải ví dụ như saline
D) Truyền thuốc kích thích thần kinh giao cảm
E) Truyền chất hoạt hoá plasminogen mô
 D) Bệnh nhân tiêm chủng influenza và nhanh chóng rơi vào tình trạng sốc, dẫn
đến có thể tin rằng anh ta đang bị sốc do quá mẫn (sốc phản vệ). Đây là tình trạng
giãn mạch quá mức do sự giải phóng histamine. Kháng histamine có thể có ích một
phần nhưng chúng có tác dụng rất chậm và bệnh nhân có thể chết trong lúc đó. Vì
vậy, một chất có tác dụng nhanh chóng phải được sử dụng, ví dụ như thuốc kích
thích thần kinh giao cảm.
TMP12 280-281
134. Tình trạng nào sau đây thường xảy ra trong sốc mất máu còn bù? Cho rằng áp
lực tâm thu là 48 mmHg.
A) Giảm nhịp tim
B) Tĩnh mạch nới lỏng
C) Giảm giải phóng ADH
D) Giảm hấp thụ dịch ruột qua hệ mao mạch
E) Đáp ứng với thiếu máu của hệ thần kinh trung ương
 E) Trong sốc mất máu còn bù có một số yếu tố ngăn chặn sự tiến triển của sốc
bao gồm tăng nhịp tim. Đồng thời xảy ra là sự đảo ngược sự giãn của hệ mạch, đặc
biệt là các tĩnh mạch, co thắt xung quanh thể tích máu còn lại. Tăng giải phóng
ADH cũng đồng thời xảy ra, gây ra sự giữ nước ở thận nhưng đồng thời gay co thắt
mạch ở hệ động mạch nhỏ. Đáp ứng với thiếu máu cục bộ của hệ thần kinh trung
ương cũng xảy ra nếu huyết áp rơi xuống một giá trị rất thấp. Điều này gây ra sự
tăng xung động giao cảm. Tăng hấp thu dịch gian bào thông qua mao mạch cũng
xảy ra làm tăng thể tích trong hệ mạch.
TMP12 275
135. Nếu một bệnh nhân đang trải qua gây mê cột sống có sự giảm mạnh huyết áp
động mạch và đi vào trạng thái sốc, liệu pháp nào sau đây sẽ được lựa chọn?
A) Truyền huết tương
B) Truyền máu
C) Truyền saline
D) Truyền glucocorticoid
E) Truyền thuốc kích thích giao cảm
 E) Gây mê cột sống đặc biệt là gây mê kéo dài cả cột sống có thể chặn đường
dẫn truyền của thần kinh giao cảm từ cột sống. Đây có thể là một nguyên nhân của
sốc do nguyên nhân thần kinh. Liệu pháp được lựa chọn là liệu pháp thay thế được
trương lực hệ giao cảm đã bị mất trong cơ thể. Cách tốt nhất để tăng trương lực
giao cảm là truyền thuốc kích thích giao cảm.
TMP12 281
136. Một người đàn ông 25 tuổi đến khoa cấo cứu sau một tai nạn moto. Quần áo
của anh ta dính đầy máu và huyết áo động mạch giảm xuống vòn 70/40. Nhịp tim
là 120 và nhịp thở là 30 lần/phút. Liệu pháp nào sau đây bác sĩ nên tiến hành?
A) Truyền máu
B) Truyền huyết tương
C) Truyền dung dịch cân bằng điện giải
D) Truyền thuốc kích thích giao cảm
E) Sử dụng glucocorticoid
 A) Bệnh nhân này đã mất một lượng máu lớn bởi tai nạn moto/ Liệu pháp có lợi
nhất là thay thế cái đã bị mất do tai nạn. Đó có thể là máu toàn phần và tốt hơn là
truyền huyết tương, trong đó bạn có cả hồng cầu, hồng cầu có khả năng vận
chuyển oxy tốt hơn thành phần huyết tương của máu. Các dây thần kinh giao cảm
hoạt động rất nhanh chóng trong tình trạng này và sự truyền chất kích thích giao
cảm sẽ chỉ có lợi rất ít.
TMP12 280-281
137. Trong loại sốc nào sau đây cung lượng tim thường tăng?
A) Sốc mất máu
B) Sốc quá mẫn
C) Sốc nhiễm khuẩn
D) Sốc do nguyên nhân thần kinh
 C) Trong sốc mất máu, sốc do mẫn cảm và sốc thần kinh, sự trở về tim của máu
tĩnh mạch giảm một cách đáng kể. Tuy nhiên, trong sốc nhiễm khuẩn, cung lượng
tim tăng ở rất nhiều bệnh nhân do sự giãn mạch trong các mô bị ảnh hưởng và bởi
tốc độ chuyển hoá cao dẫn tới sự giãn mạch trong các phần khác của cơ thế.
TMP12 280
138. Một người đàn ông 20 tuổi vào phòng cấp cứu tại địa phương vì mất máu do
trúng đạn. Anh ta có da tái nhợt, tim đập nhanh, và huyết áp động mạch là 80/50,
anh ta có vấn đề về đi lại. Không may ngân hàng máu đã hết máu. Liệu pháp nào
sau đây bác sĩ nên tiến hành để chống sốc cho bệnh nhân?
A) Sử dụng glucocorticoid
B) Sử dụng kháng histamine
C) Truyền dung dịch cân bằng điện giải
D) Truyền thuốc kích thích giao cảm
E) Truyền huyết tương
 E) Bệnh nhân này bị mất máu, vè liệu pháp tối ưu là bù được lượng máu đã mất.
Tuy nhiên không may, không còn máu có sẵn; vì vậy, chúng ta phải chọn liệu pháp
tốt nhất tiếp theo. Tăng thể tích máu là liệu pháp tốt nhất. Truyền huyết tương là
liệu pháp tốt nhất, vì áp lực thẩm thấu keo cao của nó sẽ giúp dịch được truyền ở
lại trong vòng tuần hoàn lâu hơn là dung dịch cân bằng điện giải.
TMP12 280-281
139. Một cố bé 10 tuổi vào bệnh viên do bị tắc ruột và huyết áp động mạch giảm
còn 70/40. Nhịp tim là 120 và nhịp thở là 30 lần/phút. Liệu pháp nào sau đây bác sĩ
nên tiến hành?
A) Truyền máu
B) Truyền huyết tương
C) Truyền dung dịch cân bằng điện giải
D) Truyền thuốc kích thích giao cảm
E) Sử dụng glucocorticoid
 B) Tắc ruột thường gây ra sự giảm mạnh trong thể tích huyết tương. Tắc gây ra
sự căng phồng của đoạn ruột và chặn mổ phần dòng chảy của máu tĩnh mạch trong
ruột. Điều này dễ đến sự tăng áp lực hệ mao mạch ruột, gây ra việc dịch sẽ chảy từ
mao mạch vào trong thành ruột và vào trong lòng ruột. Dịch tràn có chứ lượng
protein cao rất giống với huyết tương, làm giảm protein toàn phần huyết tương và
giảm thế tích huyết tương. Vì vậy, liệu pháp được lựa chọn sẽ là thay thế dịch đã bị
mất bởi truyền huyết tương.
TMP12 280-281
140. Điều nào sau đây thường xảy ra trong sốc tiến triển?
A) Sự chết do hoại tử chắp vá (từng phần) của gan
B) Giảm khuynh hướng đông của máu
C) Tăng chuyển hoá glucose
D) Giảm giải phóng hydrolase bởi lysosomes (?)
E) Giảm tính thấm mao mạch
 A) Trong sốc tiến triển, do dòng chảy máu giảm nên pH trong các mô của cơ
thể cũng giảm. Nhiều mạch máu bị chặn do sự kết dính mạch máu cục bộ, được gọi
là máu đặc quánh. Sự chết hoại tử chắp vá xảy ra ở gan. Hoạt động ty thể giảm và
tính thấm mao mạch tăng, CŨng sẽ có sự tăng giải phóng hydrolase bởi các tiêu
thể và sự giảm chuyển hoá glucose trong tế bào.
TMP12 280-281
141. Giải phóng chất nào sau đây gây ra sự giãn mạch và tăng tính thấm mao mạch
trong sốc do quá mẫn?
A) Histamine
B) Bradykinin
C) Nitric oxid
D) Yếu tố lợi niệu tâm nhĩ
E) Adenosine
 A) Quá mẫn là một tình trạng dị ứng gây ra do phản ứng kháng nguyên-kháng
thể diễn ra sau khi có khi phơi nhiễm của một cá thể với một chất kháng nguyên.
Tế bào basophil và tế bào mast trong các mô quanh mao mạch giải phóng
histamine hoặc chất giống với histamine. Histamine gây ra sự giãn tĩnh mạch, giãn
các tiểu động mạch và làm tăng đáng kể tính thấm mao mạch với sự mất nhanh
chóng của dịch và protein vào khoảng trống của mô. Điều này làm giảm máu tĩnh
mạch về tim và thường dẫn tới sốc do quá mẫn.
TMP12 280-281
142. Điều nào sau đây là một đặc trưng của sốc mất máu tiến triển?
A) Cục máu đông trong các mạch máu nhỏ
B) Tăng hoạt động ty thể ở gan
C) Giảm hoạt động tiêu thể
D) Giảm vận chuyển chủ động Na+
E) Nhiễm kiềm mô
 A) Trong sốc mát máu tiến triển, các cục máu đông bắt đầu xảy ra trong nhiều
mạch máu. Giảm lưu lượng chảy của máu trong toàn cơ thế gây ra nhiễm toan bởi
sự giảm việc di chuyển của CO2. Sự vận chuyển chủ động Na+ và K+ cũng giảm.
Trong gian, hoạt động ty thể giảm và hoạt động tiêu thể tăng rộng khắp các khu
vực.
TMP12 276
143. Một người đàn ông 70 tuổi vào khoa cấp cứu tại bệnh viên do tiêu chảy nặng.
Ông ấy có da xanh xao, tim đập nhanh, huyết áp động mạch là 80/50 và có vấn đề
về đi lại. Liệu phsp nào sau đây bác sĩ nên tiến hành đề chống sốc cho bệnh nhân?
A) Truyền máu
B) Sử dụng kháng histamine
C) Truyền dung dịch cân bằng điện giải
D) Truyền thuốc kích thích giao cảm
E) Sử dungh glucocorticoid
 C) Với tiêu chảy năngj, có một lượng lớn Na+ và nước bị mất khỏi cơ thể dẫn
đến sự mất nước và đôi khi là sốc. Liệu pháp tốt nhất là thay thế lượng điện giải đã
bị mất qua tiêu chảy. Vì vậy, truyền dung dịch cân bằng điện giải là liệu pháp tốt
nhất được lựa chọn.
TMP12 180-181
144. Một phụ nữ 60 tuổi bị bỏng nặng và có huyết áp động mạch là 70/40 với nhịp
tim là 130 nhịp/phút. Liệu pháp nào sau đây bác sĩ nên tiến hành với vai trò là liệu
pháp ban đầu?
A) Truyền máu
B) Truyền huyết tương
C) Truyền dung dịch cân bằng điện giải
D) Truyền thuốc kích thích giao cảm
E) Sử dụng glucocorticoid
 B) Ở bệnh nhân với bỏng nặng, có một lượng lớn chất giống với huyết tương bị
mất qua các mô bị bỏng. Vì vậy, nồng độ protein huyết tương giảm trầm trọng và
liệu pháp được lựa chọn là truyền huyết tương.
TMP12 279-280

Unit 5: Thận, dịch lọc


Câu hỏi 1 và 2:
Sử dụng các kết quả xét nghiệm sau để trả lời câu hỏi 1 và 2:
- Lượng nước tiểu trong 1 phút = 1ml/phút
- Nồng độ inulin trong nước tiểu = 100mg/ml
- Nồng độ inulin trong huyết tương = 2mg/ml
- Nồng độ ure trong nước tiểu= 50 mg/,l
- Nồng độ ure trong huyết tương = 2,5mg.ml

1. Lưu lượng lọc cầu thận bằng bao nhiêu (GFR)?


A) 25ml/phút
B) 50 ml/phút
C) 100 ml/phút
D) 125ml/phút
E) Không phải các đáp án trên
2. Tốc độ tái hấp thu ure bằng bao nhiêu?
A) 0 mg/phút
B) 25 mg/phút
C) 50 mg/phút
D) 75 mg/phút
E) 100 mg/phút
3. Dịch nào sau đây truyền vào tĩnh mạch gây ra tăng thể tích dịch ngoại bào, giảm
thể tích dịch nội bào và tăng tổng thể tích nước cơ thể sau khi áp lực thẩm thấu cân
bằng?
A) 1 L NaCl 0.9%
B) 1 L NaCl 0.45%
C) 1 L NaCl 3%
D) 1 L Dextrose 5%
E) 1 L nước cất
4. Bệnh nhân nam 65 tuổi bị nhồi máu cơ tim và bị ngừng tim khi bắt đầu được
chuyển tới phòng cấp cứu. Kết quả khí máu:
pH=7,12
PCO2 = 60mm Hg
Nồng độ HCO3 huyết tương = 19 mEq/L
Rối loạn thăng bằng kiềm toan ở bệnh nhân là gì?
A) Toan hô hấp còn bù qua thận
B) Toan chuyển hóa còn bù qua hô hấp
C) Toan hỗn hợp: Gồm cả toan chuyển hóa và toan hô hấp
D) Kiềm hỗn hợp: Gồm cả kiềm chuyển hóa và kiềm hô hấp
5.Với bệnh nhân ở câu hỏi 4, kết quả xét nghiệm thay đổi như thế nào?
A) Tăng tiết HCO3 - qua thận
B) Giảm titratable acid (H2PO4-) nước tiểu
C) Tăng pH nước tiểu
D) Tăng bài tiết NH4+
6. Ở thận bình thường, độ thẩm thấu ống lượn xa đoạn qua phức hợp cạnh cầu thận
như thế nào?
A) Thường cân bằng với áp suất thẩm thấu trong huyết tương
B) Thường có áp suất thẩm thấu thấp hơn trong huyết tương
C) Thường có áp suất thẩm thấu cao hơn trong huyết tương
D) Áp suất thẩm thấu cao hơn so với trong huyết tương trong trường hợp chống lợi
niệu.
Câu hỏi 7-9
Biểu đồ chỉ ra các tình trạng hydrat hóa bất thường. Trong mỗi biểu đồ, tình trạng
bình thường (màu vàng và màu tím) được vẽ chồng lên trên tình trạng bất thường
để thể hiện sự thay đổi về thể tích (chiều rộng hình chữ nhật) và áp suất thẩm thấu
(chiều cao hình chữ nhật) trong khoang dịch ngoại bào và dịch nội bào
7. Những biểu đồ nào thể hiện sự thay đổi (sau khi cân bằng áp suất thẩm thấu)
trong thể tích và áp suất thẩm thấu dịch nội bào và dịch ngoại bào sau khi truyền
tĩnh mạch dextrose 1%?
8. Những biểu đồ nào thể hiện sự thay đổi (sau khi cân bằng áp suất thẩm thấu)
trong thể tích và áp suất thẩm thấu dịch nội bào và dịch ngoại bào sau khi truyền
tĩnh mạch NaCl 3%?
9. Những biểu đồ nào thể hiện sự thay đổi (sau khi cân bằng áp suất thẩm thấu)
trong thể tích và áp suất thẩm thấu dịch nội bào và dịch ngoại bào ở bệnh nhân mắc
hội chứng rối loạn tiết hormone chống lợi niệu (tăng tiết quá mức hormone chống
lợi niệu)?
10. Sau khi ghép thận, bệnh nhân bị tăng huyết áp nặng (170/110 mm Hg). Chụp
động mạch thận cho thấy hình ảnh hẹp động mạch tại bên thận còn lại của bệnh
nhân, với lưu lượng lọc cầu thận giảm 25% so với bình thường. Sự thay đổi nào
dưới đây so với bình thường xảy ra với bệnh nhân này, với giả sử tình trạng bệnh
nhân ổn định?
A) Tăng mạnh nồng độ Na trong huyết tương
B) Giảm bài tiết Na qua nước tiểu 25% so với bình thường
C) Giảm bài tiết creatinin qua nước tiểu 25% so với bình thường
D) Tăng nồng độ creatinin huyết tương gấp 4 lần so với bình thường
E) Dòng máu lưu thông bình thường qua động mạch thận bị hẹp nhờ vào cơ chế tự
điều hòa
11. Tình trạng nào sau đây có xu hướng gây giảm tiết K tại ống góp?
A) Tăng nồng độ K huyết tương
B) Thuốc lợi tiểu làm giảm tái hấp thu Na tại ống lượn gần
C) Thuốc lợi tiểu kháng aldosterone (VD: spironolactone)
D) nhiễm kiềm cấp tính
E) Tăng Na vào cơ thể
12. Bệnh nhân có độ thanh thải creatinin 90 ml/min, thể tích nước tiểu 1 ml/min,
nồng độ K+ trong huyết tương 4 mEq/L, nồng độ K+ trong nước tiểu 60 mEq/L,
tốc độ bài tiết K+ khoảng bao nhiêu?
A) 0.06 mEq/phút
B) 0.30 mEq/phút
C) 0.36 mEq/phút
D) 3.6 mEq/phút
E) 60 mEq/phút
13. Những thay đổi nào dưới đây xảy ra ở bệnh nhân đái tháo nhạt do thiếu hụt bài
tiết hormone chống bài niệu (ADH)?
14. Bệnh nhân tăng huyết áp nặng (huyết áp 185/110 mmHg) đến khám. Hình ảnh
trên cộng hưởng từ cho thấy một khối u trong thận, và trên kết quả cận lâm sàng
cho thấy tăng hoạt tính renin trong huyết tương là 12 ng angiotensin 1/ml/h (bình
thường = 1). Chẩn đoán đưa ra là u tăng tiết renin. Trong tình trạng ổn định, thay
đổi nào trong số các thay đổi dưới đây bạn mong chờ sẽ thấy ở bệnh nhân này so
với bình thường?


15. Ở bệnh nhân ngộ độc qua đường tiêu hóa gây ra tình trạng suy giảm liên tục
khả năng tái hấp thu NaCl qua ống lượng gần, những thay đổi nào sẽ xảy ra sau 3
tuần? Giả sử không có thay đổi nào trong chế độ ăn hoặc đưa vào thêm các chất
điện giải qua đường tiêu hóa.


16. Bệnh nhân nữ 26 tuổi gần đây bắt đầu thay đổi chế độ ăn có lợi cho sức khỏe
hơn bằng cách ăn nhiều hoa quả và rau xanh. Do đó, lượng Kali đưa vào trong cơ
thể tăng từ 80 lên 160 mmol/ngày. Sau 2 tuần sau khi bệnh nhân này tăng lượng
Kali đưa vào cơ thể, những thay đổi nào sẽ xảy ra trên bệnh nhân, so với trước khi
tăng?


17. Trẻ nam 8 tuổi vào viện trong tình trạng bụng trướng nhiều. Bố mẹ trẻ cho biết
trẻ bị viêm họng nặng khoảng 1 tháng trước và bắt đầu xuất hiện trướng bụng từ
đó. Trẻ có phù và khi kiểm tra nước tiểu thì tháy rằng nước tiểu có chứa lượng lớn
protein. Chẩn đoán là Hội chứng thận hư sau Viêm cầu thận. Những thay đổi nào
sau đây xảy ra trên bệnh nhân, so sánh với bình thường?


18. Những thay đổi nào sẽ xảy ra sau khi sử dụng thuốc giãn mạch gây ra tình
trạng giảm 50% sức cản tiểu động mạch đến và không làm thay đổi huyết áp động
mạch?
A) Giảm lượng máu đến thận, giảm lưu lượng lọc cầu thận GFR, và giảm áp lực
thủy tĩnh mao mạch quanh ống thận
B) Giảm lượng máu đến thận, giảm lưu lượng lọc cầu thận GFR, và tăng áp lực
thủy tĩnh mao mạch quanh ống thận
C) Tăng lượng máu đến thận, tăng lưu lượng lọc cầu thận GFR, và tăng áp lực thủy
tĩnh mao mạch quanh ống thận
D) Tăng lượng máu đến thận, tăng lưu lượng lọc cầu thận GFR, và không thay đổi
áp lực thủy tĩnh mao mạch quanh ống thận
E) Tăng lượng máu đến thận, tăng lưu lượng lọc cầu thận GFR, và giảm áp lực
thủy tĩnh mao mạch quanh ống thận


19. Bệnh nhân nam 32 tuổi đến khám do đi tiểu liên tục nhiều lần. Bệnh nhân thừa
cân (nặng 126kg, cao 1,56m) và sau khi đo độ thanh thải creatinine 24h, ước tính
ra lưu lượng lọc cầu thận là 150 ml/phút. Glucose máu là 300 mg/dL. Giả sử rằng
khả năng vận chuyển glucose qua thận bình thường (cho thấy qua các chỉ số trên),
tốc độ bài tiết glucose qua nước tiểu ở bệnh nhân khoảng bao nhiêu?
A) 0 mg/phút
B) 100 mg/phút
C) 150 mg/phút
D) 225 mg/phút
E) 300 mg/phút
F) Các thông tin được đưa ra chưa đầy đủ để đánh giá tốc độ bài tiết glucose

20. Kết quả xét nghiệm máu động mạch trả về của một bệnh nhân như sau: pH máu
= 7.28, HCO3− huyết tương = 32 mEq/L, PCO2 huyết tương = 70 mm Hg. Rối
loạn thăng bằng kiềm toan ở bệnh nhân này là gì?
A) Toan hô hấp cấp không bù trừ qua thận
B) Toan hô hấp cấp còn bù một phần qua thận
C) Toan chuyển hóa cấp không bù trừ qua hô hấp
D) Toan chuyển hóa bù trừ một phần qua hô hấp

21. Thay đổi nào dưới đây làm tăng khả năng tái hấp thu dịch trong mao mạch
quanh ống thận?
A) Tăng huyết áp
B) Giảm hệ số lọc
C) Tăng sức cản của tiểu động mạch đi
D) Giảm angiotensin II
E) Tăng lượng máu tới thận

22. Nguyên nhân nào sau đây gây tăng kali máu mạnh nhất?
A) Tăng lượng Kali đưa vào từ 60 đến 180 mmol/ngày ở bệnh nhân chức năng
thận bình thường và hệ aldosteron bình thường
B) Điều trị thường xuyên bằng thuốc lợi tiểu gây ức chế aldosterone
C) Giảm lượng Natri đưa vào từ 200 xuống 100 mmol/ngày
D) Điều trị thường xuyên thuốc lợi tiểu ức chế đồng vận chuyển Na+-2Cl−-K+ tại
quai Henle
E) Điều trị lâu dài thuốc lợi tiểu ức chế tái hấp thu Na+ trong ống góp.

23. Chất nào sau đây được lọc dễ dàng nhất qua mao mạch cầu thận?
A) Albumin huyết tương
B) Dextran trung hòa với trọng lượng phân tử 25,000
C) Dextran poly-cation với trọng lượng phân tử 25,000
D) Dextran poly-anion với trọng lượng phân tử 25,000
E) Hồng cầu

24. Với điều kiện chức năng thận bình thường, nồng độ ure trong ống thận đoạn
cuối ống lượn gần như thế nào?
A) Cao hơn nồng đồ ure trong ống thận đoạn đỉnh của quai Henle
B) Cao hơn nồng độ ure trong huyết tương
C) Cao hơn nồng độ ure trong nước tiểu sau khi dùng chống bài niệu
D) Thấp hơn nồng độ ure huyết tương do hoạt hóa tái hấp thu ure trong ống lượn
gần

25. Những thay đổi nào sau đây xảy ra ở bệnh nhân mắc hội chức Liddle( tăng hoạt
động của kênh Na+ nhạy cảm với amiloride tại ống góp) trong tình trạng ổn định,
giả sử rằng lượng các chất điện giải đưa vào không thay đổi?


26. Lượng nước tiểu trong 2h của một bệnh nhân là 600 ml. Áp suất thẩm thấu
nước tiểu là 150 mOsm/L, áp suất thẩm thấu huyết tương là 300 mOsm/L. Độ
thanh thải nước là bao nhiêu?
A) +5.0 ml/phút
B) +2.5 ml/phút
C) 0.0 ml/phút
D) −2.5 ml/phút
E) −5.0 ml/phút

27. Một bệnh nhân được chuyển đến điều trị do tăng huyết áp. Sau khi xét nghiệm
nhận thấy bệnh nhân có tăng nồng độ aldosteron máu rất cao, do đó đưa ra chẩn
đoán là hội chứng Conn. Giả sử không có thay đổi về lượng điện giải đưa vào cơ
thể, những thay đổi nào sau đây có thể thấy ở bệnh nhân?
(Có bảng)
 A) Tăng tiết aldosterone tiên phát (hội chứngConn) có thể đi kèm với tình trạng
hạ Kali máu và kiềm chuyển hóa (tăng pH máu). Do aldosterone kích tích tái hấp
thu Natri và bài tiết Kali tại ống góp, có thể xảy ra giảm tạm thời bài tiết Natri và
tăng bài tiết Kali, nhưng trong tình trạng cơ thể ổn định, cả Natri và Kali bài tiết
trong nước tiểu sẽ trở về bình thường để phù hợp với lượng đưa vào của các chất
điện giải khác. Tuy nhiên, tình trạng giữ Natri cũng như tăng huyết áp đi kèm với
tăng tiết aldosterone quá mức có thể làm giảm tiết Renin
TMP12 364, 375
28. Một bệnh nhân mắc bệnh thận có nồng độ creatinine huyết tương 2 mg/dL
trong xét nghiệm 6 tháng trước. Nhận thấy huyết áp của bệnh nhân tăng hơn
30mmHg so với lần trước đến khám, và xét nghiệm cho thấy nồng độ creatine
huyết tương hiện tại là 4 mg/dL. Những thay đổi nào so với lần khám trước có thể
thấy ở bệnh nhân, giả sử bệnh nhân trong tình trạng ổn định và không có thay đổi
về lượng điện giải đưa vào cơ thể hoặc chuyển hóa?
(Có bảng)
 B) Tăng gấp đôi creatinine huyết tương cho thấy độ thanh thải creatinine và lưu
lượng lọc cầu thận GFR đã giảm khoảng 50%. Mặc dù giảm độ thanh thải creatine
ban đầu có thể gây ra giảm tạm thời tải lượng lọc creatinine, tốc độ bài tiết creatine
và tốc độ bài tiết Natri, nồng độ creatinine huyết tương sẽ tăng cho đến khi tải
lượng lọc của creatine và tốc độ bài tiết creatinine trở về bình thường. Tuy nhiên,
độ thanh thải creatine thường giữ nguyên ở tình trạng giảm, do độ thanh thải
creatinine là tốc độ bài tiết creatinine chia cho nồng độ creatinine huyết tương. Bài
tiết Natri qua nước tiểu có thể trở về bình thường và cân bằng với lượng Natri đưa
vào cơ thể là kết quả của cơ chế bù làm giảm tái hấp thu Natri trong ống thận trong
tình trạng cơ thể ổn định
TMP12 341, 404–405
29. Những thay đổi nào sau đây có thể gây tăng lưu lượng lọc cầu thận GFR?
A) Tăng áp lực cản trở của tiểu động mạch đến
B) Giảm áp lực cản trở của tiểu động mạch đi
C) Tăng hệ số lọc mao mạch cầu thận
D) Tăng áp lực thủy tĩnh bao Bowman
E) Giảm áp lực thủy tĩnh mao mạch cầu thận
 C) Hệ số lọc mao mạch cầu thận là tích của tính thấm mao mạch cầu thận và
diện tích bề mặt mao mạch. Do đó, tăng hệ số lọc mao mạch cầu thận làm tăng lưu
lượng lọc cầu thận GFR. Tăng áp lực cản trở ở tiểu động mạch đến, giảm áp lực
cản trở ở tiểu động mạch đi, tăng áp suất thủy tĩnh ở bao Bowman và giảm áp suất
thủy tĩnh mao mạch cầu thận đều làm giảm lưu lượng lọc cầu thận GFR
TMP12 314–316
30. Độ thanh thải tối đa cho một chất được lọc hoàn toàn từ huyết tương bằng giá
trị nào sau đây?
A) Lưu lượng lọc cầu thận GFR
B) Tải lượng lọc của chất đó
C) Tốc độ bài tiết qua nước tiểu của chất đó
D) Lượng huyết tương qua thận
E) Phân số lọc
 D) Nếu một chất hoàn toàn được lọc từ huyết tương, độ thanh thải của chất đó
tương đương với tổng lượng huyết tương đến thận. Nói cách khác, tổng lượng chất
đó trong máu vận chuyển tới thận (tổng lượng huyết tương đến thận × nồng độ chất
đó trong máu) sẽ bằng lượng chất đó được bài tiết qua nước tiểu.
Độ thanh thải qua thận hoàn toàn cỉa một chất sẽ phụ thuộc vào cả phân số lọc cầu
thận và bài tiết chất đó trong ống thận.
TMP12 340–343
31. Một bệnh nhân có kết quả xét nghiệm: pH động mạch = 7.13, HCO3− huyết
tương = 15 mEq/L,Nồng độ Cl huyết tương = 118 mEq/L, Pco2 động mạch = 28
mm Hg, and nồng độ Na+ huyết tương=141 mEq/L. Nguyên nhân chủ yếu gây tình
trạng toan hóa ở bệnh nhân là gì?
A) Ngộ độc acid salicylic
B) Đái tháo đường
C) Tiêu chảy
D) Khí phế thũng
 C) Bệnh nhân có pH thấp hơn so với bình thường nên có tình trạng toan hóa. Do
nồng độ bicarbonate huyết tương thấp hơn bình thường nên bệnh nhân có toan
chuyển hóa còn bù qua hô hấp (Pco2 thấp hơn bình thường). Khoảng trống anion
(Na+-Cl−-HCO3
− = 10 mEq/L) trong giới hạn bình thường cho thấy toan chuyển hóa không gây ra
bởi tình trạng quá nhiều acid không bay hơi như acid
salicylic hoặc ketoacids gây ra do ĐTĐ. Do đó, nguyên nhân chủ yếu gây toan
chuyển hóa ở bệnh nhân là tiêu chảy, do gây mất HCO3− qua phân và có thể đi
kèm với khoảng trống anion bình thường và tăng nồng độ Clo trong toan chuyển
hóa.
TMP12 392–395
32. Bệnh nhân nam 26 tuổi mắc viêm cầu thận cấp, và lưu lượng lọc cầu thận GFR
giảm 50% và giữ nguyên ở mức đó. Nồng độ chất nào sau đây tăng cao nhất trong
huyết tương?
A) Creatinine
B) K+
C) Glucose
D) Na+
E) Phosphate
F) H+
 A) Giảm 50% lưu lượng lọc cầu thận GFR có thể làm tăng gấp đôi nồng độ
creatine huyết tương, do creatine không được tái hấp thu hoặc bài tiết thêm và bài
tiết creatine phụ thuộc nhiều bào phân số lọc cầu thận. Do đó, khi GFR làm giảm
nồng độ creatine cho đến khi tăng bài tiết creatinine đủ để trở về bình thường.
Nồng độ glucose, Natri, Kali và H+ trong máu được điều hòa bởi rất nhiều cơ chế
giúp hằng định kể cả khi GFR giảm đến mức rất thấp. Nồng độ phosphate huyết
tương cũng được giữ gần như bình thường cho đến lúc GFR giảm còn 20% tới
30% bình thường.
TMP12 341, 404–405
33. Một bệnh nhân có tiền sử đau nửa đầu nặng và thường xuyên đến khám vì đau
dạ dày và thở nhanh. Bệnh nhân cho biết bị đau nửa đầu nặng 2 ngày nay và đã
uống 8 lần aspirin theo liều được khuyến cáo để giảm đau đầu. Những thay đổi nào
dưới đây có thể tìm thấy ở bệnh nhân này?
(Có bảng)
D) Sử dụng quá nhiều aspirin (salicylic acid) gây ra toan chuyển hóa đặc trưng
bởi tình trạng giảm nồng độ HCO3− huyết tương và tăng khoảng trống anion. Tình
trạng toan hóa kích thích hô hấp, gây ra giảm Pco2 huyết tương. Tình trạng toan
hóa cũng làm tăng tái hấp thu HCO3−, dẫn đến giảm bài tiết HCO3− qua nước
tiểu. Cuối cùng, tình trạng toan hóa cũng kích thích bù bằng cách tăng tạo NH4+
trong ống thận.
TMP12 392–395.
Step 1. Tình trạng ban đầu
Step 2. Ảnh hưởng của truyền 2 L of NaCl 3% sau khi cân bằng áp suất thẩm thấu
Câu 34 và 35
Cho những dữ kiện sau: thể tích dịch nội bào = 40% trọng lượng cơ thể trước khi
truyền dịch, thể tích dịch ngoại bào = 20% trước truyền dịch, trọng lượng phân tử
NaCl = 58.5g/mol, và không có bài tiết nước tiểu và điện giải.

34. Bệnh nhân nam có tình trạng dehydrat hóa(tình trạng mất nước), và sau khi có
kết quả xét nghiệm phát hiện tình trạng hạ Natri máu với nồng độ Natri huyết
tương là 130 mmol/L và áp lực thẩm thấu huyết tương 260mOsm/L. Từ đó chỉ
định truyền 2 L NaCl 3%. Trọng lượng cơ thể bệnh nhân trước truyền dịch là 60kg.
Áp suất thẩm thấu huyết tương ở bệnh nhân sau khi truyền NaCl và sau khi cân
bằng thẩm thấu là bao nhiêu. Các dữ kiện cần thiết đã đưa ra ở trên.
A) 273 mOsm/L
B) 286 mOsm/L
C) 300 mOsm/L
D) 310 mOsm/L
E) 326 mOsm/L
 C) Cách tính vận chuyển dịch và áp suất thẩm thấu sau khi truyền dung dịch
muối ưu trương được viết trong phần 25 của TMP12. Các bảng trong phần đó cho
thấy tình trạng ban đầu và tình trạng sau cùng sau khi truyền
2 L NaCl 3% và cân bằng thẩm thấu. NaCl 3% bằng 30 g NaCl/L, hoặc 0.513
mol/L (513 mmol/L). Do NaCl có 2 ion tham gia vào áp suất thẩm thấu, ảnh hưởng
tới màng lọc là thêm 2052 mmol trong 2L dịch truyền. Như vậy xấp xỉ có thể cho
rằng màng tế bào không thấm NaCl và NaCl truyền vào được giữ lại trong thành
phần dịch ngoại bào
TMP12 291–294
35. Thể tích dịch ngoại bào khoảng bao nhiêu ở bệnh nhân sau khi truyền NaCl và
cân bằng thẩm thấu?
A) 15.1 L
B) 17.2 L
C) 19.1 L
D) 19.8 L
E) 21.2 L
 B) Thể tích dịch ngoại bào được tính bằng cách chia tổng số mmol
(milliosmoles) ở thành phần dịch ngoại bào (5172 mOsm) cho nồng độ sau khi cân
bằng thẩm thấu (300 mOsm/L) được 17.2 L.
TMP12 291–294
36. Hạ Kali máu nguy hiểm xảy ra trong trương hợp nào dưới đây?
A) Giảm lượng Kali đưa vào cơ thể từ 150 xuống 60 mEq/ngày
B) Tăng lượng Natri đưa vào từ 100 lên 200 mEq/ngày
C) Tăng 4 lần lượng bài tiết aldosteron cộng với lượng Natri đưa vào cơ thể cao
D) Tăng 4 lần lượng bài tiết aldosteron cộng với lượng Natri đưa vào cơ thể thấp
E) Bệnh Addison
 C) Tăng bài tiết aldosterone số lượng lớn cùng với tăng lượng Natri đưa vào cơ
thể có thể gây ra hạ Kali máu nặng. Aldosterone kích thích bài tiết Kali và vận
chuyển Kali từ dịch ngoại bào vào trong tế bào, và tăng lượng Natri đưa vào cơ thể
làm tăng lưu lượng dịch qua ống góp giúp làm tăng tiết Kali. Ở người bình thường,
lượng Kali đưa vào có thể giảm còn ¼ so với bình thường mà chỉ gây ra giảm rất
nhẹ trong nồng độ Kali huyết tương (chi tiết tham khảo
TMP12, Hình 29-8). Lượng Natri đưa vào thấp có thể ngăn chặn tác động làm
giảm Kali máu của aldosterone, do lượng Natri đưa vào thấp có thể làm giảm tỉ lệ
dịch chảy qua ống góp và do đó làm giảm tiết Kali. Bệnh nhân mắc Addison có
suy giảm bài tiết aldosterone, do đó có nguy cơ tăng Kali máu.
TMP12 361, 364–367
37. Nếu áp suất thủy tĩnh trung bình trong mao mạch cầu thận là 50 mm Hg, áp
suất thủy tĩnh trong bao Bowman là 12 mm Hg, áp suất keo trong mao mạch cầu
thận là 30 mm Hg, và không có protein lọc qua lưới lọc cầu thận, áp lực hệ thống
thúc đẩy lọc qua cầu thận là bao nhiêu?
A) 8 mm Hg
B) 32 mm Hg
C) 48 mm Hg
D) 60 mm Hg
E) 92 mm Hg
 A) Áp lực lọc hệ thống tại mao mạch cầu thận bằng tổng của các lực hỗ trợ cho
việc lọc (áp suất thủy tĩnh cầu thận) trừ đi các lực ngăn cản việc lọc (áp lực thủy
tĩnh bao Bowman và áp suất keo trong mao mạch cầu thận). Do đó áp lực lọc hệ
thống thúc đẩy lọc ở cầu thận bằng
50 − 12 − 30 = 8 mm Hg.
TMP12 314
38. Ở bệnh nhân có ĐTĐ mạn tính không được kiểm soát, các tình trạng nào sau
đây có thể thấy ở bệnh nhân so với bình thường?
(Có bảng)
 D) ĐTĐ không được kiểm soát gây ra hậu quả làm tăng nồng độ acetoacetic
acid trong máu, từ đó gây ra toan chuyển hóa và làm giảm HCO3− and pH huyết
tương. Tình trạng toan hóa gây ra các đáp ứng bù, bao gồm tăng tần số thở nhằm
làm giảm Pco2 huyết tương; tăng sản xuất NH+ tại thận giúp tăng tiết NH+; và
tăng tác dụng hệ đệm phosphate lên H+ được tiết từ ống thận, từ đó làm tăng bài
tiết acid chuẩn độ titratable acid
TMP12 391–393
39. Truyền tĩnh mạch 1 L NaCl 0.45% (trọng lượng phân tử NaCl =58.5) có thể
xảy ra những thay đổi nào sau đây, sau khi cân bằng áp suất thẩm thấu?
 B) Truyền tĩnh mạch dung dịch NaCl nhược trương có thể ban đầu gây ra tăng
thể tích dịch ngoại bào và làm giảm áp suất thẩm thấu dịch ngoại bào. Giảm thể
tích dịch ngoại bào có thể gây ra vận chuyển dịch vào trong tế bào, do đó làm tăng
thể tích dịch nội bào và làm giảm áp suất thẩm thấu dịch nội bào sau khi cân bằng
thẩm thấu.
TMP12 292–294
40. Hình minh họa trên đây cho thấy nồng độ inulin tại các điểm khác nhau thuộc
ống thận, được thể hiện dưới dạng tỉ số nồng độ inulin trong ống thận/nồng độ
inulin trong huyết tương. Nếu inulin không được tái hấp thu, khoảng bao nhiêu
phần trăm nước được tái hấp thu trước khi tới ống lượn xa?
A) 25%
B) 33%
C) 66%
D) 75%
E) 99%
F) 100%
 D) Tỉ lệ nồng độ inulin trong ống thận/nồng độ inulin trong huyết tương= 4 ở
đoạn đầu ống lượn xa được thể hiện trong bảng. Do inulin không được tái hấp thu
ở ống thận, tái hấp thu nước sẽ cô đặc nồng độ inulin gấp 4 lần so với trong huyết
tương sau lọc. Do đó, lượng nước giữ lại trong ống thận chỉ bằng ¼ so với lượng
được lọc, cho thấy 75% nước được tái hấp thu trước khi đến ống lượn xa
TPM12 334
41. Trường hợp nào sau đây có thể làm tăng tiết Kali tại ống góp?
A) Thuốc lợi tiểu ức chế hoạt động của aldosterone (VD: spironolactone)
B) Thuốc lợi tiểu làm giảm tái hấp thu Natri tại quai Henle (VD: furosemide)
C) Giảm nồng độ Kali trong huyết tương
D) Toan chuyển hóa cấp
E) Giảm lượng Natri đầu vào
 B) Tiết Kali ở ống góp được kích thích bởi (1) aldosterone, (2) Tăng nồng độ
Kali huyết tương, (3) tăng lưu lượng dịch qua ống góp và(4) kiềm chuyển hóa. Do
đó, lợi tiểu ức chế aldosterone, làm giảm nồng độ Kali huyết tương, toan hóa cấp
và giảm thấp lượng Natri đầu vào đều có thể làm giảm tiết Kali tại ống góp. Tuy
nhiên thuốc lợi tiểu làm giảm tái hấp thu Natri ở quai Henle có thể làm tăng lưu
lượng dịch qua ống góp và do đó kích thích tiết Kali
TMP12 364–367
42. Những thay đổi nào dưới đây có thể thấy trên bệnh nhân tăng tiết aldosterone
tiên phát (hội chứng Conn) với tình trạng ổn định, giả sử rằng lượng điện giải đưa
vào không thay đổi?
(Có bảng)
 B) Tăng tiết quá mức aldosterone kích thích tái hấp thu Natri và tăng bài tiết
Kali ở ống góp, gây ra giảm tạm thời bài tiết Natri và tăng thể tích dịch ngoại bào,
cũng như tăng tốc độ bài tiết kali. Tình trạng giữ Natri làm tăng huyết áp và làm
giảm tiết renin. Tuy nhiên trong tình trạng ổn định, bài tiết Natri và Kali có thể trở
về bình thường, do đó lượng đầu vào và đầu ra của các chất này được cân bằng.
Bài tiết quá mức aldosterone có thể gây ra giảm nồng độ Kali trong huyết tương
đáng kể do tăng tạm thời bài tiết Kali cũng như tác động tăng vận chuyển Kali từ
ngoại bào vào trong tế bào của aldosterone
TMP12 337–338, 375
43. Một bệnh nhân đái tháo đường có biến chứng bệnh thận mạn tính và được
chuyển đến khoa thận. Theo giấy chuyển từ bác sĩ gia đình, độ thanh thải
creatinine ở bệnh nhân giảm từ 100 ml/phút xuống 40 ml/phút trong 4 năm nay.
Đường máu của bệnh nhân không được kiểm soát tốt, và pH huyết tương là 7.14.
Những thay đổi nào dưới đây có thể xảy ra ở bệnh nhân so với trước khi có biến
chứng ở thận, giả sử rằng bệnh nhân trong tình trạng ổn định và không có thay đổi
gì về lượng điện giải đầu vào?
(Có bảng)
 B) Bệnh nhân ĐTĐ và bệnh nhận mạn này có độ thanh thải creatinine giảm còn
tới 40% bình thường, ảnh hưởng đáng kể tới lưu lượng lọc cầu thận GFR. Bệnh
nhân cũng có tình trạng toan hóa, bằng chứng là pH huyết tương là 7.14. Giảm
thanh thải creatinin có thể gây ra giảm tạm thời bài tiết Natri và Creatinine. Do
nồng độ creatinine huyết tương tăng, tốc độ bài tiết creatine qua nước tiểu có thể
trở về bình thường, mặc dù tình trạng giảm liên tục độ thanh thải creatinine (tốc độ
bài tiết creatinine /nồng độ creatinine huyết tương).ĐTĐ đi kèm với tăng sản xuất
acetoacetic acid, từ đó gây ra toan chuyển hóa và làm giảm nồng độ HCO3−, cũng
như tăng bù do sản xuất NH4+ và tăng tốc độ bài tiết NH4+
TMP12 391–393, 404–405
44. Bệnh nhân nữ 20 tuổi vào viện vì tăng cân nhanh và phù nhiều. Huyết áp của
bệnh nhân là 105/65 mm Hg, nồng độ protein huyết tương là 3.6g/dL (bình thường
= 7.0), và bệnh nhân không có protein trong nước tiểu. Những thay đổi nào dưới
đây có thể thấy ở bệnh nhân?
(Có bảng)
 C) Giảm nồng độ protein huyết tương xuống 3.6 g/dL có thể làm tăng tốc độ lọc
ở mao mạch thận, do đó làm tăng thể tích dịch khoảng kẽ và áp lực thủy tĩnh
khoảng kẽ. Tăng áp lực khoảng kẽ làm tăng lưu lượng dịch bạch huyết và làm
giảm nồng độ protein dịch kẽ(“hao tổn” protein dịch khoảng kẽ).
TMP12 297–300
45. Bệnh nhân nữ 48 tuổi đến khám do tiểu nhiều (khoảng 0.5 L nước tiểu trong
1h) và khát nhiều (uống 2 đến 3 cốc nước mỗi tiếng). Không có glucose trong nước
tiểu của bệnh nhân, và bệnh nhân phải giới hạn lượng nước đưa vào qua một đêm.
Đến sáng hôm sau, bệnh nhân mệt mỏi và lú lẫn, nồng độ Natri trong nước tiểu là Commented [LTN1]: []
160 mEq/L, áp suất thẩm thấu nước tiểu là 80 mOsm/L. Chẩn đoán nào dưới đây
phù hợp nhất với bệnh nhân này?
A) Đái tháo đường
B) Đái tháo nhạt
C) Tăng tiết aldosterone tiên phát
D) U tăng tiết renin
E) Hội chứng tăng tiết ADH bất thường (SIADH)
 B) Chẩn đoán hợp lý nhất trên bệnh nhân này là đái tháo nhạt, tình trạng này có
thể gây ra tiểu nhiều và thực tế là áp suất thẩm thấu nước tiểu của bệnh nhân rất
thấp (80 mOsm/L) kể cả khi đã giới hạn lượng nước đưa vào qua 1 đêm. Ở các
bệnh nhân mắc đái tháo nhạt, nồng độ Natri huyết tương có thể duy trì gần như
bình thường bằng cách tăng lượng dịch đầu vào ( tăng khát). Khi lượng nước đưa
vào bị hạn chế, lượng nước tiểu nhiều có thể dẫn đến giảm nhanh chóng thể tích
dịch ngoại bào và gây hạ Natri máu nặng như đã xảy ra trên bệnh nhân này. Thực
tế là bệnh nhân không có glucose nước tiểu, loại trừ ĐTĐ. Cả tăng aldosterone
tiên phát và u tăng tiết renin có thể dẫn đến tình trạng không thể cô đặc nước tiểu
sau khi hạn chế lượng nước đưa vào qua 1 đêm. Hội chứng SIADH tăng tiết ADH
bất thường có thể gây phù nhiều và tăng áp suất thẩm thấu nước tiểu.
TMP12 354, 358–359
46. Lợi tiểu Furosemide (Lasix) là một loại lợi tiểu tăng đào thải Natri qua nước
tiểu. Vấn đề nào dưới đây là tác dụng phụ không mong muốn của thuốc lợi tiểu
furosemide do vị trí tác động của nó trên ống thận?
A) Phù
B) Tăng Kali máu
C) Tăng Canxi máu
D) Giảm khả năng cô đặc nước tiểu
E) Suy tim
 D) Lợi tiểu Furosemide (Lasix) ức chế đồng vận chuyển Na+-2Cl−-K+ ở phần
xuống của quai Henle. Nó không chỉ gây bài niệu và tăng bài tiết Natri qua nước
tiểu mà còn làm giảm khả năng cô đặc nước tiểu. Lợi tiểu Furosemide không gây
phù, trong thực tế nó thường được dùng để điều trị phù nặng và suy tim.
Furosemide cũng làm tăng bài tiết Kali và Canxi qua thận do đó gây ra hạ Kali và
hạ Canxi máu hơn là làm tăng nồng độ của chúng
TMP12 331, 348, 366–368, 397–398
47. Bệnh nhân đến khám vì đau đầu, và sau khi khám phát hiện huyết áp của bệnh
nhân là 175/112 mm Hg. Kết quả xét nghiệm cho thấy: hoạt độ renin huyết tương
= 11.5 ng angiotensin I/ml/hr (bình thường = 1), nồng độ Na+ huyết tương = 144
mmol/L, và nồng độ K+ huyết tương = 3.4 mmol/L. Kết quả cộng hưởng từ cho
thấy bệnh nhân có u tăng tiết renin tại thận. Những thay đổi nào dưới đâu có thể
xảy ra trên bệnh nhân?
(Có bảng)
 A) Tăng bài tiết renin có thể dẫn đến hình thành số lượng lớn angiotensin II, từ
đó gây ra co tiểu động mạch đi. Nó làm giảm lượng máu qua thận, tăng áp suất
thủy tĩnh ở cầu thận, và làm giảm áp suất thủy tĩnh mao mạch quanh ống thận. Do
tình trạng co tiểu động mạch đi làm giảm lượng máu qua thận nhiều hơn lưu lượng
lọc cầu thận, phân số lọc (tỉ lệ GFR/lượng huyết tương qua thận) tăng
TMP12 315–316, 318
48. Khi tăng lượng Kali đưa vào cơ thể, cân bằng Kali trong cơ thể được duy trì
nhờ tình trạng tăng bài tiết Kali ban đầu theo cơ chế nào sau đây? Commented [LTN2]: chủ yếu

A) Giảm lọc Kali qua cầu thận


B) Giảm tái hấp thu Kali ở ống lượn gần
C) Giảm tái hấp thu Kali ở đoạn xuống quai Henle.
D) Tăng tiết K+ ở đoạn cuối ống lượn xa và ống góp
E) Vận chuyển K+ vào trong tế bào
 D) Hầu hết sự thay đổi hàng ngày ở quá trình bài tiết Kali gây ra do sự thay đổi
tiết Kali ở đoạn cuối ống lượn xa và ống góp. Do đó khi lượng Kali đầu vào tăng,
cân bằng Kali trong cơ thể được duy trì chủ chủ yếu bằng tăng tiết Kali ở các đoạn
của ống thận. Tăng lượng Kali đưa vào gây ra ảnh hưởng rất nhỏ tới lưu lượng lọc
cầu thận
GFR hay tái hấp thu Kali ở ống lượn gần hay quai Henle. Mặc dù Kali đưa vào
nhiều có thể gây ra sự vận chuyển Kali vào trong nội bào mức độ ít, cân bằng giữa
đầu vào và đầu ra có thể đạt được bằng cách tăng tiết Kali nếu lượng Kali đưa vào
nhiều
TMP12 362–363
49. Bệnh nhân nữ có tình trạng tăng Natri máu nặng chưa rõ nguyên nhân (nồng độ
Na+ huyết tương= 167 mmol/L) đến khám vì đi tiểu nhiều lần và số lượng nước
tiểu nhiều. Mẫu nước tiểu cho thấy nồng độ Na+ là 15mmol/L (rất thấp) và áp suất
thẩm thấu là 155 mOsm/L (rất thấp). Xét nghiệm cho thấy: Hoạt độ renin huyết
tương = 3 ng angiotensin I/ml/giờ (bình thường = 1.0), hormone chống bài niệu
huyết tương (ADH) = 30 pg/ml
(bình thường = 3 pg/ml), và aldosterone huyết tương= 20 ng/dL (bình thường = 6
ng/dL). Nguyên nhân nào sau đây gây ra tăng Natri máu ở bệnh nhân này?
A) Mất nước đơn thuần do giảm lượng nước đưa vào cơ thể
B) Đái tháo nhạt do thận
C) Đái tháo nhạt trung ương (Đái tháo nhạt do thần kinh)
D) Hội chứng tăng tiết ADH bất thường
E) Tăng tiết aldosteron tiên phát
F) U tăng tiết renin
 B) Tăng Natri máu có thể gây ra do tăng giữ Natri quá mức hoặc mất nước.
Thực tế là bệnh nhân có thể tích nước tiểu pha loãng lớn cho thấy bài tiết nước tiểu
quá mực. Trong 2 rối loạn dưới đây có thể gây ra tăng bài tiết nước tiểu quá mức
(đái tháo nhạt do thận và đái tháo nhạt do thần kinh), đái tháo nhạt do thận là
nguyên nhân chủ yếu. Đái tháo nhạt trung ương (giảm tiết ADH) không phải là
nguyên nhân chủ yếu do nồng độ ADH huyết tương tăng đáng kể. Mất nước đơn
thuần do giảm lượng nước đưa vào không xảy ra ở bệnh nhân do bệnh nhân có
tăng bài tiết quá mức lượng nước tiểu pha loãng
TMP12 295–296, 354
50. Đái tháo đường thanh thiếu niên (type 1) thường được chẩn đoán dựa trên các
triệu chứng tiểu nhiều, khát nhiều xảy ra do nguyên nhân nào dưới đây?
A) Tăng vận chuyển glucose tới ống góp làm cản trở hoạt động của hormone
chống bài niệu
B) Tăng lọc glucose qua cầu thận làm tăng tái hấp thu Natri qua kênh đồng vận
chuyển Natri-glucose
C) Khi tải lượng lọc glucose vượt quá ngưỡng của thận, tăng nồng độ glucose ở
ống lượn gần làm giảm áp lực thẩm thấu thúc đẩy tái hấp thu nước
D) Tăng nồng độ glucose huyết tương làm giảm khát
E) Nồng độ glucose huyết tương cao kích thích hormone chống bài niệu giải phóng
từ thùy sau tuyến yên
 C) Lượng nước tiểu nhiều ở những bệnh nhân ĐTĐ type I do tải lượng lọc
glucose vượt quá ngưỡng của thận gây ra tăng nồng độ glucose trong ống thận, từ
đó làm giảm áp lực thẩm thấu thúc đẩy tái hấp thu nước. Tăng lượng nước tiểu làm
giảm thể tích dịch ngoại bào và kích thích giải phóng hormone chống bài niệu
TMP12 326–327, 355–356
51. Khi bắt đầu điều trị bệnh nhân tăng huyết áp bằng lợi tiểu quai mạnh (VD
furosemide), những thay đổi nào dưới đây sẽ xảy ra ở bệnh nhân so với trước điều
trị, khi bệnh nhân đến kiểm tra 2 tuần sau đó? Commented [LTN3]: khám lại

(Có bảng)
 C) Các thuốc lợi tiểu ức chế hấp thu Natri ở quai Henle được sử dụng để điều trị
các vấn đề về quá tải dịch (VD: Tăng huyết áp, Suy tim). Các thuốc lợi tiểu này
ban đầu làm tăng bài tiết Natri từ đó làm giảm thể tích dịch ngoại bào và huyết áp,
nhưng trong tình trạng ổn định, bài tiết Natri qua nước tiểu sẽ trở về bình thường,
một phần để hạ huyết áp. Một trong những tác dụng phụ không mong muốn của lợi
tiểu quai là tình trạng hạ Natri máu gây ra bởi quá trình ức chế đồng vận chuyển
Na+-2Cl−-K+ ở quai Henle và bởi tình trạng tăng lưu lượng dịch qua ống góp, từ
đó kích thích tiết Kali
TMP12 366, 397–398
52. Trong tình trạng toan hóa, hầu hết H+ tiết ở ống lượn gần là do quá trình nào
sau đây?
A) Bài tiết H+
B) Bài tiết NH4+
C) Tái hấp thu ion bicarbonate
D) Tái hấp thu ion phosphate
E) Tái hấp thu ion Kali
 C) Khoảng 80% đến 90% bicarbonate được tái hấp thu ở ống lượn gần trong
tình trạng bình thường cũng như khi xảy ra toan hóa. Với mỗi ion bicarbonate được
tái hấp thu thì cũng/phải có một ion H+ được tiết ra. Do đó, khoảng 80% đến 90%
H+ được tiết ra nhằm sử dụng để tái hấp thu bicarbonate tại ống lượn gần
TMP12 387, 389–390
53. Truyền lợi tiểu thiazide (VD: chlorothiazide) sẽ gây ra nhưng ảnh hưởng nào Commented [LTN4]: có thể

dưới đây được xem như cơ chế tác dụng cơ bản? Commented [LTN5]: do cơ chế tác dụng ban đầu của
thuốc
A) Ức chế đồng vận chuyển NaCl ở đầu ống lượn xa
B) Ức chế đồng vận chuyển NaCl ở ống lượn gần
C) Ức chế đồng vận chuyển Na+-2Cl−-K+ ở quai Henle
D) Ức chế đồng vận chuyển Na+-2Cl−-K+ ở ống góp
E) Ức chế hoạt động của aldosteron trên ống thận
F) Phong tỏa kênh Natri tại ống góp
 A) Lợi niệu thiazide ức chế đồng vận chuyển NaCl ở màng nội chất (luminal
membrane) ở đầu ống lượn xa Commented [LTN6]: màng đỉnh?

(Xem chi tiết tại TMP12 Bảng 31-1.)


TMP12 398
54. 2 tuần sau khi bị hẹp động mạch thận trên thận đơn độc làm giảm ban đầu áp Commented [LTN7]: làm hẹp

lực tưới máu thận xuống 20 mmHg (từ 100 xuống 80 mm Hg), những thay đổi nào Commented [LTN8]: để
có thể thấy ở bệnh nhân so với trước kia? Commented [LTN9]: []

A) Giảm bài tiết Natri nhiều (>20%)


B) Tăng tiết renin nhiều (hơn gấp đôi) Commented [LTN10]: /gấp hơn hai lần

C) Áp lực động mạch thận tăng trở lại lên gần 100 mm Hg
D) Giảm mạnh lưu lượng lọc cầu thận GFR (>20%)
E) Giảm mạnh lượng máu tới thận (>20%)
 C) Giảm áp lực tưới máu thận xuống 80 mm Hg (trong giới hạn tự điều hòa) có
thể gây ra giảm tạm thời GFR, lượng máu tới thận và bài tiết Natri, cũng như làm
tăng tạm thời tiết Renin. Giảm bài tiết Natri và tăng tiết Renin có thể gây tăng
huyết áp, do đó giúp đưa áp lực tưới máu thận và đưa chức năng thận trở về bình
thường. Miễn là áp lực tưới máu thận không giảm quá mức có thể tự điều hòa
được, GFR và lượng máu tới thận có thể trở về bình thường trong vòng vài phút
động mạch thận bị hẹp Commented [LTN11]: sau khi làm hẹp động mạch thận

TMP12 319 (xem thêm Chương 19, trang 223)


55. Thông thường do tốc độ lọc phosphate vượt quá khả năng tối đa vận chuyển Commented [LTN12]: []

phosphate tái hấp thu, trường hợp nào sau đây là đúng? Commented [LTN13]: thường

A) Tất cả phosphate được lọc đều được tái hấp thu sau đó
B) Nhiều phosphate được tái hấp thu hơn là số được lọc
C) Phosphate trong ống thận có thể góp phần quan trọng tạo nên titratable acid Commented [LTN14]: acid titratable?

trong nước tiểu


D) “Ngưỡng” phosphate thường không bị vượt quá
E) Hormon cận giáp phải được tiết để tái hấp thu phosphate
 C) Bài tiết phosphate do thận được điều hòa bởi cơ chế chảy tràn. Khi vượt quá
mức tối đa vận chuyển phosphate tái hấp thu, phosphate còn lại trong ống thận
được bài tiết vào trong nước tiểu và được sử dụng cho hệ đệm H+ và hình thành
titratable acid. Phosphate thông thường bắt đầu tràn vào nước tiểu khi nồng độ
trong dịch ngoại bào đạt tới ngưỡng trên 0.8 mmol/L, quá ngưỡng tái hấp thu.
TMP12 369
56. Thay đổi nào sau đây có thể xảy ra trong tình trạng ổn định ở bệnh nhân có
bệnh thận nặng làm giảm số lượng nephrons chức năng xuống 25% bình thường
A) Giảm mức lọc cầu thận của các nephrons còn lại
B) Giảm tốc độ bài tiết creatinine
C) Giảm lưu lượng bài tiết nước tiểu của các nephron còn lại
D) Giảm bài tiết Natri qua nước tiểu
E) Tăng khả năng cô đặc nước tiểu
 A) Giảm số lượng nephron chức năng xuống 25% bình thường có thể gây nên
tăng còn bù GFR và lưu lượng nước tiểu qua các nephrons còn lại và làm giảm khả
năng cô đặc nước tiểu.
Trong tình trạng bình thường, tốc độ bài tiết creatinine và Natri được duy trì ở mức Commented [LTN15]: Khi tình trạng ổn định

bình thường
(Xem thêm tại TMP12 Bảng 31-6.)
TMP12 404–405
57. Ở bệnh nhân mắc hội chứng SIADH (hội chứng bài tiết ADH bất thường làm
tăng bài tiết quá mức ADH), những thay đổi nào dưới đây có thể xảy ra?
Giả sử rằng trong tình trạng bình thường và lượng nước và điện giải đưa vào vẫn ở Commented [LTN16]: tình trạng bệnh nhân ổn định

mức ổn định. Commented [LTN17]: duy trì hằng định

C) Khi bài tiết quá mức ADH, sẽ có sự tăng đáng kể tính thấm nước ở đoạn sau
ống lượn xa và ống góp, gây ra tích nước, tăng thể tích tuần hoàn và giảm nồng độ
Natri và protein trong dịch ngoại bào. Hoạt độ renin trong huyết tương và nồng độ
aldosterone cũng giảm do tăng thể tích dịch. Mặc dù tăng tiết quá mức ADH ban
đầu gây giảm đáng kể lưu lượng nước tiểu, do có tăng thể tích dịch và tăng huyết
áp nên gây ra thất thoát thể tích dịch, từ đó lưu lượng nước tiểu trờ về bình thường
cân bằng với lượng dịch đưa vào.
TMP12 375–376
58. Trường hợp nào sau đây có thể dẫn tới Hạ Natri máu?
A) Tăng tiết quá mức ADH
B) Hạn chế lượng dịch đưa vào
C) Bài tiết quá mức aldosteron
D) Truyền TM 2 L NaCl 3%
E) Truyền TM 2 L NaCl 0.9%
 A) Tăng tiết quá mức ADH làm tăng tái hấp
thu nước ở ống góp, từ đó làm giảm nồng độ
Natri dịch ngoại bào (Hạ natri máu). Hạn chế
lượng dịch đưa vào, tăng tiết aldosteron quá
mức hay truyền NaCl 3% ưu trương đều có thể
làm tăng nồng độ Natri huyết tương (Tăng
Natri máu) trong khi truyền NaCl 0,9% (dịch
đẳng trương) không gây ra thay đổi đáng kể
nào áp lực thẩm thấu huyết tương.
TMP12 293–295

Câu hỏi 59–62

Chọn vị trí nephron thích hợp trong hình vẽ trên:


59. Ở bệnh nhân có đái tháo nhạt trung ương (do thần kinh) nặng gây ra do thiếu
ADH, phần nào của ống thận có áp lực thẩm thấu thấp nhất?
 E) Khi thiếu ADH, đoạn sau ống lượn xa và ống góp không thấm nước (xem ở
trên). Do đó, dịch ống thận được pha loãng khi rời khỏi quai Henle (Khoảng 100 Commented [LTN18]: , đã

mOsm/L), được pha loãng thêm khi chảy qua ống lượn xa và ống góp, trong đó các
chất điện giải được tái hấp thu. Do đó, áp suất thẩm thấu nước tiểu cuối trong
trường hợp hoàn toàn không có ADH là thấp hơn
100 mOsm/L.
TMP12 352, Bảng 28-8
60. Ở bệnh nhân có chế độ ăn chứa Kali rất thấp, phần nào của nephron giúp tái
hấp thu Kali nhiều nhất?
 A) Khoảng 65% Kali sau lọc được tái hấp thu ở ống lượn gần, và 20% đến 30%
được tái hấp thu ở quai Henle. Mặc dù hầu hết mức dao động bài tiết Kali trong
ngày gây ra do các thay đổi trong bài tiết Kali ở ống lượn xa và ống góp, chỉ một
phần trăm nhỏ tải lượng lọc Kali được tái hấp thu tại các đoạn của nephron
(Xem thêm tại TMP12 Bảng 29-2.)
TMP12 362–363
61. Phần nào của nephron thường tái hấp thu nước nhiều nhất?
 A) Ống lượn gần thường tái hấp thu khoảng 65% nước sau lọc, và một phần nhỏ
hơn còn lại được tái hấp thu ở đoạn xuống quai Henle và ống lượn xa, ống góp.
Đoạn lên của quai Henle không thấm nước, do đó tái hấp thu rất ít nước.
TMP12 329, 352–353
62. Ở thận có chức năng bình thường, phần nào của ống thận có khả năng thấm
nước thấp nhất khi có ADH?
 C) Thành của đoạn lên quai Henle không thấm nước kể cả trong trường hợp sử
dụng tối đa thuốc chống bài niệu. Ống lượn gần và đoạn xuống quai Henle thấm
nước trong cả tình trạng bình thường cũng như sử dụng chống bài niệu. Khả năng
thấm nước ở đoạn cuối ống lượn xa và ống góp tăng đáng kể khi sử dụng ADH,
phụ thuộc vào tăng nồng độ ADH
TMP12 352–353
63. Chất nào dưới đây giúp đo thể tích dịch khoảng kẽ thích hợp nhất?
A) Inulin và nước nặng ( chứa D2O, đồng vị của H)
B) Inulin và 22Na
C) Nước nặng và 125I-albumin
D) Inulin và 125I-albumin
E) 51Cr hồng cầu và 125I-albumin

64. Thay đổi nào dưới đây có thể thấy ở bệnh nhân bị mất nước trong 24h?
A) Giảm hoạt động renin huyết tương
B) Giảm nồng độ ADH huyết tương
C) Tăng nồng độ peptide natri lợi niệu tâm nhĩ (ANP)
D) Tăng tính thấm nước ở ống góp
E) Tăng tính thấm nước ở đoạn xuống quai Henle
 D) Mất nước do thiếu cung cấp nước làm giảm thể tích dịch ngoại bào, từ đó
làm tăng tiết renin và giảm peptide natri lợi niệu tâm nhĩ ANP trong huyết tương.
Mất nước cũng làm tăng nồng độ Natri huyết tương từ đó kích thích bài tiết ADH.
Tăng ADH làm tăng tính thấm nước ở ống góp. Đoạn xuống của quai Henle không
thấm nước, và tính thấm của nó không thay đổi kể cả khi thiếu cung cấp nước hay
tăng nồng độ ADH.
TMP12 349–350, 355–356

65. Thay đổi nào dưới đây xảy ra ở bệnh nhân sau truyền thuốc gây giãn mạch cấp
làm giảm 50% sức cản ở tiểu động mạch đi tại thận và không có thay đổi sức cản
tiểu động mạch đi hay huyết áp?

 E) Giảm 50% sức cản ở tiểu động mạch đi có thể làm giảm áp suất thủy tĩnh
mao mạch cầu thận (ngược dòng từ tiểu động mạch đi) và từ đó làm giảm GFR,
trong khi tăng áp suất thủy tĩnh ở mao mạch quanh ống thận (xuôi dòng từ tiểu
động mạch đi) và làm tăng lượng máu qua thận.
TMP12 315–316, 336
Các yếu tố làm thay đổi phân bố Kali ở dịch nội – ngoại bào
Các yếu tố vận Các yếu tố vận
chuy chuyển K+ vào
ển trong tế bào
+
Insulin Thiếu Insulin
K vào trong tế (Tăng [K+] (
ĐTĐ) bào)
Thiếu
bào ngoại
Aldosterone + Aldosterone
(Giảm [K ]
Kích thích β- Phong tỏa β-
ngoạ (bệnh Addison)
adrenergic
Kiếm hóa adrenergic
Toan hóa
i
bào) Phân hủy tế bào
Các bài tập nặng
Tăng áp suất
thẩm thấu dịch
66. Trường hợp nào dưới đâybào
ngoại có thể gây ra giảm nồng độ Kali dịch ngoại bào (hạ
Kali máu) ít nhất một phần bằng kích thích Kali đi vào trong tế bào?
A) Phong tỏa β-adrenergic Commented [LTN19]: Phong bế

B) Thiếu Insulin
C) Các bài tập sức mạnh kéo dài
D) Thiếu Aldosterone
(Addison’s disease) Commented [LTN20]: bệnh Addison

F) Metabolic alkalosis Commented [LTN21]: Kiềm chuyển hóa

 E) Kiềm chuyển hóa đi kèm với hạ Kali máu là do vận chuyển Kali từ dịch
ngoại bào vào trong tế bào. Chẹn β-adrenergic, thiếu hụt insulin, vận động mạnh
kéo dài và thiếu hụt aldosterone đều gây tăng kali máu do vận chuyển Kali ra khỏi
tế bào đi vào dịch ngoại bào.
TMP12 361–362, Table 29-1
67. Trường hợp nào sau đây đúng với dịch ống thận lọc qua màng nội chất (lumen) Commented [LTN22]: Nhận định

ở đầu ống lượn xa tại vùng phức hợp cạnh cầu thận? Commented [LTN23]: đi qua lòng

A) Thường đẳng trương


B) Thường nhược trương
C) Thường ưu trương
D) Nhược trương hơn nếu sử dụng chống bài niệu
E) Ưu trương hơn khi lưu lượng lọc ở nephron đó giảm xuống 50% so với bình
thường
 B) Dịch đi vào đầu gần ống lượn xa luôn nhược trương do Na+ và các ions khác
được vận chuyển tích cực qua thành đoạn lên quai Henle, trong khi đoạn này của
nephron hầu như không thấm nước. Do đó, thành đoạn lên quai Henle và đầu gần
ống lượn xa thường được gọi là đoạn pha loãng
TMP12 330–331
68. Ở bệnh nhân có mức vận chuyển glucose tối đa là 350 mg/phút, lưu lượng lọc
cầu thận GFR là 100 ml/phút, nồng độ glucose huyết tương là 150 mg/dL, lưu
lượng nước tiểu là 2 ml/phút, và không phát hiện glucose trong nước tiểu, tốc độ
tái hấp thu glucose là bao nhiêu, giả sử trong trường hợp chức năng thận bình Commented [LTN24]: []

thường?
A) Tái hấp thu glucose không thể tính được với các dữ liệu trên
B) 0 mg/phút
C) 50 mg/phút
D) 150 mg/phút
E) 350 mg/phút
 D) Trong ví dụ này, tải lượng lọc glucose = GFR (100 ml/phút) × glucose huyết
tương (150 mg/dL), hoặc 150 mg/phút. Nếu không tìm thấy glucose trong nước
tiểu, tốc độ tái hấp thu bằng tải lượng lọc glucose và bằng 150 mg/phút.
TMP12 340
69. Một bệnh nhân đến khám vì khát nhiều và hơi thở có mùi acetone. Chẩn đoán
xác định bệnh nhân mắc ĐTĐ thông qua xét nghiệm nước tiểu dương tính với
glucose và xét nghiệm nồng độ đường máu lúc đói bằng 400 mg/dL. Những thay
đổi nào trong nước tiểu dưới đây xảy ra trên bệnh nhân?

.D) Bệnh nhân có các triệu chứng điển hình của bệnh ĐTĐ: khát nhiều, hơi thở Commented [LTN25]: kinh điển

mùi acetone (do tăng acid acetoncetic trong máu), tăng nồng độ glucose máu lúc
đói và có glucose trong nước tiểu. Acetoacetic acids trong máu gây nên toan
chuyển hóa dẫn đến bù bằng cách giảm bài tiết HCO3− qua thận, giảm pH nước Commented [LTN26]: giảm

tiểu và tăng sản xuất NH3 và HCO3− qua thận. Nồng độ đường máu cao làm tăng
tải lượng lọc glucose, từ đó làm tăng quá mức mức độ vận chuyển tối đa glucose,
gây lợi niệu thẩm thấu ( tăng thể tích nước tiểu) do glucose không được tái hấp thu
trong ống thận hoạt động như một tác nhân gây lợi niệu thẩm thấu
TMP12 326–327, 391–392
70. Khẳng định nào sau đây đúng?
A) Tái hấp thu ure ở trong lòng ống góp ít hơn ở ống lượn xa khi dùng chống bài Commented [LTN27]: []

niệu
B) Nồng độ ure trong dịch kẽ ở vùng vỏ thận lớn hơn trong dịch kẽ ở vùng tủy thận
khi sử dụng chống bài niệu
C) Thành của đoạn lên quai Henle tái hấp thu ure nhiều hơn tại vùng lõi của ống
thận khi sử dụng chống bài niệu Commented [LTN28]: ống góp ở vùng tủy trong

D) Tái hấp thu ure ở ống lượn gần lớn hơn tại ống góp
 D) Khoảng 40% tới 50% urea được tái hấp thu ở ống lượn gần. Ống lượn xa và Commented [LTN29]: đã lọc

ống góp không cho ure đi qua, thậm chí kể cả sử dụng chống bài niệu, do đó, rất ít Commented [LTN30]: cả khi
ure được tái hấp thu trong các đoạn này. Tương tự như vậy, rất ít ure được tái hấp
thu ở thành đoạn lên quai Henle. Khi sử dụng chống bài niệu, nồng độ ure ở dịch
kẽ vùng tủy thận tăng lên đáng kể do tái hấp thu ure từ ống góp, từ đó góp phần
làm tăng áp suất thẩm thấu vùng tủy thận
TMP12 350–351
71. Nam giới khỏe mạnh 29 tuổi chạy bền 10 km trong thời tiết nắng nóng và bị Commented [LTN31]: Một người

mất nước. Giả sử rằng nồng độ hormone chống bài niệu ở bệnh nhân rất cao, nước
được tái hấp thu nhiều nhất ở đoạn nào trong ống thận?
A) Ống lượn gần
B) Quai Henle
C) Ống lượn xa
D) Vùng vỏ ống góp Commented [LTN32]: Ống góp ở vùng vỏ

E) Vùng tủy ống góp Commented [LTN33]: Ống góp ở vùng tủy

 A) Trong điều kiện bình thường cũng như khi sử dụng chống bài niệu, hầu hết
nước được tái hấp thu ở ống lượn gần (khoảng 60% tới 65%). Mặc dù tình trạng
mất nước làm tăng thấm nước đáng kể ở vùng vỏ và tủy ống góp, các đoạn này Commented [LTN34]: ống góp vùng vỏ và vùng tủy

vẫn tái hấp thu một tỉ lệ nhỏ (nhưng quan trọng) nước.
TMP12 352–353
Câu hỏi 72–74
Bệnh nhân có trọng lượng khối cơ thể bình thường 60 kg và thể tích dịch ngoại bào
khoảng 12.8 L, thể tích máu 4.3 L, hematocrit bằng 0.4; 57% trọng lượng cơ thể là
nước.
Trả lời 3 câu hỏi dưới đây dựa trên những thông tin đã cho
72. Thể tích dịch nội bào khoảng bao nhiêu?
A) 17.1 L
B) 19.6 L
C) 21.4 L
D) 23.5 L
E) 25.6 L
 C) Thể tích dịch nội bào được tính bằng hiệu của tổng lượng dịch cơ thể (0.57 ×
60
kilograms = 34.2 kilograms, hoặc khoảng 34.2 L) và thể tích dịch ngoại bào (12.8
L), được 21.4 L.
TMP12 289–290
73. Thể tích huyết tương khoảng bao nhiêu?
A) 2.0 L
B) 2.3 L
C) 2.6 L
D) 3.0 L
E) 3.3 L
.C) Thể tích huyết tương được tính = thể tích máu (4.3 L) × (1.0 − hematocrit), =
4.3 × 0.6 =2.58 L (làm tròn bằng 2.6).
TMP12 289–290
74. Thể tích dịch kẽ khoảng bao nhiêu?
A) 6.4 L
B) 8.4 L
C) 10.2 L
D) 11.3 L
E) 12.0 L
 C) Thể tích dịch kẽ được tính = hiệu của thể tích dịch ngoại bào (12.8 L) và thể
tích huyết tương (2.6 L), và bằng 10.2 L.
TMP12 289–290
75. Đoạn nephron nào là vị trí tái hấp thu magie chủ yếu trong điều kiện bình
thường?
A) Ống lượn gần
B) Đoạn xuống quai Henle
C) Đoạn lên quai Henle
D) Ống lượn xa
E) Ống góp
 C) Vị trí tái hấp thu magie chủ yếu ở quai Henle, ở đó khoảng 65% tải lượng
lọc của magie được tái hấp thu. Ống lượn gần thông thường chỉ tái hấp thu khoảng
25% magie, còn ống lượn xa và ống góp tái hấp thu < 5%.
TMP12 369–370
76. Thay đổi nào dưới đây có thể thấy ở trẻ 10 tuổi mới được chẩn đoán ĐTĐ type
I và tăng đường máu không được kiểm soát
(glucose máu =300 mg/dL).

 C) Nồng độ glucose huyết tương bằng 300 mg/dL có thể làm tăng tải lượng lọc
glucose/khả năng vận chuyển tối đa qua ống thận, do đó làm tăng bài tiết glucose
qua nước tiểu. Glucose không được tái hấp thu ở ống thận gây lợi niệu thẩm thấu,
làm tăng thể tích nước tiểu và làm giảm thể tích dịch ngoại bào, từ đó kích thích
tăng khát. Tăng glucose cũng gây ra giãn tiểu động mạch đến, làm tăng GRF
TMP12 321, 327, 358
Câu hỏi 77 và 78
Để đánh giá chức năng thận ở bệnh nhân nữ 45 tuổi mắc ĐTĐ type II cần đo nước
tiểu 24h. Lượng nước tiểu 24h của bệnh nhân là 3600 ml. Xét nghiệm máu và nước
tiểu cho kết quả:
Creatinine huyết tương = 4 mg/dL,
Creatinine nước tiểu = 32 mg/dL, K+ huyết tương = 5 mmol/L, K+ nước tiểu= 10
mmol/L.
77. Lưu lượng lọc cầu thận GFR của bệnh nhân khoảng bao nhiêu, giả sử bệnh
nhân đo chính xác lượng nước tiểu 24h?
A) 10 ml/phút
B) 20 ml/phút
C) 30 ml/phút
D) 40 ml/phút
 B) GFR khoảng bằng độ thanh thải creatinine. Độ thanh thải creatinine = nồng
độ creatinine nước tiểu (32 mg/dL) × lưu lượng nước tiểu (3600 ml/24 h, hoặc 2.5
ml/phút) ÷ nồng độ creatinine
huyết tương (4 mg/dL) = 20 ml/phút.
TMP12 340–341
78. Tốc độ tái hấp thu Kali ở hệ thống ống thận ở bệnh nhân là bao nhiêu?
A) 1.050 mmol/phút
B) 0.100 mmol/phút
C) 0.037 mmol/phút
D) 0.075 mmol/phút
E) Kali không được tái hấp thu trong trường hợp này
 D) Tốc độ tái hấp thu trong hệ thống ống thận = hiệu tải lượng lọc của Kali
(GFR × nồng độ Kali huyết tương) – bài tiết Kali qua nước tiểu (Nồng độ kali
trong nước tiểu × lưu lượng nước tiểu). Do đó, tái hấp thu Kali ở hệ thống ống thận
là 0.075 mmol/phút.
TMP12 340–343
Câu hỏi 79–83
Mỗi trường hợp bệnh nhân trong các câu hỏi 79-83 tương ứng với kết quả khí máu
nào dưới đây (cùng một kết quả có thể sử dụng cho nhiều bệnh nhân)

79. Bệnh nhân mắc tiêu chảy nặng


 D) Tiêu chảy nặng có thể gây ra mất HCO3− qua phân, do đó gây toan chuyển
hóa đặc trưng bởi HCO3− thấp và pH thấp. Bù qua hô hấp có thể làm giảm Pco2.
Khoảng trống anion huyết tương có thể bình thường, và nồng độ Cl- huyết tương
có thể tăng (toan chuyển hóa có tăng Cl-) trong toan chuyển hóa gây ra do mất
HCO3− qua phân.
TMP12 392, 394–395
80. Bệnh nhân mắc cường aldosterone tiên phát
 E) Tăng tiết aldosterone quá mức nguyên phát gây ra kiềm chuyển hóa do làm
tăng tiết ion H+ và tái hấp thu HCO3− ở ống góp. Do đó, kiềm chuyển hóa có thể Commented [qp35]: Tế bào kẽ

đi kèm với tăng pH máu và HCO3−, bù qua hô hấp bằng cách giảm tần số thở và
tăng Pco2. Khoảng trống anion huyết tương có thể bình thường, giảm nhẹ Cl-
huyết tương.
TMP12 393–395
81. Bệnh nhân có toan hóa ống lượn gần ở thận
 D) Toan hóa ở ống lượn xa gây ra do thiếu hụt tiết H+ qua thận, tái hấp thu
bicarbonate, hoặc cả hai, từ đó làm tăng bài tiết HCO3− qua thận và gây toan
chuyển hóa đặc trưng bởi nồng độ HCO3− , pH huyết thanh thấp, còn bù qua hô Commented [qp36]: Thấp

hấp bằng cách tăng tần số thở và Pco2 thấp, khoảng trống anion bình thường và Commented [qp37]: Cơ chế
tăng nồng độ Cl-
TMP12 392, 395
82. Bệnh nhân mắc toan ceton do ĐTĐ và khí phế thũng
 F) Bệnh nhân nhiễm toan ceton do ĐTĐ và khí phế thũng có thể có toan chuyển
hóa (do tăng quá mức
ketoacids trong máu do ĐTĐ) cũng như làm tăng Pco2 do suy giảm chức năng hô
hấp. Do đó, bệnh nhân có tình trạng giảm pH huyết tương, giảm HCO3−, tăng
Pco2, và tăng khoảng trống anion (Na+-Cl−-HCO3
− > 10−12 mEq/L) do tăng ketoacids trong máu.
TMP12 393–394
83. Bệnh nhân được điều trị kéo dài bằng thuốc ức chế enzym carbonic anhydrase
(CA)
 D) Bài tiết ion H+ và tái hấp thu HCO3− phụ thuộc chủ yếu vào sự có mặt của
enzyme CA trong ống thận. Sau khi ức chế enzyme CA, tiết ion H+ trong ống thận
và tái hấp thu HCO3− sẽ giảm, dẫn tới làm tăng bài tiết HCO3−,làm giảm nồng độ
HCO3− huyết tương, và gây toan chuyển hóa, từ đó kích thích hô hấp và làm giảm
Pco2. Khoảng trống anion huyết tương thường trong giới hạn bình thường.

TMP12 386–387, 394–395


84. Những thay đổi nào dưới đây có thể thấy ở bệnh nhân mắc suy thận cấp sau khi
ngộ độc nấm độc gây ra hoạt tử ống thận?
A) Tăng nồng độ bicarbonate huyết tương
B) Toan chuyển hóa
C) Giảm nồng độ Kali huyết tương
D) Giảm nồng độ urea nitrogen máu (BUN)
E) Giảm áp suất thủy tĩnh trong bao Bowman
 B) Suy thận cấp gây ra do hoại tử ống thận có thể gây ra toan chuyển hóa do Commented [qp38]: Nhanh chóng

suy thận làm giải phóng các acid thừa - sản phẩm sau chuyển hóa. Toan chuyển Commented [qp39]: 0
hóa có thể dẫn tới giảm nồng độ HCO3− huyết tương. Suy thận cấp cũng dẫn tới Commented [qp40]: Mất khả năng
tăng nhanh nồng độ urea nitrogen trong máu và tăng đáng kể nồng độ Kali huyết
tương do suy thận làm bài tiết điện giải hoặc các chất thải chứa nito. Hoại tử tế bào Commented [qp41]: Thận mất chức năng

biểu mô thận làm các tế bào đó bị tróc ra từ màng đáy và bịt vào ống thận, do đó
tăng áp suất thủy tĩnh trong bao Bowman và làm giảm lưu lượng lọc cầu thận GFR.
TMP12 401, 407
85. Một bệnh nhân cao tuổi đến khám do yếu cơ và lơ mơ. Xét nghiệm nước tiểu
cho thấy nồng độ Na+ là 600 mmol/L và áp suất thẩm thấu là 1200 mOsm/L. Các
xét nghiệm khác cho thấy: nồng độ Na+ huyết tương = 167 mmol/L, hoạt độ renin
huyết tương = 4 ng angiotensin
I/ml/h (bình thường = 1), nồng độ hormone chống bài niệu (ADH) = 60 pg/ml
(bình thường = 3 pg/ml), nồng độ aldosterone huyết tương
= 15 ng/dL (bình thường = 6 ng/dL). Nguyên nhân nào sau đây gây ra tăng Natri
máu trên bệnh nhân?
A) Mất nước gây ra do giảm lượng nước đầu vào
B) Hội chứng tăng tiết ADH bất thường SIADH
C) Đái tháo nhạt do thần kinh (Đái tháo nhạt trung ương)
D) Tăng tiết aldosterone tiên phát
E) U tăng tiết renin
 A) Trong ví dụ này, nồng độ Natri trong huyết tương tăng đáng kể, nhưng nồng
độ Natri trong nước tiểu tương đối bình thường, và áp suất thẩm thấu trong nước
tiểu hầu hết tăng tối đa tới 1200 mOsm/L. Thêm vào đó có sự tăng tiết renin huyết
tương, ADH, aldosterone, phù hợp với tình trạng mất nước gây ra do giảm lượng
nước đầu vào. Hội chứng tăng tiết ADH bất thường SIADH gây ra giảm nồng độ
Natri trong huyết tương, cũng như ngăn chặn tiết renin và aldosteron. Đái tháo nhạt
trung ương (do thần kinh) gây ra do suy thận gây đáp ứng với ADH, có thể đi kèm
với mất nước nhưng áp suất thẩm thấy nước tiểu thường giảm nhiều hơn là tăng.
Cường aldosterone tiên phát có xu hướng làm gây giữ nước và Natri và làm giảm
đáng kể tiết renin. Tương tự như vậy, u tăng tiết renin đi kèm với tăng nồng độ
aldosterone huyết tương và hoạt độ renin trong huyết tương, nhưng chỉ gây ra thay
đổi vừa phải nồng độ Natri huyết tương. TMP12 354, 359–360
86. Một bệnh nhân đến khám do khát nhiều và hơi thở có mùi acetone . Chẩn đoán
sơ bộ ở bệnh nhân là ĐTĐ và khẳng định qua kết quả xét nghiệm nước tiểu cho
thấy glucose dương tính và xét nghiệm máu cho thấy nồng độ glucose lúc đói là
400 mg/dL. Thay đổi nào sau đây xảy ra trong huyết tương của bệnh nhân khác với
bình thường?

 B) Bệnh nhân mắc ĐTĐ có tình trạng toan chuyển hóa đặc trưng bởi nồng độ
HCO3− thấp, giảm pH huyết tương, và tăng khoảng trống anion. Tình trạng toan
hóa kích thích hô hấp, gây ra bù bằng cách giảm Pco2 huyết tương.
TMP12 391, 395
87. Thông số nào dưới đây có giá trị tương đương ở cả dịch trong tế bào và ngoài
khoảng kẽ?
A) Nồng độ K+
B) Áp suất keo thẩm thấu
C) Nồng độ Na+
D) Nồng độ Cl-
E) Áp suất thẩm thấu tổng cộng
 E) Dịch nội bào và dịch ngoại bào có áp suất thẩm thấu tổng cộng như nhau
trong điều kiện bình thường do màng tế bào thấm nước tốt. Do đó nước dịch
chuyển nhanh qua màng tế bào cho đến khi đạt được cân bằng áp suất thẩm thấu.
Áp suất keo thẩm thấu được quyết định bởi nồng độ protein với nồng độ protein
thường cao hơn trong tế bào. Màng tế bào tương đối không thấm Natri, Kali và
Clo, các cơ chế vận chuyển tích cực giúp duy trì nồng độ Natri, Clo thấp hơn và
nồng độ Kali cao hơn trong nội bào.
TMP12 290–292
88. Ở bệnh nhân có tăng nồng độ aldosterone và các chức năng khác của thận đều
bình thường, khoảng bao nhiều phần trăm tải lượng lọc Natri được tái hấp thu ở
ống lượn xa và ống góp?
A) Hơn 66%
B) 40% tới 60%
C) 20% tới 40%
D) 10% tới 20%
E) Ít hơn 10%
 E) Mặc dù aldosteron là một trong những hormone quan trọng nhất gây giữ
Natri, nó chỉ kích thích tái hấp thu Narti ở đoạn cuối ống lượn xa và ống góp, tổng
cộng gây tái hấp thu nhỏ hơn 10% tải lượng lọc của Natri. Do đó phần trăm tải
lượng lọc tối đa của Natri có thể tái hấp thu ở ống lượn xa và ống góp, kể cả khi có
mặt aldosterone nồng độ cao là ít hơn 10%
TMP12 331–333, 337–338
89. Khẳng định nào sau đây là đúng?
A) Hormon chống bài niệu (ADH) làm tăng tái hấp thu nước từ đoạn lên quai
Henle
B) Tái hấp thu nước ở đoạn xuống quai Henle thường ít hơn đoạn lên quai Henle
C) Tái hấp thu Natri ở đoạn xuống quai Henle thường ít hơn đoạn lên quai Henle
D) Áp suất thẩm thấu ở đầu gần ống lượn xa thấp hơn 300 mOsm/L ở bệnh nhân
mất nước có chức năng thận bình thường và tăng nồng độ ADH
E) ADH làm giảm tính thấm ure ở phần tủy ống góp
 D) Ở bệnh nhân mất nước, áp suất thẩm thấu ở đoạn gần ống lượn xa thường
thấp hơn 300 mOsm/L do đoạn lên của quai Henle và đoạn gần của ống lượn xa
tương đối không thấm nước, kể cả khi có mặt ADH. Do đó, dịch trong ống thận
dần dần được pha loãng hơn trong các đoạn này so với huyết ương. ADH không
ảnh hưởng tái hấp thu nước ở đoạn lên quai Henle. Tuy nhiên đoạn lên của quai
Henle tái hấp thu Natri nhiều hơn nhiều so với ở đoạn xuống. Cơ chế hoạt động
quan trọng khác của ADH là làm tăng tính thấm ure ở ống góp, từ đó góp phần làm
tăng áp suất thẩm thấu trong dịch kẽ ở thận trong trường hợp sử dụng chống bài
niệu. TMP12 352–353
90. Những thay đổi nào dưới đây làm tăng bài tiết Ca++ qua nước tiểu?
A) Tăng thể tích dịch ngoại bào
B) Tăng nồng độ hormone cận giáp
C) Giảm huyết áp
D) Tăng nồng độ phosphate huyết tương
E) Toan chuyển hóa
.A) Ở ống lượn gần, tái hấp thu Calci thường đi kèm với tái hấp thu nước và
Natri. Khi tăng thể tích dịch ngoại bào hoặc tăng huyết áp, tái hấp thu Natri và
nước ở ống lượn gần bị giảm, và cũng giảm tái hấp thu Calci, gây ra tăng bài tiết
calci qua thận. Tăng hormone cận giáp, tăng nồng độ phosphate huyết tương và
toan chuyển hóa đều có thể làm giảm bài tiết calci qua thận. TMP12 367–369
91. Bệnh nhân còn lại một bên thận mắc hẹp động mạch thận mức độ trung bình
làm giảm áp lực động mạch thận xuống 85 mm Hg, so với mức bình thường là 100
mm Hg. Trị số nào sau đây có thể giảm sau 2 tuần xảy ra hẹp động mạch thận ở
bệnh nhân, giả sử rằng chế độ ăn của bệnh nhân không thay đổi?
A) Sức cản tiểu động mạch đi
B) Sức cản tiểu động mạch đến
C) Bài tiết renin
D) Tốc độ bài tiết Natri
E) Nồng độ aldosterone huyết tương
 B) Tình trạng hẹp động mạch thận mức độ trung bình làm giảm áp lực máu
động mạch thận xuống
85 mm Hg sẽ gây ra phản ứng tự điều hòa làm giảm sức cản tiểu động mạch đến.
Giảm áp lực tưới máu thận sẽ kích thích tiết renin, từ đó làm tăng hình thành
angiotensin II và gây co tiểu động mạch đến. Tăng hình thành angiotensin II cũng
làm tăng tiết aldosterone.
TMP12 320
92. Những thay đổi nào dưới đây có thể thấy ở bệnh nhân thực hiện chế độ ăn giàu
Natri (200 mEq/ngày) so với bệnh nhân thực hiện chế độ ăn với lượng Natri bình
thường (100 mEq/ngày), giả sử rằng bệnh nhân trong tình trạng ổn định?
A) Tăng nồng độ aldosterone huyết tương
B) Tăng bài tiết Kali qua nước tiểu
C) Giảm hoạt độ renin huyết tương
D) Giảm peptide natri lợi niệu tâm nhĩ (ANP) huyết tương
E) Tăng nồng độ Natri trong huyết tương ít nhất 5 mmol/L
 C) Tăng lượng Natri đưa vào có thể làm giảm tiết renin và hoạt độ renin trong
huyết tương cũng như làm giảm nồng độ aldosteron huyết tương và làm tăng
peptide natri lợi niệu tâm nhĩ ANP phụ thuộc vào tăng mức độ trung bình thể tích
dịch ngoại bào. Mặc dù lượng Natri đưa vào cao có thể ban đầu làm tăng vận
chuyển NaCl ở ống lượn xa, từ đó tăng bài tiết Kali, tình trạng giảm nồng độ
aldosterone có thể bù lại ảnh hưởng này, do đó không có thay đổi về bài tiết Kali
trong điều kiện bình thường ổn định. Kể cả khi tăng rất cao lượng natri đưa vào
cũng chỉ gây ra thay đổi rất nhỏ trong nồng độ Natri huyết tương cũng như
hormone chống bài niệu – cơ chế khát được vận hành hoàn toàn.
TMP12 366–367
93. Một công nhân xây dựng 26 tuổi được đưa vào khoa cấp cứu trong tình trạng
rối loạn ý thức sau khi liên tục làm việc 10 tiếng dưới thời tiết mùa hè nắng nóng
(nhiệt độ trung bình là 97°F – khoảng 37 độ C). Bệnh nhân tiết mồ hôi đầm đìa
trong cả ngày nhưng không uống nước. Bệnh nhân sốt 102°F (khoảng 39 độ C),
nhịp tim là 140 lần/phút, huyết áp là 100/55 mm Hg ở tư thế nắm. Khám thấy bệnh
nhân không tiết mồ hôi, xuất hiện niêm mạch khô, định hướng kém về không gian,
thời gian, mọi người xung quanh. Giả sử rằng chức năng thận của bệnh nhân bình
thường, những thay đổi nào về nồng độ các hormone dưới đây xảy ra trên bệnh
nhân?
A) Hormone chống bài niệu (ADH) tăng, renin tăng, angiotensin II giảm,
aldosterone giảm
B) ADH giảm, renin giảm, angiotensin II giảm, aldosterone giảm
C) ADH tăng, renin giảm, angiotensin II tăng, aldosterone giảm
D) ADH tăng, renin tăng, angiotensin II tăng, aldosterone tăng
E) ADH giảm, renin tăng, angiotensin II giảm, aldosterone tăng
 D) Bệnh nhân này bị mất nước nặng do tiết mồ hôi nhiều và thiếu lượng dịch
đưa vào phù hợp. Sự mất nước kích thích đáng kể việc giải phóng ADH và tiết
renin, từ đó kích thích hình thành angiotensin II và tiết aldosterone
TMP12 337–338, 355–356
94. Toan chuyển hóa cấp có thể _____ đối với nồng độ K+ nội bào và _____ đối
với khả năng tiết K+
tại vùng vỏ ống góp.
A) Làm tăng, Làm tăng
B) Làm tăng, Làm giảm
C) Làm giảm, Làm tăng
D) Làm giảm, Làm giảm
E) Không làm thay đổi, Làm tăng
F) Không làm thay đổi, Không làm thay đổi
.D) Toan chuyển hóa cấp làm giảm nồng độ Kali nội bào, từ đó làm giảm tiết
Kali ở các tế bào chính trên ống góp. Cơ chế chính làm giảm nồng độ ion H+ ngăn
chặn tiết Kali là làm giảm hoạt động của bơm Natri-Kali ATPase, từ đó làm giảm
nồng độ Kali nội bào và làm giảm tốc độ khuếch tán thụ động Kali qua màng tế
bào vào trong ống thận.
TMP12 367
95. Phần nào sau đây của ống thận có áp suất thẩm thấu thấp nhất ở bệnh nhân
thiếu hụt hoàn toàn hormone chống bài niệu da đái tháo nhạt trung ương (do thần
kinh)?
A) Phần tủy ống góp
B) Ống góp
C) Đoạn gần ống lượn xa
D) Đoạn xuống quai Henle
E) Ống lượn gần
 A) Do dịch pha loãng ở đầu gần ống lượn xa cho đến đoạn cuối ống lượn xa và
ống góp nên có sự tái hấp thu thêm NaCl. Khi thiếu hụt hoàn toàn hormone chống
bài niệu ADH, thành của ống thận tương đối không thấm nước và tái hấp thu thêm
các chất tan làm dịch ống thận trở nên được pha loãng hơn cả ở các đoạn pha loãng
dịch (đầu gần ống lượn xa), làm giảm ấp suất thẩm thấu xuống thấp tới
50 mOsm/L. Do đó, ở bệnh nhân thiếu hụt hoàn toàn ADH, phần của ống thận có
áp suất thẩm thấu thấp nhất là phần tủy của ống góp. TMP12 352–353, Figure 28-8
96. Nếu lưu lượng lọc cầu thận GFR đột ngột giảm từ 150 ml/phút xuống 75
ml/phút và tái hấp thu dịch ống thận đồng thời giảm xuống từ 149 ml/phút xuống
75 ml/phút, thay đổi nào dưới đây có thể thấy trên bệnh nhân (Giả sử rằng thay đổi
GFR và tái hấp thu dịch ống thận được duy trì)
A) Lưu lượng nước tiểu giảm xuống 0
B) Lưu lượng nước tiểu giảm xuống 50%
C) Lưu lượng nước tiểu không thay đổi
D) Lưu lượng nước tiểu tăng thêm 50%
 A) Lưu lượng nước tiểu được tính bằng hiệu của GFR và tốc độ tái hấp thu dịch
của ống thận. Nếu GFR giảm từ 150 xuống 75 ml/phút và tốc độ tái hấp thu dịch
trong ống thận giảm đồng thời từ 149 xuống
75 ml/phút,lưu lượng nước tiểu bằng GFR – tốc độ tái hấp thu ống thận, bằng 75–
75 ml/phút, bằng 0 ml/phút.
TMP12 323, 340–341
97. Cho các số đo dưới đây, tính phân số lọc: áp suất thủy tĩnh mao mạch cầu thận
= 50 mm Hg; áp suất thủy tĩnh bao Bowman = 15 mm Hg; áp suất keo thẩm thấu ở
mao mạch cầu thận = 30 mm Hg; hệ số lọc mao mạch cầu thận (Kf) = 12
ml/phút/mm Hg; và lưu lượng máu qua thận = 400 ml/phút.
A) 0.15
B) 0.20
C) 0.25
D) 0.30
E) 0.35
F) 0.40
 A) Phân số lọc được tính bằng GFR chia cho lượng máu qua thận. GFR bằng áp
suất lọc hệ thống (áp suất thủy tĩnh mao mạch cầu thận – áp suất thủy tĩnh bao
Bowman – áp suất keo thẩm thấu mao mạch cầu thận) nhân với hệ số lọc mao
mạch thận. Do đó, phân số lọc bằng GFR (60 ml/phút) chia cho lượng máu qua
thận (400 ml/phút), và bằng 0.15.
TMP12 314, 342
98. Bệnh nhân nam 55 tuổi thừa cân đến khám do tiểu nhiều và huyết áp của bệnh
nhân là 165/98 mm Hg. Dựa vào độ thanh thải creatine 24h đánh giá được GFR
của bệnh nhân là 150 ml/phút. Glucose máu của bệnh nhân là
400 mg/100 ml. Giả sử rằng khả năng vận chuyển tối đa của glucose qua thận là
bình thường, được cho trong biểu đồ, tốc độ bài tiết glucose qua nước tiểu ở bệnh
nhân khoảng bao nhiêu?
A) 0 mg/phút
B) 100 mg/phút
C) 150 mg/phút
D) 225 mg/ phút
E) 300 mg/ phút
F) Thông tin đưa ra không đầy đủ để đánh giá tốc độ bài tiết glucose
 E) Thận hầu như không bài tiết glucose miễn là tải lượng lọc glucose (tích của
GFR và nồng độ glucose huyết tương) không vượt quá khả năng vận chuyển tối đa
của glucose tại ống thận.
Khi tải lượng lọc của glucose vượt qua khả năng vận chuyển glucose, số lượng
đường vượng quá không được tái hấp thu và đi vào trong nước tiểu. Do đó tốc độ
bài tiết glucose qua nước tiểu có thể được tính bằng tải lượng lọc glucose trừ đi
khả năng vận chuyển tối đa. Trong ví dụ này, tải lượng lọc glucose là lưu lượng
lọc cầu thận GFR(150 ml/ phút) nhân với nồng độ glucose huyết tương (400
mg/100 ml, hoặc 4 mg/ml), và bằng 600 mg/phút. Do khả năng vận chuyển tối đa
chỉ bằng 300 mg/phút, tốc độ bài tiết glucose bằng 600-300mg/phút, và bằng 300
mg/phút.
TMP12 326–327, 340–342

99. Theo dõi bệnh nhân ĐTĐ type 2 và bệnh thận mạn thấy GFR giảm từ 80
ml/phút xuống 40 ml/phút trong 4 năm qua. Những thay đổi nào dưới đây có thể
thấy ở bệnh nhân so với 4 năm trước đó trước khi giảm GFR, giả sử bệnh nhân
trong tính trạng bình thường và không có thay đổi về lượng điện giải đưa vào hay
khả năng chuyển hóa protein?

 E) Giảm 50% lưu lượng lọc cầu thận GFR (từ 80 xuống 40 ml/phút) có thể gây
nên giảm khoảng 50% độ thanh thải creatinin, do độ thanh thải creatinin xấp xỉ
GFR. Từ đó dẫn đến tăng gấp đôi nồng độ creatinin huyết tương. Tăng nồng độ
creatinin huyết tương ban đầu là do giảm tốc độ bài tiết creatinin, nhưng khi nồng
độ creatinin huyết tương tăng, tải lượng lọc creatinin (tích của GFR× nồng độ
creatine huyết tương) và tốc độ bài tiết creatine sẽ trở về bình thường trong tình
trạng ổn định. Do đó, trong tình trạng ổn định, giảm 50% GFR đi liền với tăng gấp
đôi nồng độ creatine huyết tương, giảm 50% độ thanh thải creatine, tải lượng lọc
creatine bình thường cũng như tốc độ bài tiết creatine không thay đổi miễn là
chuyển hóa protein của bệnh nhân không thay đổi. Tương tự như vậy, tốc độ bài
tiết Natri cũng trở về bình thường kể cả khi GFR giảm do một loạt hệ thống
feedback giúp tái lập cân bằng Natri. Trong tình trạng ổn định, bài tiết Natri phải
cân bằng với lượng Natri đưa vào cơ thể để duy trì sự sống.
TMP12 341, 370
100. Ở bệnh nhân có chế độ ăn giàu Kali (200 mmol/ngày), đoạn nào của nephron
sẽ tiết nhiều Kali nhất?
A) Ống lượn gần
B) Đoạn xuống quai Henle
C) Đoạn lên quai Henle
D) Đầu gần ống lượn xa
E) Ống góp
 E) Hầu hết tiết Kali xảy ra ở ống góp. Chế độ ăn giàu Kali kích thích tiết Kali ở
ống góp qua một loạt cơ chế, bào gồm tăng nhẹ nồng độ Kali nội bào cũng như
tăng nồng độ aldosterone.
TMP12 364–365
101. Trường hợp nào dưới đây có thể làm giảm nồng độ Kali huyết tương bằng
cách gây ra vận chuyển Kali từ ngoại bào vào trong tế bào?
A) Tập cường độ cao kéo dài
B) Thiếu hụt aldosterone
C) Toan hóa
D) Phong tỏa Beta adrenergic
E) Tăng tiết insulin quá mức
 E) Tăng nồng độ insulin gây ra sự dịch chuyển Kali từ ngoại bài vào trong tế
bào. Tất cả các điều kiện còn lại đều gây ra tác động ngược làm Kali dịch chuyển
ra ngoài tế bào ra ngoại bào.
TMP12 361–362
102. Bệnh nhân nam 23 tuổi chạy đường chạy 10 km vào tháng 7 và mất 2 L dịch
do tiết mồ hôi. Bệnh nhân cũng uống 2L nước trong suốt đường chạy. Những thay
đổi nào có thể xảy ra ở bệnh nhân này, sau khi bệnh nhân hấp thụ nước và giả sử
rằng áp suất thẩm thấu được cân bằng và không có bài tiết nước hay điện giải?

 E) Sau khi thực hiện được chạy và mất dịch cũng như điện giải, bệnh nhân này Commented [qp42]: 0

thay thế thể tích dịch bị mất bằng cách uống 2L nước. Tuy nhiên bệnh nhân không
bù điện giải. Do đó bệnh nhân có thể bị giảm nồng độ Natri huyêt tương gây ra
giảm áp suất thẩm thấu ở cả dịch ngoại bào và nội bào. Giảm áp suất thẩm thấu
dịch ngoại bào dẫn tới tăng thể tích dịch nội bào do dịch khuyếch tán vào trong tế
bào thông qua khoảng ngoại bào.
Do đó sau khi uống và hấp thụ nước, thể tích dịch toàn bộ cơ thể có thể bình
thường nhưng dịch nội bào tăng thể tích và thể tích dịch ngoại bào giảm.
TMP12 292
Câu hỏi 103–105
Kết quả xét nghiệm dưới đây cho thấy: lưu lượng nước tiểu = 2.0 ml/phút; nồng độ
inulin nước tiểu = 60 mg/ml; nồng độ inulin huyết tương = 2 mg/ml; nồng độ Kali
nước tiểu = 20 μmol/ml; nồng độ Kali huyết tương = 4.0 μmol/ml; áp suất thẩm
thấu nước tiểu = 150 mOsm/L; áp suất thẩm thấu huyết tương = 300mOsm/L.

103. GFR khoảng bao nhiêu?


A) 20 ml/phút
B) 25 ml/phút
C) 30 ml/phút
D) 60 ml/phút
E) 75 ml/phút
F) 150 ml/phút
 D) GFR bằng độ thanh thải inulin. Độ thanh thải inulin = nồng độ inulin nước
tiểu (60 mg/ml)× lưu lượng nước tiểu(2 ml/phút)/nồng độ inulin huyết tương (2
mg/ml) = 60 ml/phút.
TMP12 340–342
104.Tốc độ tái hấp thu Kali trong hệ thống bằng bao nhiêu?
A) 0 μmol/phút
B) 20 μmol/phút
C) 60 μmol/phút
D) 200 μmol/phút
E) 240 μmol/phút
F) 300 μmol/phút
.D) Tốc độ tái hấp thu Kali qua hệ thống ống thận bằng hiệu của tải lượng lọc
Kali (GFR × nồng độ Kali huyết tương) và tốc độ bài tiết Kali qua nước tiểu (nồng
độ Kali trong nước tiểu× lưu lượng nước tiểu). Do đó, tốc độ tái hấp thu Kali qua
hệ thống ống thận là 200 μmol/min.
TMP12 340–342
G) Kali không được tái hấp thu trong trường hợp này.
105. Tốc độ thanh thải nước tự do bằng bao nhiêu?
A) +1.0 ml/phút
B) +1.5 ml/phút
C) +2.0 ml/ phút
D) −1.0 ml/ phút
E) −1.5 ml/ phút
F) −2.0 ml/ phút
 A) Độ thanh thải nước tự do được tính bằng lưu luuwongj nước tiểu (2.0
ml/phút) – độ thanh thải thẩm thấu (áp suất thẩm thấu nước tiểu × lưu lương nước
tiểu/áp suất thẩm thấu huyết tương). Do đó, độ thanh thải nước tự do bằng +1.0
ml/phút.
TMP12 354
106. Một bệnh nhân có kết quả xét nghiệm dưới đây: pH máu động mạch = 7.04,
HCO3− huyết tương = 13 mEq/L, nồng độ Cl- huyết tương = 120 mEq/L, PCO2
động mạch = 30 mm Hg, nồng độ Natri huyết tương = 141mEq/L. Nguyên nhân
chính nào dưới đây gây ra tình trạng toan hóa trên bệnh nhân này?
A) Khí phế thũng
B) Ngộ độc methanol
C) Ngộ độc acid salicylic
D) Tiêu chảy cấp
E) Đái tháo đường
 D) Bệnh nhân có tình trạng toan chuyển hóa thể hiện qua giảm nồng độ HCO3−
huyết tương (bình thường= 24 mEq/L) và giảm PCO2 động mạch (bình thường
khoảng 40 mm Hg). Do khoảng trống anion
(Na+ huyết tương − HCO3− − Cl-) bình thường (khoảng 10 mEq/L), tình trạng
toan hóa không gây ra bởi tình trạng quá mức các acid không bay hơi do ngộ độc Commented [qp43]: Xảy ra trong trường hợp thừa

acid salicylic, đái tháo đường hay ngộ độc methanol. Do đó, nguyên nhân có thể
gây ra toan chuyển hóa là tiêu chảy cấp, dẫn đến mất
bicarbonate qua phân. Trong trường hợp khí phế thũng, toan chuyển hóa có thể đi
kèm với tăng PCO2.
TMP12 390–391, 395
107. Một bệnh nhân nam trẻ tuổi được phát hiện trong tình trạng hôn mê, đã uống
một lượng thuốc ngủ không rõ liều và thời gian sử dụng trước đó. Khí máu động
mạch cho thấy: pH = 7.02, HCO3− = 14 mEq/L, và PCO2 = 68 mm Hg. Tình trạng
rối loạn cân bằng acid-base ở bệnh nhân này là gì?
A) Toan chuyển hóa mất bù
B) Toan hô hấp mất bù
C) Đồng thời cả toan chuyển hóa và toan hô hấp
D) Toan hô hấp còn bù một phần qua thận
E) Toan hô hấp còn bù hoàn toàn qua thận
 C) Trong ví dụ, tình trạng toan hóa đi kèm với giảm nồng độ bicarbonate, biểu
thị tình trạng toan chuyển hóa. Hơn nữa, bệnh nhân có tăng PCO2 biểu thị toan hô
hấp. Do đó bệnh nhân này có đồng thời cả toan chuyển hóa và toan hô hấp
TMP12 391–394
108. Trong tình trạng toan hô hấp còn bù một phần qua thận, có thể thấy những
thay đổi như sau so với bình thường:___ bài tiết NH4+ qua thận; ______ nồng độ
HCO3−
huyết tương; và _____ pH nước tiểu.
A) tăng, tăng, giảm
B) tăng, giảm, giảm
C) không thay đổi, tăng, giảm
D) không thay đổi, không thay đổi, giảm
E) tăng, không thay đổi, tăng
 A) Toan hô hấp mãn tính gây ra do hạn chế thông khí phổi, dẫn đến tăng PCO2.
Từ đó toan hóa kích thích tiết H+ vào dịch ống thận và làm tăng sản xuất NH4+ tại
thận, góp phần giúp bài tiết H+ và sản xuất HCO3− qua thận, do đó làm tăng nồng
độ bicarbonate huyết tương. Tăng tiết H+ qua ống thận cũng làm giảm pH nước
tiểu. TMP12 391
109. Ở vị trí nào trên ống thận có nồng độ creatinine cao nhất trong tình trạng mất
nước ở người bình thường?
A) Nồng độ creatinine như nhau ở các vị trí ở ống thận, do creatine không được
tiết ra hay tái hấp thu.
B) Sau khi lọc qua cầu thận
C) Đoạn cuối ống lượn gần.
D) Đoạn cuối quai Henle
E) Ống lượn xa
F) Ống góp
 F) Do creatine không được tái hấp thu đáng kể ở ống thận, nồng độ creatine dần
dần tăng lên do nước được tái hấp thu dọc theo các đoạn ống thận. Do đó, trong
tình trạng mất nước xảy ra ở người bình thường, nồng độ creatine cao nhất ở ống Commented [qp44]: Hydrate hóa

góp.
TMP12 334, Figure 27-14

Câu hỏi 110 và 111


Biểu đồ dưới đây cho thấy các trường hợp có thể xảy ra khi mất nước bất thường. Commented [qp45]: Sự phân bố nước

Trong mỗi biểu đồ, đường thể hiện tình trạng bình thường (các đường nét liền)
được đặt chồng lên các đường thể hiện tình trạng bất thường của cơ thể (đường nét
đứt) nhằm minh họa sự dịch chuyển thể tích (chiều ngang của các hình chữ nhật)
và tổng áp suất thẩm thấu (chiều cao của các hình chữ nhật) giữa hai thành phần
dịch nội bào và dịch ngoại bào.
110. Biểu đồ nào cho thấy sự thay đổi (sau khi cân bằng áp suất thẩm thấu) ở thể
tích và áp suất thẩm thấu dịch ngoại bào và dịch nội bào sau khi truyền 2 L
dextrose 3.0%?
A)
B)
C)
D)
 B) Dextrose 3% là dịch nhược trương. Do đó, truyền dextrose 3% sẽ làm giảm
áp suất thẩm thấu dịch ngoại bào, từ đó dẫn đến đưa nước vào trong tế bào. Trong
tình trạng bình thường sẽ không có tình trạng giảm áp suất thẩm thấu dịch nội,
ngoại bào cũng như tăng thể tích dịch ở cả 2 khoang này. TMP12 292–294
111. Biểu đồ nào cho thấy sự thay đổi (sau khi cân bằng áp suất thẩm thấu) ở thể
tích và áp suất thẩm thấu dịch ngoại bào và dịch nội bào ở bệnh nhân mắc đái tháo
nhạt trung ương mức độ nặng?
A)
B)
C)
D)
 D) Ở bệnh nhân mắc đái tháo nhạt trung ương xảy ra tình trạng thiếu hụt tiết
hormone chống bài niệu, dẫn đến tăng bài tiết nước nhiều. Tình trạng đó gây ra
mất nước và tăng Natri máu. Tăng Natri máu có thể gây ra giảm thể tích dịch nội
bào. Do đó, mất nước ban đầu có thể làm tăng áp suất thẩm thấu cả dịch nội bào và
ngoại bào cũng như làm giảm thể tích dịch nội, ngoại bào.
TMP12 293–296
112. Ở bệnh nhân bị toan ceton do đái tháo đường mạn tính, những thay đổi nào
dưới đây có thể xảy ra?
A) Giảm bài tiết HCO3− qua thận, tăng bài tiết NH4+, tăng khoảng trống anion
huyết tương
B) Tăng nhịp thở, giảm PCO2 động mạch, giảm khoảng trống anion huyết tương
C) Tăng bài tiết NH4+, tăng khoảng trống anion huyết tương, tăng pH nước tiểu
D) Tăng sản xuất HCO3− qua thận, tăng bài tiết NH4+ , giảm khoảng trống anion
huyết tương
E) Giảm pH nước tiểu, giảm bài tiết HCO3− qua thận, tăng PCO2 động mạch
 A) Toan ceton do đái tháo đường dẫn đến tình trạng toan chuyển hóa đặc trưng
bởi tình trạng giảm nồng độ bicarbonate, tăng khoảng trống anion (do tăng lượng
anion không định lượng được đi kèm acid cetonic), và đáp ứng bù qua thận (làm
tăng tiết NH4+). Ngoài ra có tình trạng tăng nhịp thở và giảm PCO2 máu động
mạch, cũng như giảm pH nước tiểu và giảm bài tiết HCO3− qua thận.
TMP12 392–395
113. Tăng cả lưu lượng máu qua thận và lưu lượng lọc cầu thận GFR gây ra do tình
trạng nào dưới đây?
A) Giãn tiểu động mạch đến
B) Tăng hệ số lọc mao mạch cầu thận
C) Tăng áp suất thẩm thấu keo huyết tương
D) Giãn tiểu động mạch đi
E) Tăng độ nhớt máu do tăng thể tích khối hồng cầu hematocrit
 A) Giãn tiểu động mạch đến dẫn tới làm tăng áp suất thủy tĩnh mao mạch cầu
thận, do đó làm tăng GFR cũng như tăng lưu lượng máu qua thận. Tăng hệ số lọc
mao mạch cầu thận cũng làm tăng GFR nhưng không làm ảnh hưởng tới lưu lượng
máu qua thận. Tăng áp suất keo huyết tương hoặc giãn tiểu động mạch đi đều làm
giảm GRF. Tăng độ nhớt máu có thể làm giảm lưu lượng máu đến thận và GFR.
TMP12 314–317
114. Trong trường hợp nồng độ inulin trong dịch tại ống góp là 40 mg/100 ml và
nồng độ inulin huyết tương là 2.0 mg/100 ml, khoảng bao nhiêu phần trăm nước
được giữ lại tại ống góp?
A) 0%
B) 2%
C) 5%
D) 10%
E) 20%
F) 100%
 C) Do inulin không được tái hấp thu hay tiết ở ống thận, tăng nồng độ inulin
trong ống thận phản ánh tình trạng tái hấp thu nước. Do đó, tăng nồng độ inulin từ
mức 2
mg/100 ml trong huyết tương tới 40 mg/100 ml tại ống góp cho thấy nồng độ
inulin đã tăng gấp 20 lần. Hay nói cách khác, chỉ 1/20 (5%) nước trong dịch lọc
còn lại trong ống góp.
TMP12 334
115. Những thay đổi nào dưới đây xảy ra khi tăng tái hấp thu Ca2+ tại ống thận?
A) Tăng thể tích dịch ngoại bào
B) Tăng nồng độ hormone cận giáp trong huyết tương
C) Tăng huyết áp
D) Giảm nồng độ phosphate huyết tương
E) Kiềm chuyển hóa
 B) Tăng nồng độ hormone cận giáp kích thích tái hấp thu ở thành đoạn lên quai
Henle và ống lượn xa. Tăng thể tích dịch ngoại bào, tăng huyết áp, giảm nồng độ
phosphate huyết tương và kiềm chuyển hóa đều đi cùng với giảm tái hấp thu
calcium tại ống thận. TMP12 368–369
116. Thuốc lợi tiểu nào dưới đây có cơ chế tác dụng chính là ức chế đồng vận
chuyển Na+-2Cl−-K+ tại quai Henle?
A) Lợi tiểu thiazide
B) Furosemide
C) Ức chế men CA carbonic anhydrase
D) Lợi niệu thẩm thấu
E) Amiloride
F) Spironolactone
 B) Furosemide là tác nhân ức chế mạnh mẽ đồng vận chuyển Na+-2Cl−-K+ tại
quai Henle. Lợi tiểu thiazide chủ yếu ức chế tái hấp thu NaCl vào ống lượn xa,
trong khi các thuốc ức chế men CA carbonic
Andydrase làm giảm tái hấp thu bicarbonate tại ống thận. Amiloride ức chế hoạt
động kênh Natri trong khi spironolactone ức chế hoạt động của hormone điều hòa
thăng bằng điện giải tại ống thận. Lợi niệu thẩm thấu ức chế tái hấp thu nước và
điện giải bằng cách làm tăng áp suất thẩm thấu dịch ống thận.
TMP12 398
117. Bệnh nhân nam 55 tuổi bị huyết áp cao có huyết áp tương đối kiểm soát tốt Commented [qp46]: Được kiểm soát huyết áp tương đối
tốt
bằng lợi tiểu thiazide. Trong suốt 6 tháng trước đó, huyết áp của bệnh nhân là
130/75 mm Hg và nồng độ creatine huyết tương là 1 mg/100 ml. Bệnh nhân tập thể
dục đều đặn trong 2 năm trước đó, nhưng gần đây xuất hiện đau khớp gối nên bắt
đầu sử dụng số lượng lớn thuốc NSAIDS. Khi tới khám, huyết áp của bệnh nhân là
155/85 và creatine huyết tương là 2.5 mg/100 ml. Tăng creatine huyết tương ở
bệnh nhân gây ra do nguyên nhân chủ yếu nào dưới đây?
A) Tăng sức cản tiểu động mạch đi làm giảm GFR
B) Tăng sức cản tiểu động mạch đến làm giảm GFR
C) Tăng hệ số lọc mao mạch cầu thận làm giảm GFR
D) Tăng hình thành angiotensin II làm giảm GFR
E) Tăng khối lượng cơ do tập luyện
 B) Các thuốc NSAIDS ức chế tổng hợp prostaglandins, từ đó làm co tiểu động
mạch đến làm giảm GFR. Tình trạng giảm GFR làm tăng creatine huyết tương.
Tăng sức cản tiểu động mạch đi, và tăng hệ số lọc mao mạch cầu thận đều làm tăng
GFR. Tăng khối cơ do tập luyện gây ra thay đổi không đáng kể creatine huyết
tương.
TMP12 314–316, 321
118. Tình trạng nào dưới đây làm giảm GFR trên 10% trong trường hợp thận hoạt
động bình thường?
A) Giảm huyết áp động mạch thận từ 100 xuống 85 mm Hg
B) Giảm 50% sức cản tiểu động mạch đến
C) Giảm 50% sức cản tiểu động mạch đi
D) Tăng 50% hệ số lọc mao mạch cầu thận
E) Giảm áp suất thẩm thấu keo huyết tương từ 28 xuống 20mm Hg
 C) Giảm 50% sức cản tiểu động mạch đi có thể làm giảm đáng kể GFR hơn
10%. Giảm huyết áp máu động mạch thận từ 100 xuống 85 mm Hg có thể làm
giảm nhẹ GFR trong tình trạng thận hoạt động bình thường tự điều hòa. Giảm sức
cản tiểu động mạch đến, giảm áp suất thẩm thấu keo huyết tương hay tăng hệ số
lọc mao mạch cầu thận đều làm tăng GFR.
TMP12 314–316, 319, Hình 26-15 và 26-17

Unit 6: Máu
1. Trong quý thứ 2 của thai kỳ, đâu là nơi chủ yếu sản xuất hồng cầu?
A. Túi noãn hoàng
B. Tủy Xương
C. Hạch Lympho
D. Gan
Commented [TL47]: Quý đầu tiên của thai kì
 D) quá trình sản xuất hồng cầu bắt đầu trong túi noãn hoàng từ thai kỳ thứ nhất.
Commented [TL48]: Đầu quý hai của thai kì
quá trình sản xuất trong túy noãn hoàng giảm xuống ở thai kỳ thứ 2 và gan trở
thành nguồn sản xuất chủ yếu trong thời gian này. Quý ba của thai kì qua trình sản
xuất ở tủy xương tăng lên và tiếp tục trong suốt cuộc đời
TMP12 414

2. Sau khi hiến máu, sản xuất hồng cầu của tủy xương bắt đầu tăng lên trong
A. 30 phút
B. 24 giờ
C. 2 ngày
D. 5 ngày
E. 2 tuần
 B) quá trình sản xuất tế bào máu tăng lên một cách nhanh chóng trong
vòng 24h, nhưng tế bào máu mới chỉ xuất hiện trong vòng 5 ngày
TMP12 416
câu hỏi 3-6
Những điểm nào trong đồ thị sau đây định nghĩa gần nhất với các điều kiện
sau? Biết nồng độ erythropietin (EPO) bình thường xấp xỉ 10
A. Vận động viện chạy marathon olympic
B. suy tủy xương
C. bệnh thận giai đoạn cuối
D. đa hồng cầu nguyên phát

3 D) một vẫn động viên điền kinh luyện tập tốt sẽ có lượng EPO tăng lên nhẹ và
lượng hematocrit có thể tăng lên đến 50%. một lượng hematocrit cao hơn 50% gợi
ý điều trị EPO
4.--> E) thiếu máu do bất sản tuỷ là tình trạng mà tủy xương giảm sản xuất hồng
cầu nhưng không đáp ứng với erythropoietin. Vì vậy, một người thiếu máu bất sản
tuỷ sẽ có lượng hematocrit thấp và tăng lượng erythropoietin.
TMP12 420
5.--> A) Trong suy thận giai đoạn cuối, nồng độ erythropoietin giảm vì giảm giải
phóng từ thận bị bệnh. Hậu quả của việc giảm nồng độ erythropoietin là giảm
hematocrit.
TMP12 416
6.--> B) với một trường hợp đa hồng cầu nguyên phát, tủy xương sản xuất tế bào
máu mà không cần sự kích thích của erythropoietin. Hematocrit sẽ rất cao, có thể
đên 60%. Vứi sự tăng lên của hematocrit, sẽ xuất hiện ức chế ngược âm tính
erythropoietin và lượng EPO sẽ rất thấp
TMP12 421

7. Một bệnh nhân nam 62 tuổi đến để khám sức khỏe định kỳ. Cô ấy
phàn nàn về việc ngứa ở tay cùng với đau đầu và chóng mặt. Công
thức máu thường quy: RBCs: 8.2M/uL, WBCs: 37,000/uL và tiểu cầu
640.000/uL. Nồng độ erythropoietin của cô ấy thấp hơn so với bình
thường. Chẩn đoán đầu tiên là:
A. Giam tiểu cầu
B. đa hồng cầu tương đối
C. đa hồng cầu thứ phát
D. đa hồng cầu nguyên phát
E. leukemia thể tủy
 D) Sự tăng lên của cả hồng cầu, WBC, PLT gợi ý răng bệnh nhân đang có một
đa hồng cầu nguyên phát. Bệnh thận có thể dẫn đến lượng EPO thấp, nhưng RBC
cũng sẽ thấp. Leukemia thể tủy có thể dẫn đến tăng WBCs nhưng không làm tăng
RBCs. Đa hồng cầu thứ phát sẽ có tăng lượng EPO. Đa hồng cầu tương đối liên
quan đến mất nước.
TMP12 412

8. một phụ nữ 40 tuổi đến khám phàn nàn về việc mệt mỏi. Cô ấy vừa
mới được điều trị một bệnh truyền nhiễm. Xét nghiệm của cô ấy: RBC
1.8x 10^6/uL, Hb 5.2 g/dL, hematocrit (Hct) 15, WBC 7.6 x 1063/uL,
PLC 320.000/uL, thể tích hông cầu trung bình (MCV) 92 fL, và hồng
cầu lưới 24%. Giai thích phù hợp nhất cho những biểu hiện này là?
A. thiếu máu do suy tủy xương
B. thiếu máu tan máu
C. bệnh hồng cầu hình cầu di truyền
D. thiếu vitamin b12
 B) bệnh nhân này có giảm sản xuất hồng cầu vì có tình trạng thiếu máu (giảm
số lượng, Hb, Hct) trong khi tế bào máu được sản xuất với kích thước bình thường,
MCV=90. Bởi vậy, bệnh nhân này không có hồng cầu hình cầu (Hồng cầu nhỏ)
hay thiếu vitamin B12 (hồng cầu lớn). WBC bình thường và tăng hồng cầu lưới gợi
ý rẳng tủy xương vẫn còn chức năng. Tăng lượng hồng cầu lưới có nghĩa là một
lượng lớn hồng cầu đang được sản xuất. các giá trị xét nghiệm càng khẳng định
thiếu máu là do một dạng mất máu nào đó, trong trường hợp này là thiếu máu do
tan máu
TMP 12 420.

9.--> C) Ở bệnh nhân thiếu máu do bất sản tuỷ, sản xuất hồng cầu rất ít hoặc không
sản xuất. Hct và hemoglobin sẽ thấp, MCV sẽ bình thường ( tế bào máu vẫn bình
thường, chỉ là bị giảm sản xuất) và tăng lượng EPO
TMP12 420

10.một bệnh nhân tâm thần phân liệt 34 tuổi bị mệt mỏi kéo dài trong 6
tháng gần đây. Anh ấy vẫn ăn ngon, nhưng không chịu ăn rau vì nghe thấy
ảo thanh răng rau có thuốc độc. Khám cơ thể và thần kinh anh ấy đều bình
thường. Anh ấy có nồng độ hemoglobin là 9.1 g/dL, WBC 10000/uL, và
MCV là 122. Chẩn đoán ban đầu là
A. Mất máu cấp
B. hồng cầu hình liềm
C. thiếu máu do suy tủy
D. thiếu máu tan máu
E. thiếu acid folic
 E) bệnh nhân bị thiếu máu Hb<14 g/dL, Trong khi số lượng bạch cầu là bình
thường, gợi ý tuỷ xương bình thường. Kích thước tế bào hồng cầu lớn hơn bình
thường rất nhiều (bình thường MCV= 90). Anh ấy ăn thiếu rau gợi ý đến việc thiếu
cả vitamin B12 và acid folic. Tuy nhiên, cơ thể vẫn dự trữ đủ lượng B12 trong 4
đến 5 năm, nên anh ấy không có biểu hiện thiếu vitamin B12. Cơ thể chỉ dự trữ
acid folic được trong 3 đến 6 tháng nên sau 1 năm không ăn rau, bệnh nhân đã bị
thiếu acid folic
TMP 12 417, 420

11. một bệnh nhân Mỹ gốc Phi đến phòng cấp cứu 3h sau khởi phát của đau
lưng và ngực nặng. Những vấn đề này bắt đầu sau khi anh ấy đã trượt tuyết.
Anh ấy sống ở LA (miền biển) và có nhiều đợt triệu chứng như thế này 5
năm trước đây khi đến thăm Wyoming (miền núi). Anh ấy trông đau thực
sự, kết quả xét nghiệm:
Hb 11g/dL
WBC 22000/uL
Hồngcầulưới 25%
chẩn đoán của bệnh nhân này là
A. mất máu cấp
B. hồng cầu hình liềm
C. thiếu máu của một bệnh mãn tính
D. bệnh thận giai đoạn cuối
 B) Người Mỹ gốc Phi này có thiếu máu thể hiện qua giảm nồng độ hemoglobin
và tăng lượng hồng cầu lưới. Anh ấy có đáp ứng với một số tình trạng viêm/nhiễm
khuẩn được nhận biết qua việc tăng bạch cầu. Khu vực độ cao cao đã kịch thích
cho một đợt thiếu oxi và gây ra các tế bào máu bị biến thành hình liềm. Bệnh nhân
này có thiếu máu hồng cầu hình liềm.
TMP12 418,420

12. môt bệnh nhân nam 62 tuổi phàn nàn về đau đầu, khó nhìn, và đau ngực. Khám
bệnh cho thấy bệnh nhân có màu da đỏ ửng và lách to. CBC của anh ấy:
Hematocrit: 58%, WBC 13300/uL, PLC 600000/uL. Oxi bão hòa động mạch
(arterial SaO2) 97% khí phòng. Bạn sẽ đề xuất biện pháp điều trị nào
A. hóa trị liệu
B. chích máu (??)
C. bố sung sắt
D. thở oxi
 B) bệnh nhân này có đa hồng cầu nguyên phát: tăng RBCs, WBCs, và tiểu cầu.
Tăng Hct cũng làm tăng độ quánh của máu, dẫn đến tăng hậu gánh cho tim. Điều
này có thể là lý do của việc đau ngực. Do đó, một đợt trích máu là cần thiết để
giảm việc tăng các tế bào máu
TMP12 421

13. một bệnh nhân nữ 45 tuổi bị mệt mỏi tiến triển trong tháng 7 và có CBC được
báo cáo lại là bình thường. Cô ấy nhập viện vào tháng 12 vì đau đầu rất nhiều, và
được phát hiện huyết áp là 175/90 mmHg. Cận lâm sàng: Hb (8,3 g/dL), RBCs
(2,2x10^g/uL), MCV (89fL), WBCs (5100/uL), PLC (262x10^3/uL) và hồng cầu
lưới 0.8%. chẩn đoán ở bệnh nhân này là:
A. Thiếu acid folic
B. thiếu sắt
C. thiếu máu tan máu
D. bệnh thận giai đoạn cuối
 D)bệnh nhân này bị thiếu máu, nhưng các tế bào hồng cầu đã được sản xuất lại
bình thường (chú ý MCV bình thường). Sản xuất hồng cầu nhìn chung là giảm
(hồng cầu lưới giảm). WBCs và PLTs bình thường, gợi ý rằng tủy xương bình
thường. Thiếu máu do thiếu acid folic và thiếu sắt có thể gây ra giảm RBC và
MCV. Thiếu máu tan máu có thể gây tăng hồng cầu lưới. Tăng huyết áp cho thấy
bằng chứng của bệnh thận. Bệnh nhân này có bệnh thận giai đoạn cuối và giảm sản
xuất erythropoietin.
TMP12 416
14. Một phụ nữ 38 tuổi khỏe mạnh đến khám định kỳ ở chỗ bạn. Cô ấy đã
dành 2 tháng gần đây để leo núi Himalayas và cắm trại trên đỉnh Everest.
Kết quả nào sau đây bạn trông đợi ở công thức máu của cô ấy?

 C) Cô gái này có một đa hồng cầu thứ phát do tiếp xúc với môi trường
oxi thấp. Cô sẽ có tăng HCT, và do đó tăng RBC, nhưng WBC bình thường.
Các tế bào cũng là bình thường nên MCV sẽ bình thường
TMP12 421
15.Một bệnh nhân đến phòng khám của bạn than phiền về việc rất mệt mỏi và
khó thở khi gắng sức tăng dần trong 2 tuần vừa qua. Khám lâm sàng, bạn
quan sát thấy cô ấy là một phụ nữ có thể trạng tốt, toàn trạng thoải mái mặc
dù hơi có khó thở. Dấu hiệu sinh tồn: mạch: 120, nhịp thở: 20, huyết áp:
120/70. Khi cô ấy đứng dậy huyết áp hạ xuống 80/50. Xét nghiệm máu: Hb
7g/dL, Hct 20%, Hồng cầu: 2x10^6/uL, tiểu cầu: 400000/uL. Trong phiến
đồ máu ngoại vi, hồng cầu của cô ấy là hồng cầu nhỏ và nhược sắc. Chẩn
đoán của bệnh nhân này là?
a. Thiều máu do suy tủy
b. Suy thận
c. Thiếu máu thiếu sắt
d. Thiếu máu hồng cầu liềm
e. Thiếu máu hồng cầu khổng lồ
 C) Xét nghiệm tế bào máu chỉ ra rằng cô gái này có thiếu máu. Tủy
xương của cô ấy vẫn còn chức năng và cô ấy có số lượng tiểu cầu bình
thường, nhưng tủy xương đang tạo ra số lượng hồng cầu bất thường.
Hồng cầu nhỏ nhược sắc là một triệu chứng kinh điển của thiếu máu
thiếu sắt. nếu bệnh nhân suy thận sẽ có hồng cầu bình thường. Bệnh
hồng cầu hình liềm sẽ tạo ra các hồng cầu bị biến dạng. thiếu máu
hồng cầu khổng lồ sẽ đặc trưng bởi hồng cầu lớn
TMP12 418
16.Một người được đặt trong môi trường với nồng độ oxi thấp, sẽ mất một
khoảng thời gian là bao lâu để có sự tăng lên của hồng cầu lưới:
a. 6 giờ
b. 12 giờ
c. 3 ngày
d. 5 ngày
e. 2 tuần
 C) Nồng độ erythropoietin tăng sau khi có sự giảm nồng độ oxy
động mạch với lượng erythropoetin được sản xuất nhiều nhất xảy ra
trong vòng 24h. cần 5 ngày để sản xuất ra tế bào hồng cầu mới. Tuy
nhiên, vì cần 1-2 ngày để hồng cầu lưới trở thành hồng cầu nên đáp ản
đúng sẽ là 3 ngày cho đến khi có tăng số lượng hồng cầu lưới
TMP12 414-146
17.Một bệnh nhân nam 75 tuổi mắc bệnh hở chủ mức độ vừa. Trong vòng 12
tuần vừa bệnh nhân xuất hiện khó thở và đau ngực khi gắng sức. Toàn trạng
của bệnh nhân nhợt nhạt. Test máu trong phân dương tính. Kết quả cận lâm
sàng: hemoglobin: 7.2 g/dL, MCV: 75. Phiến đồ máu cho kết quả hồng cầu
nhỏ, nhược sắc. Chẩn đoán cho bệnh nhân này có thể là?
a. Thiếu vitamin B12
b. Thiếu máu do tan máu tự miễn
c. Thiếu máu do thiếu folate
d. Thiếu máu thiếu sắt
 d) Bệnh nhân này có thiếu máu và có nồng động hemoglobin thấp
với hồng cầu nhỏ. Thiếu vitamin B12 và acid folic sẽ dẫn đến hồng
cầu to. Bạch cầu và tiểu cầu của anh ấy bình thường, gợi ý rằng tủy
xương bình thường. Kết quả xét nghiệm phân dương tính cho thấy có
mất máu trong đường tiêu hóa. Một bệnh nhân có thể thiếu máu vì
mất máu và hồng cầu bình thường miễn là có đủ sắt. thiếu máu hồng
cầu nhỏ nhược sắc là một dấu hiệu của thiếu máu thiếu sắt
TMP12 417-417
18.Một bệnh nhân nam 24 tuổi đến phòng cấp cứu vì lý do gãy chân. Xét
nghiệm máu được chỉ định với kết quả bạch cầu là 22x10^3/uL. 5 giờ sau,
xét nghiệm máu thứ 2 có kết quả và số lượng bạch cầu chỉ còn 7x10^3/uL.
Nguyên nhân gây tăng bạch cầu trong xét nghiệm đầu tiên là gì?
a. Tăng sản xuất bạch cầu do tủy xương
b. Dịch chuyển bạch cầu từ hệ viền sang hệ tuần hoàn
c. Giảm phá hủy bạch cầu
d. Tăng sản xuất các selectin
 B) Phần lớn bạch cầu được dự trữ ở tủy xương, chờ đến khi nồng
độ cytokines tăng lên để kịch tích giải phóng chúng ra khỏi tủy
xương. Tuy nhiên, chần thươn ở xương có thể dẫn đến giải phóng
bạch cầu ra vào tuần hoàn. sự tăng lên của bạch cầu không phải do bất
cứ đáp ứng viêm nào mà do cơ chế chấn thương.
TMP12 424
19.Sự bám dính của bạch cầu lên các tế bào nội mô có đặc điểm nào sau đây?
a. Nguyên nhân do sự giảm xuống của các selectin
b. Phụ thuộc vào sự hoạt hóa của các integrin
c. Nguyên nhân do sự ức chế giải phóng histamine
d. Nhiều hơn ở khu vực động mạch hơn là tĩnh mạch ở trong vòng tuần
hoàn
 B) Hoạt hóa của các selection hay các integin sẽ dẫn đến sự bám
dính của bạch cầu lên tế bào nội mô
TMP12 425,429
20.Trong quá trình đáp ứng viêm, đâu là thứ tự đúng của các sự kiên xảy ra ở
cấp độ tế bào?
a. Loại bỏ bạch cầu đơn nhân từ máu, tăng sản xuất bạch cầu đa nhân
trung tính, hoạt hóa đại thực bào mô, bạch cầu đa nhân trung tính xâm
nhập vào máu.
b. Hoạt hóa đại thực bào mô, bạch cầu đa nhân trung tính xâm nhập vào
máu, bạch cầu đơn nhân xâm nhập vào máu, tăng sản xuất bạch cầu
đa nhân trung tính.
c. Tăng sản xuất bạch cầu đa nhân trung tính, hoạt hóa đại thực bào mô,
bạch cầu đa nhân trung tính xâm nhập vào máu, bạch cầu đơn nhân
xâm nhập vào máu.
d. Bạch cầu đa nhân trung tính xâm nhập vào máu, hoạt hóa đại thực bào
mô,bạch cầu đơn nhân xâm nhập vào máu, tăng sản xuất bạch cầu đa
nhân trung tính.
 B) Sự kiên tuần hoàn đầu tiên sau khi có tình trạng viêm là kích
thích các đại thực bào mô. Sau đó sẽ có sự xâm nhậm của các bạch
cầu đa nhân trung tính và bạch cầu mono theo thứ tự. cuối cùng là
tăng sự sản xuất bạch cầu ở tủy xương.
TMP12 428
21.Trên một người bình thường khoẻ mạnh, thành phần nào sau đây trong máu
có đời sống ngắn nhất?
a. Đại thực bào
b. Tế bào T nhớ
c. Hồng cầu
d. Tế bào Lympho B nhớ
 C) Đại thực bào tồn tại đến nhiều năm. Tế bào T và B nhớ tồn tại
suốt đời của bệnh nhân. Hồng cầu tồn tại khoảng 120 ngày và sau đó
bị phá hủy trong khi đi qua lách
TMP12 430
22.Một bệnh nhân nam 45 tuổi vào phòng cấp cứu vì lý do tiêu chảy trong 2
tuần nay. Bệnh tiến triển nặng dần trong một vài ngày gần đây. Bệnh nhân
có hiện tượng thiểu niệu và nhập viện do mất nước. Xét nghiệm phân dương
tính với một loại trứng của ký sinh trùng. Loại bạch cầu nào sẽ tăng lên
trong trường hợp này?
a. Bạch cầu ái toan
b. Bạch cầu đa nhân trung tính
c. Lympho T
d. Lympho B
e. Bạch cầu đơn nhân
 A) Bạch cầu ái toan chiếm khoảng 2% của tổng bạch cầu, nhưng
chỉ sản xuất với số lượng lớn ở những người bị nhiễm ký sinh trùng
TMP12 430
23.Bệnh nhân nam 8 tuổi thường xuyên đến với phòng khám vì viêm da dai
dẳng (thời gian tổn thương hồi phục không nằm trong giới hạn bình thường).
Bệnh nhân vừa hồi phục lại sau bệnh sởi. Kiểm tra hiệu giá các kháng thể
sau tiêm phòng cho kết quả phản ứng kháng thể bình thường. Sự thiểu hụt
loại tể bào nào sau đây có khả năng cao nhất gây ra nhiễm trùng liên tục?
a. Tế bào lympho B
b. Các tế bào trong huyết tương
c. Bạch cầu đa nhân trung tính
d. Đại thực bào
e. Tế bào Lympho T CD4
 C) Tế bào lymphocytes T và B, và các tế bào trong huyết tương
cùng với đại thực bào là cần thiết với đáp ứng miễn dịch thu được.
Bạch cầu ái kiềm không cần để chống lại các nhiễm trùng nhỏ. Bạch
cầu đa nhân trung tính là cần thiết với mọi nhiễm trùng.
TMP12 428-429
24. Quá trình xâm nhập của bạch cầu diễn ra ở đâu khi có đáp ứng với các tác
nhân gây viêm?
a. Tiểu động mạch
b. Kênh lympho
c. Tiểu tĩnh mạch
d. Động mạch bị viêm
 C) Sự dịch chuyển của bạch cầu xảy ra qua một phần của hệ mạch
có thành mạch mỏng và có số lượng cơ trơn tối thiểu, bao gồm mao
mạch và tiểu tĩnh mạch.
TMP12 425 429
25.Một bệnh nhân nghiện rượu 65 tuổi có đau ngực và ho có khạc đờm. Xét
nghiệm máu cho kết quả bạch cầu 42.000/uL. Bạch cầu này có nguồn gốc từ
đâu?
a. Động mạch phế nang
b. Tiểu phế quản
c. Phế quản
d. Khí quản
e. Tủy xương
 E) Tất cả các tế bào bạch cầu đều có nguồn gốc từ tủy xương từ
lymphocyte hay myelocytes
TMP12 424
26.--> B) Có nhiều yếu tố có thể gây nên đau hay bắt đầu quá trình đau.
Bao gồm histamine, bradykinin, và prostaglandins. Yếu tố hoạt hóa tiểu
cầu (PAF) hoạt hóa tiểu cầu trong quá trình hình thành cục máu đông.
Interleukin và TNF là các yếu tố trong quá trình đáp ứng viêm và điều
khiển đại thực bào.
TMP12 428
27.Một bệnh nhân đến khám phòng khám răng-hàm-mặt, nha sĩ phát hiện ra
một vết loét trên môi của bệnh nhân này. Vết loét này không bình thường ở
chỗ nó không đau hay có dịch chảy. Bệnh nhân sau đó được nhập viện với
một cơn gai lạnh rùng mình dữ dội. Kết quả xét nghiệm là Hct 30%, tiểu cầu
400.000/uL, bạch cầu 4200/uL, 68% lympho, và 20% trung tính. Chẩn đoán
của bệnh nhân này là?
a. Một nhiễm trùng nhẹ, không được điều trị
b. Giảm bạch cầu hạt
c. Thiếu máu do suy tủy
D. Leukemia cấp
 B) bệnh nhân này có giảm hồng cầu nhẹ cà số lượng tiểu cầu bình
thường. điều này gợi ý rẳng tủy xương hoạt động bình thường. Bạch cầu của
anh ta trong giới hạn bình thường nhưng tỷ lệ tế bào thì không. Anh ấy nên
có 60% trung tính. tỷ lệ lymphocyte 66% (bình thường 30%) gợi ý rằng anh
ấy đang có leukemia cấp TMP12 431
28.Điều gì diễn ra tiếp theo sau hoạt hóa bạch cầu ái kiềm?
a. Giảm sự xuyên mạch của bạch cầu đa nhân trung tính
b. Giảm quá trình hình thành chân giả
c. Co mạch
d. Tăng tính thấm thành mạch
 D)Bạch cầu ái kiềm giải phóng heparin, histamine, và một loạt các yếu tố hoạt
hóa. Histamine hoạt động để tăng tính thấm thành mạch trong khi heparin chống
hình thành cục máu đông. Các chất giải phóng từ bạch cầu ái kiềm cũng thu hút
bạch cầu ái toan và tăng tinhst thấm của mao mạch. TMP12 431
29.Sự tăng tiết dịch vào trong mô xảy ra trong quá trình viêm cấp là do?
a. Giảm huyết áp
b. Giảm protein trong khoảng kẽ
c. Tắc mạch bạch huyết
d. Tăng các yếu tố đông máu
e. Tăng tính thấm thành mạch
 E) Dịch rỉ ra vào trong mô là vì tăng tính thấm thành mao mạch TMP12
428

30.Điều gì sau đây có ở bệnh nhân AIDS?


a. Có khả năng tạo ra đáp ứng miễn dịch bình thường
b. Tăng tế bào T hỗ trợ
c. Tăng tiết interleukins
d. Giảm tế bào T hỗ trợ
 D) Tế bào t hỗ trợ bị phá hủy bởi virut AIDS, làm cho bệnh nhân không đươc
bảo vệ trước các bệnh nhiễm trùng TMP12 440

31.Sự trình diin kháng nguyên trong phhn hhh hòa hha mô I (MHC-I) sI dI đđ :
a. Tạo kháng thể
b. Hoạt hóa tế bào T độc
c. Tăng thực bào
d. Giải phóng histamine bởi tế bào mast
 B) Sự trình diện của một kháng nguyên trên một tế bào bị nhiễm trùng sẽ
dẫn đến hoạt hóa tế bào T độc để điến giết té bào đó. Trình diện của một
kháng nguyên bởi đại thực bào sẽ hoạt hóa tế bào T hỗ trợ, dẫn đến hình
thành kháng thế TMP12 441
32.Quá trình gắn mmn tác nhân vi sinh xâm nhh vào cơ thh vvh IgG và bb thh
đđ hoob hóa quá trình nhhn biin đưưn ggư là?
a. Chemokenisis
b. Opsonin hóa
c. Thực bào lysosome lan tỏa
d. Truyền tín hiệu
 B) một trong các sản phẩm của phức hợp bổ thể hoạt hóa quá trình thực
bào vi khuẩn được gắn phức hợp kháng nguyên kháng thể, quá trình này
được gọi là opsonin hóa TMP12 439
33.Interleukin 2 (IL-2) là một phân tử quan trọng trong quá trình đáp ứng miễn
dịch. Chức năng của IL-2 là gì?
a. Gắn và trình diện kháng nguyên
b. Kích thích sự tăng sinh của tế bào T độc
c. Tiêu diệt các tế bào bị nhiễm virut
d. Là điều kiện cho quá trình nhân lên của tế bào T hỗ trợ
 B) interleukin-2 (IL-2) được tiết bởi tế bào T hỗ trợ khi thế bào T được
hoạt hóa bởi kháng nguyên đặc hiệu. IL-2 đóng một vai trò quan trọng trong
quá tình phát triển và kéo dài sự sống của cả T độc và T ức chế. TMP12
440-441
34.CD4 là một marker của?
a. Tế bào B
b. Tể bào T độc
c. Tế bào T hỗ trợ
d. Một đại thực bào được hoạt hóa
e. Một tiền thân của bạch cầu hạt trung tính
 C) Tế bào T CD4 hỗ trợ nhận ra phức hợp hòa hợp mô chủ yếu (MCH) lớp II và
peptide trên các tế bào trình diện. tế bào T CD8 nhận ra phức hợp hòa hợp mô chủ
yếu (MCH) lớp I và peptide trên các tế bào bị nhiễm. TMP12 440-441

35.Điều gì xảy ra sau quá trình trình diện kháng nguyên của một đại thực bào?
a. Trực tiếp tạo ra các kháng thể
b. Hoạt hóa tế bào T độc
c. Tăng quá trình thực bào
d. Hoạt hóa tế bào T hỗ trợ
 D) quá trình trình diện kháng nguyên trên bề mặt một đại thực bào hay các tế
bào liên võng dẫn đến hoạt hóa các tế bào T hỗ trợ. hoạt hóa tế bào T hỗ trợ sau đó
dẫn đến giải phóng ác lymphokine kích thích các tế bào T độc hoạt hóa cùng với
quá trình hoạt hóa tế bào B và sản sinh kháng thế TMP12 440-441

36.Hoạt hóa hệ thống bổ thể sẽ dẫn tới quá trình nào sau đây?
a. Gắn yếu tố vi sinh xâm nhập vào IgG
b. Bất hoạt tế bào bạch cầu ưa acid
c. Giảm nồng độ phức hợp bổ thể mô
d. Tạo ra các chất độc tế bào
 D) Hoạt hóa hệ thống bổ thế dẫn đến một loạt các hoạt động. Bao gồm
opsonin hóa và thực bào bởi bạch cầu đa nhân trung tính, ly giải vi khuẩn,
làm kế dính các yếu tố vi sinh, hoạt hóa bạch cầu ái kiềm và tế bào mass và
gây hóa hướng động. Mảnh C5a của bổ thể gây ra hóa hướng động với bạch
cầu trung tính và đại thực bào.
37.Một bé nữ 9 tuổi có chảy dịch mũi và ngứa mắt mỗi mùa xuân. Một bác sĩ dị
ứng tiến hành test da sử dụng một phức hợp phấn hoa. Chỉ trong một vài
phút cô ấy đã xuất hiện đỏ ở trung tâm và phù tại vùng da được test. Nguyên
nhân của đáp ứng này có thể là?
a. Phức hợp kháng nguyên kháng thể bắt đầu được hình thành trong mạch máu
của da
b. Hoạt hóa bạch cầu trung tính do việc tiêm kháng nguyên
c. Hoạt hóa tế bào CD4 hỗ trợ và kết quả là tạo ra các kháng thể đặc hiệu
d. Hoạt hóa tế bào lympho độc để phá hủy các kháng nguyên
A) một khi một người đã được chưgns minh là có dị ứng, phản ứng đầu
tiên sẽ gây ra do phản ứng phức hợp kháng nguyên kháng thể, và sự hoạt
hóa của hệ thống bổ thế. Qúa trình di chuyển của bạch cầu đa nhân trung
tính, hoạt hóa tế bào T hỗ trợ và gây mẫn cảm vởi tế bao lympho cần có
thêm thời gian TMP12 443
38.Điều nào sau đây đúng với tế bào T độc?
a. Tế bào T độc yêu cầu sự nhận diện của một hệ thống tế bào B có thẩm
quyền miễn dịch
b. Tế bào T độc yêu cầu sự nhận diện của một hệ thống ức chế Lympho T có
thẩm quyền miễn dịch
c. Tế bào T độc được hoạt hóa bởi sự trình diện của kháng nguyên của tế bào
viêm
d. Tế bào T độc phá huỷ vi khuẩn nhờ sự bắt đầu quá trình thực bào của đại
thực bào
 C) Các tế bào gây độc hoạt động trên các tế bào bị nhiễm khi các tế bào ấy có
kháng nguyên phù hợp trên bề mặt. Tế bào T độc đơ]cj kích thích bới các
lymphokine tạo ra do sự hoạt hóa của các tế bào T hỗ trợ. Tế bào T độc phá huyer
các tế bào bị nhiễm bằng việc tiết ra các protein tạo ra những lỗ lớn trên màng các
tế bào bị nhiễm. Không có sự tương tác giữa tế bào T độc và B độc. TMP12 441

39.Tế bào T hỗ trợ


a. Được hoạt hóa bởi sự trình diện của kháng nguyên bởi một tế bào bị viêm
b. Yêu cầu sự nhận diện của một hệ thống tế bào B có thẩm quyền miễn dịch
c. Phá hủy vi khuẩn nhờ quá trình thực bào
d. Được hoạt hóa nhờ sự trình diện của kháng nguyên bởi đại thưc bào hay các
tế bào hình sao
 D) Tế bào T hỗ trợ được hoạt hóa bởi sự trình diện các kháng nguyên trên
bề mặt của các tế bào trình diện kháng nguyên. Tế bào T hỗ trợ hoạt hóa tế
bào B để hình thành nên kháng thế, nhưng chúng không cần tế bào B để hoạt
hóa tế bào T hỗ trợ. Tế bào T hỗ trợ giúp các đại thực bào thực bào nhưng
không có khả năng tự thực bào vi khuẩn TMP12 440-441

40.Quá trình truyền máu nào sau đây sẽ gây ra phản ứng truyền máu ngay lập tức?
A. Truyền O Rh- toàn phần cho một bệnh nhân O Rh+
B. Truyền A Rh- toàn phần cho một bệnh nhân B Rh-
C. Truyền AB Rh- toàn phần cho bệnh nhân AB Rh+
D. Truyền B Rh- toàn phần cho một bệnh nhân B Rh-
 B) Truyên máu Rh- cho người nhận Rh+ cùng nhóm máu ABO sẽ
không gây ra một phản ứng nào. Máu nhóm A có kháng nguyên A
trên bề mặt và kháng thể kháng B. Máu nhóm B có kháng nguyên B
trên bề mặt và kháng thể kháng A. Vì thế, truyền máu nhóm A cho
một người nhóm B sẽ gây ra kháng thể kháng A trong người nhóm B
này phản ứng với máu được cho
TMP12 445-448
41.Điều nào sau đây là đúng?
A. Trong phản ứng truyền máu, có sự kết dính của máu người nhận
B. Mất chức năng thận sau khi có phản ứng truyền máu diễn ra một cách
chậm chạp
C. Truyền máu Rh+ cho mọi người nhận Rh- sẽ dẫn đến phản ứng truyền
máu ngay lập tức
D. Một người có nhóm máu AB được coi là một người nhận vạn năng
 D). Người nhận máu sẽ có một lượng lớn huyết tương và kháng thể ở trong đó.
Những kháng thế này sẽ hoạt động trên hồng cầu của máu được cho. Huyết tương
của máu được cho sẽ được pha loãng và vì thế sẽ có tác dụng tối thiểu với máu của
người nhận. Với mọi phản ứng truyền máy do kháng nguyên-kháng thế sẽ có sự
phá vỡ nhanh chóng của hồng cầu, giải phóng hemoglobin vào trong huyết tơ]ng,
có thể gây nên suy thận cấp. Truyền máu Rh+ sé chỉ gây ra phản ứng truyền máu
nến như người Rh- trước đó có được truyền máu hay tiếp xúc với kháng nguyên
Rh+. Một người nhóm AB không có kháng thế AB trong huyết tương, vì thế họ có
thế nhận mọi loại máu.
TMP12 448
42.Nhóm máu nào được mô tả trong hình dưới đây?

A. A
B. B
C. O
D. AB
 A) Có phản ứng kháng nguyên kháng thể giữa các kháng thế
akháng A và những tế bào hồng cầu. Không có phản ứng giữa những
kháng thế anti-B và tế bào hồng cầu. Vì vậy những tế bào hồng cầu có
kháng nguyên A và những tế bào này phải là nhóm A.
TMP12 446-447
43.Một cặp vợ chồng yêu cầu định nhóm máu cho đứa con 2 tuổi của họ ( bố
AB, Rh- âm tính, mẹ B, Rh- âm tính). Kết quả ngưng tập máu đứa trẻ được
trình bày ở hình dưới. Đâu là kết luận liên quan đến yếu tố bố mẹ của đứa trẻ
này là đúng?

A. Đứa trẻ là con ruột của cặp vợ chồng


B. Mẹ có thể là mẹ ruột, còn người bố không phải là bố ruột
C. Bố có thể là bố ruột, nhưng người mẹ không thể là mẹ ruột
D. Cả bố và mẹ đều không phải là bố mẹ ruột của đứa trẻ
 A) Không có phản ứng kháng nguyên và kháng thể giữa kháng thể kháng B và
hồng cầu. Có một phản ứng giữa kháng thế kháng B và hồng cầu. Do đó hồng cầu
phải có kháng nguyên B và phải là máu nhóm B. Không có phản ứng kháng
nguyên - kháng thể giữa kháng thế kháng D và hồng cầu, vì thế hồng cầu phải là
Rh-. đứa trẻ có nhóm máu B-. vì cha đứa trẻ có AB- và mẹ đứa trẻ có B- nên đứa
trẻ có thể là B-.
TMP12 446-447

44.Một phụ nữ 21 tuổi, nhóm máu B, phải trải qua phẫu thuật. Tiểu cầu của cô
ấy là 75000/uL. Cô ấy cần truyền tiểu cầu trước và trong phẫu thuật. Nhóm
máu nào sau đây có thể dùng để lấy tiểu cầu tương thích với bệnh nhân?
a. Chỉ nhóm A
b. Chỉ nhóm B
c. Chỉ nhóm O
d. Nhóm B và O
e. Nhóm A và B
f. Chỉ nhóm A và B
g. Chỉ nhóm A,B và AB
 B) Vì huyết tương có chứa kháng thể, bịch huyết tương sai có thể
có chứa kháng thể chống lại kháng nguyên B. Vì vậy, anh ấy có thể
nhận huyết tương B (Chứa kháng thế kháng A) hoặc AB (không chứa
cả kháng thể kháng A hay kháng B)
TMP12 448
45.Điều nào sau đây đúng khi nói về tan máu sơ sinh (HDN)
A. Nó diễn ra khi mẹ Rh+ có con Rh-
B. Điều này được phòng tránh nhờ truyền máu cho mẹ
C. Một việc truyền máu hoàn toàn sau khi sinh lần đầu sẽ ngăn chặn
HDN
D. Bố đứa trẻ phải có Rh+
 D) HDN (tan máu sơ sinh) xảy ra khi một người mẹ Rh- sinh con Rh+ thứ 2.
Vì thế bố phải là Rh+. Người mẹ trở nên mẫn cảm với kháng nguyên Rh sau khi
sinh đứa con Rh+ thứ nhất. Tan máu sơ sinh được ngăn chặn nhờ điều trị cho mẹ
với kháng thế kháng Rh sau khi sinh một đứa con Rh+. Điều này sẽ phá hủy tất cả
hồng cầu của bào thai trong mẹ và ngăn chặn người mẹ trờ nên mẫn cảm với kháng
nguyên Rh. Một lần chuyền máu toàn phần cho mẹ sẽ được yêu cầu để ngăn chặn
sự hình thành kháng thể Rh, nhưng không có tính thực tế. Một lần chuyền máu cho
đứa con đầu tiên sau khi sinh sẽ không được đạt được điều gì nếu như mẹ đã tiếp
xúc với kháng nguyên Rh+ trong quá trình sinh.
TMP12 447-448
46.Quá trình truyền máu nào sau đây sẽ xảy ra phản ứng truyền máu, nếu cho
rằng bệnh nhân chưa bao giờ được truyền máu trước đây?
a. Truyền nhóm O Rh- cho bệnh nhân AB Rh+
b. Truyền nhóm A Rh+ cho bệnh nhân A Rh-
c. Truyền nhóm AB Rh+ cho bệnh nhân AB Rh+
d. Truyền nhóm type A Rh+ cho bệnh nhân O Rh+
 D) Nhóm O được coi là nhóm máu cho toàn năng. Những phản ứng xảy ra giữa
kháng thể người nhận và kháng nguyên người cho như bảng dưới đây.
TMP12 445-447

47.Một người mẹ nhóm máu A+ luôn khỏe mạnh vừa sinh đứa con thứ 2, bố
đứa trẻ có nhóm máu O-. Biết rằng nhóm máu đứa trẻ là O- (O,Rh-), điều gì
có thể xảy ra ở đứa trẻ này?
A. Đứa trẻ sẽ bị tan máu sơ sinh do bất đồng yếu tố Rh
B. Đứa trẻ sẽ bị tan máu sơ sinh do bất đồng nhóm máu ABO
C. Đứa trẻ sẽ bị cả A. và B.
D. Đứa trẻ sẽ không có cơ hội để bị tan máu sơ dinh
 D) Phản ứng tan máu sơ sinh xảy ra khi mẹ là Rh-, bố là Rh+, dẫn đến một đứa
con Rh+. Vì đứa trẻ là O- và bố là Rh-, sẽ không có cơ hội cho HDN.
TMP12 445-448
48.Đơn vị máu nào sau đây có nguy cơ thấp nhất gây ra phản ứng truyền máu
ngay lập tức cho người nhận nhóm máu B+ (B Rh+)?
A. Máu toàn phần A+
B. Máu toàn phần O+
C. Máu toàn phần AB+
D. Bịch hồng cầu O+
E. Bịch hồng cầu AB-
 D) Trong mọi bệnh nhân, truyền túi tế bào nhóm O sẽ giảm thiểu một phản ứng
truyền máu vì kháng thể sẽ bị loại ra cùng với huyết thanh bị loại. Nếu yếu tố Rh
giống nhau, thì điều này cũng sẽ giảm thiểu phản ứng truyền máu. Bởi vậy, với
một bệnh nhân B+, một lần truyền máu B+ hay O+ sẽ không có phản ứng truyền
máu.
TMP12 445-448

49.Một phụ nữ có thai đến khám, cô ấy AB Rh- và chồng cô ấy A Rh+. Đây là


lần mang thai lần đầu của cô ấy, phải làm gì vào lần này?
A. Không gì cả
B. Chỉ định kháng thể Anti-D cho mẹ vào lần này
C. Chỉ định kháng thể Anti-D cho mẹ sau sinh
D. Chỉ định kháng thể Anti-D cho con sau sinh
E. Chỉ định kháng thể Anti-D cho con nếu con là Rh+
 A) Một người mẹ Rh- sẽ tạo ra kháng thế với nhóm Rh+ sau khi sinh đứa con
đầu tiên là Rh+. Trong trường hợp này, người mẹ chưa từng tiếp xúc với hồng cầu
Rh+ nên cô ấy sẽ không có kháng thể phát triển từ trước. Tuy nhiên, sau khi sinh
đứa trẻ, nếu đứa trẻ được nhận thấy là Rh+, kháng thể kháng D nên được chỉ định
cho mẹ để phá hủy mọi hồng cầu mà cô ấy đã tiếp xúc và ngăn cô ấy hình thành
nên kháng thể kháng Rh+ (D)
TMP12 447-448

50.Con đường đúng của đông máu ngoại sinh là?


A. Kết dính máu với collagen, hình thành phức hợp hoạt hóa
prothrombin, chuyển prothrombin thành thrombin, chuyển fibrinogen
thành sợi fibrin
B. Tổn thương mô, hình thành phức hợp hoạt hóa prothrombin, chuyển
prothrombin thành thrombin, chuyển fibrinogen thành sợi fibrin
C. Hoạt hóa tiểu cầu, hình thành phức hợp hoạt hóa prothrombin, chuyển
prothrombin thành thrombin, chuyển fibrinogen thành sợi fibrin
D. Tổn thương của máu, hình thành phức hợp hoạt hóa prothrombin,
chuyển prothrombin thành thrombin, chuyển fibrinogen thành sợi
fibrin
 B) Con đường đông máu ngoại sinh bao gồm sự tổn thương của mô, và
hậu quả là sự hình thành yếu tố hoạt hóa prothrombin. Tổn thương mô dẫn
đến giải phóng yểu tố mô hay yếu tố thrombinplastin, thứ có chức năng là
một enzyme ly giải protein. Yếu tố mô gắn với yếu tố VII dẫn đến hoạt hóa
yếu tố X. Có một hậu quả là sự hoạt hóa yếu tố hoạt hóa prothrombin, sự
chuyển đổi của prothrombin thành thrombin, sao đó chuyển đổi fibrinogen
thành mạng lưới fibrin. Hoạt hóa con đường ngoại sinh sẽ rất nhanh bởi chỉ
có một số nhỏ các phản ứng xúc tác bởi enzyme xảy ra.
TMP12 453-455
51.Điều nào sau đây dẫn đến thiếu yếu tố IX có thể điều chỉnh bởi một liều tiêm
vitamin K?
A. Hemophilia cổ điển
B. Viêm gan B
C. Tắc ống mật
D. Khiếm khuyết gen tổng hợp antithrombin III
 C) Hemophilia gây ra bởi mất đoạn gen của yếu tố đông máu VIII.
Hầu hết các yếu tố đông máu đều hình thành tại gan. Nếu một liều
vitamin K có thể cải thiện được tình hình, điều đó biểu hiện rằng gan hoạt
động tốt, và bệnh nhân không có viêm gan. Vitamin K là vitamin tan
trong dầu và được hấp thu tại ruột cùng với chất béo. Mật tiết ra bởi túi
mật là cần thiết cho sự hấp thu chất béo. Nếu bệnh nhân thiếu vitamin K,
thì sự thiếu hụt đông máu có thể được cải thiện nhờ một liều tiêm vitamin
K. Antithrombin II không có liên quan gì đến yếu tố IX.
TMP12 457-458
52.Một bệnh nhân bị thiếu yếu tố XIII bẩm sinh (yếu tố ổn định fibrin). Xét
nghiệm máu sẽ cho kết quả như thế nào?
A. Kéo dài thời gian prothrombin (PT time)
B. Kéo dài thời gian đông máu (clotting time)
C. Kéo dài thời gian thromboplastin cục bộ (APTT time)
D. Cục máu đông dễ vỡ
 D) Các monomer của fibrin được polyme hóa để hình thành cục
máu đông. Để tạo ra một cục máu động chắc chắn cần phải có sự hiện
diện của yếu tố ổn định fibrin được giải phóng từ tiểu cầu bên trong
cục máu đông. Những xét nghiệm đông máu khác thể hiện sự hoạt hóa
của con đường nội sinh hay con đường ngoại sinh hay số lượng tiểu
cầu.
TMP12 452,454,460-461
53.Một bé traI 2 tuổi dễ bị bầm tím và từng bị chảy máu lợi trước đây. Ông
ngoại cũng có một rối loạn đông máu. Khám lâm sàng cho thấy nhiêu vết
bầm tím nhỏ ở chân. Bạn nghi ngờ rằng bệnh nhân này bị thiếu yếu tố đông
máu nào?
A. Yếu tố hoạt hóa prothrombin
B. Yếu tố II
C. Yếu tố VIII
D. Yếu tốt X
 C) Một nam giới trẻ tuổi với rối loạn đông máu và tiền sử rối loạn đông máu
của những người đàn ông trong gia đình có thể dẫn đến sự nghi ngờ hemophilia A,
một bệnh với sự thiếu hụt yếu tố VIII.
TMP12 458
51. Điều nào sau đây dẫn đến thiếu yếu tố IX có thể điều chỉnh bởi một liều tiêm
tĩnh mạch vitamin K?
a. Hemophilia cổ điển
b. Viêm gan B
c. Tắc ống mật
d. Khiếm khuyết gen tổng hợp antithrombin III

52. Một bệnh nhân bị thiếu yếu tố XIII bẩm sinh (yếu tố ổn định fibrin). Xét
nghiệm máu sẽ cho kết quả như thế nào?
a. Kéo dài thời gian prothrombin (PT time)
b. Kéo dài thời gian đông máu (clotting time)
c. Kéo dài thời gian thrombopastin một phần (APTT time)
d. Cục máu đông dễ vỡ
53. Một bé trai 2 tuổi hay dễ bị bầm tím và từng bị chảy máu lợi trước đây. Ông
ngoại cũng có một rối loạn đông máu. Khám lâm sàng cho thất nhiêu vết bầm tím
nhỏ ở chân. Bạn nghi ngờ rằng bệnh nhân này bị thiếu yếu tố đông máu nào?
a. Yếu tố hoạt hóa prothrombin
b. Yếu tố II
c. Yếu tố VIII
d. Yếu tố X
54. Một bé gái chưa dậy thì 11 tuổi có biểu hiện đau ở khớp gổi sau một chấn
thương nhẹ. Quan sát kỹ hơn có phát hiện các vết bầm tím tại mô mềm. Đứa trẻ bị
mồ côi và không có tiền sử gia đình. Mẹ nuôi báo lại không có bất cứ vấn đề nào
khác. aPTT kéo dài và PT bình thường. Một xết nghiệm máu hoàn thiện sẽ cho kêt
quả nào sau đây?
a. Giảm Ca++ huyết tương
b. Tăng plasmin
c. Thiếu yếu tố VIII
d. Giảm số lượng tiểu cầu
55. Con đường đông máu bắt đầu bằng thromboplastin mô là?
a. Ngoại sinh
b. Nội sinh
c. Con đường chung
d. Ổn định fibrin
56. Một người phụ nữ 63 tuổi quay lại làm việc sau kỳ nghỉ ở New Zealand. Nhiều
ngày sau khi về nhà, cô ấy tỉnh dậy có sưng và đau ở chân phải và chân cô ấy bầm
xanh. Cô ấy ngay lập tức đến phòng cấp cứu nơi mà khám lâm sàng phát hiện có
huyết khối tĩnh mạch sâu diện rộng bao gồm tĩnh mạch đùi và chậu ở bên phải. Sau
khi giải quyết cục huyết khối, bệnh nhân cần phải điều trị gì tiếp trong tương lai?
a. Tiêp tục chuyền heparin
b. Warfarin
c. Aspirin
d. Vitamin K
57. Thuốc nào sau đây có khả năng cao nhất được sử dụng để dự phòng thiếu máu
cơ tim thoáng qua?
a. Heparin
b. Warfarin
c. Aspirin
d. Streptokinase
58. Liệu pháp nào sau đây phù hợp cho nhồi máu phổi diện rộng?
a. Heparin
b. Wafarin
c. Aspirin
d. Yếu tố hoạt hóa plasmingen mô
59. Điều nào sau đây giải thích tốt nhất cho việc kéo dài thời gian máu chảy?
a. Hemophilia A
b. Hemophilia B
c. Thiếu tiểu cầu
d. Sử dụng coumarin

60. Tại sao một số bệnh nhân suy dinh dưỡng chảy máu rất nặng khi chấn thương?
a. Thiếu vitamin K
b. Tiểu cầu bị giam giữ do gan nhiễm mỡ
c. Bilirubin tăng lên trung hòa thrombin
d. Protein huyết thương thấp gây ra các vấn đề của yếu tố XIII
61. Một cậu bé tuổi vị thành niên bị chảy máu mũi nhiều lần được giới thiệu đến
khám một bác sĩ để khám trước tiểu phẫu. PT là 11s ( bình thường là 11-15s), PTT
là 58s ( bình thường là 25-40s), và thời gian máu chảy là 6.5 min (bình thường là
2-7 phút). Điều nào sau đây có khả năng bị bất thường nhất trên bệnh nhân trẻ này?
a. Giảm số lượng tiểu cầu
b. Khiếm khuyết của tiểu cầu
c. Con đường nội sinh
d. Con đường ngoại sinh
e. Sản sinh yếu tố đông máu do gan

Unit 7: Hô Hấp
1. Một người đàn ông 45 tuổi khỏe mạnh đang đọc báo. Những cơ nào sẽ được
sử dụng cho thở yên tĩnh:
A) cơ hoành và cơ liên sườn ngoài
B) cơ hoành và cơ liên sườn trong
C) chỉ cơ hoành
D) cơ liên sườn trong và cơ thẳng bụng
E) cơ bậc thang
F) cơ ức đòn chũm
 C) Phổi có thể giãn ra và co lại bằng việc tăng và giảm thể tích của lồng
ngực. Thể tích của lồng ngực có thể thay đổi nhờ 2 cách: (a) Sự di chuyển
lên và xuống của cơ hoành tăng và giảm độ dài của lồng ngực. và (b) nâng
và ép khung sườn tăng và giảm đường kính trước sau của lồng ngực. Thở
bình thường trong điều kiện thư giãn đạt được hoàn toàn nhờ cơ hoành. Cơ
hoành co gây ra sự hít vào và giãn gây ra sự thở ra. Những cơ khác được liệt
kê trong câu hỏi nâng và ép khung sườn và được sử dụng trong thở mạnh
liên quan đến vận động cũng như các bất thường hô hấp đặc trưng bởi hô
hấp gắng sức quá mức
TMP12 465-466

2. Một sinh viên y khoa 25 tuổi khỏe mạnh tham gia vào cuộc chạy từ thiện
10km cho Hiệp hội tim mạch Mỹ. Cơ trong những cơ sau đây anh ấy dùng
(cơ co lại) trong thì thở ra?
A) Cơ hoành và cơ liên sườn ngoài
B) cơ hoành và cơ liên sườn trong
C) Chỉ cơ hoành
D) Cơ liên sườn trong và cơ thẳng bụng
E) Cơ bậc thang
F) Cơ ức đòn chũm
 D) Co cơ liên sườn trong và cơ thành bụng đẩy khung sườn đi xuống
trong thì thở ra. Cơ thành bụng và các cơ bụng khác ép các thành phần ổ
bụng lên trên về phía cơ hoành, điều này cũng giúp loại khí ra khỏi phổi.
Cơ hoành giãn trong thì thở ra. Cơ liên sườn ngoài, cơ ức đòn chũm và
cơ bậc thang tăng đường kính của lồng ngực trong khi vận động và do đó
hỗ trợ thì thở vào, nhưng chỉ cơ hoành là cần thiết cho thở vào trong khi
thở im lặng.
TMP12 465-466
3. Áp lực màng phổi bình thường ở một phụ nữ 56 tuổi là xấp xỉ -5cm H2O
trong điều kiện thư giãn ngay trước khi hít vào ( ví dụ, tại thể tích cặn chức
năng của phổi). Áp lực màng phổi (tính bằng cm H2O) trong kỳ hít vào là:
A) +1
B) +4
C) 0
D) -3
E) -7
 E) Áp lực khoang màng phổi (đôi khi gọi là áp suất trong khoang
màng phổi) là áp lực của dịch trong khoang hẹp giữa là thành của phổi
và là tạng của thành ngực. Áp suất khoang màng phổi thông thường
khoảng -5cm H2O ngay trước thì hít vào (ví dụ, tại dung tích cặn chức
năng, FRC) khi tất cả các cơ hô hấp đều giãn. Trong thì hít vào, thể tích
của lồng ngực tăng lên và áp lực khoang màng phổi trở nên âm hơn. Áp
lực khoang màng phổi trung bình khoảng -7.5 cmH2O ngay trước khi
thở ra khi phổi giãn hết cỡ. Áp lực khoang màng phổi sau đó trở về giá
trị tại trạng thái thư giãn là -5 cmH2O khi cơ hoành giãn và thể tích phổi
trở về thành FRC. Vì thế, áp lực trong khoang màng phổi luôn ở dưới áp
lực khí quyển trong điều kiện bình thường ở khoảng giữa -5 và -7
cmH2O khi thở yên tĩnh
TMP12 466
4. Áp lực tại phế nang bình thường ở một phụ nữ 77 tuổi là xấp xỉ 1cm H2O
trong kỳ thở ra. Đâu là áp lực phế nang trong kỳ hít vào (tính bằng cm
H2O)?
A) +0.5
B) +1
C) +2
D) 0
E) -1
F) -5
 E) Áp lực phế nang là áp lực của không khí trong lòng phế nang. Khi
nắp thanh môn mở và không có luồng không khí vào trong hay ra ngoài
phổi, áp lực của tất cả các phần của cây khí phế quản tương đương với
không. Sự giãn nở của khoang lồng ngực trong khi hít vào gây ra áp lực
phế nang ở dưới mức áp suất khí quyền, trung bình khoảng -1 cmH2O
khi thờ yên tĩnh và tạo ra một gradient áp lực +1 cmH2O để không khí đi
vào phổi. Sự co lại của khoang lồng gây ra áp lực phế nang để đạt được
giá trị áp suất dương khoảng +1 cmH2O, thứ tạo ra một gradient áp suất
+1cmH2O cho không khí đi ra khỏi phổi. Áp suất phế nang trở nên âm
hơn khi hít vào và dương hơn khi thở ra gắng sức liên quan đến vận động
cũng như trong nhiều tình trạng bệnh phổi khác.
TMP12 466
5. Một người đàn ông hút vào 1000ml từ một phế dung kế. Áp lực trong
khoang màng phổi là -4cm H2O trước khi hít vào và -12 cm H2O vào cuối
kỳ hít vào. Đâu là khả năng giãn nở của phổi?
A) 50mm/cm H2O
B) 100 ml/cm H2O
C) 125 ml/cm H2O
D) 150 ml/cm H2O
E) 250 ml/cm H2O
 C) Độ giãn nở là sự thay đổi của thể tích/sự thay đổi của áp suất.
Công thức tính là 1000ml/8 cm H2O tương đương với 125.
TMP12 467
6. Biểu đồ trên đây chỉ ra 3 đường cong giãn nở (S, T và U) tương ứng
của một phổi được cô lập chịu các áp lực xuyên phổi khác nhau. Điều nào
sau đây mô tả tốt nhất sự giãn nở tương đối của 3 đường cong?

A) S<T<U
B) S<T>U
C) S=T=U
D) S>T<U
E) S>T>U
 E) Độ giãn nở (C) là sự thay đổi của thể tích (ΔV) xảy ra khi có một
sự thay đổi áp suất xuyên phổi (ΔP), tức là, C = ΔV/ΔP. (Áp suất xuyên
màng là sự khác biệt giữa áp suất phế nang và áp suất khoang màng
phổi). Bởi độ giãn nở tương đương với độ dốc của đường cong quan hệ
thể tích-áp suất, nên rõ ràng đường cong S đại diện cho độ giãn nở cao
nhất, và đường cong U biểu thị độ giãn nở thấp nhất
TMP12 467
7. Một phổi thông khí qua chất lỏng so với một phổi thông khí qua chất khí
thì?
A) giảm sức cản của đường thở
B) tăng thể tích cặn
C) nó phức tạp hơn
D) cần áp lực lớn hơn để giãn ra
 D) Đô giãn nở là sự thay đổi thể tích với mỗi sự thay đổi về áp lực.
Độ giãn nở gây ra do "lực đàn hổi của nhu mô phổi và … lực đàn hồi của
nhu mô gây ra do lực căng bề mặt của dung dịch nằm ở bện trong thành
phế nang". Nếu một người được thông khí với dịch thì sẽ thiếu lực đàn
hồi do sức căng bề mặt. Vì thế độ giãn nở sẽ giảm.
TMP12 467-468
8. Một phụ nữ 22 tuổi có độ giãn nở của phổi là 0.2l/cmH2O và áp lực màng
phổi là -4 cmH2O. Áp lực màng phổi (tính bằng cm H2O) khi người phụ nữ
này hít vào 1.0 L không khí là?
a. -6
b. -7
c. -8
d. -9
e. -10
8.  D) Vì độ giãn nở là 0.2 L/cmH2O, nên rõ ràng là cứ 1.0L tăng lên trong
thể tích sẽ gây ra 5cmH2O giảm đi trong áp suất khoang màng phổi (1.0L/
0.2 L/cm H2O = 5.0 cm H2O) ; và, bởi vì áp lực màng phổi ban đầu là -4
cmH2O trươc khi hít vào, áp lực đã giảm 5cm H2O ( thành -9 cmH2O) khi
1.0 L khí được hít vào.
TMP12 466-467

9. Một trẻ sơ sinh thiếu tháng bị thiếu surfactant. Không có surfactant, nhiều
phế nang bị xẹp vào cuối mỗi kỳ thở ra, thứ dẫn đến suy hô hấp. Nhóm
những thay đổi nào sau đây xuất hiện trên đứa trẻ sơ sinh thiếu tháng này?
Áp lực bề mặt Độ giãn nở của
phế nang phổi
A) giảm giảm
B) giảm tăng
C) giảm không đổi
D) tăng giảm
E) tăng tăng
F) tăng không đổi
G) không đổi không đổi
 D) Surfactant được hình thành một cách tương đối trong giai đoạn cuối của thời
kỳ bào thai. Trẻ sinh sớm không có đầy đủ lượng sufactant có thể bị suy hô hấp và
tử vong. Surfactant là một chất hoạt động bề mặt có tác dụng giảm sức căng của
nước trong lòng phế nang rất lớn. Nước thường có sức hút lẫn nhau và điều này
làm cho giọt nước có hình cầu. Nhờ giảm sức căng bề mặt của mặt nước ở trong
lòng phế nang (và nhờ đó giảm sức căng bề mặt của nước với nhau), surfactant
giảm công hô hấp. vd: cần lượng áp suất xuyên phổi ít hơn cần để hít vào cùng một
lượng khí cho trước. Vì độ giãn nở của phổi bằng sự thay đổi thể tích với một áp
lực cho trước, rõ ràng rằng độ giãn của phổi giảm nếu có sự vắng mặt của
surfactant
TMP12 467-468
10.Một bệnh nhân có khoảng chết là 150ml, dung tích cặn chức năng là
3L, thể tích khí lưu thông là 650ml, thể tích dự trữ thở là là 1.5L, tổng dung
tích phổi là 8L, và nhịp thở là 15 lần/phút
Thể tích cặn là?
a. 500ml
b. 1000ml
c. 1500ml
d. 2500ml
e. 6500ml
 C) Thể tích cặn= FRC -ERV= 3L - 1,5L =1,5L
TMP12 469-471
Câu 11 và 12
11.Một người đàn ông 27 tuổi khỏe mạnh thở một cách yên tĩnh. Anh ta
sau đó hít vào hết mức có thể và thở ra hết mức có thể, tạo ra một đường khí
phế dung ở bên dưới. Thể tích thở ra dự trữ của anh ấy là (tính bằng Lít)?
a. 2.0
b. 2.5
c. 3.0
d. 3.5
e. 4.0
f. 5.0
 A) Thể tích dự trữ thở ra (ERV) là lượng thể tích của khí
tối đa có thể thở ra nhờ thở ra gắng sức sau khi kết thúc một
chu kỳ thông khí phế nang thở ra bình thường. ERV tương
đương với sự chệnh lệch giữa dung tích cặn chức năng
(FRC, 3L) và thể tích cặn (RV, 1L). Mặc dù cả FRC và RV
đều không thể xác định được bằng chỉ test khí phế dung, sự
khác biệt tương đối của hai thể tích này vẫn có thể xác định
được từ test khí phế dung và do đo có thể dùng để tính ERV
TMP12 469-471
12.Một người phụ nữ 22 tuổi hít vào hết mức có thể và thở ra hết mức có thể,
tạo ra một đường khí phế dung ở bên dưới. Thể tích cặn của cô ấy là 1.0l
được xác định bằng kỹ thuật hấp thu heli. Thể tích cặn chức năng của cô ấy
là (tính bằng lít)?
A) 2.0
B) 2.5
C) 3.0
D) 3.5
E) 4.0
F) 5.0
 C) Dung tích cặn chức năng (FRC) bằng thể tích dự trữ thở ra (2L)
cộng với thể tích cặn (1.0 L). Đây là lượng khí còn lại trong phổi tại lúc
kết thúc một lần thở ra bình thường. FRC được xem là phần thể tích còn
lai của phổi vì không có một cơ hô hấp nào còn được co tại FRC. Điều
này mô tả một điểm quan trọng: một test khí phế dung có thể đo được sự
thay đổi thể tích của phổi, nhưng không đo được thể tích tuyệt đối của
phổi. vì thể, chỉ test khí phế dung không thể dùng để đo thể tích cặn,
dung tích cặn chức năng và tổng dung tích của phổi
TMP12 469-471
13.Có nhiều thể tích và dung tích phổi như tổng thể tích của phổi (TLC), dung
tích sống (VC), dung tích hít vào (IC), thể tích khí lưu thông (VT), dung tích
thở ra (EC), thể tích dự trữ thở ra (ERV), thể tích dự trữ hít vào (IRV), dung
tích cặn chức năng (FRC) và thể tích cặn (RV). Những dung tích và thể tích
nào có thể đo trực tiếp bằng phương pháp khí phế dung mà không cần các
phương pháp bổ trợ khác?
T V I V E E I F R
L C C T C R R R V
C V V C
AK K c k c kh c k kh
h h ó h ó ôn ó h ôn
ô ô ô g ô g
n n n n
g g g g
Bk có c c c có c k kh
h ó ó ó ó h ôn
ô ô g
n n
g g
Ck có c c c có c c kh
h ó ó ó ó ó ôn
ô g
n
g
Dc có c c c có c k có
ó ó ó ó ó h
ô
n
g
Ec có c c c có c c có
ó ó ó ó ó ó
 B) Một test khí phế dung có thể dùng để xác định sự thay đổi thể tích của phổi,
nhưng không thể xác định thể tích tuyệt đối của phổi. Nó bao gồm một cái trống
chứa đầy khí nhúng vào một bình nước. Khi một người thở vào và ra, cái trống di
chuyển lên và xuống ghi lại sự thay đổi thể tích. Test khí phế dung không thể được
sử dụng để đo thể tích cặn (RV) vì dung tích cặn là lượng khí trong phổi không thể
vào máy khí phế dung. Dung tích cặn chức năng (FRC) là lượng khí còn lại trong
phổi sau khi thở ra bình thường. FRC không thể đo được sử dụng khí phế dung vì
nó có chứa RV. Tổng dung tích phổi (TLC) là tổng lượng khí mà phổi có thể giữ
sau khi hít vào tối đa. Bởi vì TLC bao gồm RV nên nó không thể đo được bằng
cách sử dụng test khí phế dung. TLC, FRC, và RV có thể được xác định sử dụng
phương pháp hấp thụ helium hoặc đo bằng plethysmography cơ thể.
TMP12 469-471
14.Một bệnh nhân có khoảng chết là 150ml, dung tích cặn chức năng là 3L, thể
tích khí lưu thông là 650ml, thể tích dự trữ thở là là 1.5L, tổng dung tích
phổi là 8L, và nhịp thở là 15 lần/phút.
Thông khí phế nang là?
A) 5 L/phút
B) 7.5 L/phút
C) 6.0 L/phút
D) 9.0 L/phút
14. B) Thông khí phế nang = Tần số * (VT - VD) = 15/phút * (650 - 150) =
7.5 L/phút
TMP12 472
15.Cuối kỳ hít vào, với thanh môn để mở, áp lực khoang màng phổi là
A) Lớn hơn áp suất khí quyển
B) Bằng áp suất khí quyển
C) Nhỏ hơn áp suất phế nang
D) Bằng áp suất phế nang
E) Lớn hơn áp suất phế nang
15. C) Ap lực trong khoang màng phổi trong thì hít vào luôn nhỏ hơn áp lực
phế nang hay áp suất khi quyển
TMP12 465-466
16.Một thí nghiệm tiến hành trên 2 đối tượng ( Đối tượng T và V) với thể tích
thí lưu thông xác định (1000ml), thể tích khoảng chết (200 ml) và tần số thở
(20 lần thở/phút). Đối tượng T tăng gấp đôi thể tích khí lưu thông và giảm
tần số thở 50%. Đối tượng V tăng gấp đôi tần số thở và giảm thể tích khí lưu
thông 50%. Điều nào sau đây mô tả chính xác nhất tổng thể tích khí lưu
thông (hay còn gọi là thông khí phút) và thông khí phế nang của đối tượng T
và V?
thông khí thông khí phế
phút nang
A T<V T=V
B T<V T>V
C T=V T<V
D T=V T=V
E T=V T>V
F T>V T<V
G T>V T=V
16. E) Tổng thông khí phổi bằng thể tích khí luu thông (Vt) nhân với tần số
hô hấp ( Freq). Thông khí phế nang = (Vt - Vd) x tần số, với Vd là thế tích
khoảng chết. Cả 2 thông số này có đặc điểm chung vớ tổng thông khí: Ðối
tượng T, 1000X10 = 10L/phút, Ðối tượng V, 500x20 = 10L/phút. Tuy nhiên,
đối tượng T có thông khí phế nang là 18L (là (2000-200)x10); trong khi đối
tượng V có thông khí phế nang chỉ 12L ( Ðó là (500-200)x40). VVấn đề
này một ta sâu hơn một trong số những biện pháp để tăng thông khí phế
nang là tăng thể tích khí lưu thông chứ không phải tăng tần số thở
TMP12 471-472
17.Một bé trai 10 tuổi khỏe mạnh thở một cách yên tĩnh dưới điều kiện thư
giãn. Thể tích khí lưu thông của bé là 400 ml và tần số thở là 12 lần/phút.
Điều nào sau đây miêu tả thông khí ở khu vực trên, giữa và dưới phổi?

Khu vực khu vực khu vực


trên giữa dưới
A Cao nhất thấp nhất trung
bình
B cao nhất trung thấp nhất
bình
C trung thấp nhất cao nhất
bình
D thấp nhất trung cao nhất
bình
E như nhau như như nhau
nhau
17. D) Vùng thấp hơn của phổi thông khí tốt hơn vùng cao và vùng giữa chỉ
có thông khí ngắt quãng. Sự khác biệt của 3 khu vực thông khí có thể giải
thích nhờ sự khác biệt khu vực của áp lực màng phổi. Áp lực màng phổi
thông thường ở khoảng -10 cmH2O ở khu vực trên và khỏang -2,5 cmH2O ở
khu vực dưới. Một áp lơcj âm ít hơn của màng phổi ở khu vực thấp hơn của
lồng ngực gây ra ít sự giãn nở của vùng thấp hơn trong điều kiện nghỉ ngơi.
Do đó, vùng thấp hơn ở phổi chỉ bị ép tương đối trong khi nghỉ nhưng giãn
tốt hơn khi thở ra so với vùng đỉnh
TMP12 493-494
18.Một người đàn ông 34 chịu đựng một vết thương do đạn bắn vào ngực gây
tràn khí màng phổi. Điều nào sau đây miêu tả chính xác nhất sự thay đổi của
thể tích phổi và lồng ngực cùa người đàn ông này, so với bình thường?
thể tích phổi thể tích lồng
ngực
A Giảm giảm
B giảm tăng
C giảm không đổi
D tăng giảm
E tăng tăng
F không đổi giảm
18. B) Cả lồng ngực và phổi đều có độ đàn hồi. Dưới điều kiện bình thường,
độ đàn hồi của phổi làm xẹp phổi cân bằng đúng với độ đàn hổi của lồng
ngực làm lồng ngực nở ra. Khi khong khí đi vào trong khoang màng phổi, áp
lực khoang màng phổi trở nên cân bằng với áp suất khí quyển- lồng ngực co
ra ngoài và phổi bị xẹp
TMP12 466-467
19.Sức cản của cây dẫn khí thấp đến mức một gradient áp lức 1cm nước
có thể đủ để gây ra dòng khí trong điều kiện nghỉ. Vị trí nào sau đây thường
có sức cản quan trong trong các tình trạng bệnh lý hô hấp gây nên hạn chế
thông khí phế nang?
a. Phế nang
b. tiểu phế quản
c. phế quản lớn
d. phế quản nhỏ
e. khí quản
19. B) phế quản lớn lớn hơn gần với khí quản có sức cản lớn hơn với dòng
khí so ở phổi bình thường. Tuy nhiên, trong tường hợp bệnh lý, các tiểu phế
quản nhỏ có vai trò lớn hơn nhiều trong việc hình thành sức cản vì (a) chúng
dễ bị tắc nghẽn hơn vì kích thướng bé và (b) chúng có một lượng cơ trơn
trên thành và vì thế dễ bị co lại
TMP12 473
20.Biểu đồ sau cho thấy sức cản của đường dẫn khí được biểu hiện như
một chức năng của thể tích phổi. Mối quan hệ nào mô tả chính xác nhất một
phổi bình thường?
 A) Sự tăng lên của thể tích phổi gây ra sự giảm sức cản của đường dẫn
khí, tức là đường kính đường dẫn khí tăng lên. Đường dẫn khí bị dính vào
các mô xung quanh, thứ làm cho chúng bị kéo mở khi phổi giãn ra. Điều này
được gọi là sự "keo dãn lan tỏa" có thể giải thích vì sao một người với bệnh
phổi tắc nghẽn thở dễ hơn tại thể tích phổi lớn hơn bình thường
TMP12 473,516
21.Đường thở có các cơ trơn ở trên thành của nó. Điều nào sau đây miêu
tả chính xác nhất tác dụng của acetylcholine và epinephrine lên đường thở?
Acetycholine Epinephrine
A Co co
B co giãn
C co không tác
dụng
D giãn co
E giãn giãn
F giãn không tác
dụng
G không tác co
dụng
H không tác giãn
dụng
 B) Trương lực cơ trơn trong đường thở bị điều khiển vởi hệ thần kinh tự
động cingx như epinephrine tuần hoàn. Kích thích vận động và bởi thần kinh
lang thang. Sự kích thích vào các receptor bởi norepinephrine và epinephrine
gây ra giãn phế nang. Hoạt động hệ đối giao cảm (cũng như acetycholine)
gây ra sự co phế quản. Chú ý rằng tác dụng của hệ thần kinh tự động lên
đường thở đối nghịch với tác động của chúng lên hệ mạch máu ngoại vi
TMP12 473
22.Một người đàn ông 67 tuổi nhập viện khoa cấp cứu của một bệnh viện đại
học vì đau ngực dữ dội. Một catheter Swan-Ganz được đặt vào trong động
mạch phổi, một quả bóng được thổi căng và áp lực phổi bít được đo. Áp lực
phổi bít được dùng trên lâm sàng để đo áp lực nào sau đây?
a. Áp lực nhĩ trái
b. Áp lực thất trái
c. Áp lực động mạch phổi thì tâm trương
d. Áp lực động mạch phổi thì tâm thu
e. Áp lực mao mạch phổi
 A) Thường sẽ không khả thi để đo áp lực nhĩ trái trực tiếp trên người bình
thường do sẽ khó khăn để đưa catheter qua buồng tim vào nhĩ trái. Một
catheter gắn bóng, có dẫn đường ( Catheter Swan-Ganz) được phát triển gần
30 năm trước để ước tính áp lực nhĩ trái nhằm kiểm soát các trường hợp nhồi
máu cơ tim>???. Khi quả bóng được bung trên một catheter Swan-Ganz, áp
lực được đo qua catheter, được gọi lá áp lực bít, xấp xí áp lực thất trái vì
những lý do sau: dòng máu chảy ở đoạn xa của catheter bị chặn tất cả các
đường về nhĩ trái, cho phép áp lực nhĩ trái được ước tính. Aps lực bít thực ra
cao hơn áp lực nhĩ trái một vài mmHg, phụ thuoojcv ào nơi đo áp lực bít,
nhưng vẫn cho phép sự thay đổi áp lực trong nhĩ trái được đo ở bệnh nhân bị
suy thất trái.
TMP12 478
23.Đáp án nào sau đây mô tả tốt nhất huyết động ở tuần hoàn phổi so với vòng
tuần hoàn hệ thống?
dòng sức cản Áp lực động
chảy mạch
A cao hơn cao cao hơn
hơn
B cao hơn thấp thấp hơn
hơn
C thấp cao thấp hơn
hơn hơn
D thấp thấp thấp hơn
hơn hơn
E bằng cao thấp hơn
nhau hơn
F bằng thấp thấp hơn
nhau hơn
 F) Hệ tuần hoàn phổi và hệ thống để nhận cùng một lượng máu vì
phổi nhận toàn bộ cung lượng tim (tuy nhiên, lượng máu chảy ra ở
thất trái thật ra 1%-2% cao hơn thất Phải vì máu từ tĩnh mạch phế
quản chảy vào tĩnh mạch phổi.) Tĩnh mạch phổi có độ cản tương đôi
thấp cho phép toàn bộ cung lượng tim đi qua mà không tăng áp k]ngj
lên quá cao. Áp lực động mạch phổi trung bình khoảng 15 mmHg,
thấp hơn nhiều so với áp lực tuần hoàn hệ thống 100mmHg
TMP12 477-479
24.Biểu đồ nào mô tả chính xác nhất tưới máu phổi khi tăng cung lượng
tim lên tối đa?

A) A
B) B
C) C
D) D
E) E.
 a) Dòng máu của phổi có thể tăng lên nhiều lần mà không gây ra sự tăng
áp lưc động mạch phổi lên nhiều vì 2 lý do: các động mạch trước đây đóng
mở ra (sự tăng thêm) và các động mạch giãn ra (sự giãn nở). Sự tăng thêm
và sự giãn nở của mạch phổi đều để giảm sức cản thành mạch (và vì thế để
duy trì áp suất mạch phổi thấp) khi cung lượng tim tăng lên
TMP12 480
25.Một người phụ nữ 30 tuổi tiến hành nghiệm pháp valsava 30 phút sau khi ăn
trưa. Điều nào sau đây mô tả sự thay đổi thể tích tuần hoàn hệ thống và tuần
hoàn phồi xảy ra trên người phụ nữ này?
tuần hoàn tuần hoàn hệ
phổi thống
A giảm giảm
B giảm tăng
C giảm không đổi
D tăng giảm
E tăng tăng
F tăng không đổi
G không đổi giảm
H không đổi tăng
I không đổi không đổi
 B) khi một người tiến hành nghiệm pháp valsalva (Đẩy khí ra khi đóng
nắp thanh môn), áp suất cao trong phổi có thể đẩy đến 250ml máu từ tuần
hoàn phổi vào tuần hoàn hệ thống. Phổi có một chức năng dự trữ máu quan
trọng, tự động chuyển máu sang tuần hoàn hệ thống như một đáp ứng bù
trừ cho việc chảy máu và những điều kiện khác khiến cho thể tích tuần
hoàn hệ thống quá thấp
TMP12 478
26.Một người đàn ông 32 tuổi lái xe lên đỉnh của Pikes Peak nơi áp lực oxi là
85mm Hg. Điều nào sau đây mô tả chính xác nhất tác động của môi trường
oxi thấp lên sức cản của tuần hoàn phổi và hệ thống?
Sức cản tuần Sức cản tuần
hòa hệ thống hoàn phổi
A giảm giảm
B giảm tăng
C giảm không đổi
D tăng giảm
E tăng tăng
F tăng không đổi
G không đổi giảm
H không đổi tăng
I không đổi không đổi
 D)Sẽ hết sức quan trọng để máu được phân phối đến những khoảng của
phổi nơi mà phế nang được oxi hóa nhiều nhất. Khi phân áp oxi của phế
nang bị giảm xuống dưới mức bình thường, các mạch máu liền kề sẽ co lại
gây nên sức cản tăng lên đến 5 lần khi oxi giảm rất nhiều. Điều ngược lại
được quan sát thấy ở tuần hoàn hệ thống khi mà mạch máu lại giãn ra đáp
ứng với nồng độ oxi thấp (ví dụ: giảm sức cản).
TMP12 479

27.Đi lên từ tư thế đứng yên lặng để trèo lên một cái cầu thang, điều nào sau
đây sẽ xuất hiện?
A) ↑ ↑
B) ↑ ↓
C) ↓ ↑
D) ↓ ↓
E) ↑ ↔
F) ↓ ↔
 A) trong khi đứng sẽ có sự tăng dòng máu đến đáy phổi và giảm dòng máu đến
đỉnh phổi. Khi thể dục sẽ có sự tăng dòng máu song song ở toàn phổi
TMP12 479-480
28.Một người đàn ông 65 tuổi bị khí phế thũng do 34 năm hút thuốc nhập viện
vì khó thở. Với những xét nghiệm sâu hơn áp lực động mạch phổi trung bình
được xác định là 45 mm Hg lúc nghỉ. Ông ấy bị thiếu oxi (PO2=49 mmHg),
tăng giữ CO2 (85 mmHg) và bị toan máu nhẹ. Thay đổi của hệ tim mạch và
oxi gây ra do điều nao sau đây?
a. Tăng Pco2 động mạch
b. Tăng hoạt động hệ đối giao cảm
c. giảm Po2 phế nang
d. giảm pH
e. giảm sức cản của phổi
 C) giảm PO2 phế nang sẽ gây ra sự tăng sức cản mạch phổi, gây ra
tăng áp mạch phổi
TMP12 479
29.Bệnh nào sau đây sẽ gây tăng sức cản dòng máu của phổi?
a. Tiêm noreprnephrine tĩnh mạch
b. Hít vào đến dung tích phổi toàn phần
c. thở oxi 5%
d. Phổi ở dung tích cặn chức năng
 B) Sự thở vào cho đến TLC và thở ra cho đến thể tích cặn sẽ tăng
sức cản dòng máu phổi. Giamr oxi phế nang sẽ tăng sức cản dòng
máu. Phổi ở trạng thái FRC sức cản sẽ ở mức thấp nhất.
30.Một người đàn ông 19 tuổi bị bòng nặng hoàn toàn hơn 60% bề mặt cơ thể
của anh ấy. Một nhiễm trùng hệ thống Pseudomonas aeruginosa xảy ra và
phù phổi nặng xảy ra 7 ngày tiếp theo. Dữ liệu có được từ bệnh nhân này
bao gồm: Áp lực keo huyết tương, 19mmHg; Áp lực thủy tĩnh khoan màng
phổi, 17mmHg; Áp lực thủy tĩn trong lòng ruột, 1mmHg. Điều gì sau đây
xảy ra trong phổi của bệnh nhân là kết quả của việc bỏng và hậu quả của
nhiễm trùng?
Dòng Áp lực Tính thấm
lympho keo huyết mao mạch
tương phổi
A giảm giăm giảm
B tăng giăm giảm
C tăng giảm tăng
D tăng tăng giăm
E tăng tăng tăng
30. C) Một nhiễm trùng Pseudomonas có thể tăng tính thấm mao mạch của
phổi và những nơi khác của cơ thể, điều dẫn đến sự mất quá mức của protein
huyết tương vào khoảng kẽ. Sự mất các protein huyết tương từ hệ mạch gây
ra giảm áp suất keo huyết tương từ mức bình thường là 28mmHg xuống còn
19mmHg. áp lực thủy tính mao mạch vẫn giữ ở mức 7mmHg, nhưng đôi
khi tăng cao hơn mức bình thường, làm nặng thêm tình trạng phù. Aps lực
thủy tĩnh của dịch ở trong khoảng kẽ tăng từ giá trị bình thường khoảng -
5mmHg lên 1mmHg, điều làm giảm lượng dịch mất từ mao mạch. Sự quá
mức dịch ở khoảng kẽ (Phù) làm tăng dòng Lympho
TMP12 481-483

31.Thể tích khí lưu thông bình thường của một bệnh nhân là 400ml với khoảng
chết là 100ml. Nhịp thở là 12 lần/phút. Người này được đặt thông khí để
phẫu thuật và thể tích khí lưu thông là 700 với nhịp thở là 12. PCO2 phế
nang ước tính của bệnh nhân này là?
a. 10
b. 20
c. 30
d. 40
e. 45
 B) PCO2 phế nang bình thường là 40mmHg. Thông khí phế nang
bình thường cho người này là 3,6L/phút, khi thông khí nhân tạo thông
khí phế nang là 7,2L/phút. Sự gấp đôi của thông khí phế nang dẫn đến
PCO2 còn một nửa. Vì thế PCO2 là 20
TMP12 488
32.Các lực chi phổi sự khuyeechs tán của khí qua màng sinh học bao gồm sự
chênh lệch áp lực qua màng (ΔP), diện tích cắt ngang của màng (A), độ hấp
thu của khí (S), khoảng cách khuyeechs tán (d) và trọng lượng phân tử của
khí (MW). Những thay đổi nào sau đầy tăng sự khuyêchs tán của một khí
qua một màng sinh học?
ΔP A S d MW
A tăng tăng tăng tăng tăng
B tăng tăng tăng tăng giả
m
C tăng giả tăng giả giẳ
m m m
D tăng tăng tăng giả tăng
m
E tăng tăng tăng giả giả
m m
 E) Luật Fick của tình trạng khuuyeechs tán cho rằng tốc độ
khuyeechs tán của một khí qua một màng sinh học tỷ lệ thuận với ΔP,
A, và S và tỷ lệ nghịch với d và căn bậc 2 của MW của khí
ví dụ: D α (ΔP × A × S) / (d × MW−2). Gradient áp suất càng lớn
thì tốc độ khuyeechs tán càng lớn. Diện tích cắt ngang của màng
càng lớn thì tổng số phân tử có thể khuyeechs tán qua màng càng
lớn. Độ hòa tan của khí càng lớn thì sẽ có càng nhiều số phân tử
có sẵn để khuyeechs tán qua một áp suất cho trước. Khi khoảng
cách của đường khuyeechs tán càng ngắn, thời gian cần sẽ ít hơn
để các phân thử khuyeecsh tán qua toàn bộ khoảng cách. Khi
trọng lượng phân tử giảm, tốc độ của chuyển động nhiệt của phân
tử sẽ cao hơn, cũng làm tăng tốc độ khuếch tán.
TMP12 486-487
33.Một người có phổi bình thường tại mực nước biển (760mmHg) thở 50% oxi.
PO2 xấp xỉ tại phế nang là bao nhiêu?
a. 100
b. 159
c. 268
d. 330
e. 380\
31. c) Để tính PO2 lý tưởng cần nhớ rằng không khí được làm ẩm khi nó đi
vào cơ thể. Vì thế không khí được làm ẩm sẽ ảnh hưởng đến tổng áp suất khí
quyển (760) - áp suất hơi nước (47). áp suất tính ra là (760 - 47) =713
mmHg. oxi là 50% tổng lượng khí nên áp suất riêng phần của oxi là 716*0.5
=316 mmHg. Để điều chỉnh CO2 trong phế nang, sau đso cần trờ đi áp suất
riêng phần cua CO2 chia cho phân số hô hấp (thường là 0.8). Vì thế, PO2
phế nang= PiO2 - (PCO2/R) =318 - (40/0.8) = 318 − 50 = 268 mm Hg.
TMP12 487–489
34.Một đứa trẻ ăn một viên kẹo tròn đường kính xấp xỉ 1 và 1,5 cm và hit vào
đường thở làm chặn phế quản trái của đứa bé ấy. Những thay đổi này được
mô tả sau đây sẽ xảy ra?
PCO2 phế PO2 phế Po2
nang phổi nang phổi động
trái trái mạch
hệ
thống
A ↑ ↑ ↔
B ↑ ↔ ↑
C ↓ ↓ ↓
D ↑ ↑ ↑
E ↑ ↓ ↓
 E) Khi có sự cản trở của một đường thở thì sẽ không có sự lưu
thông của không khí. Vì thế không khí trong phế nang sẽ đạt đến
tương đương với khí máu động mạch. Vì thế PO2 giảm từ 100 xuống
40, PCO2 tăng từ 40 lên 45 và PO2 hệ thống sẽ giảm vì có sự giảm
hấp thu từ phế nang và vì thế giảm sự khuếch tán O2 từ phế nang
TMP12 492-493
35.Trong quá trình tập luyện, sự gắn oxi của máu tăng không chỉ bởi tăng thông
khí phế nang mà còn bởi khả năng khuyeechs tán lớn hơn của màng tế bào
để vận chuyển oxi vào máu. Những sự thay đổi nào sau đây sẽ xảy ra trong
quá trính tập luyện?
Diện tích bề mặt tỷ số thông
của màng hô hấp khí-
khuyeechs
tán
A Giamr cải thiện
B tăng cải thiện
C tăng không đổi
D không đổi cải thiện
E không đổi không đổi
 B) Khả năng khuếch tán của khí là lượng khí có thể khuếch tán
được qua một màng trong mỗi phút với sự chênh lệch áp suất
1mmHg. Khả năng khuếch tán của O2 tăng trong khi thể dục bằng
cách (a) mở các mao mạch trước đây đóng (sự tăng thêm) và giãn các
mao mạch đã mở trước đây (sự giãn nở), cả 2 đều tăng diện tích tiếp
xúc của máu với vùng oxi có thể khuếch tán, và (b) cải thiện tỷ số
thông khí- tưới máu tức là tăng khả năng kết nối giữa thống khí phế
nang và tuố máu của mao mạch phế nang
TMP12 491-492
36.Khả năng khuyeechs tán của khí là thể tích của một khí được khuyeechs tán
qua một màng mỗi phút với một chêch lệch áp lực là 1mmHg. Loại khí nào
sau đây được sử dụng để đánh giả khả nang khuyeechs tán của oxi của phổi?
a. carbon dioxide
b. carbon monoxide
c. khí cyanide
d. nitrogen
e. oxi
 B) Sẽ không có tính thực tế nếu đo khả năng khuếch tán oxi trực
tiếp vì sẽ không thể nào đo được một cách chính xác phân áp oxi ở
mao mạch phổi. Tuy nhiên, khả năng khuếch tán của carbon
monoxide (CO) có thể đo một cách chính xác vì phân áp CO ở mao
mạch phổi là không ở điều kiện bình thường. Khả năng khuếch tán
CO sau đó được sử dụng để tính toán khả năng khuếch tán của oxi
nhờ tính dựa trên sự khác biệt trong hệ số khuếch tán giữa oxi và CO.
Biết được tốc độ trao đổi của CO qua màng thường giúp ích cho việc
ước tính sự có mặt của cac bệnh nhu mô phổi khi mà test khí phế
dung/ đo thể tích phổi gợi ý có sự giảm dung tích sống, thể tích cặn
hay tổng dung tích phổi
TMP12 492
37.Một sinh viên y 23 tuổi có áp lực trộn lẫn oxu và carbon dioxide lần lượt là
40 và 45 mmHg. Một nhóm các phế nang không được thông khí trên sinh
viên này vì đờm dãi đã chặn một nhánh đường thở cục bộ. Áp lực oxi và
carbon dioxide phế nang phần sau chỗ bị chặn là? (tính bằng mmHg)
carbon oxy
dioxide
A 40 100
B 40 40
C 45 40
D 50 50
E 90 40
 C) Vì máu tưới ở mao mạch phổi là máu tĩnh mạch quay về phổi
(vd: máu hỗn hợp tĩnh mạch) từ tuần hoàn hệ thống, khi đó khí trong
máu này và trong phế nang cân bằng. Vì thế, khi một đường thở bị tắc
nghẽn, khí trong phế nang cân bằng với khí trong tĩn mạch và áp lực
riêng phần của khí trong cả máu và phế nang được xác định
TMP12 492-494
38.Một người đàn ông 45 tuổi ở mức nước biển với phân áp oxi lý tưởng là
149mmHg, phân áp nitrogen là 563mmHg, và áp lực hơi nước là 47mmHg.
Một khối y nhỏ chèn vào một nhánh mạch phổi và chặn hoàn toàn dòng máu
chảy đến một nhóm phế nang nhỏ. Phân áp oxi và carbon dioxide của phần
phế nang không được tưới máu là (tính bằng mmHg)
Carbon dioxide Oxi
A 0 0
B 0 149
C 40 104
D 47 149
E 45 149
 B) Khí trong phế nang thường cân bằng với khi trong máu tĩnh
mạch hỗn hợp tưới cho nó nên hỗn hợp khí tron phế nang và mao
mạch phổi được xác định. Khi một nhóm phế nang không được thông
khí, phức hôn khí trong phế nang trở nên cân bằng với hỗn hợp khí lý
tưởng, có áp lực riêng phần oxi là 149mmHg và áp lực riêng phần
carbon dioxide khoảng 0 mmHg
TMP12 492-494
39.Đồ thị O2-CO2 dưới đây chỉ ra đường tỷ số thông khí-tưới máu của một
phổi bình thường. Điều nào sau đây mô tả chính xác nhất ảnh hưởng của
việc tăng tỷ số thông khí-tưới máu lên PO2 và PCO2 phế nang?

Carbon dioxide Oxi


A Giamr giảm
B giảm tăng
C giảm không đổi
D tăng giảm
E tăng tăng
 D) Một sự giảm tỷ số thông khí- tưới máu (Va/Q) được mô tả bằng
việc di chuyển đường cong sang bên trái của đường cong thông khí,
tưới máu bình thường. Mỗi khi Va/Q giảm dưới mức bình thường, sẽ
có sự mất cân bằng của thông khí để cung cấp lượng oxi cần thiết để
oxi hóa dòng máu chảy qua mao mạch phế nang (vd: PO2 giảm). Vì
thế, một phần ổn định máu tĩnh mạch đi qua mao mạch phổi không
được oxi hóa. Khu vực kém thông khí của phổi cũng tích tụ Carbon
dioxide khuếch tán vào phế nang từ máu tĩnh mạch. Kết quả của việc
giảm Va/Q (Dịch chuyển đường Va/Q sang trái) lên PO2 và PCO2
phế nang được chỉ ra trong biểu đồ, ở đó, PO2 giảm và PCO2 tăng
TMP12 492-494
40.Trong điều kiện nào sau đây PO2 phế nang tăng và PCO2 phế nang giảm?
a. tăng thông khí phế nang và không thay đổi chuyển hóa
b. giảm thông khí phế nang và không thay đổi chuyển hóa
c. tăng chuyển hóa và không thay đổi thông khí phế nang
d. có sự tăng tương ứng chuyển và thông khí phế nang
 a) PO2 phế nang phụ thuộc vào khí lý tưởng và PCO2 phế nang.
PCO2 phế nang là sự cân bằng giữa thông khí phế nang và sự sản sinh
CO2. Để giảm PCO2 phế nang cần phải tăng thông khí phế nang liên
quan đến chuyển hóa. PO2 thấp không trực tiếp ảnh hưởng đến PCO2
nhưng có thể kích thích hô hấp (Nếu PO2 đủ thấp). Một sự tăng
chuyển hóa với thông khí phế nang không đổi sẻ tăng PCO2. Một sự
gấp đôi chuyển hóa đi kèm với gấp đôi thông khí phế nang sẽ không
ảnh hưởng đến PCO2
TMP12 488-489
Câu 41 và 42
41.Một người đàn ông 67 tuổi có một khối u đặc đẩy vào đường thở và làm tắc
một phần dòng khí vào các phế nang ở phía sau. Điểm nào trên đồ thì thông
khí-khuyeechs tán của CO2-O2 tương ứng với khí trong phế nang của các
phế nang ở xa này?

a. A
b. B
c. C
d. D
e. E
 A) Khi thông khí phế nang giảm đến không (Va/Q=0) khí phế nang
sẽ cân bằng với máu động mạch đi vào phổi, thứ làm cho phức hợp
khí của phế nang trở nên xác định với máu. Điều này xảy ra ở điểm A,
nơi PO2 phế nang là 40mmHg và PCO2 phế nang là 45mmHg, được
chỉ ra trên đồ thị A. Một sự giảm xuống của Va/Q (gây ra bởi sự tắc
nghẽn một phần đường thở) gây ra PO2 và PCO2 tiến dần đến giả trị
này khi Va/Q=0
TMP12 492-494
42.Một người đàn ông 55 tuổi có huyết khối phổi làm tắc một phần dòng máu
đến phổi phải của ống ý. Điểm nào trên đồ thì thông khí-khuyeechs tán của
CO2-O2 tương ứng với khí trong phế nang của các phế nang phổi phải của
ông ấy?
a. A
b. B
c. C
d. D
e. E
 E) Một huyết khối phổi giảm dòng máu đế phần phổi bị ảnh hưởng
gây nên thông khí trở nên quá mức với dòng máu. Khi huyết khổi
chặn hoàn toàn tất cả dòng máu đến một khu vực phổi, phức hợp khí
lý tưởng đi vào phổi cân bằng với dòng máu bị chặn lại trong mao
mạch phế nang trong một thời gian ngắn, phức hợp khí của phế nang
được xác định là khí lý tưởng, Tình huống này khi mà Va/Q cân bằng
đến vô giới hạn tương ứng với điểm E trên đồ thị (khí lý tưởng). Một
sự tăng Va/Q gây ra do sự tắc nghẽn một phần dòng máu trong trường
hợp nàY gây ra PO2 và PCO2 đạt đến giá trị khi VA/Q = ∞.
TMP12 49
43.Sơ đồ sau cho thấy một phổi với một shunt lớn, tại đấy máu của tĩnh mạch bị
trộn lẫn đi qua các khu vực trao đổi khí của phổi. Thở tại khí phòng với áp
suất riêng phần oxi như hình. Áp lực oxi của bệnh nhân trong máu động
mạch (tính bằng mmHg) khi người này thở 100% oxi và trong môi trường áp
lực oxi lý tưởng hơn 600 mmHg là?

a. 40
b. 55
c. 60
d. 175
e. 200
f. 400
g. 600
 C) Thở 100% oxi đã làm giới hạn tác động lên PO2 động mạch khi
nguyên nhân gây ra hạ oxi máu là một shunt mạch. Tuy nhiên, thở oxi
100% tăng PO2 động mạch lên quá 600mmHg trên một đối tượng
bình thường. Với một shunt mạch, PO2 động mạch được xác định bởi
(a) máu được oxi hóa cao tại đầu cuổi mao mạch (PO2 >600mmHg)
phần đã đi qua phần được thông khí của phổi, và (b) máu qua shunt đã
đi tắt qua phần được thông khí của phổi và do đó có áp lực riêng phần
oxi tương đương với máu hòa trộn tĩnh mạch (PO2=40mmHg). Một
hỗn hợp 2 máu gây ra sự hạ PO2 lớn vì đường cong phân ly oxi quá
dẹt ở phần trên của nó
TMP12 493-494
44.Biểu đồ tiếp theo cho thấy 2 đơn vị phổi (S và T) với nguồn cấp máu của nó.
Đơn vị phổi S có quan hệ lý tưởng giữa dòng máu và thông khí. Đơn vị phổi
T có một sự chèn ép vào dòng màu. Quan hệ giữa khoảng chết phế nang
(Dalv), khoảng chế sinh lý (DPHY) và khoảng chết giải phẫu (DANT) của
những đơn vị này là?

Đơn vị phổi S Đơn vị phổi T


A DPHY < DPHY =
DANAT DANAT
B DPHY = DPHY >
DALV DALV
C DPHY = DPHY <
DANAT DANAT
D DPHY = DPHY >
DANAT DANAT
E DPHY > DPHY <
DANAT DANAT
 D) Khoảng chết giải phẫu (DANAT) là không khí mà một người
thở vào được chứa trong đường thở nhưng không bao giờ chạm đến
phế nang. Khoảng chết phế nang (DALV) là không khí trong phế
nang được thông khí nhưng không được tưới máu. Khoảng chết sinh
lý là tổng của khoảng chết giải phẫu và khoảng chết phế nang (ví dụ:
DPHY = DANAT + DALV). DALV là không ở đơn vị phối S (đơn vị
phổi lý tưởng) và DANAT và DPHY là bằng nhau. Biểu đồ chỉ ra một
nhóm phế nang được tưới máu kém đơn vị phổi T), điều có nghĩa là
DALV là quan trọng. Vì thế, DPHY lớn hơn DANAT hay DALV ở
đơn vị phổi T
TMP12 489,493-494
45.Một sinh viên viên Y 32 tuổi đã tăng gấp 4 cung lượng tim khi tập thể dục
nặng. Đường cong nào trong sơ đồ sau có khả năng cao nhất biểu hiện sự
thay đổi áp suất oxi xảy ra trong dòng máu từ đầu động mạch đến đầu tĩnh
mạch của mao mạch phổi ở trên sinh viên này?
 E) PO2 của máu trộn tĩnh mạch đi vào mao mạch phổi thông
thường là 40mmHg và PO2 tại máu đầu tận mao tĩnh mạch thông
thường tương đương với khí trong phế nang (104 mmHg). PO2 của
máu phổi thông thường tăng lên bằng khí phế nang khi máu đi được
1/3 chiều dài của mao mạch, gần bằng 104 mmHg. Vì thế, đường
cong B biểu thị ở trạng thái nghỉ. Khi thế dục, cung lượng tim có thể
tăng lên đến 7 lần, nhưng máu mao mạch phổi vẫn gần như bão hòa
oxi khi đi qua phổi. vì dòng chảy càng nhanh khi thể dục, oxi càng ít
có thời gian khuếch tán vào máu mao mạc phổi, vì thế PO2 của máu
mao mạch phổi không chạm đếng giá trị tối đa cho đến khi nó chạm
đến đầu tận mao động mạch. Mặc dù đường D và E đểu chỉ ra rằng sự
bão hòa oxi của máu xảy ra ở đầu tận mao mạch, chú ý tằng chỉ đường
E chỉ một PO2 thấp tại 25 mmHg tại đầu mao động mạch của mao
mạch phổi. thứ thường xảy ra với máu hỗn hợp mao mạch trong khi
thể dục
TMP12 495-496
46.Đồ thị cho thấy sự thay đổi của áp suất riêng phần của oxi và carbon
dioxide trong dòng máu từ đầu tĩnh mạch đến đầu động mạch của mao mạch
phổi. Đồ thì nào mô tả tốt nhất quan hệ bình thường giữa PO2 (đường đỏ) và
PCO2 ( đường xanh trong điều kiện nghỉ)
 A) PO2 của máu trộn tĩnh mạch vào mao mạch phổi tăng lên trong khi
nó được di chuyển qua mao mạch (từ 40 mmHg lên 104 mmHg) và
PCO2 giảm đồng thời từ 45 xuống 40 mmHg. Vì thế, PO2 được biểu
diễn bởi đường đỏ và PCO2 được biểu diễn bởi đường xanh trong đồ thị.
Trong điều kiện nghỉ, oxi có gradient áp lực 64 mmHg (104 - 64 = 64) và
carbon dioxide có gradient áp lực 5 mmHg ( 45 - 40 = 5 mmHg) giữa
máu tại mao động mạch và phế nang. Mặc dù sự khác biệt lớn giữa
gradient áp lực giữa oxi và carbon dioxide, cả 2 khí đều cân bằng với khí
phế nang khi máu đi hết 1 phần 3 khoảng cách qua mao mạch tại điều
kiện nghỉ (lựa chọn A). Điều này có thế là do CO2 khuếch tán nhanh hơn
oxi 20 lần.
TMP12 496-497
47.Một phụ nữ 17 tuổi đang đạp xe mà không có mũ bảo hiểm. Cô ấy ngã và
đập đầu. Trong phòng cấp cứu, cô ấy bất tỉnh và được nhận hỗ trợ thông khí.
Khí máu của cô ấy là:
PaO2 =52 mmHg, PaCO2 = 75 mmHg, pH=7,15 và HCO3- = 31 mM.
Phần lớn CO2 được chuyển đi dưới dạng:
a. CO2 gắn vào các protein huyết tương
b. CO2 gắn vào hemoglobin
c. các Ion bicarbonate
d. Phân ly
 C) CO2 được vận chuyển dưới 3 dạng, phân ly (7% của toàn bộ), Gắn
trực tiếp với hemoglobin (23%), và nó được chuyển đổi thành acid carbonic
và vận chuyển dưới dạng HCO3- với H+ gắn với hemoglobin (70%). Vì vậy
phần lớn CO2 được vận chuyển dưới dạng ions bicarbonate
TMP12 501-502
48.Đồ thị dưới đây cho thấy đường cong phân ly oxi-hemoglobin bình thường.
Đường nào sau đây cho thấy giá trị xấp xỉ của hemoglobin bão hòa (% Hb-
O2), áp suất riêng phần của oxi ( PO2) và thành phần oxi cho máy được gắn
oxi rời khỏi phổi và máu bị khử quay về phổi từ các mô?

 C) Máu tĩnh mạch phổi gần như 100% bão hòa oxi, có PO2 khoảng 104
mHg, và mỗi 100 ml máu mang khoảng 20 m/s oxi (vd: nồng độ oxi là
khoảng 20 vol%). Xấp xỉ 25% oxi được vận chuyển trong máu động mạch
được sử dụng bởi tế bào trong điều kiện nghỉ. Vì thế, giảm lượng máu về
phổi khoảng 75% bão hòa oxi, có PO2 khoảng 40mmHg, và có thành phần
oxi khoảng 15 vol%. Chú ý rằng cần thiết để biết một giá trị cho máu được
oxi hóa và khử và 2 giá trị này được yêu cầu trong câu hỏi có thể đọc được
từ đường cong phân ly oxi-hemoglobin
TMP12 496, 498-499
49.PO2 động mạch là 100mmHg và Pco2 động mạch là 40 mmHg. Tổng dòng
máu của tất cả các cơ là 700 ml/phút. Có một sự hoạt đông của thần kinh
giao cảm dẫn đến giảm dòng máy còn 350 ml/phút. Điều nào sau đây xảy
ra?
PO2 tĩnh PCO2 tĩnh
mạch mạch
A ↑ ↓
B ↓ ↑
C ↓ ↔
D ↔ ↑
E ↑ ↑
F ↓ ↓
G ↔ ↔
 B) PO2 mô là sự cân bằng giữa việc vận chuyển và sử dụng. Với sự giảm
của dòng máu, mà không có sự thay đổi trong chuyển hóa sẽ giảm PO2 động
mạch (giảm vận chuyển nhưng không thay đổi chuyển hóa) và tăng PCO2
tĩnh mạch (giảm thải trừ)
TMP12 496-497
50.Điểm nào trong các hình sau đại diện cho máu động mạch ở một người thiếu
máu nặng?
hình 1 hình 2
A D D
B E E
C D E
D E D
 D) Khi một người thiếu máu, sẽ có sự giảm các thành phần. Oxi bão hòa
trong hemoglobin và phân áp riêng phần oxi máu động mạch không ảnh
hưởng đến nồng độ hemoglobin trong máu.
TMP12 498-499
51.Một phụ nữ 34 tuổi bị thiếu máu với nồng độ hemoglobin là 7.1 g/dL.
Những sự thay đổi bào sau đây xảy ra trên người phụ nữ này, so sánh với
bình thường?
PO2 PO2 trộn 2,3
động lẫn trong diphophos-
mạch mao glycerate
mạch
A giảm giảm tăng
B giảm giảm bình
thường
C giảm bình giảm
thường
D tăng giảm bình
thường
E tăng tăng tăng
F tăng bình giảm
thường
G bình giảm giảm
thường
H bình giảm tăng
thường
I bình bình bình
thường thường thường
 H) Khả năng mang oxi của máu giảm ở một người thiếu máu, nhưng PO2
động mạch và oxi bão hòa của hemoglobin đề bình thường. Sự giảm của
thành phần oxi động mạch được bù trừ bởi sự tăng thải trừ của oxi trong
hemoglobin, thứ giảm PO2 của máu tĩnh mạch. Sự tăng thải trừ oxi tại mô
được tăng lên nhờ sự tăng nồng độ 2,3 DPG ở một người thiếu máu vì 2,3
DPG gây ra sự dịch chuyển đường cong phân ly oxi-hemoglobin sang phải.
TMP12 498-500
52.Đường cong phân ly oxi-hemoglobin nào sau đây tương ứng với máu bình
thường (đường đỏ) và máu chứa Carbon monoxide (đường xanh)
 E) Carbon monoxide (CO) kết hợp với hemoglobin tại cùng vị trị trên
phân tử hemoglobin như oxi và vì thay thế vị trí của oxi từ hemoglobin,
giảm nồng độ bão hòa oxi. Vì CO gắn với hemoglobin (dưới dạng
carbonhemoglonin) có độ bền gấp 250 lần so với oxi, kể cả một lượng nhỏ
CO trong máu có thể giới hạn nghiêm trọng khả năng mang oxi của máu. Sự
xuất hiện của carboxyhemoglobin cũng dịch chuyển đường cong phân ly
sang trái, thứ sau đó làm giới hạn khả năng vận chuyển oxi của mô.
TMP12 499-501
53.Đường cong phân ly oxi-hemoglobin nào sau đây tương ứng với máu bình
thường (đường đỏ) và máu trong khi thể dục (đường xanh)

 B) Trong luyện tập, nhiều yếu tố làm dịch chuyển đường cong phân ly
oxi-hemoglobin sang phải, thứ phục vụ cho việc tăng thêm một lượng oxi
đến các sợi cơ đang vận động. Những yếu tố này bao gồm tăng lượng
carbondioxide được thải trừ, tăng nồng độ ion hydrogen trong máu mao
mạch cơ, và tăng nhiệt độ do sự tạo nhiệt từ các cơ đang vận động. Sự dịch
chuyển sang phải của đường cong oxi-hemoglobin cho phép thêm oxi được
giải phóng từ cơ tại một phân áp riêng phần oxi trong máu cho trước.
TMP12 499-500
54.Đường cong phân ly oxi-hemoglobin nào sau đây tương ứng với máu người
lớn (đường đỏ) và máu trẻ em (đường xanh)

 C) Sự khác biệt về cấu trúc giữa hemoglobin bào thai và hemoglobin


người lớn làm cho hemoglobin của bào thai không có khả năng phản ứng với
2,3-DPG và do đó có ái lực cao hơn với oxi tại một phân áp riêng phần oxi
cho trước. Đường cong phân li của bào thai do đó dịch chuyển sang trái một
cách tương đối so với đường cong của người lớn. Thông thường, áp suất oxi
của bào thai thường thấp, và do đó sự dịch chuyển sang trái tăng khả năng
lấy oxi của nhau thai.
TMP12 499-500
55.Một người bị thiếu máu có nồng độ hemoglobin (Hb) là 12g/dL. Anh ấy bắt
đầu thể dục và sử dụng 12ml O2. PO2 trong máu hòa trộn tại tĩnh mạch là
bao nhiêu?
a. 0 mmHg
b. 10 mmHg
c. 20 mmHg
d. 40 mmHg
e. 100 mmHg
 C) Mỗi gram hemoglobin có thể bình thường mang 1,34 mililiters
oxi. Hb=12g/Dl. Lượng oxi động mạch = 12 * 1.34 = 16 ml O2/dL.
Sử dụng oxi là 12ml O2/Dl gây ra nồng độ bão hòa máu động mạch
25%. Với nồng độ bão hòa động mạch 25% PO2 tĩnh mạch nên gần
với 20 mmHg
TMP12 499-500
56.Carbon dioxide được vận chuyển trong máu dưới trạng thái phân ly, dưới
dạng ion bicarbonate, và dưới dạng kết hợp với hemoglobin (
Carbaminohemoglobin). Điều nào sau đây mô tả tốt nhất mối quan hệ về số
lượng giữa 3 cơ chế vận chuyển carbon dioxide trong máu động mạch dưới
điều kiện bình thường (tính theo phần trăm)?
dạng phân ion carbamin
ly bicarbona o-
te hemoglob
in
A 7 70 23
B 70 23 7
C 23 70 7
D 7 23 70
E 70 7 23
F 23 7 70
 A) Phần lớn carbon dioxide (70%) được vận chuyển trong máu
dưới dạng ion bicarbonate. Carbon dioxide phân ly phản ứng với nước
tạo thành carbonic acid (hầu hết ở trong tế bào hồng cầu), thứ phân ly
vào thành ion bicarbonate và hydrogen. Carbon dioxide cũng phản
ứng với các đuôi amin của phân tử hemoglobin để hình thành nên
phức hợp carbaminohermoglobin, thứ chiếm 23% của lượng
carbondioxide được vận chuyển trong máu. Phần carbon dioxide còn
lại (7%) được vận chuyển dưới dạng phân ly
TMP12 502-503
57.Một sinh viên Y 26 tuổi với chế độ ăn bình thường có tỷ số trao đổi hô hấp
là 0.8. Bao nhiêu oxy và carbon dioxide được trao đổi giữa phổi và mô ở
sinh viên này (Tính bằng mm khí/ 100ml máu)?
oxi carbon dioxide
A 4 4
B 5 3
C 5 4
D 5 5
E 6 3
F 6 4
 C) Tỷ số trao đổi hô hấp (R ) bằng với lượng carbon dioxide thở ra
chi cho lượng oxi hít vào. Một giá trị là 0.8 vì thế tức là lượng carbon
dioxie sản sinh ra trong mô là 80% lượng oxi sử dụng bởi mô, thứ
cũng có nghĩa là lượng carbondioxe được vận chuyển trong mô đến
phổi mỗi 100ml máu là 80% lượng oxi được vận chuyyeern đến mô
mỗi 100 ml máu. Lựa chọn C là câu trả lời duy nhất ở đó tỷ số của
carbon dioxide chia cho oxi là 0.8 (4/5-0.8). Mặc dù R thay đổi dưới
các điều kiện chuyển hóa khác nhau, thay đổi từ 1.00 ở những người
chỉ tiêu thụ carbonhydrate đến 0.7 ở những người chỉ tiêu thụ chất
béo, giá trị trung bình của R gần với 0.8.
TMP12 503
58.Carbon dioxide được vận chuyển từ mô về phổi chủ yếu dưới dạng ion
bicarbonate. So sánh với tế bào hồng câu trong động mạch, điều nào sau đây
mô tả chính xác tế bào hồng cầu trong máu tĩnh mạch?
nồng độ Clo nội thể tích tế bào
bào
A giảm giảm
B giảm tăng
C giảm không đổi
D tăng giảm
E tăng không đổi
F tăng tăng
G không đổi giảm
H không đổi tăng
I không đổi không đổi
 F) Carbon dioxide được phân ly kết hợp với nước ở trong tế bào
hồng cầu để hình thành acid carbonic, Thứ phân ly thành bicarbonate
và ion hydrogen. Nhiều ion Bicarbonate khuếch tán ra khỏi hồng cầu
trong khi ions Chloride khuếch tán vào hồng cầu để duy trì sự cân
bằng điện tích. Hiện tượng, gọi là sự dịch chuyển chloride, được tạo
ra có thể do protein mang đặc biết bicarbonate- chloride trên màng
hồng cầu thứ dịch chuyển 2 ion theo hướng ngược nhau. Nước dịch
chuyển vào hồng cầu để duy trì cân bằng thẩm thấu, thứ làm cho hồng
cầu hơi phù lên trong máu động mạch.
TMP12 502-503
59.Nhipj cơ bản của hô hấp được tạo ra nhờ các tế bào thần kinh nằm tại hành
nào. Điều nào sau đây giới hạn thời gian thì hít bào và tăng nhịp hô hấp?
a. Trung tâm ức chế hô hấp
b. nhóm nhân hô hấp lưng
c. Nhân đơn độc
d. Trung tâm điều hòa hô hấp
e. nhóm nhân hô hấp bụng
 D) trung tâm điều chỉnh thở vận chuyển tín hiệu đến nhóm nhân
lưng thứ “làm tắt) tín hiệu thở vào, vì thế điều chỉnh độ dài của pha
làm đầy của phổi. Nó có hiệu ứng thứ phát làm tăng nhịp thở, vì sự
giới hạn của thì hít vào cũng làm ngắn thì thở ra và toàn bộ chu trình
hô hấp
TMP12 505-506
60.Khi điều hòa hoạt độn hô hấp cho thông khí phổi đã được tăng lên trở nên
lớn hơn nữa so với bình thường, một nhóm tế bào thần kinh bị bất hoạt khi
thở yên tĩnh trở nên hoạt động, đóng góp vào điều hòa hoạt động hô hấp.
Nhóm tế bào thần kinh đó nằm trong cấu trúc nào sau đâ?
a. Trung tâm ức chế hô hấp
b. nhóm nhân hô hấp lưng
c. Nhân đơn độc
d. Trung tâm điều hòa hô hấp
e. nhóm nhân hô hấp bụng
 E) Nhịp cơ bản của hô hấp được tạo ra ở nhóm nhân bụng, nằm
hầu hết trong nhóm nhân bó đơn độc. Khi hệ hô hấp điều chỉnh để
tăng hô hấp lớn hơn bình thường, tín hiệu tan ra toàn bộ nhóm nhân
lưng, gây ra khu vực nhân lưng cũng tham gia vào điều chỉnh quá
trình thở. Tuy nhiên, nhóm nhân lưng duy trì hầu như không hoạt
động trong khi thở bình thường
TMP12 505-506
61.Sự tăng lên của phản xạ Hering-Breuer là cơ chế bào vệ chủ yếu điều hòa
hoạt động thông khí dưới điều kiện bình thường. Điều nào sau đây miêu tả
chính xác nhất hiệu ứng của phản xạ này trong kỳ hít vào và thở ra cũng như
vị trí của các receptor sức căng để bắt đầu phản xạ này?
Vị trí của Hít vào Thở ra
các
receptor
sức căng
A Thành phế Không tắt
nang tác dụng
B Thành phế tắt không tác
nang dụng
C Thành phế mở mở
nang
D phế Không tắt
quản/tiểu tác dụng
phế quản
E phế tắt không tác
quản/tiểu dụng
phế quản
F phế mở mở
quản/tiểu
phế quản
G thành Không tắt
ngực tác dụng
H thành tắt không tác
ngực dụng
I thành mở mở
ngực
 E) Thành cơ của phế quản và tiểu phế quản chứa các receptors áp
suất thứ truyền tín hiệu qua thần kinh lang thang đến nhóm nhân hô
hấp bụng khi phổi quá dãn. Tín hiệu này “tắt” sự hít vào và do đó
ngăn chặn sự giãn nở của phổi quá nhiều cùng một cách với trung tâm
điều chỉnh thở. Sự phản hồi không có tác dụng trực tiếp lên thì thở ra.
TMP12 506
62.Tại một bữa tiệc đầu năm, một bạn nam 17 tuổi đặt một cái túi vào miệng và
liên tục thở vào và ra trong cái túi. Khi anh ấy thớ liên tục vào cái túi, nhịp
thở liên tục tăng lên. Điều nào sau đây chịu trách nhiệm cho sự tăng thông
khí.
a. Tăng PO2 phế nang
b. Tăng PCO2 phế nang
c. giảm PCO2 động mạch
d. Tăng pH
 B) Ở một người bình thường các khí trong phế nang sẽ giống với
trong máu động mạch. Với qua trình thở lại, nồng độ CO2 thở ra
không bao giờ được remove và tích tụ ở trong túi. Điều này làm tăng
PCO2 trong phế nang và do đó tăng trong máu kích thích tăng thở.
Anh ta sẽ có sự giảm PO2, không tăng, với sự giảm PO2 kích tích thở.
Một sự giảm PCO2 sẽ không kích thích hô hấp. Sự tăng pH, toan hóa
máu. Sẽ không kích thích hô hấp.
TMP12 507-509
Điều nào sau đây xảy ra khi hít carbon monoxide?
PO2 phế PCO2 Hoạt động
nang phế recpetor
nang hóa học
ngoại vi
A ↑
B
C ↑
D
E ↑
F ↑
 B) Với carbon monoxide sẽ hỉ cần một lượng CO nhỏ để gắn vào
hemoglobin. Vì thế chỉ có một sự thay đổi tối thiểu trong PO2. Vì
vậy, sẽ không có sự kích thích trung tâm hô hấp và sẽ không làm thay
đổi Pco2.
TMP12 501-502, 508-509
64.Đồ thị nào mô tả tốt nhất mối quan hệ giữa thông khí phế nang (VA) và áp
suất carbondioxie động mạch (PCO2) khi PCO2 thay đổi một cách độ ngột
quá khoảng 35-75 mmHg?

 F) Thông khí phế nang có thể tăng đến hơn 8 lần khi áp suất carbon
dioxide động mạch tăng quá mức sinh lý từ 35 lên 75 mmHg. Điều
này biểu thị tác dụng rất lớn của sự thay đổi carbon dioxide trong điều
hòa hô hấp. Ngược lại, sự thay đổi của hô hấp gây ra do thay đổi PH
trong một ngưỡng bình thường từ 7.3 đến 7.5 ít hiệu quả hơn đến hơn
10 lần.
TMP12 508
65.Đồ thị nào mô tả tốt nhất mối quan hệ giữa thông khí phế nang (VA) và áp
suất oxi động mạch (PO2) khi PO2 thay đổi một cách độ ngột quá khoảng 0-
160 mmHg và PCO2 động mạch và nồng độ ion Hydro vẫn bình thường?
 D) Phân áp oxi động mạch về cơ bản không có tác dụng lên thông
khí phế nang khi nó cao hơn khoảng 100mmHg, nhưng thông khí xấp
xỉ gấp đôi khi phân áp oxi giảm xuống còn 60 mmHg và có thể tăng
đến 5 lần tại phân áp oxi rất thấp. Mối quan hệ về lượng giữa phân áp
oxi động mạch và thông khí phế nang được biểu thị trong một thí
nghiệm tại đó phân áp carbondioxide và Ph được giữ ổn định. Thí
nghiệm có thể mô tả rằng đáp ứng của thông khí với hạ oxi máu có
thể bị làm giảm nếu phân áp carbon dioxide được giảm.
TMP12 509
66.Một người đàn ông bị gây mê đang thở không cần hỗ trợ. Anh ấy sau đó
được thông khí nhân tạo trong 10 ph tại thể tích khí lưu thông bình thường
của anh ấy nhưng nhịp thở gấp đôi. Anh ấy được thông khí với khó trộn lẫn
giữa 60% O2 và 40% N2. Sự thông khí nhân tạo bị ngừng lại và anh ấy
không thể thở trong nhiều phút. Đợt ngừng thở này là vì nguyên nhân nào
sau đây?
a. PO2 động mạch cao ức chế hoạt động của các receptor hóa học ngoại
biên
b. giảm pH động mạch ức chế hoạt động của các receptor hóa học ngoại
biên
c. PCO2 động mạch cao ức chế hoạt động của các receptor hóa học ở
hành não
d. PCO2 động mạch thấp ức chế hoạt động của các receptor hóa học
ngoại biên
 C) Bệnh nhân này có sự tăng thông khí phế nang, vì thế dẫn đến
một sự giảm PCO2 động mạch. Hiệu ứng mà ở đó sự giảm PCO2
động mạch có thể là một ức chế vùng điều chỉnh thở cho đến khi
PCO2 được trở lại bình thường. Thở oxi cao không giảm các hoạt
động thần kinh đủ để giảm hô hấp. Đấp ứng với các receptor hóa học
với CO2 và PH là ít, và không đóng vai trò chính trong việc điều hòa
hô hấp.
TMP12 507-509
67.Trong thể dục mạnh, tiêu thụ oxi và hình thành carbon dioxide có thể tăng
đến 20 lần. Thông khí phế nang tăng gần như chính xác trong các bước
tương đương với tăng tiêu thụ oxi. Điều nào sau đây mô tả chính xác nhất
điều gì xảy ra với phân áp Oxi động mạch (PO2) và phân áp carbon dioxide
(PO2) và pH của một vận động viên khỏe mạnh khi tập thể dục mạnh?
PO2 động PCO2 pH động
mạch động mạch
mạch
A giảm giảm giảm
B giảm tăng giảm
C tăng giảm tăng
D tăng tăng tăng
E không đổi không đổi không
đổi
 E) Có một điều đáng ghi nhận là PO2 đông mạch, PCO2 và Ph giữ
ở mức gần như bình thường tại một vận động viên trong khi vận động
mạnh mặc dù sự tăng gấp 20 lần sự tiêu thụ oxi và tạo carbondioxide.
Hiệ tượng thú vị này đặt ra câu hỏi: thứ gì trong khi vận động mạnh
gây nên sự tăng cường hô hấp?
TMP12 511-512
68.Thông khí phế nang tăng nhiều lần khi thể dục mạnh. Yếu tố nào sau đây có
khả năng kích thích thông khí trong khi thể dục mạnh nhất?
a. Xung tín hiệu từ các trung tâm não cao cấp hơn cùng cùng bên
b. giảm pH trung bình động mạch
c. giảm PO2 trung bình động mạch
d. giảm PO2 trung bình tĩnh mạch
e. Tăng PCO2 trung bình động mạch
 A) Bởi vì vận động mạnh không thay đổi một cách một cách rõ rệt
PO2. PCO2, hay Ph trung bình động mạch, sẽ ít có khả năng chúng
đóng vai trò chính trong việc kích thích nổi trội trong sự thông khí.
Mặc dù PO2 trung bình tĩnh mạch giảm trong khi thể dục, tĩnh mạch
không chứ các receptor hóa học để cảm nhận PO2. Não, trong khi
truyền các tín hiệu vận động đến các cơ đang co, được cho là chuyền
tín hiệu tương ứng đến thân não để kích thích trung tâm hô hấp. Cùng
với đó, chuyển động của các phần cơ thể trong khi vận động cũng
được cho là kích thích các repceptor áp lực tại khớp thứ sau đó truyền
tín hiệu kích thích đến trung tâm hô hấp.
TMP12 511-512
69.Đồ thị sau cho thấy độ sâu của nhịp hô hấp ở một người đàn ông 45 tuổi trải
qua một chấn thương đầu bởi một tai nạn xe máy. Hình dạng "lên cao-xuông
thấp" của nhịp thở này được gọi là gì trong những điều sau?

a. Ngừng thở
b. Thở kiểu Biot
c. thở kiểu Cheyne-Stokes
d. thở sâu
e. tăng nhịp thở
 C) Thở Chayne-Strokes là kiểu thở có chu kỳ phổ biến nhất. Người
thử sâu trong một khoảng ngắn và sau đó thở nhẹ hoặc không thở
trong một khoảng tiếp theo. Hình dạng thở này lặp lại mỗi phút.
Ngừng thở là dang cắt ngang tạm thời của sự hô hấp nên đúng là thở
kiểu Cheyne-Stokes liên quan đến các chu kỳ ngừng thở. Thở kiểu
Biot liên quán đến các chuỗi thống nhất thở ngáp sâu, ngừng thở, sau
đó thở ngáp. Tăng thông khí có nghĩa là tăng thở, thường liên quan
đến tăng thể tích khí lưu thông với hoặc không với tăng tần số thở.
Thở nhanh có nghĩa là tăng tần số thở.
TMP12 512-513
70.Thở Cheyne-Stokes là một kiểu thở bất thường đặc trưng cho sự tăng dần độ
sâu của nhịp thở, tiếp theo bởi sự giảm dần độ sâu của nhịp thở xảy ra lặp lại
khoảng mỗi một phút, như được trình bày ở đồ thị dưới đây. Thời điểm nào
trong các thời điểm sau liên quan đến PCO2 cao nhất trong máu phổi và
PCO2 cao nhất trong các tế bào thần kinh của trung tâm hô hấp?
Máu phổi Trung tâm hô hấp
A V V
B V W
C W W
D X Z
E Y Z
 B) Cơ chế cơ bản của kiểu thở Cheyne-stokes có thể được đóng
góp bởi sự tạo ra carbon dioxide thứ làm kích thích sự thông khí quá
mức, tiếp theo bởi một sự ức chế trung tâm hô hấp do PCO2 thấp của
các neuron hô hấp. Rõ rang rằng độ sâu của việc thở lớn hơn xảy ra
nếu các neuron của trung tâm hô hấp tiếp xúc với nồng độ carbon
dioxide cao (điểm W). Sự tăng lên trong việc thở làm cho
carbondioxide bị thổi ra và vì thế PCO2 của máu phổi tại giá trị thấp
nhất của nó ở khoảng điêm Y của đồ thị. PCO2 của máu phổi tăng lên
từ từ từ điểm Y đến điểm Z, chạm đến giá trị tối đa tại điểm V. Vì thế,
có sự lệch pha giữa PCO2 tại trung tâm hô hấp và PCO2 của máu phổi
thứ dẫn đến kiểu thở này. Sự lệch pha này xảy ra khi có suy tim trái,
vì sự phì đại của thất trái, thứ làm tăng thời gian cần thiết để máu đến
được trung tâm hô hấp. Một nguyên nhân khác gây ra kiểu thở
Chayne-Stokes là sự feedback âm tính trong khu vực điều chỉnh hô
hấp, thứ có thể gây ra do chấn thương đầu, đột quỵ và những kiểu
chấn thương đầu khác.
TMP12 512-513.
71.Một người đàn ông 45 tuổi thở vào hết mức có thể sau đó thở ra hết sức đến
khi không còn không khí để thở. Điều này tạo ra đồ thì vận tốc-thể tích thở
ra tối đa trình bày ở dưới đây. Đâu là dung tích sống gắng sức của người này
(tính bằng liters)?
a. 1.5
b. 2.5
c. 3.5
d. 4.5
e. 5.5
f. 6.5
 D) Dung tích thở gắng sức (FVC) bằng sự khác biệt giữa dung tích
phổi toàn phần (TLC) và thể tích cặn (RV). TLC và RV là điểm của
đường cắt giữa đường ngang và đường cong vận tốc-thể tích, vì thế,
TLC = 5.5L, và RV =1.0 L. Vì thế, FVC =5.5 -1.0 =4.5 L.
TMP 12 516
72.Đường cong vận tốc-thể tích thở ra tối đa được trình bày đồ thị dưới đây
được sử dụng như một công cụ chẩn đoán bệnh phổi tắc nghẽn và hạn chế.
Tại điểm nào trong các điểm của đường cong đường thở xẹp xuống giới hạn
tốc độ thở ra tối đa?

 D) Đường cong vận tốc tối đa - thể tích (MEFV) được tạo ra khi
một người hít vào hết sức có thể (điểm A, tổng thể tích phổi = 5.5L)
và sau đó thở ra khí với nỗ lực tối đa cho đến khi không còn khí để
thở (điểm E, thể tích cặn = 1,0 L). Phần ngang của đường cong biểu
thị bởi mũi tên chỉ xuống biểu thị vận tốc thở ra tối đa tại mỗi thể tích.
Phần ngang của đường cong đôi khi được nhắc đến như là phần
“không phụ thuộc nỗ lực” của đường cong bởi vì bệnh nhân không thể
tăng vận tốc thở ra đến mức cao hơn kể cả khi nỗ lực thở ra tối đa
được tăng lên
TMP12 516
73.Đường cong vận tốc-thể tích thở ra tối đa trong đồ thị sau đạt được từ một
người khỏe mạnh (đường đỏ) và một người đàn ông phàn nàn về việc khó
thở ( Đường xanh). Rối loạn nào sau đây có khả năng xuất hiện nhất ở người
đàn ông này?

a. Bệnh bụi phổi


b. Khí phế thũng
c. Gù cột sống
d. Vẹo cột sống
e. Bệnh bụi phổi silic
f. Lao
 B) Trong bệnh phổi tắc nghẽn như khí phế thũng hay hen, đường
cong vận tốc thở ra tối đa – thế tích (MEFV) bắt đầu và kết thúc tại
những thể tích phổi cao bất thường, và vận tốc dòng khí thấp hơn bình
thường tại mọi thể tích phổi cho trước. Đường cong có thể có độ dốc
cao hơn so với bình thường, như được chỉ ra trên hình. Những bệnh
khác được liệt kê trên lựa chọn đáp án là bệnh phổi co thắt (thường
được gọi là bệnh phổi hạn chế). Thể tích phổi thường thấp hơn bình
thường trong bệnh phổi co thắt.
TMP12 516
74.Một người đàn ông 62 tuổi phàn nàn với bác sỹ của ông ấy rằng ông ấy bị
khó thở. Đồ thị sau trình bày một đường cong vận tốc-thể tích thở ra tối đa
(MEFV) của bệnh nhân (đường xanh) và của người khỏe mạnh (đường đỏ).
Điều nào sau đây giải thích cho đường cong MEFV của bệnh nhân?
a. Bệnh bụi phổi
b. Hen
c. Co thắt phế quản
d. Khí phế thũng
e. tuổi già
 A) Bệnh bụi phổi là một bệnh phổi co thắt đặc trưng bởi xơ hóa
khoảng kẽ lan tỏa. Trong bệnh phổi co thắt (Thường được gọi phổ
biến hơn là bệnh phổi hạn chế), đường cong MEFV bắt đầu và kết
thúc tại thế tích phổi thấp một cách bất thường, và vận tốc dòng khí
thường cao hơn bình thường tại mọi thể tích phổi cho trước, như được
chỉ ra trên đồ thị. Thê tích phổi thường được kỳ vọng là cao hơn trong
hen, co thắt phế quản, khí phế thũng, tuổi già, và những trường hợp
khác đường dẫn khí bị hẹp hay lực kéo xuyên tâm của đường dẫn khí
bị giảm cho phép chúng đóng lại dễ dàng hơn.
TMP12 516
75.Đường cong vận tốc-thể tích thở ra tối đa tình bày trong đồ thị sau (đường
đỏ) thu được từ một người đàn ông 75 tuổi hút 40 điều thuốc mỗi ngày trong
vòng 60 năm qua. Đường cong vận tốc- thể tích màu xanh thu được từ người
đàn ông này trong điều kiện nghỉ. Những thay đổi nào sau đây có khả năng
áp dụng cho người đàn ông này?

 B) Đồ thị chỉ ra rằng nỗ lực thở ra tối đa là cần thiết trong điều
kiện nghỉ vì vận tốc thở ra tối đa đạt được ngay trong điều kiện nghỉ.
Rõ rang rằng khả năng của anh ấy để tập luyện bị giảm xuống rất
nhiều. Người đàn ông này hút thuốc 60 năm và có khả năng bị khí phế
thũng. Vì thế, nghiên cứu có thể dự đoán được rằng tổng thể tích phổi,
thể tích cặn chức năng, và thể tích cặn lớn hơn bình thường. Thể tích
sống là khoảng 3.4 L, như được chỉ ra trên đồ thị.
TMP12 516-517
76.Đồ thị dưới đây trình bày một thì thở ra gắng sức của một người khỏe mạnh
(Đường cong X( và của một người với bệnh phổi (đường cong Z). Đâu là tỷ
lệ FEV1/FVC (tính theo phần trăm) ở những người này

Người X Người Z
A 80 50
B 80 40
C 100 80
D 100 60
E 90 50
F 90 60
 A) Một dung tích thở ra gắng sức (FVC) là dung tích sống được đo
với một sự thở ra gắng sức. Thể tích thở ra gắng sức tại giây thứ nhất
(FEV1) là lượng khí có thể thở ra từ phổi trong giây đầu tiên với sự
thở ra gắng sức. FEV1/FVC cho người bình thường (Đường cong X)
là 4L/5L = 80% và 2L/4L =50% cho bệnh nhân (đường cong Z). tỷ số
FEV1/FVC có giá trị chẩn đoán cho những trường hợp khác biệt khác
nhau giữa các hình dạng bình thường, tắc nghẽn, co thắ của một sự
thở ra gắng sức.
TMP12 517
77.Đồ thị tiếp theo trình bày thì thở ra gắng sức của một người có phổi khỏe
mạnh ( Đường X) và của một bệnh nhân (Đường Z). Điều gì sau đây có khả
năng có mặt nhất trên bệnh nhân này?
a. Hen
b. Co thắt phế quán
c. Khí phế thũng
d. Tuổi già
e. Bệnh phổi silicon
 E) Dung tích thở ra gắng sức là dung tích sống đo được với sự thở
ra gắng sức. Dung tích thở ra gắng sức trong giây đầu tiên (FEV1) là
lượng khí có thể thở ra của phổi trong giây đầu tiên. Tỷ số FEV1/FVC
cho người khỏe mạnh (X) là 4L/5L = 80%; FEV1/FVC cho bệnh nhân
là 3.0L/3,5L = 86%. FEV1/FVC thường tăng trong bệnh bụi phổi và
những bệnh khác đặc trưng bởi xơ hóa khoảng kẽ vì sự tăng lực kéo
xuyên tâm của đường thở, thứ mà, đường thở được giữ mở ra với độ
mở lớn hơn ở mọi thế tích phổi, giảm sức cản của nó với dòng khí.
Sức cản đường thở tăng (và vì thế tỷ số FEV1/FVC giảm) trong hen,
co thắt phế quản, khí phế thũng và tuổi già.
TMP12 517
78.Đồ thì tiếp theo trình bày một thì thở ra gắng sức của một nguồ có phổi khỏe
mạnh (Đường X) và một bệnh nhân (Đường Z). Điều nao sau đây có thể giải
thích cho kết quả của bệnh nhân này?
a. Bệnh bụi phổi
b. Khí phế thũng
c. Viêm màng phổi xơ hóa
d. Tràn khí màng phổi
e. Bệnh phổi silicon
f. Lao
 B) Dung tích thở ra gắng sức (FVC) là dung tích đo được khi thở
ra gắng sức (FVC = 4.0L cho bệnh nhân Z). Dung tích thở ra trong
giây đầu tiên (FEV1) là lượng khí có thể được thở ra từ phổi trong
giây đầu tiên của sự thở ra gắng sức (FVC = 2.0 L cho bệnh nhân Z).
FEV1/FVC là một chức năng của sức cản đường thở. Sức cản đường
thở thường tăng trong bệnh khí phế thũng, thứ làm cho FEV1/FVC
giảm. Chú ý rằng FEV1/FVC là 50% ở bệnh nhân Z và 80% ở những
người bình thường biểu thị bằng đường X. Tỷ số FEV1/FVC thường
không bị ảnh hưởng bới tràn dịch màng phổi và tràn khí màng phổi vì
sức cản đường thở là bình thường. FVC thường giảm trong bệnh bụi
phổi, dày dính màng phổi, bệnh phổi silicon và lao và FEV1/FVC có
thể bình thường hay tăng nhẹ.
TMP12 517
79.Một đường con thể tích-áp lực trình bày trong đồ thị tiếp theo thu được từ
một người trẻ, một đối tường khỏe mạnh và một bệnh nhân. Điều nào sau
đây mô tả chính xác nhất tình trạng của bệnh nhân?
a. Hen
b. Co thắt phế quản
c. Khí phế thũng
d. Tuổi già
e. Bệnh phổi silicon
 E) Tiếp xúc lâu ngày với silica gây ra xơ hóa khoảng kẽ, thứ lại
gây ra giảm sức đàn hồi của phổi. Sức đàn hồi là sự thay đổi của thể
tích phổi với một áp suất xuyên phổi cho trước cần đến giãn phổi ra.
Đường cong thể tích- áp suất thể hiện rằng bệnh nhân có độ giãn nở
của phổi nhỏ hơn bình thường, thứ thích hợp với bệnh phổi silicon.
Lực đàn hổi của phổi tăng khi chất xơ hình thành trong khoảng kẽ và
thành phế nang, giảm độ giãn nở (sức đàn hồi) của phổi. Độ giãn nở
của phổi tăng trong khí phế thũng và tuổi già. Hen và những bệnh
khác đặc trưng bởi co thắt phế quản cũng gây ra sự tăng lên của độ
giãn nở của phổi.
TMP12 467
80.Một đường con thể tích-áp lực ở dưới đây thu được từ một đối tượng bình
thường và một bệnh nhân đang trải qua một bệnh phổi. Bất thường nào sau
đây có khả năng biểu hiện nhất trên bệnh nhân này?
a. Bệnh bụi phổi
b. Khí phế thũng
c. Tắc van hai lá ???
d. Bệnh thấp tim
e. Bệnh phổi silicon
f. Lao
 B) Mất thành phế nang với sự phá hủy giường mao mạch trong
bệnh bụi phổi làm giảm độ đàn hồi của phổi và tăng độ giãn nở.
Nghiên cứu nên nhắc lại rằng độ giãn nở bằng sự thay đổi của thể tích
phổi với một áp suất xuyên màng cho trước, vì thế, độ giãn nở của
phổi bằng độ dốc của đường cong thể tích – áp suất chỉ ra trên hình.
Bệnh bụi phổi, bệnh phổi silicon, và leo liên quán đến sự ứ đọng của
mô xơ tại phổi, thứ làm giảm độ giãn nở. Tắc van hai là và bệnh tim
khớp có thể gây nên phù phổi, thứ cũng làm giảm độ giãn nở của phổi
TMP12 467
81.Một sinh viên y 34 tuổi tạo ra một đường cong vận tốc-thể tích trình bày ở
đồ thì dưới. Đường cong W là đường cong vận tốc- thể tích thở ra tối đa khi
sinh viên còn khỏe mạnh. Điều gì có thể giải thích cho đường cong X?

a. Cơn hen
Nuốt phải thịt vào trong khí quản
b.
Tập thể dục nặng
c.
Tập thể dục nhẹ
d.
Thở bình thường lúc nghỉ
e.
Viêm phổi
f.
g.
Lao
 C) Đường cong X biểu thị sự vận động gắng sức với thể tích khí
lưu thông khoảng 3L. Chủ ý rằng vận tốc thở ra đạt đến giá trị tối đa ở
gần 4.5 L/giây khi thể dục gắng sức. Điều này diễn ra bởi một vận tốc
thở ra tối đa được yêu cầu để di chuyển khí trong đường thở với thông
khí vận tốc cao liên quan đến vận động gắng sức. Thở bình thường ở
trạng thái nghỉ biểu thị bởi đường cong Z; để ý rằng thể tích khí lưu
thông nhỏ hơn 1L trong điều kiện nghỉ. Đường cong Y được co lại
trong khi vận động nhẹ. Một cơn hen hay khó thở có thể tăng sức cản
dòng khí từ phổi, làm cho nó không có khả năng thở ra với vận tốc có
thể chạm đến vận tốc tối đa ở một thể tích phổi cho trước. Thể tích khí
lưu thông không nên tăng lên quá nhiều với viêm phổi hay lao và
không nên có thể đại được vận tốc khí thở ra tối đa tại một thể tích
phổi cho trước với những bệnh này.
TMP12 516-517
82.Một người đàn ông 78 tuổi hút 60 điếu thuốc một ngày trong 55 năm than
phiền về việc khó thở. Bệnh nhân được chẩn đoán là khí phế thũng mãn tính.
Những thay đổi nào sau đây xuất hiện tại bệnh nhân này, so với người khỏe
mạnh không hút thuốc?
Độ giãn Lực đàn Dung
nở của hồi nhu tích toàn
phổi mô phổi phần của
phổi
A Giamr giảm giảm
B giảm giảm tăng
C giảm tăng tăng
D tăng giảm giảm
E tăng giảm tăng
F tăng tăng tăng
 E) Sự mất mô của phổi trong khí phế thũng dẫn đến sự tăng độ
giãn nở của phổi và một sự giảm lực đàn hồi của phổi. Độ giãn nở của
phổi là độ đàn hồi của phổi thay đổi theo hai hướng khác nhau, thứ
mà, độ giãn nở của phổi tỷ lệ thuật với 1/độ đàn hổi của phổi, thể tích
cặn, và dung tích cặn chức năng tăng lên trong khí phế thũng, nhung
dung tích sống lại giảm xuống.
TMP12 467
83.Một người đàn ông 73 tuổi đã làm việc 5 năm trong một nhà máy khi ông
ấy đầu những năm 40 tuổi nơi mà amiang được dùng làm vật liệu cách nhiệt,
Người đàn ông này được chẩn đoán là bị bệnh bụi phổi. Những thay đổi nào
xuất hiện trên bệnh nhân này, so sáng với người bình thường?
Độ giãn Lực đàn Dung
nở phổi hổi nhu tích toàn
mô phổi phần của
phổi
A giảm giảm giảm
B giảm tăng tăng
C giảm tăng giảm
D tăng giảm giảm
E tăng giảm tăng
F tăng tăng tăng
 C) Bệnh bụi phổi liên quan đến sự lắng đọng chất sợi trong phổi,
thứ gây nên độ giãn nở giảm xuống, và độ đàn hồi tăng lên. Độ giãn
nở của phổi thay đổi theo hai hướng ngươc nhau vì độ giãn nở tỷ lệ
thuận với 1/độ đàn hổi. Đôi khí đáng ngạc nhiên để biết rằng độ đà
hồi của một hòn đá còn lớn hơn của một sợi dây cao su, bởi vì càng
khó hơn để thay đổi một vật, độ đàn hồi càng cao. Tổng dung tích
phổi, dung tích cặn chức năng, thể tích cặn, và thể tích sống giảm
trong tất cả các dạng của bệnh phổi do xơ hóa.
TMP12 467
84.So sánh phổi của trẻ sinh non có hội chứng suy hô hấp với một trẻ sinh đủ
tháng bình thường, độ giãn nở của phổi và nồng độ surfactant như thế nào?
độ giãn nở của surfactant ở trẻ
trẻ đẻ non so đẻ non so với
với trẻ đủ tháng trẻ đủ tháng
A ↑ ↓
B ↑ ↑
C ↓ ↓
D ↓ ↑
E ↔ ↑
F ↔ ↓
 C) Một trẻ sinh non với hội chứng suy hô hấp bị mất hay giảm
nồng độ surfactant. Mất surfactant tạo ra áp suất bề mặt lớn hơn. Vì áp
suất bề mặt đóng một phần lớn vai trò trong độ giãn nở của phỏi, tăng
áp xuất bề mặt sẽ làm tăng độ đàn hồi của phổi làm phổi trở nên cứng
hơn và độ giãn nở thấp hơn
TMP12 519
85.Điều nào sau đây tăng lên với bệnh khí phế thũng?
a. PO2 phế nang
b. cung lượng tim
c. Diện tích khuyech tán
d. Ap lực động mạch phổi
 C) vì sự phá hủy thành phế nang, sẽ có sự tăng tỷ lệ bề mặt/tưới
máu. Cung lượng tim sẽ bình thường. Áp lực mạch phổi sẽ tăng lên vì
sự phá hủy phế nang dẫn đến mất các mao mạch xung quanh làm cho
dòng máu vẫn còn ở trong mao mạch, thứ có xu hướng tăng làm tăng
áp lực động mạch phổi. PCO2 phế nang sẽ tăng vì sự khó thở ra
TMP12 517-518
86.Liệu pháp oxi có lợi ích nhất trong những tình huống nào sau đây? Chức
năng phổi vẫn bình thường
a. Thiếu máu
b. ứ đọng CO2 (COPD)
c. Ngộ độc Cyanide
d. Độ cao cao
 D) Khả năng khuếch tán liên quan trực tiếp đến bề mặt phế nang.
Nó tăng lên khi tập luyện nhờ khả năng mở các mao mạch và tỷ số
V/Q gần hơn. Khả năng khuêch tán của CO2 cao gấp 20 lần so với
O2. Khi một người đi đến độ cao lớn hơn, sự mở mạch máu và tăng
thông khí diễn ra để tăng sự khuếch tán oxi, dẫn đến tăng khả năng
khuêch tán.
TMP12 523-525
87.So sánh với một người bình thường, thể tích phổi toàn phần và vẫn tốc thở ra
tối đa của bệnh nhân với bệnh phổi hạn chế thay đổi như thế nào?
Tông thể tích Vận tốc thở ra
phổi tối đa
A ↑ ↓
B ↓ ↓
C ↑ ↑
D ↓ ↑
 B)Tổng dung tích tối đa và vận tốc thở ra tối đa giảm trong bệnh
phổi co thắt
TMP12 516

Unit 9: thần kinh:sinh lý cảm giác


1. Trong một nơron với điện thế nghỉ của màng là -65mV, sự phân bố ion nào qua
màng tế bào thần kinh cho lực điện thế lớn nhất (EMF)?
A. K+
B. Cl-
C. Na+
D. Ca2+
 C. Điện thế khuếch tán của Natri là +61 mV. Một điện thế nghỉ +61 mV là cần
thiết để ngăn dòng Natri di chuyển qua màng tế bào. Giá trị này lớn hơn điện thế
khuếch tán của Kali (-86 mV) và Clo (-70 mV).

Do vậy, sự phân bố Natri qua màng tế bào sẽ cho lực điện thế lớn nhất.
2. Kiềm hoá máu do thở nhanh gắng sức sẽ dẫn đến những thay đổi nào trong hoạt
động của hệ thần kinh?
A. Giảm hoạt động của hệ thần kinh
B. Tăng hoạt động của hệ thần kinh
C. Ban đầu giảm, sau đó tăng
D. Hoạt động của hệ thần kinh không thay đổi
 B. Một pH kiềm sẽ gây tăng hoạt động của hệ thần kinh
3. Việc giải phóng các chất truyền đạt thần kinh tại một synap hoá học trong hệ
thần kinh trung ương phụ thuộc vào yếu tố nào sau đây?
A. Sự tổng hợp của acetycholinesterase
B. Sự ưu phân cực của đầu tận cùng synap
C. Sự mở của các kênh calci (loại cổng đóng mở theo chất kết nối)
D. Di chuyển calci vào các cúc tận cùng
 D. Việc giải phóng các chất truyền đạt thần kinh phụ thuộc vào dòng
chảy Calci qua cổng điện thế. Khi hiện tượng này xảy ra, túi synap sẽ hoà
vào màng trước synap và giải phóng các chất truyền đạt thần kinh vào khe
synap.
4. Là một receptor có vỏ bọc, thích nghi chậm được tìm thấy ở đỉnh các gai da của
vùng da không có nang lông (glabrous skin) và đặc biệt phong phú ở môi, đầu
ngón tay. Mô tả trên đúng với receptor nào dưới đây?
A. Đĩa Merkel
B. Một số đầu dây thần kinh tự do
C. Tiểu thể Meissner
D. Tiểu thể Ruffini
 C. Tiểu thể Meissner được tìm thấy ở các vùng da không có nang lông
5. Các receptor đau trên da là điển hình của nhóm receptor nào sau đây?
A. Một số đầu dây thần kinh có vỏ bọc
B. Nhóm các có khả năng thích nghi bằng việc thay đổi hình thái
C. Cùng loại với các repceptor cảm giác bản thể
D. Các đầu dây thần kinh tự do
 D. Các receptor đau trên da là các đầu dây thần kinh tự do
6. Là một receptor xúc giác có đầu mở rộng nằm trên lớp hạ bì của vùng da có
chứa các nang lông (hairy skin), được biệt hoá để nhận thông tin về các nhận cảm
xúc giác liên tục?
A. Một số đầu dây thần kinh tự do
B. Đĩa Merkel
C. Tiểu thể Pacini
D. Tiểu thể Ruffini
 B. Đĩa Merkel nằm trên lớp hạ bì của vùng da có chứa các nang lông
(hairy skin) và nhận thông tin về các nhận cảm xúc giác liên tục
7. Tình trạng giảm thông khí có ảnh hưởng gì đến hoạt động hệ thần kinh?
A. Hệ thần kinh bị ức chế
B. Hệ thần kinh bị kích thích
C. Tăng hiện tượng châm synap
D. Tăng giải phóng các chất truyền đạt thần kinh
 A. Tình trạng giảm thông khí dẫn tới có tình trạng toan máu. Kết quả là
giảm hoạt động thần kinh
8. Nguyên lý nào sau đây giải thích tính chất đặc hiệu của các sợi thần kinh cảm
giác nghĩa là mỗi sợi thần kinh cảm giác chỉ dẫn truyền một loại cảm giác?
A. Nguyên lý mã hoá tần số
B. Học thuyết năng lượng thần kinh chuyên biệt
C. Nguyên lý kì dị
D. Nguyên lý đường truyền gắn nhãn
 D. Mối liên hệ giữa đường truyền cảm giác và một loại sợi thần kinh là nền tảng
cho lý thuyết đường truyền gắn nhãn

9. Là một loại tiểu thể ngoài nằm ngay dưới da và cả trong lớp sâu của da, chúng
rất nhạy cảm với sự biến đổi của sự biến dạng và rung động của da. Mô tả trên
đúng với receptor nào dưới đây?
A. Tiểu thể Pacini
B. Tiểu thể Meissner
C. Một số đầu dây thần kinh tự do
D. Tiểu thể Ruffini
 A. Tiểu thể Pacini nhạy cảm với sự biến đổi của sự biến dạng và rung
động của da. Là một loại tiểu thể ngoài nằm ngay dưới da và cả trong lớp
sâu của da
10. Chất nào sau đây làm tăng tính nhạy cảm của các receptor đau nhưng không
trực tiếp trực tiếp kích thích các receptor đó?
A. Bradykinin
B. Serotonin
C. Ion K+
D. Prostaglandins
 D. Prostaglandins được cho là làm tăng tính nhạy cảm của các receptor
đau nhưng không trực tiếp trực tiếp kích thích các receptor đó
11. Nhận định nào sau đây mô tả đúng đặc trưng chức năng quan trọng của các
receptor đau?
A. Ít hoặc không có khả năng thích nghi
B. Không bị ảnh hưởng bởi sự co thắt cơ
C. Cảm nhận đau chỉ được dẫn truyền khi gập bao khớp
D. Có thể tự bị ức chế
 A. Receptor đau ít hoặc không có khả năng thích nghi
12. Sự kích thích hay ức chế một chất truyền đạt thần kinh được quyết định bởi yếu
tố nào sau đây?
A. Chức năng của receptor sau synap của chất truyền đạt thần kinh đó
B. Cấu tạo phân tử
C. Hình dạng túi synap chứa chất truyền đạt thần kinh đó
D. Khoảng cách giữa màng trước và sau synap
 A. Chức năng của một chất truyền đạt thần kinh phụ thuộc vào chức năng
receptor sau synap của chất truyền đạt thần kinh đó
13. Nhận định nào sau đây mô tả đúng nhất về sự dẫn truyền tín hiệu đau đến thần
kinh trung ương?
A. Các sợi đau nhanh dẫn truyền với tốc độ khoảng 6-30 m/sec và được xếp
vào loại C
B. Các sợi đau loại A-delta có thể xác định vị trí kích thích đau
C. Sau khi qua sừng sau của tuỷ sống, các sợi đau nhanh và chậm liên hợp thần
kinh với các nơron tại đây
D. Bó gai đồi thị cũ được biệt hoá để dẫn truyền nhanh chóng các tín hiệu đau
tới đồi thị
 B. Các sợi đau loại A-delta và không phải loại C chịu trách nhiệm cho
việc xác định vị trí kích thích đau
14. Hệ thống nào sau đây dẫn truyền cảm giác bản thể chính xác nhất về không
gian và thời gian?
A. Bó gai thị trước bên
B. Bó gai thị sau
C. Bó vỏ tuỷ (bó tháp)
D. Bó tuỷ tiểu não
 B. Độ chính xác nhất về không gian và thời gian được tăng cường tại bó
gai thị sau so với bó gai thị trước
15. Đường dẫn truyền nào sau đây đi qua cột trắng trước bên qua một số đoạn
đường vào và tận cùng ở đồi thị đối bên với vị trí xuất phát?
A. Bó gai thị trước bên
B. Bó gai thị sau
C. Bó vỏ tuỷ (bó tháp)
D. Bó tuỷ tiểu não
 A. Các sợi thần kinh ở bó gai thị trước bên đi qua cột trắng trước bên
trước khi tận cùng ở đồi thị đối bên. Các sợi thần kinh của bó gai thị trước
bên bắt chéo ở mép trắng trước trong một vài đoạn tuỷ trước khi đi lên ở bên
đối diện. Các tín hiệu đi lên trong bó gai thị sau không bắt chéo cho tới khi
chúng đến các nhân thon và nhân chêm ở hành não.
16. Nhận định nào sau đây mô tả đúng nhất về điện thế receptor thụ cảm với cơ
học?
A. Tăng năng lượng kích thích dẫn đến tăng điện thế receptor
B. Khi điện thế receptor tăng trên ngưỡng kích thích nhất định thì điện thế hoạt
động sẽ xuất hiện trên nơron gắn receptor đó
C. Tần số điện thế hoạt động trên nơron gắn receptor tỉ lệ thuận với điện thế
receptor
D. Tất cả các đáp án trên
 D. Điện thế receptor được định nghĩa là điện thế màng của receptor thần
kinh. Khi đạt được ngưỡng kính thích được đạt đến, tần số điện thế hoạt
động được tạo ra sẽ tăng tỉ lệ thuận với độ lớn của kích thích, và sau đó
chuyển sang đi theo đường ngang dạng bình nguyên.
17. Một số synap hoá học có chứa chất truyền tin thứ 2, điển hình là protein G, một
protein liên kết với receptor sau synap. Khi chất truyền đạt thần kinh gắn vào
receptor, protein G sẽ được hoạt hoá. Nhận định nào sau đây mô tả đúng nhất hoạt
động của chất truyền tin thứ 2 khi bị hoạt hoá?
A. Đóng các kênh Natri và hoặc Kali trên màng
B. Hoạt hoá cAMP và cGMP
C. Bất hoạt các enzym khởi đầu cho các phản ứng hoá sinh xảy ra tại nơron sau
synap
D. Bất hoạt quá trình phiên mã gen của nơron sau synap
 B. Việc Gắn vào protein G có thể dẫn tới sự hoạt hoá cAMP và cGMP.
Các protein này không đóng kênh Kali hay Natri, cũng như không bất hoạt
các enzyme, quá trình phiên mã gen hay chất truyền đạt thần kinh (nhìn hình
ở trang bên)
18. Những nơron nào sau đây nằm ở vùng giải phóng ra chất truyền đạt thần kinh
serotonin?
A. Vùng chất xám quanh rãnh Sylvius
B. Các nơron trung gian của tuỷ sống
C. Vùng quanh não thất
D. Nhân raphe magnus
 D. Những nơron nằm ở vùng nhân raphe magnus sẽ giải phóng serotonin
tại các đầu tận cùng của chúng. Trong hệ thống ức chế cảm giác đau nội
sinh, các neuron này tận cùng ở các neuron trung gian trong tuỷ sống, sau
đó, chính các neuron trung gian này lại giải phóng encephalin và ức chế các
tín hiệu đến từ các sợi thần kinh dẫn truyền cảm giác đau.
19. Hệ thống nào sau đây dẫn truyền cảm giác xúc giác tinh tế (xác định chính xác
vị trí bị kích thích) và cảm giác về vị trí cơ thể?
A. Bó gai thị trước bên
B. Bó gai thị sau
C. Bó vỏ tuỷ (bó tháp)
D. Bó tuỷ tiểu não
 B. Cảm giác xúc giác tinh tế (xác định chính xác vị trí bị kích thích) và
cảm giác về vị trí cơ thể được dẫn truyền bởi bó gai thị sau
20. Trên một bệnh nhân đau nặng, triệu chứng khó ngủ thường xuất hiện nếu bệnh
nhân không được sử dụng thuốc giảm đau. Nguyên nhân của hiện tượng trên là gì?
A. Vùng vỏ não cảm giác bản thể cho nhận cảm đau chặn chu kỳ tạo giấc ngủ
B. Các sợi đau đi vào sừng sau và đường dẫn truyền cảm giác đau đi lên chặn
chu kỳ tạo giấc ngủ
C. Đường dẫn truyền cảm giác đau đưa các kích thích tới hệ cấu tạo lưới, các
kích thích này góp phần duy trì tỉnh táo và trạng thái thức của não
D. Các chất truyền đạt thần kinh trong đường dẫn truyền cảm giác đau chậm
khuếch tán sang các tế bào bên cạnh và thường làm tăng trạng thái kích thích
của não
C. Các bệnh nhân trải qua cảm giác đau dai dẳng và thường mất ngủ do
đường dẫn truyền cảm giác đau đưa các kích thích tới hệ cấu tạo lưới. Các
kích thích này góp phần duy trì tỉnh táo và trạng thái thức của não
21. Các thân nơron đầu tiên (hướng tâm nguyên thuỷ) của bó gai thị sau được tìm
thấy trong cấu trúc nào sau đây?
A. Sừng sau tuỷ sống
B. Sừng trước tủy sống
C. Hạch gai
D. Nhân chêm
 C. Các thân nơron đầu tiên (hướng tâm nguyên thuỷ) của bó gai thị sau
được tìm thấy trong hạch gai
22. Cấu trúc nào sau đây đưa các sợi trục từ nhân thon đến đồi thị?
A. Bó thon
B. Bó chêm
C. Bó tuỷ đồi thị bên
D. Dải Reil giữa
 D. Dải Reil giữa gồm các sợi trục từ nhân thon và nhân chêm đến đồi thị?
23. Nhận định nào sau đây mô tả đúng nhất sự biến đổi của một kích thích cảm
giác thành các xung động thần kinh?
A. Thay đổi trong tính thấm với ion của màng receptor
B. Tạo ra một điện thế hoạt động
C. Bất hoạt phản ứng qua trung gian protein G
D. Tổng hợp protein
 A. Nguyên lý cơ bản của việc dẫn truyền cảm giác và sự thay đổi (tăng)
trong tính thấm với ion của màng receptor
24. Cấu trúc nào sau đây đưa sợi trục từ các nơron ở nhân bụng sau bên của đồi thị
đến hồi sau trung tâm (vùng cảm giác bản thể nguyên thuỷ S-I)?
A. Dải Reil giữa
B. Bao ngoài
C. Bao trong
D. Bao ngoài cùng
 C. Bao trong đưa sợi trục từ các nơron ở nhân bụng sau bên của đồi thị
đến hồi sau trung tâm (vùng cảm giác thân thể nguyên thuỷ S-I)
25. Đặc trưng của các hiện tượng xảy ra tại một synap hưng phấn là?
A. Có sự rò rỉ một lượng lớn Calci từ cúc tận cùng
B. Túi synap hoà vào màng sau synap
C. Kênh Kali (kênh đóng mở do điện thế) bị đóng
D. Các kênh đóng mở do chất kết nối được mở ra, cho phép Natri đi vào nơron
sau synap.
 D. Các kênh đóng mở do chất kết nối được mở ra, cho phép Natri đi vào.
Hiện tượng này xảy ra đồng thời với sự rò rỉ Calci vào trong bào tương, sự
hoà màng của các túi synap vào màng trước synap, và sự thay đổi điện thế ở
màng sau synap
26. Trong một nơron với điện thế nghỉ của màng là -65mV, sự phân bố ion nào qua
màng tế bào thần kinh cho lực điện thế nhỏ nhất (EMF)?
A. K+
B. Cl-
C. Na+
D. Ca2+
 B. Điện thế Nernst của ion Cl- là -70 mV. Trong một nơron điển hình, giá
trị này gần nhất với điện thế nghỉ trung bình của màng. Do đó, ion Cl- cho
lực điện hoá nhỏ nhất.
27. Sự kích thích vùng não nào có thể gây ra cảm giác đau?
A. Phức hợp trám trên
B. Nhân lục
C. Chất xám quanh rãnh Sylvius
D. Hạch hạnh nhân
 C. Chất xám quanh rãnh Sylvius ở vùng não giữa chứa các nơron có vai
trò giảm hoạt động của hệ ức chế đau
28. Hình chiếu của bộ phận nào trên cơ thể nằm ở cao, phía giữa của hồi sau trung
tâm?
A. Chi trên
B. Chi dưới
C. Bụng
D. Sinh dục
 B. Hình chiếu của chi dưới được tìm thấy ở cao và phía giữa của hồi sau
trung tâm
29. Nhóm nơron nào trong đường ức chế cảm giác đau sử dụng serotonin như một
chất truyền đạt thần kinh?
A. Hồi sau trung tâm
B. Nhân raphe magnus
C. Chất xám quanh rãnh Sylvius
D. Các sợi cảm giác loại AB
 B. Các nơron trong nhân raphe magnus sử dụng serotonin như một chất
truyền đạt thần kinh.
30. Khi điện thế receptor tăng cao hơn ngưỡng kích thích, tần số điện thế hoạt động
có đặc trưng cơ bản nào sau đây?
A. Giảm
B. Tăng
C. Không đổi
D. Chỉ tăng khi điện thế receptor tăng gấp đôi ngưỡng kích thích
 B. Khi điện thế receptor tăng trên ngưỡng kích thích, tần số điện thế hoạt
động sẽ tăng.
31. Loại nơron trung gian nào sau đây sử dụng enkephalin để ức chế dẫn truyền
cảm giác đau?
A. Nhân raphe magnus
B. Hồi sau trung tâm
C. Sừng sau của tuỷ sống
D. Sợi cảm giác loại C
 C. Nơron trung gian ở sừng sau của tuỷ sống sử dụng enkephalin như một
chất truyền đạt thần kinh để ức chế dẫn truyền đau một cách hiệu quả.
32. Vị trí đau được xác định chính xác nhất trong điều kiện nào?
A. Khi kích thích đồng thời cả các đầu dây thần kinh tự do và các sợi xúc giác
B. Khi kích thích các đầu dây thần kinh tự do bằng bradykinin
C. Khi các sợi thần kinh đi tới đồi thị theo con đường của bó gai đồi thị cũ
D. Khi kích thích các sợi loại A-delta
 A. Thông thường cảm giác đau được xác định vị trí kém. Tuy nhiên, khi
receptor xúc giác và receptor đau được kích thích đồng thời, cảm giác đau sẽ
được xác định vị trí với độ chính xác cao hơn.
33. Sự ức chế các tín hiệu đau bởi kích thích xúc giác trên bề mặt da liên quan đến
nhóm sợi thần kinh nào sau đây?
A. Các sợi thần kinh loại A-alpha trong hệ thần kinh ngoại vi
B. Các sợi thần kinh loại A-beta trong hệ thần kinh ngoại vi
C. Các sợi thần kinh loại A-delta trong hệ thần kinh ngoại vi
D. Các sợi thần kinh loại C trong hệ thần kinh trung ương
 B. Kích thích xúc giác trên bề mặt da liên quan đến các sợi thần kinh loại
A, các sợi này có thể ức chế cảm giác đau.
34. Tại hồi sau trung tâm (vùng cảm giác bản thể nguyên thuỷ S-I), hình chiếu của
các phần cơ thể đối bên trên vùng S-I có kích thước thay đổi tương ứng với?
A. Kích thước tương đối của các phần cơ thể
B. Mật độ receptor ngoại biên trên từng phần cơ thể
C. Kích thước của các cơ trên phần cơ thể đó
D. Vận tốc dẫn truyền của các sợi hướng tâm nguyên thuỷ
 B. Kích thước hình chiếu của các phần khác nhau trên cơ thể ở vùng cảm
giác bản thể nguyên thuỷ S-I thay đổi tương ứng với mật độ receptor ngoại
biên trên từng phần cơ thể.
35. Chất xám tại hồi sau trung tâm (vùng cảm giác bản thể nguyên thuỷ S-I) chứa 6
lớp tế bào. Lớp nào sau đây nhận các tín hiệu đến từ nhân cảm giác bản thể của đồi
thị?
A. Lớp I
B. Lớp II và III
C. Chỉ lớp III
D. Lớp IV
 D. Lớp IV của vùng cảm giác bản thể nhận các tín hiệu đến từ nhân cảm
giác bản thể của đồi thị.
36. Nhận định nào sau đây mô tả đúng nhất về màng tế bào thần kinh ở trạng thái
nghỉ?
A. Nồng độ ion Natri trong dịch ngoại bào thấp hơn nồng độ ion Natri trong
dịch nội bào
B. Ion Clo có nồng độ cao nhất trong tế bào
C. Nếu điện thế nghỉ tụt xuống giá trị âm, tế bào sẽ bị kích thích nhiều hơn
D. Chiều bậc thang nồng độ của ion Kali Natri có xu hướng khuếch tán từ trong
ra ngoài màng
 D. Ở trạng thái nghỉ, nồng độ ion Kali trong dịch nội bào cao hơn trong
dịch ngoại bào, và ion Kali có xu hướng khuếch tán ra ngoài tế bào. Trong
khi đó, nồng độ ion Natri và Clo trong dịch ngoại bào cao hơn nồng độ của
chúng trong dịch nội bào.
37. Giải thích nào sau đây là đúng cho hiện tượng đau đối chiếu?
A. Tín hiệu đau tạng và tín hiệu đau nông liên hợp thần kinh với các nhóm
nơron riêng biệt ở sừng sau tuỷ sống
B. Dẫn truyền cảm giác đau tạng và dẫn truyền cảm giác đau nông được tiếp
nhận bởi một nhóm nơron chung ở đồi thị
C. Tín hiệu đau tạng hiếm khi có cường độ đủ vượt qua ngưỡng kích thích của
các nơron ở sừng sau
D. Một số tín hiệu đau tạng và tín hiệu đau nông hội tụ đến một nhóm nơron
chung ở sừng sau tuỷ sống
 D. Các sợi đau tạng có thể cung cấp đầu vào đến các tế bào trong bó gai
thị trước, các tế bào này cũng nhận tín hiệu đau bản thể từ bề mặt da. Sự hội
tụ của 2 loại tín hiệu đau này vào các nơron riêng lẻ ở tuỷ sống là cơ sở cho
hiện tượng đau đối chiếu.
38. Sự thuận hoá sau khi co cơ được cho là kết quả của hiện tượng nào sau đây?
A. Sự mở các kênh Na+ phụ thuộc điện thế
B. Sự mở các kênh K+ phụ thuộc chất truyền đạt thần kinh
C. Sự tích luỹ ion Ca2+ ở cúc tận cùng
D. Sự dẫn truyền xung động điện
 C. Sự thuận hoá sau khi co cơ là hiện tượng thần kinh mà trong đó một
nơron bị kích thích dễ dàng hơn sau một thời gian chịu kích thích. Điều này
được giải thích là do sự tích luỹ ion Ca2+ ở cúc tận cùng nhờ hoạt động của
hệ thần kinh trước đó. Sự tích luỹ ion Ca2+ từ các kích thích trước tạo điều
kiện cho những xung động sau đó giải phóng chất truyền đạt thần kinh dễ
dàng hơn.
39. Đau xuất phát từ vùng bụng được liên hệ với vùng nào sau đây trên cơ thể?
A. Vùng trên vai bên phải
B. Vùng bụng trên rốn
C. Vùng gốc chi ở trước và trong đùi
D. Vùng bụng dưới rốn
- B. Đau xuất phat từ vùng bụng được liên hệ với vùng bụng trên. Thông
thường, nó sẽ nằm ở vùng bụng trên rốn.
40. Khẳng định nào sau đây là đúng cho các tín hiệu đau tạng?
A. Chúng được dẫn truyền bởi các sợi cảm giác đi trong dây thần kinh giao cảm
của các tạng trong ổ bụng và lồng ngực
B. Chúng không bị kích thích bởi sự thiếu máu cục bộ của các tạng
C. Chúng được dẫn truyền chỉ bởi các sợi cảm giác nhỏ, có bao myelin thuộc
loại A
D. Chúng thường được cảm nhận chính xác về vị trí
 A. Các tín hiệu đau tạng từ các tạng trong ổ bụng và lồng ngực đi qua tuỷ
sống và liên hệ với các sợi thần kinh trong hệ giao cảm.
Câu hỏi 41-43
Mỗi một loại rối loạn trong 3 câu hỏi sau đây đều được đặc trưng bởi sự truyền
cảm giác đau mạnh hơn (chứng đau tăng) hoặc mất cảm giác đau.
41. Rối loạn nào sau đây được đặc trưng bởi sự truyền cảm giác đau mạnh hơn trên
một vùng tổn thương da do nhiễm virus tại hạch gai của rễ sau tuỷ sống?
A. Đau dây thần kinh sinh ba
B. Hội chứng đồi thị
C. Hội chứng Brown-Séquard
D. Zona thần kinh
 D. Zona thần kinh là một rối loạn được đặc trưng bởi cảm giác đau mạnh
hơn trên một vùng tổn thương da do nhiễm virus tại hạch gai của rễ sau tuỷ
sống.
42. Rối loạn nào sau đây liên quan tới sự mất cảm giác đau ở một bên cơ thể kết
hợp với mất sự nhận cảm trong cơ thể, mất sự nhận cảm chính xác về vị trí xúc
giác và mất sự nhận cảm rung động ở đối bên cơ thể?
A. Zona thần kinh
B. Hội chứng đồi thị
C. Hội chứng tuỷ bên (Hội chứng Wallenberg)
D. Hội chứng Brown-Séquard
D. Hội chứng Brown-Séquard được đặc trưng bởi sự mất cảm giác đau ở
một bên cơ thể kết hợp với mất sự nhận cảm trong cơ thể, ví dụ mất sự nhận
cảm chính xác về vị trí xúc giác và mất sự nhận cảm rung động ở đối bên cơ
thể
43. Rối loạn nào sau đây được đặc trưng bởi sự mất cảm giác đau ở toàn bộ một
bên cơ thể và nửa mặt ở bên đối diện?
A. Hội chứng Brown-Séquard
B. Hội chứng đồi thị
C. Zona thần kinh
D. Hội chứng tuỷ bên (Hội chứng Wallenberg)
 D. Hội chứng tuỷ bên là một trong những rối loạn mất cảm giác trong
thần kinh lâm sàng – cảm giác đau bị mất ở một bên cơ thể từ chân tới cổ và
nửa mặt bên đối diện. Hơn nữa, mất cảm giác đau một bên mặt gợi ý bên tổn
thương.
44. Hiện tượng điện học nào sau đây là đặc trưng của các tương tác ức chế synap?
A. Khi một chất truyền đạt thần kinh gây mở chọn lọc kênh Cl-, điện thế ức chế
sau synap sẽ được hình thành
B. Do điện thế Nernst của ion Clo là khoảng -70 mV nên ion Clo có xu hướng
khuếch tán vào trong màng tế bào theo chiều bậc thang điện hoá của nó
C. Ion Kali khuếch tán vào trong màng tế bào do một chất truyền đạt thần kinh
gây mở chọn lọc kênh K+
D. Sự tăng nồng độ ion Natri ở bên ngoài màng tế bào thường trực tiếp sinh ra
một điện thế ức chế sau synap
 A. Sự mở các kênh Clo và sự di chuyển của ion Clo vào trong tế bào gây
ra hiện tượng ưu phân cực của màng. Sự tăng nồng độ ion Natri trong dịch
ngoại bào hay sự di chuyển của ion Kali vào trong tế bào đều không thể gây
ra hiện tượng ưu phân cực màng.
45. Sự thiếu hụt cảm giác bản thể nào sau đây không điển hình cho các tổn thương
liên quan đến hồi sau trung tâm?
A. Không có khả năng nhận cảm xúc giác rời rạc (xác định chính xác vị trí bị
kích thích) ở vùng mặt và chi trên của bên đối diện
B. Không có khả năng đánh giá đúng trọng lượng của những đồ vật dễ nhận
dạng
C. Không có khả năng nhận biết được tính chất bề mặt của những đồ vật thông
dụng khi chạm vào chúng bằng các đầu ngón tay
D. Không có khả năng vận động vùng vai và chân của bên đối diện
 D. Chứng liệt vai và chân bên đối diện là sự thiếu hụt vận động và sự
thiếu hụt này sẽ không điển hình trong các tổn thương của hồi sau trung tâm
(vùng cảm giác bản thể nguyên thuỷ)
46. Khả năng xác định 2 điểm được kích thích đồng thời trên da dựa trên cơ chế
nào sau đây?
A. Sự ức chế trước synap
B. Sự ức chế bên
C. Sự ức chế giữa
D. Sự ức chế truyền thẳng
 B. Quá trình ức chế bên được minh hoạ trong hình, là nền tảng cho khả
năng phân biệt 2 điểm được kích thích đồng thời trên da.
47. Kích thích bằng cách chạm hoặc kéo lên cấu trúc nào sau đây ít có khả năng
gây ra cảm giác đau nhất?
A. Hồi sau trung tâm
B. Vùng màng cứng nằm trên hồi sau trung tâm
C. Các nhánh của động mạch não giữa nằm trên màng cứng của hồi sau trung
tâm (vùng màng cứng nằm trên hồi sau trung tâm)
D. Các nhánh của động mạch não giữa cung cấp máu cho hồi sau trung tâm
 A. Kích thích bằng cách chạm hoặc kêos lên hồi sau trung tâm ít có khả
năng gây ra cảm giác đau nhất bởi vì các mô não tại đây thiếu receptor đau.
48. Sự rung động phụ thuộc vào khả năng tiếp nhận các cảm giác thay đổi nhanh
và lặp lại. Vùng tần số cao theo phân độ kích thích lặp lại được tiếp nhận bởi
receptor xúc giác nào sau đây?
A. Đĩa Merkel
B. Tiểu thể Meissner
C. Tiểu thể Pacini
D. Một số đầu dây thần kinh tự do
 C. Kích thích lặp lại, tần số cao (sự va chạm/áp suất) lên da được tiếp
nhận bởi các tiểu thể Pacini
49. Những biến đổi kéo dài trong hoạt động của hệ thần kinh thường đạt được
thông qua việc hoạt hoá kênh nào sau đây?
A. Các kênh Cl-
B. Các kênh Na+
C. Các kênh gắn protein G
D. Các kênh K+
 C. Sự hoạt hoá protein G thường thay đổi những đặc điểm đáp ứng dài
hạn của nơron.
50. Sự dẫn truyền các tín hiệu điện từ các đuôi gai đến thân nơron xảy ra do cơ chế
nào sau đây?
A. Dòng đoản mạch
B. Một cơ thế điện thế hoạt động
C. Sự dẫn truyền xung động điện
D. Sự phóng điện
C. Các đuôi gai có một vài kênh Natri. Khả năng tạo ra điện thế hoạt động
của các cấu trúc này là vô cùng hạn chế. Do vậy, sự dẫn truyền các tín hiệu
điện trong đuôi gai dựa trên cơ chế dẫn truyền xung động điện mà không
phải là điện thế hoạt động
51. Khẳng định nào sau đây là đúng khi nói về nơron cảm giác và các tính chất
chức năng của chúng?
A. Tất cả các sợi cảm giác đều không được bọc myelin
B. Khi cộng kích thích trong không gian, cường độ tín hiệu đang tăng sẽ được
dẫn truyền bằng cách tăng sử dụng các sợi cảm giác
C. Cường độ kích thích đã tăng sẽ được tín hiệu hoá bằng cách giảm dần điện
thế receptor
D. Sự kích thích dưới ngưỡng liên tục của một mạng nơron cảm giác dẫn đến
sự mất thuận hoá các nơron đó
E. Cộng kích thích theo thời gian liên quan đến sự tín hiệu hoá của cường độ
kích thích đã tăng bằng cách giảm tần số điện thế hoạt động trong các sợi
cảm giác
 B. Khi cộng kích thích trong không gian, cường độ tín hiệu đang tăng
được dẫn truyền bằng cách tăng sử dụng các sợi cảm giác.
52. Khi một sợi thần kinh cảm giác tiếp nối với một tiểu thể Pacini trên bề mặt gan
bàn tay phải, sự tiếp nối synap với các nơron tiếp theo trên con đường dẫn truyền
cảm giác sẽ được đặt tại vị trí nào sau đây?
A. Nhân bó cột sau bên phải
B. Nhân bó cột sau bên trái
C. Sừng sau bên phải của tuỷ sống
D. Sừng sau bên trái của tuỷ sống
 A. Tiểu thể Pacini dẫn truyền một loại cảm giác (rung động), loại cảm
giác này được truyền đến bó gai thị sau. Sự tiếp nối synap đầu tiên trên con
đường dẫn truyền cảm giác nằm ở nhân bó cột sau cùng bên cơ thể.
53. Chứng đau nửa đầu thường bắt đầu với các triệu chứng báo trước như nôn, mất
thị lực, tiền triệu thị giác hoặc các ảo giác giác quan khác. Giải thích nào sau đây là
đúng về nguyên nhân gây ra các tiền triệu đó?
A. Tăng dòng máu đến các mô não trong vùng vỏ não thị lực và vùng vỏ não
giác quan khác
B. Sự mất chọn lọc các nơron GABA trong nhiều vùng vỏ não giác quan
C. Chứng táo bón
D. Sự co thắt mạch dẫn tới thiếu máu cục bộ và gián đoạn hoạt động của hệ
thần kinh trong nhiều vùng vỏ não giác quan liên quan
 D. Co thắt mạch và cuối cùng là thiếu máu cục bộ trong vùng vỏ não giác
quan là cơ sở cho các triệu chứng báo trước (tiền triệu) mà bệnh nhân mắc
chứng đau nửa đầu phải trải qua.
54. Khẳng định nào sau đây là đúng về sự phát sinh một điện thế hoạt động?
A. Khi điện thế màng trong thân/ gò sợi trục (nơi tiếp xúc giữa thân nơron và
sợi trục) giảm xuống dưới ngưỡng, một điện thế hoạt động được sinh ra
B. Điện thế hoạt động được sinh ra tại các cúc tận cùng
C. Trong một sợi trục, vùng gần eo Ranvier có số lượng kênh Na+ ít nhất
D. Ngay khi một điện thế hoạt động được sinh ra, nó sẽ luôn luôn hoàn tất quá
trình dẫn truyền
 D. Điện thế hoạt động tuân theo quy luật tất cả hoặc không. Ngay khi
được sinh ra, điện thế hoạt động sẽ hoàn tất quá trình dẫn truyền của nó.
55. Cảm giác vị trí, hay thông dụng hơn là cảm giác về vị trí cơ thể liên quan đến
các suốt cơ và các receptor xúc giác nào sau đây?
A. Các receptor xúc giác ở da
B. Các receptor nằm sâu trong bao khớp
C. Cả các receptor xúc giác và bao khớp
D. Các tiểu thể Pacini
 C. Cảm giác vị trí phụ thuộc vào các receptor xúc giác và bao khớp
56. Cảm giác nhiệt độ được tiếp nhận chủ yếu bởi các receptor nóng và lạnh, các
sợi cảm giác tại đây sẽ đi cùng với các sợi cảm giác dẫn truyền tín hiệu đau. Khẳng
định nào sau đây mô tả chính xác đặc trưng của sự dẫn truyền tín hiệu từ các
receptor nóng?
A. Các receptor nóng được biệt hoá cao về mô học
B. Các tín hiệu từ receptor nóng chủ yếu dẫn truyền dọc theo các sợi cảm giác
chậm loại C
C. Các receptor nóng được đặt dưới bề mặt da trong các mô liên kết dưới da
D. Trong hầu hết các vùng cơ thể, số lượng các receptor nóng nhiều gấp 3 đến
10 lần số lượng các receptor lạnh
 B. Các receptor nóng chủ yếu dẫn truyền tín hiệu dọc theo các sợi dẫn
truyền chậm, loại C.
57. Không giống như các hệ thần kinh cảm giác khác, hệ cảm giác bản thể có một
bộ phận có khả năng điều chỉnh toàn bộ độ nhạy của hệ thống. Khẳng định nào sau
đây là đúng khi mô tả chức năng của tín hiệu corticofugal được dẫn truyền từ vùng
cảm giác bản thể xuống đồi thị và nhân bó cột sau bên?
A. Tăng hoặc giảm sự nhận biết của cường độ tín hiệu
B. Giảm khả năng tiếp nhận cảm giác vị trí cơ thể
C. Sự loại bỏ đồi thị trong quá trình xử lý các tín hiệu cảm giác bản thể
D. Cho phép tăng thông tin bỏ qua nhân chêm và nhân thon
 A. Bộ phận điều chỉnh hoạt động vỏ não của hệ cảm giác bản thể có chức
năng tăng và giảm sự nhận biết của cường độ tín hiệu.
58. Khẳng định nào sau đây là đúng khi mô tả đặc điểm nhận cảm nhiệt độ của cơ
thể?
A. Các receptor lạnh tiếp tục bị hoạt hoá ngay cả khi nhiệt độ da thấp hơn nhiều
so với điểm đông của nó
B. Cả các receptor lạnh và nóng đếu rất biệt hoá, các khoảng không chồng chéo
của độ nhạy nhiệt độ
C. Các receptor nóng và lạnh tương ứng với nhiệt độ trạng thái ổn định và sự
thay đổi nhiệt độ
D. Chức năng của các receptor nhiệt độ là kết quả của sự thay đổi dẫn truyền
ion và không thay đổi tốc độ chuyển hoá của chúng
 C. Cả receptor nóng và lạnh đều có khả năng đáp ứng với nhiệt độ trạng
thái ổn định cũng như sự thay đổi nhiệt độ.
59.Khẳng định nào sau đây là đúng khi nói về sự dẫn truyền qua synap?
A. Khi một tập hợp riêng của các tận cùng synap lan truyền trên bề mặt của một
nơron, hiệu quả của tập hợp trên không thể cộng hợp trong không gian và
sinh ra một điện thế hoạt động
B. Ngay cả khi sự phát xung liên tục của một synap kích thích xảy ra trong thời
gian đủ gần, chúng cũng không thể cộng hợp theo thời gian và sinh ra một
điện thế hoạt động
C. Một nơron được thuận hoá khi điện thế màng hạ xuống giá trị âm hơn hoặc
bị khử cực
D. Ngay cả khi bị kích thích với tần số cao bởi nguồn kích thích synap trong
một thời gian dài, các nơron cũng thường không có hiện tưởng mỏi synap
 C. Một nơron được thuận hoá là nơron có điện thế màng ở trạng thái nghỉ
gần hơn với ngưỡng kích thích, nghĩa là hạ xuống giá trị âm hơn hoặc bị khử
cực.
60. Khẳng định nào sau đây là đúng khi mô tả về quá trình xử lý các tín hiệu cảm
giác bởi một mạng nơron?
A. Sự hội tụ của các tín hiệu đầu vào đến một nơron riêng trong mạng nơron,
khi mà mỗi nơron đều cùng kích thích vào một kênh đầu ra, có thể dẫn đến
sự khuếch đại tín hiệu
B. Sự phân kì của các tín hiệu đầu vào đến các nơron khác nhau trong mạng
nơron, khi mà mỗi nơron trong mạng đều kích thích vào một kênh đầu ra
khác nhau, có thể dẫn đến sự khuếch tán tín hiệu
C. Sự kết hợp giữa các tín hiệu đầu vào khác nhau (xuất phát từ các nguồn khác
nhau) đến một nơron đơn lẻ trong mạng nơron là một ví dụ của sự phân kì
D. Sự phân bố của các tín hiệu đầu vào khác nhau (xuất phát từ cùng một
nguồn) đến nhiều nơron mạng nơron là một ví dụ của sự hội tụ
 D. Sự hội tụ xảy ra khi các tín hiệu đầu vào được gửi đến các nơron khác nhau
trong mạng nơron, và mỗi nơron sinh ra một tín hiệu gửi đến kênh đầu ra của
chúng.

Unit 10: Giác quan đặc biệt


1. Khẳng định nào sau đây là đúng khi nói về nhân gối ngoài?
A. Lớp thứ nhất, còn được gọi là lớp tế bào parvo
B. Lớp thứ nhất nhận tín hiệu từ một nửa bên của võng mạc
C. Lớp thứ nhất nhận tín hiệu xuất phát từ các tế bào que
D. Lớp thứ tư nhận tín hiệu từ võng mạc cùng bên
E. Lớp thứ tư nhận tín hiệu từ các tế bào hạch loại Y
 C. Lớp I và II của các nhân gối ngoài được gọi là lớp tế bào magno, và nhận tín
hiệu đầu vào từ các tế bào que đến các tế bào hạch loại Y. Lớp III đến IV được gọi
là lớp tế bào parvo và nhận tín hiệu đầu vào từ các tế bào nón đến các tế bào hạch
TMP12 623–624
2. Chất nào sau đây gây cảm giác chua?
A. Aldehyd
B. Alkaloid
C. Amino acid
D. Ion H+
E. Ketone
 D. Nhận cảm cảm giác chua tỉ lệ với logarit của nồng độ ion H+, gây ra bởi các
acid. Cảm giác ngọt gây ra bởi một loạt các chất hoá học như đường, rượu,
aldehyd, ketone, và amino acid.
TMP12 645
3. Khẳng định nào sau đây là đúng khi nói về sự khúc xạ của ánh sáng?
A. Các sóng ánh sáng trong môi trường rắn trong suốt sẽ có bước sóng dài hơn
so với trong môi trường không khí
B. Các sóng ánh sáng trong môi trường rắn trong suốt sẽ lan truyền với vận tốc
lớn hơn so với trong môi trường không khí
C. Chỉ số khúc xạ của một chất rắn trong suốt là tỉ số của vận tốc ánh sáng
trong không khí trên vận tốc ánh sáng trong trong chất đó
D. Chỉ số khúc xạ của không khí là 0
E. Khi các sóng ánh sáng đi vào môi trường rắn trong suốt, chúng luôn luôn
phản xạ ngược lại chứ không đi qua môi trường đó.
 C. Các tia sáng lan truyền trong không khí với tốc độ khoảng 300,000 km/sec,
nhưng chậm hơn nhiều so với trong môi trường rắn trong suốt. Do vậy, chỉ số khúc
xạ của không khí sẽ là 1, và chỉ số khúc xạ của bất cứ chất rắn trong suốt nào cũng
sẽ > 1
TMP12 597
4. Khẳng định nào sau đây là đúng khi so sánh vùng trung tâm võng mạc với
vùng ngoại vi (vùng rìa)
A. Trung tâm võng mạc chứa nhiều tế bào nón hơn vùng rìa
B. Trung tâm võng mạc chứa nhiều tế bào hạch hơn vùng rìa
C. Trung tâm võng mạc chứa nhiều tế bào ngang hơn vùng rìa
D. Trung tâm võng mạc chứa nhiều nhiều tế bào que hơn vùng rìa
E. Trung tâm võng mạc chứa nhiều mạch máu hơn vùng rìa
 A. Vùng trung tâm võng mạc tập hợp hầu như toàn bộ các tế bào nón. Mạch
máu, các tế bào hạch và những lớp tế bào khác đều nằm ở một bên, do đó cho phép
ánh sáng đi thẳng vào các tế bào nón mà không bị cản trở
TMP12 609
5. Xương nhỏ nào của tai giữa gắn với màng nhĩ?
A. Trụ ốc tai
B. Xương đe
C. Xương búa
D. Cột ốc tai
E. Xương bàn đạp
 C. Xương búa được gắn với màng nhĩ và xương bàn đạp được gắn với cửa sổ
bầu dục. Xương đe tiếp khớp với cả 2 xương trên.
TMP12 633
6. Khi đi vào mắt, ánh sáng sẽ qua lớp võng mạc nào đầu tiên?
A. Lớp nhân trong
B. Lớp nhân ngoài
C. Lớp rối ngoài
D. Lớp tế bào nhận cảm ánh sáng
E. Lớp hạch
 E. Ánh sáng qua mắt vào võng mạc nằm ở phần sau của mắt. Lớp trước nhất
của võng mạc (lớp cho ánh sáng đi qua đầu tiên), là lớp hạch. Ánh sáng sau đó đi
qua các lớp tế bào khác của võng mạc cho đến khi gặp các tế bào nhận cảm ánh
sáng ở vùng sau võng mạc.
TMP12 609
7. Các tế bào hạch gắn với tế bào nhận cảm ánh sáng ở võng mạc sẽ truyền tín
hiệu đến cấu trúc nào sau đây?
A. Nhân gối ngoài đối bên
B. Nhân gối ngoài cùng bên
C. Nhân gối giữa cùng bên
D. Rãnh cựa
E. Nhân gối giữa đối bên
 B. Các sợi trục của tế bào hạch tạo nên các sợi của thần kinh thị giác. Synap đầu
tiên trong hệ thống thị giác nằm ở nhân gối ngoài. Các tế bào hạch gắn vào các tế
bào nhận cảm ánh sáng trên võng mạc phía thái dương, đi lên các nhân gối ngoài
cùng bên. Các sợi từ võng mạc phía mũi đi lên các nhân gối ngoài đối bên trong
giao thoa thị giác. Các nhân gối giữa là đường truyền cho hệ thống thính giác.
TMP12 623
8. Khẳng định nào sau đây là đúng khi nói về điểm mù của mắt?
A. Điểm mù nằm lệch 5 độ về phía bên (phía thái dương) so với điểm trung tâm
của thị giác (điểm vàng)
B. Vị trí đi ra của dây thần kinh thị giác
C. Chỉ chứa tế bào que và do vậy có thị giác đơn sắc
D. Không chứa mạch máu
E. Là vị trí chứa nhiều cấu trúc đơn sắc nhất
 B. Điểm mù nằm lệch 15 độ về phía bên so với điểm trung tâm của thị giác
(điểm vàng). Nó là nơi các sợi thần kinh thị giác đi ra ngoài nhãn cầu. Do vậy,
không có các tế bào nhận cảm ánh sáng tại vị trí này.
TMP12 627
9. Khi chùm tia sáng song song đi qua một thấu kính phân kỳ, hiện tượng nào
sau đây sẽ xảy ra?
A. Các tia hội tụ gần nhau
B. Các tia phân tán xa nhau
C. Các tia vẫn chạy song song với nhau
D. Các tia phản xạ ngược lại hướng chúng đến
E. Các tia khúc xạ đến một tiêu điểm
 B. Một thấu kính phân kỳ sẽ phân tán các tia sáng và ngược lại một thấu kính
hội tụ sẽ tập trung các tia sáng gần nhau. Nếu một thấu kính hội tụ có độ cong
thích hợp, các tia song song sẽ bị bẻ cong để tập trung tại một điểm gọi là tiêu
điểm.
TMP12 598
10.Khẳng định nào sau đây là đúng khi nói về hiện tượng giảm phản xạ?
A. Có thể làm tăng cường độ dẫn truyền các sóng âm có tần số thấp lên 30 đến
40 decibel
B. Làm tăng sự ‘cứng nhắc’ của hệ thống xương con, qua đó giảm dẫn truyền
các sóng âm có tần số thấp
C. Lấp các sóng âm có tần số cao trong môi trường ồn ào nên các sóng có tần
số thấp hơn sẽ dễ dàng được nghe hơn
D. Xảy ra theo một chu kì ẩn từ 4 đến 8 giây sau một âm thanh lớn
E. Bảo vệ ốc tai khỏi những dao động do các sóng âm có tần số cao nhưng
tương đối im lặng gây ra
 B. Cơ căng màng nhĩ kéo cán xương búa vào phía trong, trong khi cơ bàn đạp
kéo xương bàn đạp ra ngoài. Hai lực này đối lập nhau và do vậy làm cho hệ thống
xương con vững chắc hơn, giảm cường độ của các sóng âm có tần số thấp xuống
mức 30 đến 40 decibel
TMP12 634
11.Chất nào sau đây gây cảm giác đắng?
A. Aldehyd
B. Alkaloid
C. Amino acid
D. Ion H+
E. Ketone
 B. Nhận cảm cảm giác đắng được gây ra bởi những chất hữu cơ chứa nitrogen
cũng như bởi các alkaloid.
TMP12 645
12.Tổn thương dây thần kinh VI sẽ làm giảm vận động nhãn cầu như thế nào?
A. Mất khả năng di chuyển mắt lên xuống theo trục dọc
B. Mất khả năng xoay mắt trong ổ mắt
C. Mất khả năng di chuyển mắt từ phía bên vào đường giữa
D. Mất khả năng di chuyển mắt từ đường giữa ra xa 2 bên
E. Lác dọc
 D. Dây thần kinh sọ VI còn được gọi là dây vận nhãn ngoài. Dây vận nhãn
ngoài phân bố thần kinh cho cơ vận thẳng bên (là cơ gắn với mặt bên của nhãn
cầu). Sự co giãn của cơ thẳng bên giúp di chuyển mắt từ đường giữa mặt ra xa 2
bên, nên còn được gọi là dây thần kinh vận nhãn ngoài.
TMP12 627–628
13.Khẳng định nào sau đây là đúng khi nói về tiêu cự của một thấu kính hội tụ?
A. Khi đi qua một thấu kính hội tụ, các tia sáng hội tụ sẽ tập trung tại một tiêu
điểm nằm xa tiêu cự thấu kính hơn
B. Khi đi qua một thấu kính hội tụ, các tia sáng phân kỳ sẽ hội tụ tại một tiêu
điểm nằm gần tiêu cự thấu kính hơn
C. Khi đi qua một thấu kính hội tụ, các tia sáng song song sẽ tập trung tại một
tiêu điểm bằng tiêu cự thấu kính
D. Hình ảnh được tạo ra bởi một thấu kính hội tụ nằm ở phía trên, bên phải
nhưng hai bên vẫn được giữ nguyên như vật
E. Thấu kính với độ cong lớn nhất sẽ có tiêu cự lớn nhất
 C. Khoảng cách từ tiêu điểm (nơi các tia sáng song song hội tụ) đến quang tâm
của thấu kính hội tụ được gọi là tiêu cự. Do vậy, tiêu điểm sẽ tương ứng tiêu cự
trong một thấu kính hội tụ. Khi thấu kính lồi có độ cong lớn hơn, tiêu cự sẽ nhỏ
hơn tiêu điểm (vị trị các tia sáng song song hội tụ)
TMP12 597–598
14.Nếu một thấu kính hội tụ có tiêu cự là 1 cm (0.01 m), độ khúc xạ của thấu
kính này sẽ là bao nhiêu?
A. +0.01
B. +0.1
C. +1
D. +10
E. +100
 E. Thấu kính hội tụ càng bẻ cong chùm tia song song, độ khúc xạ của nó càng
lớn, độ khúc xạ được đo bằng đơn vị diop. Theo định nghĩa, một thấu kính hội tụ
với tiêu cự 1m sẽ có độ khúc xạ là 1 diop. Nếu một thấu kính hội tụ có thể bẻ cong
chùm tia song song gấp đôi thì thấu kính đó sẽ là độ khúc xạ gấp đôi (2 diop) và
tiêu cự 0.5 m. Do vậy, tiêu cự sẽ tỷ lệ nghịch với độ khúc xạ. Trong câu hỏi này,
thấu kính hội tụ có tiêu cự 1 cm (0.01 m) nên sẽ có độ khúc xạ gấp 100 lần, là 100
diop.
TMP12 600
15.Quá trình hay điều kiện nào sau đây dẫn tới bệnh lý đục thuỷ tinh thể?
A. Biến tính protein trong nhân mắt
B. Nhãn cầu dài
C. Đồng tử giãn và không đáp ứng
D. Đông đặc protein trong nhân mắt
E. Tăng áp suất nội nhãn
 D. Bệnh đục thuỷ tinh thể làm nhân mắt trở nên mờ đục (giống như khi ta nhìn
vào một dòng thác nước hoặc một con sông chảy xiết), nên còn được gọi là đục
nhân mắt. Các protien cấu tạo nên nhân mắt bị đông đặc tiến triển làm nhân mắt
mờ đục. Có thể ví quá trình này giống như lòng trắng trứng trở nên đông đặc khi bị
nấu chín. Việc nấu chín lòng trắng trứng sẽ dẫn tới quá trình đông đặc protien của
nó.
TMP12 604
16.Nhận cảm vị giác nào sau đây là nhạy cảm nhất (VD: có ngưỡng kích thích
thấp nhất)?
A. Acid
B. Đắng
C. Mặn
D. Chua
E. Ngọt
 B. Cảm giác đắng nhạy cảm hơn nhiều so với các cảm giác vị giác khác, đây có
thể là một chức năng bảo vệ quan trọng của cảm giác này, giúp cơ thể tống nhiều
chất gây độc trong thức ăn ra ngoài.
TMP12 646
17.Khẳng định nào sau đây là đúng khi nói về màng đáy?
A. Rung động với tần số cao xảy ra ở gần đáy ốc tai, trong khi rung động với
tần số thấp xảy ra ở phía đỉnh ốc tai
B. Hạch xoắn ốc nằm trên bề mặt màng đáy
C. Chứa các sợi nền với đường kính tăng dần từ đáy ốc đến đỉnh ốc tai
D. Chứa các sợi nền với đường kính giảm dần từ đáy ốc đến đỉnh ốc tai
E. Ngăn cách thang giữa với thang tiền đình
 A. Màng đáy (nhìn trên hình) chứa các sợi nền với độ dài tăng dần từ đáy
đến đỉnh ốc tai, trong khi sự rung động của các sợi nền giảm dần. Do đó, các
sợi rung động với tần số cao sẽ ở gần đáy ốc tai và các sợi rung động với tần
số thấp sẽ ở gần đỉnh ốc tai.

Hình ảnh minh hoạ


TMP12 635–636
18.Chất nào sau đây gây ra cảm giác ngọt thịt?
A. Acid acetic
B. Kali tartrate
C. Các chất hữu cơ chuỗi dài có chứa nito
D. Fructose
E. Glutamate
 E. Khái niệm ngọt thịt bắt nguồn từ một từ trong tiếng Nhật Bản, nghĩa là có
mùi vị hoặc ngon, và thường được mô tả như vị của thịt. Glutamate được cho là
chất hoá học gây ra cảm giác ngọt thịt này.
TMP12 646

19.Quá trình phân tích chi tiết hình ảnh xảy ra ở vị trí nào trong vùng thị giác
thứ cấp?
A. Vùng 18 theo bản đồ vỏ não của Brodmann
B. Vùng bụng dưới (trước-dưới) và vùng giữa của thuỳ thái dương và thuỳ
chẩm
C. Thuỳ trán
D. Vỏ não vùng đỉnh chẩm
E. Vùng trước-giữa của thuỳ thái dương
 B. Thông tin thị giác từ vùng thị giác sơ cấp (vùng 17) được gửi tới vùng 18 và
rồi đi vào trong các vùng khác của vỏ não để tiếp tục quá trình phân tích. Quá trình
phân tích hình thể-hình dạng ba chiều và chuyển động của vật xảy ra ở vùng sau
giữa thuỳ thái dương và vỏ não vùng đỉnh chẩm. Quá trình phân tích chi tiết và
màu sắc của vật xảy ra ở vùng trước-dưới và vùng giữa của thùy chẩm và thuỳ thái
dương.
TMP12 624,626
20.Khẳng định nào sau đây là đúng khi nói về vai trò của melanin trong lớp sắc
tố võng mạc?
A. Tiền chất của rhodopsin (chất nhạy cảm với ánh sáng)
B. Đóng vai trò như chất dinh dưỡng cho tế bào nón và tế bào que của võng
mạc
C. Lớp sắc tố đen không cho ánh sáng phản xạ trong nhãn cầu
D. Đảm bảo sự toàn vẹn của ống Schelemm
E. Ánh sáng phản xạ qua lớp sắc tố melanin đóng vai trò quan trọng trong việc
thích nghi của nhân mắt
 C. Melanin là sắc tố đen có nhiều trong lớp sắc tố của võng mạc. Lớp sắc tố có
chức năng giới hạn sự tán xạ ánh sáng trong nhãn cầu và tăng độ tương phản và rõ
ràng của ảnh.
TMP12 609–611
21.Rhodopsin là phức hợp của 2 chất nào sau đây?
A. Bathorhodopsin và 11-cis-retinal
B. Bathorhodopsin và all-trans-retinal
C. Bathorhodopsin và scotopsin
D. Scotopsin và 11-cis-retinal
E. Scotopsin và all-trans-retinal
 D. Rhodopsin là chất nhạy cảm với ánh sáng trong tế bào que. Scotopsin và all-
trans retinal là sản phẩm phân huỷ của rhodopsin, có khả năng hấp thụ năng lượng
ánh sáng. All-trans retinal được chuyển thành 11-cis retinal, là chất có thể kết hợp
lại với scotopsin để tạo thành rhodopsin.
TMP12 611
22.Sự thiếu hụt vitamin nào sau đây sẽ ngăn cản sự tổng hợp đầy đủ lượng
retinal, và cuối cùng gây bệnh quáng gà?
A. Vitamin A
B. Vitamin C
C. Vitamin D
D. Vitamin E
E. Vitamin K
 A. Một dạng của vitamin A là all-trans retinol, là chất được chuyển dạng qua 2
con đường khác nhau thành 11-cis-retinal, sau đó kết hợp với scotopsin để tạo
thành rhodopsin. Vitamin A được dự trữ với lượng lớn trong gan. Tuy nhiên, sự
thiếu hụt vitamin A trong chế độ ăn trong nhiều tháng có thể dẫn tới bệnh quáng
gà, do rhodopsin cần thiết cho chức năng của tế bào que.
TMP12 612
23.Ở những người trên 70 tuổi, nhân mắt hầu như không thể thích nghi được
nữa. Tên gọi của hiện tượng này là?
A. Giảm thị lực
B. Mắt bình thường (chính thị)
C. Tật viễn thị
D. Tật cận thị
E. Tật lão thị
 E. Trong tật lão thị, mỗi mắt chỉ còn khả năng tập trung vĩnh viễn tại một
khoảng cách nhất định. Mắt không thể thích nghi với nhìn gần cũng như nhìn xa
nữa. Tật viễn thị và tật cận thị liên quan đến khả năng nhìn xa và nhìn gần. Chính
thị là thị giác bình thường. Giảm thị lực là nguyên nhân dẫn tới sự giảm hoặc mất
thị lực cả 2 mắt.
TMP12 601
24.Ngăn nào trong ốc tai chứa cơ quan Corti?
A. Bóng ống bán khuyên
B. Cầu nang
C. Thang giữa
D. Thang hòm nhĩ
E. Thang tiền đình
 C. Bóng ống bán khuyên và cầu nang là một bộ phận của hệ thống tiền đình,
nhưng không thuộc hệ thống ốc tai. Ốc tai có 3 phần chính, với sự chuyển động
của dịch xảy ra trong thang giữa và thang tiền đình tương ứng với các dao động
âm. Cơ quan Corti nằm trong thang giữa.
TMP12 634–635
25.Khẳng định nào sau đây là đúng khi nói về sự dẫn truyền thông tin vị giác từ
lưỡi đến vỏ não?
A. Đa số nơron vùng đồi thị thuộc đường dẫn truyền cảm giác vị giác sẽ tạo
synap trong thuỳ chẩm
B. Các sợi thần kinh mang thông tin vị giác từ lưỡi sẽ không tạo synap trong
thân não
C. Các sợi thần kinh mang thông tin vị giác từ lưỡi sẽ tạo synap trong các nhân
đơn độc
D. Nhân đồi thị thuộc đường dẫn truyền cảm giác vị giác là các nhân lưng giữa
E. Nhân đồi thị thuộc đường dẫn truyền cảm giác vị giác là các nhân bụng sau
bên
 C. Tất cả các sợi vị giác đều tạo synap trong nhân đơn độc và gửi nơron thứ hai
vào nhân bụng sau giữa của đồi thị. Nơron thứ ba đi lên hồi đỉnh lên, gần vùng
cảm giác lưỡi trên vùng cảm giác thân SI.
TMP12 648
26.Tế bào nào trong lớp IV của vùng thị giác sơ cấp có vai trò xác định hướng
của các đường thẳng và các đường ranh giới?
A. Các tế bào rìa
B. Các tế bào phức tạp
C. Các tế bào hạch
D. Các tế bào quá phức tạp
E. Các tế bào đơn giản
 E. Tế bào đơn của vùng thị giác sơ cấp có vai trò xác định hướng của các đường
thẳng và các đường ranh giới, trong khi tế bào phức tạp có vai trò xác định các
đường thẳng nằm cùng hướng chứ không nằm ở các vị trí riêng biệt. Điều này có
nghĩa đường thẳng có thể bị dịch chuyển sang bên hoặc tính tiến theo chiều dọc với
một khoảng cách phù hợp, và một số nơron giống nhau sẽ bị kích thích miễn sao
các đường thẳng đều nằm cùng một hướng.
TMP12 626
27.Quá trình truyền sóng âm trong ốc tai xảy ra khi nào?
A. Khi nền xương bàn đạp đẩy vào trong cửa sổ bầu dục và cửa sổ tròn lồi ra
ngoài
B. Khi nền xương bàn đạp đẩy vào trong cửa sổ tròn và cửa sổ bầu dục lồi ra
ngoài
C. Khi chỏm xương búa đẩy vào trong cửa sổ bầu dục và cửa sổ tròn lồi ra
ngoài
D. Khi xương đe đẩy vào trong cửa sổ bầu dục và cửa sổ tròn lồi ra ngoài
E. Khi xương đe đẩy vào trong cửa sổ tròn và cửa sổ bầu dục lồi ra ngoài
 A. Xương búa được nối với màng nhĩ, xương con tiếp khớp với xương búa và
xương bàn đạp, và xương bàn đạp được nối với cửa sổ bầu dục.
TMP12 633
28.Trong điều kiện ánh sáng yếu, hợp chất hoá học nào sau đây có vai trò bơm
dòng Natri từ màng đoạn ngoài của tế bào nhận cảm ánh sáng vào trong tế
bào?
A. Metarhodopsin II
B. Cyclic GMP

C. 11-cis retinal

D. Cyclic AMP
E. 11-trans retinol

 B. Trong điều kiện ánh sáng yếu, nồng độ cGMP cao. Các kênh Natri phụ thuộc
cGMP trên đoạn ngoài của tế bào que và nón cho phép ion Natri di chuyển từ
ngoài vào trong tế bào nhận cảm ánh sáng. Quá trình này làm sinh ra một điện thế
màng, thấp hơn điện thế màng của một nơron điển hình khi ở trạng thái nghỉ. Sự di
chuyển của ion Natri và sự thay đổi điện thế sau đó (kết quả của sự tăng tính thấm
màng tế bào) được gọi là dòng tối.
TMP12 612–613
29.Khẳng định nào sau đây là đúng khi nói về sự phân bố thần kinh sọ của
lưỡi?
A. Cảm giác vị giác từ 2/3 trước của lưỡi theo dây thần kinh hạ thiệt truyền đến
nhân đơn độc
B. Cảm giác vị giác từ vùng hầu theo dây thần kinh mặt truyền đến nhân đơn
độc
C. Cảm giác vị giác từ 1/3 sau của lưỡi theo dây thần kinh hạ thiệt truyền đến
nhân đơn độc
D. Cảm giác vị giác từ 1/3 sau của lưỡi ban đầu đi qua dây lưỡi
E. Cảm giác vị giác từ 1/3 sau của lưỡi ban đầu đi qua thừng nhĩ
 C. Các xung vị giác từ 2/3 trước của lưỡi đầu tiên đi qua dây thần kinh lưỡi, rồi
qua thừng nhĩ vào dây thần kinh mặt, và cuối cùng đến nhân đơn độc. Cảm giác vị
giác từ 1/3 sau lưỡi được truyền qua dây thần kinh hạ thiệt vào nhân đơn độc. Các
tín hiệu vị giác từ vùng hầu được dẫn truyền qua các dây thần kinh số X.
TMP12 647–648
30.Tế bào nhận cảm khứu giác thuộc nhóm tế bào nào sau đây?
A. Nơron 2 cực
B. Các nguyên bào sợi
C. Tế bào biểu mô biến đổi
D. Nơron đa cực
E. Nơron đơn cực
 A. Các tế bào nhận cảm cho cảm giác vị giác là tế bào hai cực xuất phát từ hệ
thần kinh trung ương của chính nó.
TMP12 649
31.Khẳng định nào sau đây là đúng khi mô tả tế bào lông?
A. Các tế bào lông khử cực khi sợi lông của chúng đổ rạp về hướng của sợi
lông ngắn nhất
B. Các sợi thần kinh bị kích thích bởi tế bào lông có thân tế bào nằm trong nhân
ốc tai ở thân não
C. Càng xa trục ốc tai (trung tâm), các sợi lông của tế bào lông càng dài
D. Trong cơ quan Corti, số tế bào lông ở trong nhiều hơn số tế bào lông ở ngoài
E. Sự dẫn truyền các tín hiệu âm thanh được thực hiện chủ yếu nhờ các tế bào
lông ở trong hơn là các tế bào lông ở ngoài
 E. Mặc dù số lượng tế bào lông ở ngoài nhiều gấp 3 đến 4 lần số tế bào lông ở
trong, khoảng 90% sợi thần kinh thính giác bị kích thích bởi các tế bào lông ở
trong.
TMP12 637
32.Sự thích nghi để nhìn xa (tập trung vào một vật ở khoảng cách xa mắt) yêu
cầu quá trình nào sau đây?
A. Sự co của đồng tử mắt
B. Sự giãn của đồng tử mắt
C. Sự tăng tạo rhodopsin
D. Quá trình làm nhân mắt cong hơn (to và dày hơn)
E. Quá trình làm nhân mắt bớt cong hơn (mỏng hơn)
 E. Các tia sáng xuất phát từ một vật ở xa mắt không yêu cầu sự khúc xạ nhiều
(sự uốn cong) như các tia sáng xuất phát từ một vật gần trong phạm vi có thể với
tới được. Do vậy, sự uốn cong của hệ thấu kính cần cho sự hội tụ các tia sáng này
trên võng mạc cũng nhỏ hơn.
TMP12 601
33.Hiện tượng nào sau đây xảy ra với các tế bào nhận cảm ánh sáng, nhằm đáp
ứng với ánh sáng trong quá dẫn truyền hình ảnh?
A. Giảm hoạt động của enzym phosphodiesterase
B. Giảm hoạt động của transducin
C. Tăng hoạt động thuỷ phân cGMP
D. Tăng giải phóng chất dẫn truyền thần kinh
E. Tăng số lượng kênh Calci (kênh dẫn truyền theo điện thế)
C. Trong bóng tối, cGMP giúp đảm bảo sự mở các kênh Natri trên
màng đoạn ngoài của tế bào que. Sự thuỷ phân cGMP bởi ánh sáng làm
các kênh Natri này đóng lại. Ít Natri có khả năng đi vào đoạn ngoài của tế
bào que hơn, do vậy xảy ra quá trình ưu phân cực ở tế bào que
34.Trong quá trình nhận cảm ánh sáng, tất cả các chất sau đây đều tăng, TRỪ:
A. cGMP phosphodiesterase

B. Transducin

C. cAMP
D. Metarhodopsin II
E. Dòng Natri đi vào đoạn ngoài của tế bào que
 E. Trong quá trình nhận cảm ánh sáng, hợp chất metarhodpsin hoạt hoá được
hình thành. Chất này hoạt hoá một protein có tên là transducin. Transducin hoạt
hoá một cGMP phosphodiesterase làm phân huỷ cGMP. Các kênh Natri phụ thuộc
cGMP đóng và dòng ion Natri đi vào trong đoạn ngoài của các tế bào nhận cảm
ánh sáng cũng giảm theo.
TMP12 612–614
35.Khẳng định nào sau đây là đúng khi nói về tật loạn thị?
A. Các tia sáng không cùng rơi vào một tiêu điểm chung
B. Các tia sáng xuất phát từ những vật ở xa mắt được hội tụ sau võng mạc
C. Các tia sáng xuất phát từ những vật ở xa mắt được hội tụ trước võng mạc
D. Các tia sáng xuất phát từ những vật ở xa mắt được hội tụ ngay trên võng
mạc
E. Có một hoặc nhiều vùng đục hoặc mờ trên hệ thấu kính
 A. Tật loạn thị thường gặp nhất khi độ cong giác mạc không đồng đều làm cho
độ hội tụ của thấu kính không đồng đều theo các trục. Bởi vì độ cong của thấu kính
loạn thị theo một trục nhỏ hơn độ cong của thấu kính theo một trục khác, các tia
sáng đi theo 2 trục sẽ bị bẻ cong ở các mức độ khác nhau. Do vậy, các tia sáng đi
qua một thấu kính loạn thị không cùng rơi vào một tiêu điểm chung.
TMP12 603
36.Các sợi lông của tế bào lông được gắn vào màng nào?
A. Màng đáy
B. Màng Reissner
C. Màng mái
D. Màng nhĩ
E. Màng tiền đình
 C. Thang giữa được bao bọc bởi màng đáy và màng Reissner, và có chứa màng
mái. Phần đỉnh của các tế bào lông có các sợi lông được gắn vào màng mái.
TMP12 637
37.Các dây thần kinh sọ nào sau đây được ghép cặp chính xác với cơ nhãn cầu
mà nó chi phối?
A. Dây thần kinh số VI – cơ thẳng giữa
B. Dây thần kinh số III – cơ chéo dưới
C. Dây thần kinh số III – cơ thẳng bên
D. Dây thần kinh số III – cơ chéo trên
E. Dây thần kinh số IV – cơ thẳng trên
 B. Dây thần kinh số VI chi phối cho cơ thẳng bên. Dây thần kinh số IV chi phối
cho cơ chéo trên. Dây thần kinh số III chi phối cho cơ thẳng giữa, chéo dưới, thẳng
trên, và thẳng dưới.
TMP12 637
38.Sau khi các tế bào nhận cảm khứu giác gắn vào các phân tử mang mùi, một
chuỗi các phản ứng xảy ra trong tế bào, làm mở các kênh ion chuyên biệt
gây khử cực tế bào khứu giác. Các ion nào tham gia vào quá trình trên?
A. Ion Calci
B. Ion Clo
C. Ion Hydro
D. Ion Kali
E. Ion Natri
 E. Ngay cả nồng độ nhỏ nhất của một chất thơm cũng có thể khởi phát một
chuỗi các phản ứng (hiệu ứng phân tầng) làm mở một số lượng lớn các kênh Natri.
Điều này giải thích cho sự nhạy cảm tuyệt vời của thần kinh khứu giác với ngay cả
lượng nhỏ nhất của chất thơm.
TMP12 650
39.Khi mắt đáp ứng với ánh sáng cường độ lớn, hiện tượng nào sau đây có thể
xảy ra?
A. Các tế bào hai cực sẽ liên tục truyền tín hiệu với tốc độ lớn nhất có thể
B. Các chất nhạy cảm với ánh sáng ở cả tế bào que và tế bào nón sẽ chuyển
thành retinal và các opsin
C. Nồng độ rhodopsin sẽ rất cao
D. Sẽ có sự tăng kích thước đồng tử
E. Vitamin A sẽ chuyển thành retinal
 B. Sự giảm số lượng rhodopsin và sắc tố nón bởi ánh sáng làm giảm nồng độ
các chất nhạy cảm với ánh sáng ở tế bào que và tế bào nón. Do vậy, sự nhạy cảm
của mắt với ánh sáng sẽ giảm tương ứng. Quá trình này được gọi là sự thích ứng
ánh sáng.
TMP12 614–615
40.Tật cận thị thường được chữa bằng cách đeo loại thấu kính nào sau đây?
A. Thấu kính phức hợp
B. Thấu kính hội tụ
C. Thấu kính cầu
D. Thấu kính phân kỳ
E. Thấu kính trụ
 D. Trong tật cật thị, tiêu điểm của hệ thấu kính mắt nằm trước võng mạc. Một
thấu kính phân kì sẽ giúp phân tán các tia sáng. Bằng cách đặt một thấu kính phân
kì phù hợp ở trước mắt, sự phân tán của các tia sáng sẽ chuyển tiêu điểm từ vị trí
trước võng mạc đến một vị trí trên võng mạc.
TMP12 603
41.Đây là một vùng diện tích nhỏ nằm ở cả 2 bên vỏ não, chi phối cho các vận
động tự chủ cố định. Vùng này nằm trên thuỳ nào của vỏ não?
A. Thuỳ trán
B. Thuỳ viền
C. Thuỳ chẩm
D. Thuỳ đỉnh
E. Thuỳ thái dương
 A. Vùng tiền vận động của 2 bên thuỳ trán vỏ não chi phối vận động cố định tự
chủ. Tổn thương vùng này sẽ gây khó khăn trong việc đưa mắt ra khỏi một điểm cố
định và di chuyển đến điểm khác.
TMP12 628–629
42.Hệ giác quan nào sau đây có dải nhận cảm hẹp nhất?
A. Thính giác
B. Vị giác
C. Khứu giác
D. Cảm giác bản thể
E. Thị giác
 C. Nồng độ chất gây mùi chỉ cần cao hơn 10 đến 50 lần ngưỡng gây mùi sẽ gây
ra cảm giác mùi tối đa, điều này trái ngược với hầu hết các hệ giác quan của cơ thể,
nơi mà dải nhận cảm từ 1 đến 1 tỷ (ví dụ thị giác có dải từ 1 đến 50000, thính giác
có dải từ 1 đến 1 tỷ). Điều này có thể được giải thích do khứu giác quan tâm tới
việc dò tìm sự có mặt hay vắng mặt của phân tử mùi hơn là việc dò tìm sự khác
biệt của cường độ phân tử mùi đó.
TMP12 650
43.Các phân tử nào sau đây đi từ chất nội bạch huyết vào sợi lông của tế bào
lông và gây khử cực tế bào lông?
A. Ion Calci
B. Ion Clo
C. Ion Hydro
D. Ion Kali
E. Ion Natri
 D. Mặc dù hầu hết các tế bào trong hệ thần kinh trung ương khử cực qua kênh
Natri, các tế bào lông là một nhóm tế bào khử cực qua kênh Kali.

TMP12 637
44.Thay đổi nào sau đây giúp hệ thính giác nhận biết được tiếng ồn?
A. Giảm số lượng tế bào lông ở trong bị kích thích
B. Giảm số lượng tế bào lông ở ngoài bị kích thích
C. Các tế bào lông kích thích các tận cùng thần kinh với tốc độ chậm hơn
D. Giảm biên độ dao động của màng đáy
E. Tăng biên độ dao động của màng đáy
 E. Có ít nhất 3 cách cho hệ khứu giác để xác định tiếng ồn. Thứ nhất, biên độ
dao động của màng nhĩ tăng để các tế bào lông kích thích các tận cùng thần kinh
với tốc độ nhanh hơn. Thứ hai, càng ngày càng nhiều tế bào lông trên rìa của bộ
phận cộng hưởng của màng nhĩ bị kích thích. Thứ ba, các tế bào lông ở ngoài bị
kích thích với tốc độ đáng kể.
TMP12 636, 638
45.Khẳng định nào sau đây là đúng khi nói về các thành phần của võng mạc?
A. Tổng số lượng các tế bào nón trong võng mạc nhiều hơn tổng số lượng tế
bào que
B. Mỗi tế bào nón đều có khả năng đáp ứng với tất cả bước sóng ánh sáng
C. Sự hoạt hoá các tế bào nhận cảm ánh sáng (tế bào que và tế bào nón) dẫn
đến sự ưu phân cực của các tế bào nhận cảm đó
D. Điểm trung tâm (điểm vàng) chỉ chứa tế bào que
E. Lớp sắc tố của võng mạc chứa các tế bào nhận cảm ánh sáng
 C. Không giống như hầu hết các receptor cảm giác khác (khử cực khi bị hoạt
hoá), các tế bào nhận cảm ánh sáng khi bị hoạt hoá sẽ dẫn đến sự ưu phân cực.
Tổng số lượng tế bào que nhiều hơn số lượng tế bào nón. Các tế bào nón tương
ứng với một dải bước sóng ánh sáng rất riêng biệt. Lớp sắc tố nằm sau lớp võng
mạc nơi các tế bào nhận cảm ánh sáng.
TMP12 612
46.Dị tật nào sau đây của mắt thường gây ra tật viễn thị?
A. Giảm sản xuất melanin
B. Độ cong của giác mạc không đồng đều
C. Nhãn cầu ngắn hơn bình thường
D. Nhãn cầu dài hơn bình thường
E. Hệ thấu kính của mắt quá mạnh (độ hội tụ của nhân mắt tăng) và tập trung
ảnh của vật ở trước võng mạc
 D. Trong tật viễn thị, tiêu điểm của hệ thấu kính mắt nằm sau võng mạc. Điều
này thường là kết quả của việc nhãn cầu quá ngắn theo trục trước ra sau.
TMP12 602
47.Câu nào sau đây mô tả chính xác thứ tự các chặng trong đường dẫn truyền
thính giác từ tai tới hệ thần kinh trung ương?
A. Nhân ốc tai – nhân trám trên – qua bó lemnicus bên để tận cùng ở collius
dưới (củ trung não dưới) – nhân gối giữa – vùng nghe trên vỏ não
B. Nhân ốc tai – nhân trám dưới – qua bó lemnicus giữa để tận cùng ở collius
dưới (củ trung não dưới) – nhân gối giữa – vùng nghe trên vỏ não
C. Nhân ốc tai – nhân trám trên – qua bó lemnicus bên để tận cùng ở collius
trên (củ trung não trên) – nhân gối bên – vùng nghe trên vỏ não
D. Nhân ốc tai – nhân trám dưới – qua bó lemnicus bên để tận cùng ở collius
dưới (củ trung não dưới) – nhân gối bên – vùng nghe trên vỏ não
E. Nhân ốc tai – thể hình thang – vân thính lưng – qua nhân gối giữa để tận
cùng ở collius dưới (củ trung não dưới) – vùng nghe trên vỏ não
 A. Các sợi thính giác đi vào nhân ốc tai. Các sợi từ nhân ốc tai qua bó lemnicus
bên để tận cùng ở collius dưới. Các sợi từ collius dưới qua nhân gối giữa và từ đây
đi đến vùng nghe sơ cấp trên vỏ não.
TMP12 639
48.Khẳng định nào sau đây là đúng khi nói về sự dẫn truyền các tín hiệu thính
giác từ tai tới vỏ não?
A. Các nơron của collius dưới liên hợp thần kinh trong nhân ốc tai của thân não
B. Các nơron với thân nơron nằm trong hạch xoắn Corti liên hợp thần kinh
trong collius dưới
C. Đa số các nơron từ nhân ốc tai liên hợp thần kinh trong nhân trám trên đối
bên
D. Không có các sợi bắt chéo giữa đường dẫn truyền thính giác bên phải và bên
trái tại thân não
E. Các nơron của thể hình thang liên hợp thần kinh trong nhân ốc tai của thân
não
 C. Các tế bào thần kinh với thân nằm trong hạch xoắn Corti liên hợp thần kinh
trong nhân ốc tai. Đa số các nơron của nhân ốc tai liên hợp thần kinh với các nhân
trám đối bên. Các sợi bắt chéo tại ít nhất 3 vị trí trên đường dẫn truyền thính giác,
và đóng vai trò dẫn truyền chủ yếu ở bên đối diện. Từ các nhân trám trên, đường
dẫn truyền thính giác đi qua bó lemnicus bên để tận cùng ở collius dưới. Từ đây,
đường dẫn truyền tiếp tục đến các nhân gối giữa, rồi đi đến vùng nghe sơ cấp của
vỏ não.
TMP12 639
49.Khẳng định nào sau đây là đúng khi nói về cơ chế nhìn màu?
A. Màu lục được nhìn thấy khi chỉ các tế bào nón nhạy cảm với màu lục bị kích
thích
B. Tỷ lệ kích thích của 3 loại tế bào nón cho phép nhìn thấy các màu khác nhau
C. Bước sóng ánh sáng tương ứng với màu trắng ngắn hơn bước sóng ánh sáng
tương ứng với màu lục
D. Khi không có sự kích thích các tế bào nón nhạy cảm với màu đỏ, lục hay
lam, sẽ cho cảm giác màu trắng
E. Màu vàng được nhìn thấy khi các tế bào nón nhạy cảm với màu vàng và màu
lam bị kích thích với tỷ lệ như nhau
 B. Các nghiên cứu đã chỉ ra rằng hệ thần kinh trung ương nhận cảm giác màu
riêng biệt bởi tỉ lệ kích thích của 3 loại tế bào nón. Các nhà nghiên cứu chỉ sử dụng
3 ánh sáng đơn sắc đỏ, lục, và lam; 3 màu này được phối hợp thành các màu khác
nhau. Tất cả những phân độ màu sắc mà mắt người có thể nhìn thấy được đều chỉ
sử dụng 3 màu này.
TMP12 615–616

50.Khẳng định nào sau đây mô tả chính xác nhất chức năng của cửa sổ tròn?
A. Tạo vị trí kết nối với xương bàn đạp
B. Giúp tránh các âm thanh có tần số thấp như giọng nói của chính bạn
C. Truyền các thông tin về tần số từ màng nhĩ vào trong ốc tai
D. Có chức năng như một van giảm áp cho ốc tai
E. Truyền các thông tin về biên độ từ màng nhĩ vào trong ốc tai
 D. Ốc tai là một cấu trúc ống và xoang được lấp đầy bởi dịch. Dịch này không
nén được. Khi xương bàn đạp đẩy vào trong cửa sổ bầu dục, sự tăng và giảm áp
suất gây ra bởi sự di chuyển vào trong và ra ngoài của cửa sổ bầu dục làm cửa sổ
tròn di chuyển theo hướng ngược lại.
TMP12 635
51.Sự co cơ nào sau đây góp phần gây ra phản xạ của đồng tử với ánh sáng?
A. Cơ thể mi
B. Cơ giãn đồng tử
C. Cơ co đồng tử
D. Các sợi tia (cơ tia) của mống mắt
E. Cơ chéo trên
 C. Trong một người bình thường, khi chiếu ánh sáng vào một mắt, cơ co đồng
tử co sẽ dẫn tới sự co đồng tử ở cả 2 mắt. Ngược lại, cơ giãn đồng tử làm giãn
đồng tử. Cơ thể mi liên quan đến độ hội tụ của mắt (khả năng điều tiết).
TMP12 632
52.Cơ chế nào sau đây cho phép cơ quan thị giác xác định chính xác khoảng
cách từ một vật đến mắt (nhận thức chiều sâu)?
A. Cơ chế nhìn bằng một mắt
B. Vị trí của hình ảnh võng mạc trên võng mạc
C. Hiệu ứng parallax tĩnh
D. Hiệu ứng ảo giác lập thể
E. Kính thước của hình ảnh võng mạc nếu vật không có kích thước xác định
 D. Do hai mắt cách nhau một khoảng hơn 2 inches nên hình ảnh trên 2 võng
mạc cũng khác nhau. Hiệu ứng ảo giác lập thể này cho phép cho một người có 2
đánh giá khoảng cách tương đối từ một vật ở gần tốt hơn một người chỉ có một
mắt.
TMP12 605
53.Nguyên nhân nào sau đây hay gây ra chứng thiên đầu thống?
A. Các thuốc làm giảm sự bài tiết của thuỷ dịch
B. Tăng sức cản với dòng chảy của thuỷ dịch từ bên ngoài qua các khoảng bè
(trabecular) vào trong ống Schlemm
C. Chức năng bình thường của các đại thực bào trên bề mặt bè
D. Sự thực bào các protein và các mảnh vụn của biểu mô mống mắt
E. Sự hoạt hoá các tế bào võng nội mô trong dịch kẽ ra ngoài ống Schlemm
 B. Bệnh thiên đầu thống là một bệnh của mắt trong đó nhãn áp tăng cao một
cách bệnh lý, đôi khi tăng cấp tính lên 60 đến 70 mmHg. Áp lực trên 30 mmHg
trong một thời gian dài có thể dẫn đến mất thị lực. Nguyên nhân thường gặp nhất
của hiện tượng tăng nhãn áp này là do dịch từ bên ngoài đi vào ống schlemm bị
cản trở.
TMP12 607–608
54.Khẳng định nào sau đây là đúng khi nói về hai loại điếc?
A. Thính đồ của một người điếc dẫn truyền sẽ cho thấy sự mất dẫn truyền âm
thanh trong môi trường khí nhiều hơn trong môi trường xương
B. Thính đồ của một người điếc thần kinh sẽ cho thấy sự mất dẫn truyền âm
thanh trong môi trường xương nhiều hơn trong môi trường khí
C. Điếc dẫn truyền xảy ra khi ốc tai hoặc dây thần kinh ốc tai bị tổn thương
D. Điếc thần kinh xảy ra khi các cấu trúc vật lý có vai trò trong dẫn truyền âm
thanh vào ốc tai bị tổn thương
E. Sự tiếp xúc lâu dài với các âm thanh rất lớn thường gây điếc với các âm
thanh có tần số cao hơn là các âm thanh có tần số thấp
 A. Trong điếc thần kinh, có sự tổn thương dây ốc tai, dây thần kinh thính giác,
hoặc đường dẫn truyền thần kinh. Khả năng nghe các âm thanh (được kiểm tra
trong cả môi trường khí và môi trường xương) sẽ giảm đáng kể hoặc mất hoàn toàn
trong điếc thần kinh. Tuy nhiên, trong điếc dẫn truyền, một người vẫn có khả năng
nghe âm thanh được dẫn truyền trong môi trường xương, nhưng không nghe được
trong âm thanh dẫn truyền trong môi trường khí.
TMP12 642
55.Tưởng tượng khi một người đang quay đầu về bên trái trục của cơ thể. Cử
động bắt đầu khi cằm nằm trực tiếp trên vai phải và kết thúc khi cằm nằm
trực tiếp trên vai trái. Khẳng định nào sau đây là đúng khi mô tả sự di
chuyển của mắt với kiểu quay đầu này trên một người bình thường?
A. Trong khi đầu quay, hai mắt sẽ di chuyển sang phải và sự rung giật nhãn cầu
sẽ hướng sang trái
B. Trong khi đầu quay, hai mắt sẽ di chuyển cùng với hướng đầu xoay và sự
rung giật nhãn cầu sẽ hướng sang trái
C. Trong khi đầu quay, hai mắt sẽ di chuyển sang phải và sự rung giật nhãn cầu
sẽ hướng về bên phải
D. Trong khi đầu quay, hai mắt sẽ giữ nguyên vị trí trong quỹ đạo và sự rung
giật nhãn cầu sẽ hướng về bên phải
E. Trong khi đầu quy, hai mắt sẽ di chuyển sang trái và sự rung giật nhãn cầu
sẽ hướng sang phải
 A. Trong tình huống kể trên, hai mắt sẽ cố định vào vật trong thị trường và tiếp
tục cố định vào vật đó trong quá trình đầu quay sang trái. Điều này dẫn tới kết quả
là mắt di chuyển sang phải khi đầu quay sang trái. Khi vật không còn trong thị
trường bình thường của 2 mắt nữa thì hai mắt sẽ liếc nhanh sang trái (theo hướng
đầu quay) và cố định vào vật ở vị trí mới trong thị trường. Sự liếc nhanh này được
gọi là một rung giật. Quá trình này sẽ lặp lại cho tới khi đầu hoàn toàn quay sang
trái. Trong quá trình nhìn, hiện tượng rung giật nhãn cầu sẽ bị ức chế.
TMP12 629–630
56.Hội chứng Horner xảy ra khi các sợi thần kinh giao cảm tới mắt bị gián
đoạn, dẫn đến các triệu chứng nào trên mặt của bên bị tổn thương?
A. Các mạch máu của mặt bên tổn thương bị co lại liên tục
B. Mồ hôi bài tiết nhiều
C. Mi mắt trên của bên tổn thương luôn ở trạng thái mở
D. Sự sản xuất quá mức nước mắt từ tuyến lệ
E. Xảy ra hiện tượng co đồng tử liên tục làm đường kính đồng tử bên tổn
thương nhỏ hơn bên đối diện
 E. Hội chứng Horner điển hình xảy ra khi các sợi thần kinh giao cảm xuất phát
từ tuỷ sống đoạn ngực lên mắt bị gián đoạn trong chuỗi hạch giao cảm cổ.
TMP12 632
57.Chất dẫn truyền thần kinh nào sau đây được giải phóng bởi cả tế bào que và
tế bào nón tại vị trí chúng liên hợp thần kinh với các tế bào 2 cực?
A. Acetylcholine
B. Dopamine
C. Glutamate
D. Glycine
E. Serotonin
 C. Có ít nhất 6 loại chất dẫn truyền thần kinh được xác định trong các tế bào
amacrine. Các chất dẫn truyền thần kinh được sử dụng cho tế bào hai cực và tế bào
ngang vẫn còn chưa sáng tỏ, tuy nhiên, đã có những cơ sở vững chắc cho việc tế
bào nón và tế bào que giải phóng glutamte tại vị trí chúng tiếp xúc synap với tế bào
2 cực (hình minh hoạ sau đây).
TMP12 617
58.Thông tin thính giác được truyền đến vỏ não trán ổ mắt qua nhân đồi thị nào
sau đây?
A. Lưng giữa
B. Gối bên
C. Gối giữa
D. Bụng sau bên
E. Bụng giữa bên
 A. Một đường dẫn truyền thính giác mới hơn đã được tìm ra. Đường dẫn truyền
này đi qua các nhân lưng giữa của đồi thị và lên vùng vỏ não trán ổ mắt. Tuy
nhiên, đường dẫn truyền thính giác cũ không đi qua đồi thị mà lên thẳng vỏ não,
ngược lại với các hệ giác quan khác đều có trạm dừng ở đồi thị.
TMP12 651
59.Cấu trúc nào sau đây tạo nên 2:3 công suất khúc xạ của mắt (59 diop)?
A. Mặt trước của giác mạc
B. Mặt trước của thuỷ tinh thể
C. Mống mắt
D. Mặt sau của giác mạc
E. Mặt sau của thuỷ tinh thể
 A. Nguyên lý cơ bản giải thích hiện tượng mặt trước giác mạc tạo nên phần lớn
công suất khúc xạ của mắt là do chỉ số khúc xạ của giác mạc khác biệt đáng kể so
với chỉ số khúc xạ của không khí.
TMP12 600
60.Sự dẫn truyền các tín hiệu thị giác từ võng mạc đến vùng nhìn sơ cấp của võ
não bao gồm quá trình liên hợp thần kinh xảy ra tại cấu trúc nào sau đây?
A. Nhân gối bên
B. Nhân gối giữa
C. Nhân trước mái
D. Collius trên (củ trung não trên)
E. Nhân trên giao thoa thị giác (nhân suprachiasmatic)
 A. Các tế bào hạch của võng mạch liên hợp thần kinh ở một vài vị trí, nhưng tất
cả sự dẫn truyền tín hiệu thần kinh tại đó đều liên hợp thần kinh tại nhân gối bên và
tận cùng tại vùng thị giác sơ cấp trên vỏ não.
TMP12 623–624
61.Khẳng định nào sau đây là đúng khi nói về các tế bào hạch của võng mạc?
A. Một tế bào hạch loại W từ vùng ngoại vi võng mạc thường truyền tín hiệu
cho một tế bào nón
B. Một tế bào hạch loại X từ vùng trung tâm võng mạc thường truyền tín hiệu
cho 200 tế bào nón
C. Các tế bào hạch loại W đáp ứng tốt nhất với sự chuyển động hoặc nhìn theo
hướng trong điều kiện rất sáng
D. Các tế bào hạch loại X đáp ứng tốt nhất với các hình ảnh màu và chiếm số
lượng lớn nhất trong 3 loại tế bào hạch
E. Các tế bào hạch loại Y đáp ứng tốt nhất với sự thay đổi nhanh của hình ảnh
thị giác và chiếm số lượng lớn nhất trong 3 loại tế bào hạch
 D. Có ba loại tế bào hạch, được gọi là tế bào hạch loại W, X và Y. Các tế bào
hạch loại W dẫn truyền tín hiệu thị giác từ tế bào nón. Các tế bào hạch loại Y có số
lượng ít nhất và dẫn truyền tín hiệu về những thay đổi nhanh của hình ảnh thị giác.
Các tế bào X có số lượng nhiều nhất và nhận tín hiệu đi vào từ các tế bào nón liên
quan tới thị giác hình ảnh và cơ chế nhìn màu.
TMP12 619
62.Thông tin thính giác được tiếp nhận bởi nhân đồi thị nào sau đây?
A. Nhân lưng giữa
B. Nhân gối bên
C. Nhân gối giữa
D. Nhân bụng sau bên
E. Nhân bụng sau giữa
 C. Nhân gối giữa là nhân đồi thị có chức năng dẫn truyền các tín hiệu thần kinh
thính giác từ thân não lên vùng nghe sơ cấp ở võ não.
TMP12 639
63. Hiện tượng ưa thích một vị là?
A. Cơ chế phản xạ của hệ thống thần kinh trung ương
B. Kết quả của sự kính thích gai lưỡi hình vòm có từ lúc mới sinh
C. Một tập tính hình thành qua quá trình học tập
D. Kết quả của sự trưởng thành nụ vị giác
E. Kết quả của sự tăng sinh nụ vị giác khi tiếp xúc với acid glutamic
 A. Sự thích một vị giác, chưa hoàn toàn được hiểu rõ, nhưng được cho là có liên
quan đến cơ chế phản xạ của hệ thần kinh trung ương
TMP12 648
64. Các tế bào nhận cảm ánh sáng nào sau đây đáp ứng với quang phổ rộng
nhất của các bước sóng ánh sáng?
A. Các tế bào nón
B. Các tế bào nón nhạy cảm với màu lục
C. Các tế bào nón nhạy cảm với màu lam
D. Các tế bào nón nhạy cảm với màu đỏ
E. Các tế bào chứa melanin trong lớp sắc tố
 D. Theo trực giác, một người sẽ đoán rằng tế bào que có khoảng nhạy cảm ánh
sáng (quang phổ) rộng nhất. Tuy nhiên, tế bào que nhạy cảm với màu đỏ mới là tế
bào có quang phổ rộng nhất, sau đó là tế bào que nhạy cảm với màu lục, và cuối
cùng là tế bào que nhạy cảm với màu làm có quang phổ hẹp nhất.
TMP12 614
65.Cấu trúc nào sau đây có chức năng bài tiết thuỷ dịch trong mắt?
A. Các tế bào biểu mô của thể mi
B. Giác mạc
C. Mống mắt
D. Thuỷ tinh thể
E. Bè
 A. Các tế bào biểu mô của thể mi bài tiết tất cả thể dịch, với tốc độ trung bình 2
đến 3 μl/phút. Các tế bào này nằm trên các nếp thể mi (đi từ các cơ thể mi đến
phần sau mống mắt). Thuỷ dịch chảy ra từ sau mống mắt qua đồng tử vào tiền
phòng của mắt.
TMP12 606
66.Loại gai lưỡi nào nằm ở phần sau của lưỡi?
A. Gai lưỡi hình vòm
B. Gai lưỡi hình lá
C. Gai lưỡi hình nấm
D. Gai lưỡi hình nấm và hình vòm
E. Nhú vater
 A. Các gai lưỡi hình vòm nằm ở phần sau của lưỡi, gai lưỡi hình nấm nằm ở
phần trước của lưỡi, và gai lưỡi hình lá nằm dọc hai mặt bên của lưỡi. Núm vater
là vị trí dịch tuỵ và dịch mật đổ vào tá tráng.
TMP12 647
67.Cấu trúc nào sau đây có chức năng đảm bảo sự phân bố thần kinh đối lập
của 3 nhóm cơ nhãn cầu, sao cho khi một cơ trong cặp cơ giãn thì cơ còn lại
sẽ co?
A. Nhân Edinger-Westphal
B. Bó dọc giữa
C. Nhân trước mái
D. Collius trên (củ trung não trên)
E. Nhân trên giao thoa thị giác (nhân suprachiasmatic)
 B. Bó dọc giữa là con được dẫn truyền cho các sợi thần kinh đi vào và đi ra
nhân dây vận nhãn chung, nhân dây ròng dọc và nhân dây vận nhãn ngoài của thân
não. Điều này cho phép liên hệ và phối hợp hoạt động co giãn của nhiều cơ nhãn
cầu.
TMP12 628
68.Các tế bào võng mạc nào sau đây có điện thế hoạt động?
A. Các tế bào amacrin
B. Các tế bào 2 cực
C. Các tế bào hạch
D. Các tế bào ngang
E. Các tế bào nhận cảm ánh sáng
 C. Chỉ các tế bào hạch có điện thế hoạt động. Các tế bào nhận cảm ánh sáng, tế
bào hai cực, tế bào amacrine, và tế bào ngang đều có mặt để có tác dụng tạo ra điện
thế phân bậc (graded potentials)
TMP12 617
69.Loại gai lưỡi nào nằm dọc hai mặt bên của lưỡi?
A. Gai lưỡi hình vòm
B. Gai lưỡi hình lá
C. Gai lưỡi hình nấm
D. Gai lưỡi hình nấm và hình vòm
E. Nhú vater
 A. Các gai lưỡi hình lá nằm dọc hai mặt bên của lưỡi, gai lưỡi hình nấm nằm ở
phần trước của lưỡi, và gai lưỡi hình vòm nằm ở phần sau của lưỡi. Núm vater là
vị trí dịch tuỵ và dịch mật đổ vào tá tráng.
TMP12 647
70.Vùng nghe sơ cấp nằm ở thuỳ nào của vỏ não?
A. Thuỳ trán
B. Thuỳ viền
C. Thuỳ chẩm
D. Thuỳ đỉnh
E. Thuỳ thái dương
 E. Hầu hết vùng thính giác sơ cấp nằm trên thuỳ thái dương, nhưng vùng thính
giác liên hợp lại mở rộng sang vùng thuỳ đảo và thậm chí sang phần bên của thuỳ
đỉnh.
TMP12 640
71.Thuỷ dịch chảy từ ống Schlemm vào trong cấu trúc nào sau đây?
A. Tiền phòng
B. Tĩnh mạch ngoài nhãn cầu
C. Thuỷ tinh thể
D. Hậu phòng
E. Bè (trabeculae)
 B. Thuỷ dịch chảy từ tiền phòng của mắt, giữa giác mạc và mống mắt qua một
khoang của trabeculae vào trong ống Schlemm, các ống này sau đó sẽ đổ vào tĩnh
mạch ngoài nhãn cầu (xem hình).
TMP12 607
72.Sự liên hợp thần kinh đầu tiên của các nơron truyền cảm giác vị giác ngọt
xảy ra trong cấu trúc nào sau đây?
A. Nhân lưng (nhân cảm giác) của dây thần kinh phế vị
B. Nhân của bó đơn độc
C. Nhân của dây thần kinh khứu giác
D. Nhân của dây thần kinh hạ thiệt
E. Nhân của dây thần kinh mặt
 B. Tận cùng của các sợi vị giác nhận cảm cho tất cả cảm giác vị đều nằm trong
nhân bó đơn độc của tuỷ sống.
TMP12 647–648
73.Cấu trúc thân não đóng vai trò quan trọng trong việc định hướng một âm
thanh xuất phát từ?
A. Nhân ốc tai
B. Collius dưới (củ trung não dưới)
C. Lemnicus bên
D. Nhân trám trên
E. Thể hình thang
 D. Các nhân trám trên nhận tín hiệu thính giác từ cả 2 tai và bắt đầu quá trình
xác định hướng xuất phát của các âm thanh. Phần bên của nhân trám trên thực hiện
chức năng này bằng cách so sánh sự khác biệt trong cường độ âm đến 2 tai, trong
khi phần giữa của nhân trám trên xác định sự trễ thời gian giữa các tín hiệu khi đi
vào 2 tai
TMP12 641
74.Sự tương phản của thị giác được tăng cường nhờ quá trình ức chế bên của
các tế bào nào?
A. Các tế bào amacrin
B. Các tế bào 2 cực
C. Các tế bào hạch
D. Các tế bào ngang
E. Các tế bào nhận cảm ánh sáng
 B. Tế bào amacrine có nhiều loại và ít nhất 6 loại chức năng. Trong một nhận
cảm thị giác, các tế bào amacrine bắt đầu phân tích các tín hiệu thị giác trước khi
chúng rời khỏi võng mạc. Tuy nhiên, các tế bào ngang (các tế bào luôn luôn có
tính ức chế) có tác dụng chuyển tiếp tín hiệu bên giữa các tế bào nhận cảm ánh
sáng và tế bào hai cực. Sự chuyển tiếp tín hiệu bên gây ra hiện tượng ức chế giống
nhau ở cả hai bên (ức chế bên), đóng vai trò quan trọng trong tất cả các hệ giác
quan, giúp đảm bảo sự tương phản của thị giác.
TMP12 618
75.Khẳng định nào sau đây là đúng khi nói về cơ sở của dòng tối trong đoạn
ngoài của các tế bào nhận cảm ánh sáng?
A. Dòng tối là kết quả của dòng ion Natri đi vào qua các kênh Natri phụ thuộc
cAMP
B. Dòng tối là kết quả của dòng ion Natri đi vào qua các kênh Natri phụ thuộc
cGMP
C. Dòng tối là kết quả của dòng ion Kali đi ra qua các kênh Kali phụ thuộc
cGMP
D. Dòng tối là kết quả của dòng ion Natri đi ra qua các kênh Natri phụ thuộc
cGMP
E. Dòng tối là kết quả của dòng ion Natri đi ra qua các kênh Natri phụ thuộc
cAMP
 B. Kênh Natri phụ thuộc cGMP trong đoạn ngoài của tế bào que và tế bào nón
cho phép ion Natri đi từ ngoài vào trong các tế bào nhận cảm ánh sáng. Hiện tượng
này làm phát sinh một điện thế hoạt động có giá trị thấp hơn điện thế nghỉ của
màng ở một nơron điển hình. Sự di chuyển của ion Natri và sự thay đổi điện thế
sau đấy là kết quả của sự tăng tính thấm của màng, hay còn gọi là dòng tối
TMP12 612

Unit 11: TK vận động

1.Vỏ não mới (neocortex, phylogenetically new cerebral cortex) gồm 6 lớp tế bào
nằm tiếp tuyến với màng mềm của bán cầu đại não. Khẳng định nào sau đây liên
quan tới sắp xếp của 6 lớp tế bào này là đúng:
A) Các neuron ở lớp I, II và III thực hiện hầu hết các liên hệ đồi thị-vỏ não
trong cùng 1 bán cầu
B) Các neuron trong lớp II và III tạo liên hệ với các nhân ở nền não
C) Các tín hiệu đặc hiệu đến từ tiểu não tận cùng chủ yếu ở lớp IV
D) Các neuron ở lớp V có sợi trục kéo dài từ lớp V tới các vùng dưới vỏ và tuỷ
sống.
E) Các neuron ở lớp VI cho sợi trục đi tới hồi hải mã.
 D) Các neuron ở lớp V đi vào chất trắng dưới vỏ, rồi từ đây chúng lan ra rất
nhiều vị trí khác nhau dưới vỏ, bao gồm: các nhân ở nền não, thân não và tuỷ sống.
Các tế bào ở lớp VI đi vào đồi thị. Các tế bào ở lớp I, II và III tạo các loại khác
nhau của liên hệ nội vỏ, và các tế bào ở lớp IV nhận các sợi từ đường liên hệ đồi
thị-vỏ não
TMP12 697-698
2. Khi rời tuỷ sống đi tới các cơ vân ở ngoai vi, sợi trục của các neuron vận động
phải đi qua các cấu trúc nào sau đây:
A) Cột sau tuỷ sống
B) Rễ sau tuỷ sống
C) Mép trắng trước
D) Sừng sau tuỷ sống
E) Sừng trước tuỷ sống.
 E) Sợi trục của các neuron vận động ở sừng trước tuỷ sống rời tuỷ sống qua rễ
trước. Rễ sau là đường vào của các sợi cảm giác đi vào sừng sau của tuỷ sống. Cột
sau và mép trắng trước gồm các bó sợi chỉ nằm trong tuỷ sống.
TMP12 655-656
3. Loại neuron nào trong các loại sau có sợi trục tạo synapse với các tế bào cơ vân
(sợi ngoại suốt), là thành phần tạo nên phần lớn sợi cơ?
A) Neuron vận động alpha
B) Các tế bào tháp
C) Neuron vận động gamma
D) Các tế bào hạt
E) Các tế bào Purkinje
 A) Neuron vận động alpha tạo synapse trực tiếp với sợi ngoại suốt, trong khi đó
neuron vận động gamma tạo synapse với sợi nội suốt. Các tế bào tháp, tế bào hạt
và tế bào Purkinje nằm ở hệ thần kinh trung ương và không có liên hệ trực tiếp với
cơ vân.
TMP12 656
4. Các sợi đi lên từ vùng kích thích của hệ lưới hoạt hoá đến các nhân gian tấm của
đồi thị (intralaminar nuclei of the thalamus), và từ đây chúng phân bố tới các vị trí
nào trong các vị trí sau:
A) Các nhân cảm giác thân thể của đồi thị.
B) Các vùng khác nhau của vỏ não
C) Các nhân vận động của đồi thị
D) Chủ yêu đi tới hồi trước trung tâm.
E) Chủ yếu đi tới hồi sau trung tâm
 B) Một đặc điểm của hệ lưới hoạt hoá là nó tạo ra sự hoạt hoá rộng khắp các
vùng của vỏ não. Có được đặc điểm này là nhờ các nhân gian tấm của đồi thị tạo
liên kết lan toả tơi các vùng khác nhau của vỏ não.
TMP12 711
5. Khẳng định nào sau đây về chức năng của tiểu não là đúng?
A) Tiểu não điều khiển trực tiếp các neuron vận động cần thiết để tạo ra động tác
B) Tiểu não không thể điều chỉnh tính chính xác của động tác một khi nó được
thực hiện
C) Tiểu não không nhân feedback từ các cơ thực hiện động tác thực sự.
D) Tiểu não không liên quan trong việc dự kiến động tác mà chỉ thực hiện nó.
E) Tiểu não đóng vai trò tích cực trong việc phối hợp các cơ cần thiết để tạo nên
động tác.
 E) Hoạt động của tiểu não khác biệt khá nhiều so với các neuron vận động và sự
kích thích cơ, đặc biệt tại các chi và các cơ cạnh trục. Mặc dù, hoạt động của tiểu
não có ảnh hưởng rõ ràng tới hoạt động cơ, nhưng đó là ảnh hưởng một cách gián
tiếp. Các nhân ở tiểu não cho sợi trục tới các nhân vận động của đồi thị và nhân đỏ,
do đó, nó có thể tác động lên hoạt động của dải vỏ-tuỷ và dải đỏ-tuỷ.
TMP12 681
6. Đoạn tuỷ sống nào sau đây chứa toàn bộ neuron giao cảm trước hạch?
A) C5-T1
B) C3-C5
C) S2-S4
D) T1-L2
E) T6-L1
D) Tất cả neuron giao cảm trước hạch đều nằm ở cột trung gian bên (sừng bên); và
nhóm các tế bào này đi từ T1 đến L2
TMP12 730
7. Khẳng định nào sau đây mô tả đúng nhất chức năng của bán cầu bên tiểu não?
A) Điều khiển và phối hợp động tác của các cơ quanh trục (đầu và thân mình) cũng
như vai và hông.
B) Điều khiển động tác liên quan tới các cơ đầu chi
C) Phối hợp cùng với vỏ đại não dự kiến động tác.
D) Điều hoà neuron vận động thông qua liên hệ với tuỷ sống.
 C) Bán cầu bên tiểu não phối hợp cùng vỏ đại não trong việc dự kiến các động
tác phức tạp
TMP12 682
8. Yếu tố nào sau đây làm tăng tuần hoàn máu não?
A) Tăng nồng độ CO2
B) Tăng nồng độ O2
C) Giảm hoạt động của neuron vỏ đại não
D) Giảm nồng độ CO2
E) Giảm áp lực máu động mạch từ 120 mmHg xuống 90 mmHg
 A) Yếu tố kích thích mạnh mẽ nhất lên tuần hoàn máu não là sự tăng nồng độ
CO2 cục bộ, tiếp theo sau là sự giảm nồng độ O2 và sự tăng hoạt động neuron địa
phương.
TMP12 744
9. Vùng vận động sơ cấp tương ứng với vùng số mấy của Brodmann?
A) 6
B) 5
C) 4
D) 3
E) 1
 C) Vùng vận động sơ cấp tương ứng với vùng 4 của Brodmann và nằm trong
hồi trước trung tâm. Vùng 6 là vùng tiền vận động, vùng 5 là một phần của thuỳ
đỉnh trên, và vùng 3 và 1 tạo nên vùng cảm giác thân thể sơ cấp nằm trong hồi sau
trung tâm.
TMP12 667
10. Phần nào của cơ thế được biểu thị chủ yếu ở vùng bên và vùng dưới của vỏ não
vận động sơ cấp?
A) Mặt
B) Tay
C) Cổ
D) Bụng
E) Chi dưới
 A) Vùng mặt của vỏ não vận động nằm chủ yếu ở vùng dưới và vùng bên, được
cấp máu bởi ĐM não giữa, trong khi đó chi dưới nằm ở tiểu thuỳ cạnh trung tâm,
được cấp máu bởi ĐM não trước
TMP12 668
11. Neuron nào trong các loai sau có sợi trục tạo synapse với tế bào cơ vân (sợi nội
suốt) nằm trong thoi cơ?
A) Neuron vận động alpha
B) Các tế bào tháp
C) Neuron vận động gamma
D) Các tế bào hạt
E) Các tế bào Purkinje
 C) Neuron vận động Gamma tạo synapse trực tiếp với tế bào cơ vân, còn được
gọi là sợi nội suốt. Sợi ngoại suốt được chi phối thần kinh bởi neuron vận động
alpha, trong khi đó các tế bào Purkinje, tế bào hạt và tế bào tháp không có kết nối
synapse với các cơ ở ngoại vi.
TMP12 656
12. Sợi trục của neuron giao cảm trước hạch đi qua cấu trúc nào trong các cấu trúc
sau:
A) Rễ sau tuỷ sống
B) Nhánh sau của thần kinh sống.
C) Nhánh thông trắng
D) Nhánh thông xám
E) Nhánh trước của thần kinh sống
 C) Sợi trục giao cảm trước hạch đi qua nhánh thông trắng để vào thân giao cảm.
Sợi trục giao cảm sau hạch đi qua nhánh thông xám và có thể nằm trong nhánh
trước và nhánh sau của thần kinh sống
TMP12 729
13. Khẳng định nào mô tả đúng nhất vai trò của thuỳ nhộng tiểu não?
A) Điều khiển và phối hợp động tác của các cơ quanh trục (đầu và thân mình) cũng
như vai và hông.
B) Điều khiển động tác liên quan tới các cơ ngọn chi
C) Phối hợp cùng với vỏ đại não dự kiến động tác.
D) Điều hoà neuron vận động thông qua liên hệ với tuỷ sống.
 A) Thuỳ nhộng tiểu não liên quan tới chức năng điều khiên các cơ quanh trục
cũng như các cơ gốc chi ở vai và hông
TMP12 682
14. Khẳng định nào sau đây về giấc ngủ là đúng?
A) Mặc dù giai đoạn ngủ sâu (fast-wave sleep, deep sleep, REM sleep) thường liên
quan tới ngủ không mơ, nhưng giấc mơ và đôi khi ác mộng thường xảy ra trong
thời điểm này
B) Một người sẽ ít khi thức giấc một cách tự phát khi đang ở giai đoạn ngủ có cử
động mắt nhanh (REM sleep)
C) Trương lực cơ toàn cơ thể giảm đáng kể trong giai đoạn ngủ có cử động mắt
nhanh (REM sleep)
D) Tần số tim và tần số hô hấp trở nên rất đều trong giai đoạn REM sleep.

15. Khẳng định nào sau đây về cung phản xạ nội tủymạng nằm trong tuỷ sống là
đúng?
A) Neuron vận động lớn hơn rất nhiều neuron trung gian
B) Hầu hết các sợi cảm giác đến từ ngoại vi sẽ tạo synapse với neuron vận động và
không tạo synapse với neuron trung gian
C) Hầu hết sợi trực của hệ vận động trên tuỷ tạo synapse trưc tiếp với neuron vận
động
D) Neuron trung gian ở tuỷ sống chỉ nằm trong sừng trước tuỷ sống
E) Cả neuron trung gian kích thich và ức chế đều có trong tuỷ sống
16. Khẳng định nào về vai trò của vùng trung gian của tiểu não là đúng nhất?
A) Điều khiển và phối hợp động tác của các cơ quanh trục (đầu và thân mình) cũng
như vai và hông.
B) Điều khiển động tác liên quan tới các cơ đầu chi
C) Phối hợp cùng với vỏ đại não dự kiến động tác.
D) Điều hoà neuron vận động thông qua liên hệ với tuỷ sống.
17. Tế bào thần kinh khổng lồ của cấu tạo lưới giải phóng chất nào trong các chất
dẫn truyền thần kinh sau:
A) Norepinephrine
B) Serotonin
C) Dopamine
D) Acetylcholine
E) Glutamate
18. Phần lớn vỏ đai não không phù hợp như định nghĩa quy ước của vỏ não vận
động và cảm giác. Thuật ngữ nào dưới đây được sử dụng để chỉ loại vỏ não nhận
các tín hiệu đầu vào tiên phát từ các vùng khác của vỏ não?
A) Vùng vỏ não vô hạt
B) Vùng cảm giác thân thể thứ cấp
C) Vùng vỏ não liên hợp.
D) Vùng vận động bổ sung
E) Vùng thị giác thứ cấp
19. Khẳng định nào về vùng tiền vận động là đúng?
A) Vùng tiền vận động nằm ngay sau vùng vận động sơ cấp
B) Sự sắp xếp hình thể người lùn cân đối từ phía bên vào giữa ở vùng tiền vận
động, đối ngược hẳn với sự sắp xếp ở vùng vận động thứ phát.
C) Sự kích thích một nhóm neuron nhỏ rải rác ở vùng tiền vận động sẽ tạo ra động
tác co cơ của một cơ nào đó.
D) Sự kích thích vùng tiền vận động không dẫn tới bất kì sự hoạt hoá cơ nào
E) Vùng tiền vận động tạo nên một tư thế đặc biệt cần thiết cho chi thể tạo nên
động tác mong muốn.
20. Đặc điểm nào trong số các đặc điểm sau là của vùng vận đông bổ sung?

A) Không có người lùn cân đối


B) Kích thích vùng vận động bổ sung tạo nên động tác của cả 2 bên cơ thể, đặc
biệt là cả 2 tay
C) Nằm ngay trước vùng tiền vận động ở mặt bên của bán cầu đại não
D) Giống như vùng tiền vận động, kích thích vùng vận động bổ sung dẫn đến
các động tác rời rạc của các cơ
E) Chức năng vùng vận động bổ sung độc lập hoàn toàn với vùng tiền vận động
và vùng vận động sơ cấp
21. Tiêm thuốc ức chế giải phóng serotonin sẽ gây nên tác dụng nào sau đây lên
giấc ngủ?
A) Sự khởi phát giấc ngủ (buồn ngủ)Đi vào giấc ngủ gần như ngay tức khắc
B) Giai đoạn ngủ có cử động mắt nhanh (REM) sẽ bị chặn
C) Sự khởi phát giấc ngủĐi vào giấc ngủ sẽ kéo dài đáng kể hoặc bị chặn
D) Giai đoạn ngủ có cử động mắt nhanh sẽ ngay lập tức xuất hiện.
22. Cấu trúc nào của não có vai trò điều khiển hệ viền (limbic system)?
A) Dưới đồiHạ đồi thị
B) Hồi hải mã
C) Thể hạnh nhân
D) Thể vú
E) Vòm
23. Đường dẫn truyền li tâm nào trong các đường sau nằm trong cuống tiểu não
trên?
A) Các sợi cầu – tiểu não
B) Các sợi tiểu não – đồi thị
C) Bó tuỷ tiểu não sau
D) Bó vỏ - tuỷ
24. Cấu trúc bao gồm một neuron vận động với tất cả các sợi cơ vân có tiếp xúc
với neuron vận động đó thì được gọi là?
A) Thụ cảm thể Golgi ở gân của cơ
B) Đơn vị vận động
C) Các neuron liên đốt tuỷ
D) Các sợi cơ vân
25. Quá trình tạo trí nhớ có thể bị gián đoạn bởi hoạt động nào trong các hoạt động
sau?
A) Sự phosphoryl hoá của kênh Kali nhằm ngăn chặn hoạt động
B) Sự hoạt hoá adenylate cyclase
C) Mất ý thức giả tạo
D) Tăng tổng hợp protein
E) Sự hoạt hoá GMP vòng phosphodiesterase.
26. Tình trạng mất nhận thức khuôn mặt thường xuất phát từ tình trạng rối loạn hay
tổn thương vùng nào của vỏ não?
A) Vùng trước trán
B) Vùng nối giữa thuỳ đỉnh và thuỳ thái dương ở bán cầu não không ưu thế
C) Vùng xoay mắt thuỳ trán
D) Mặt dưới trong của thuỳ chẩm và thuỳ thái dương.
E) Các vùng liên quan hệ viền của thuỳ trán và phần trước của thuỳ thái dương.
27. U ở vùng nàoTổn thương vùng nào trong các vùng sau của não sẽ gây nên ảnh
hưởng nghiêm trọng nhất lên trí thông minh hình tượng và lời nói?
A) Hồi hải mã
B) Hạch hạnh nhân
C) Vùng Wernicke ở bán cầu không ưu thế
D) Vùng Broca
E) Vùng Wernicke ở bán cầu ưu thế

43. Bệnh lý thần kinh có liên quan tới tiểu não có biểu hiện ra triệu chứng nào
trong các triệu chứng sau?
A) Run khi nghỉ ngơi
B) Múa vớn
C) Co cứng (tăng trương lực)
D) Thất điều
E) Bất động (mất vận động)
44. Neuron phó giao cảm trước hạch chi phối thần kinh cho đại tràng xuống và trực
tràng, nằm trong cấu trúc nào sau đây?
A) Hạch cổ trên
B) Nhân vận động lưng của TK X
C) Hạch mạc treo tràng trên
D) Hạch mi
E) Tuỷ sống ngang đốt S2, S3.
45. Phức hợp điện thế dạng gai (complex spike pattern) của tế bào Purkinje ở tiểu
não có thể xuất hiện do kích thích từ vùng nào của đại não?
A) Phức hợp nhân trám dưới
B) Các nhân lưới ở thân não
C) Các neuron ở nhân đỏ
D) Phức hợp nhân trám trên
E) Nhân tiền đình lưng
46. Hoạt động nào trong các hoạt động sau làm tăng tính nhạy cảm của phản xạ
căng cơ?
A) Cắt các sợi của rễ sau có liên quan tới cơ trong kiểm tra phản xạ căng cơ
B) Tăng hoạt động của các nhân lưới hành não
C) Cúi đầu về phía trước
D) Các hoạt động tăng cường trong hệ nội suốt (neuron vận động gamma)
E) Kích thích bán cầu bên của tiểu não
47. Cấu trúc nào trong các cấu trúc sau hoạt động như một con đường thay thế cho
các tín hiệu từ vỏ não vận động đến tuỷ sống?
A) Nhân đỏ
B) Các nhân nền tiểu não
C) Nhân đuôi
D) Đồi thị
E) Nhân thon và nhân chêm.
48. Hiện tượng duỗi cứng mất vỏ có thể được giải thích, ít nhất phần nào đó, bởi ý
nào trong các ý sau đây?
A) Sự kích thích của neuron cảm giác loại 1b
B) Mất tín hiệu hướng tâm tiểu não tới nhân đỏ
C) Tăng quá mức hoạt động của các nhân lưới hành não liên quan tới điều hoà vận
động
D) Các hoạt động không có đối kháng của các nhân lưới cầu não
E) Thoái triển con đường dẫn truyền nằm giữa chất đen và thể vân
49. Tương tự như vùng thị giác sơ cấp, vùng vận động sơ cấp cũng được tổ chức
thành các cột dọc bao gồm các tế bào liên kết với nhau qua 6 lớp của vỏ não. Các
tế bào mà đóng góp sợi trục cho bó vỏ-tuỷ tập trung ở lớp nào của vỏ não?
A) Lớp I
B) Lớp II
C) Lớp III
D) Lớp IV
E) Lớp V
50. Khi một cơ bất ngờ bị kéo căng, một tín hiệu được đường truyền từ suốt cơ
thông qua sợi cảm giác Ia. Khẳng định nào đúng nhất khi mô tả đáp ứng phát sinh
bởi tín hiệu ly tâm từ suốt cơ?
A) Co cơ có chứa suốt cơ hoạt động
B) Giãn cơ có chứa suốt cơ hoạt động
C) Co cơ đối vận với cơ có chứa suốt cơ hoạt động
D) Giãn cơ đối vận với cơ có chứa suốt cơ hoạt động
E) Kích thích trực tiếp synapse của neuron vận động gamma
51. Lỗ nào cho phép dịch não tuỷ đi qua trực tiếp từ hệ thống não thất vào khoang
dưới nhện?
A) Lỗ Magendie
B) Cống Sylvius
C) Não thất ba
D) Não thất bên
E) Hạt màng nhện
52. Có một vùng nằm trong bán cầu ưu thế, khi nó bị tổn thương cảm giác nghe có
thể bình thường nhưng không thể sắp xếp các từ thành một ý hoàn chỉnh. Tên của
vùng đó là gì?
A) Vùng thính giác sơ cấp
B) Vùng Wernicke
C) Vùng Broca
D) Hồi góc
E) Vỏ não liên quan hệ viền
53. Các sợi li tâm từ tiểu não xuất phát từ?
A) Các nhân xám dưới vỏ (Các nhân tiêu não sâu)
B) Lớp tế bào Purkinje của vỏ tiểu não
C) Lớp hạt của vỏ tiểu não
D) Lớp phân tử của vỏ tiểu não
E) Vỏ thuỳ nhung của tiểu não
54. Các tín hiệu hướng tâm từ phần ngoại vi của thân thể đi tới tiểu não trong bó
nào sau đây?
A) Bó tuỷ tiểu não trước
B) Bó mái – chất lưới
C) Bó tiền đình-tiểu não
D) Bó chất lưới-tiểu não
E) Bó tuỷ tiểu não sau
55. Các neuron của vỏ não vận động nhận các tín hiệu feedback từ các cơ được
kích thích bởi hệ vỏ-tuỷ. Feedback này xuất phát từ cấu trúc nào trong các cấu trúc
sau:
A) Nhân đỏ
B) Các bó tuỷ-tiểu não
C) Bề mặt da của các ngón tay dùng để cầm nắm đồ vật
D) Suốt cơ trong các cơ đối kháng với những cơ tạo nên động tác
E) Nhân tiền đình
56. Tình trạng động kinh nào có liên quan tới tình trạng trầm cảm sau cơn động
kinh kéo dài từ vài phút hay có thể cho tới vài giờ?
A) Động kinh cơn lớn
B) Đông kinh cơn bé
C) Cơn động kinh vận động khu trú (Động kinh Jackson)
D) Cơn co giật pha thoát
E) Động kinh thuỳ thái dương
57. Các tuyến mồ hôi và các cơ dựng lông của vùng da có lông được chi phối thần
kinh bởi loại sợi nào trong các loại sợi sau?
A) Các sợi phó giao cảm hậu hạch Cholinergic
B) Các sợi giao cảm hậu hạch Cholinergic
C) Các sợi phó giao cảm tiền hạch Adrenergic
D) Các sợi giao cảm hậu hạch Adrenergic
E) Các sợi giao cảm tiền hạch Adrenergic
58. Trong việc điều khiển các cơ tinh tế của bàn tay và ngón tay, các sợi trục của
bó vỏ-tuỷ có thể tạo synapse trực tiếp với cấu trúc nào?
A) Neuron sừng sau tuỷ sống
B) Neuron trung gian tuỷ sống
C) Neuron vận động tuỷ sống
D) Các tế bào Purkinje
E) Các tế bào Renshaw.
59. Khẳng định nào sau đây là đúng khi mô tả cung phản xạ tuỷ?
A) Các neuron vận động gamma động chi phối thần kinh cho các sợi túi nhân tĩnh
B) Sợi trục trên tuỷ đi xuống sẽ tạo synapse với cả neuron vận động alpha và
gamma.
C) Hiện tượng rung giật xuất hiện do hậu quả của phản xạ căng cơ quá mức
D) Các thành phần có khả năng co giãn của các sợi nội suốt được tìm thấy ở vùng
trung tâm của sợi
E) Cả 2 loại sợi cảm giác trong suốt cơ là các thụ thể cơ học thông báo tín hiệu kéo
căng của 2 đầu không co giãn của sợi nội suốt.
60. Tế bào nào trong các tế bào sau nhận tín hiệu đầu vào trực tiếp từ thụ thể Golgi
ở gân của cơ?
A) Neuron trung gian ức chế type Ia
B) Neuron vận động gamma động
C) Neuron vận động alpha
D) Neuron trung gian ức chế type Ib
E) Neuron trung gian kích thích type II.
61. Sợi trục của neuron nhân lục phân bố xuyên suốt nhiều vùng của não sử dụng
chất dẫn truyền thần kinh nào sau đây?
A) Norepinephrine
B) Dopamine
C) Serotonin
D) Acetylcholine
62. Khẳng định nào sau đây là đúng khi mô tả các sợi rêu và sợi leo?
A) Các sợi rêu cung cấp tín hiệu đầu vào kích thích cho các tế bào Purkinje
B) Các sơi leo tạo nên một chồi đơn trong tế bào Purkinje
C) Tất cả các sợi trục của bó tuỷ-tiểu não đều tận cùng như các sợi rêu ở vỏ tiểu
não
D) Tất cả các sợi trục của bó cầu-tiểu não đều tận cùng như các sợi leo ở vỏ tiểu
não
E) Tất cả các sợi leo đều bắt nguồn từ nhân đỏ
63. Động tác vận động tinh tế của ngón trỏ có thể được thực hiện bởi sự kích thích
vùng vỏ não bộ nào sau đây?
A) Vùng vỏ não vận động sơ cấp
B) Bán cầu tiểu não bên
C) Vỏ não tiền vận động
D) Vùng vận động bổ sung
E) Nhân đỏ
 A) Một vùng lớn của vỏ não vận động sơ cấp chi phối sự hoạt hoá các cơ điều
khiển vận động của các ngón tay. Sự kích thích vùng vỏ não vận động sơ cấp
thường dẫn tới sự co riêng biệt của các nhóm cơ nhỏ. Sự kích thích vùng tiền vận
động dẫn tới co một lượng lớn các nhóm cơ và kích thích vùng vận động bổ sung
dẫn tới vận động cả 2 bên cơ thể.
TMP12 667-668
64. Khoảng quanh mạch (khoảng Virchow – Robin) ở trong não bộ được tạo thành
giữa thành của các mạch xuyên nhỏ và cấu trúc nào trong các cấu trúc sau?
A) Màng cứng
B) Màng nhện
C) Màng mềm
D) Đám rối mạch mạc
E) Các tế bào màng ống nội tuỷ
 C) Khoảng quanh mạch (còn được biết là khoảng Virchow-Robin) được tạo
thành giữa thành ngoài của các mạch máu nhỏ đâm xuyên vào trong não và màng
mềm, là lớp màng nằm bên ngoài bề mặt của não và chỉ được gắn một cách lỏng
lẻo vào não bộ
TMP12 747

65. Loại động kinh nào trong các loại động kinh sau có liên quan tới sóng điện não
đồ có dạng vòm và dạng gai trong cơn động kinh?
A) Động kinh cơn lớn
B) Động kinh thuỳ thái dương
C) Động kinh Jackson
D) Động kinh cơn nhỏ
E) Động kinh có liên quan tới xuất huyết
 D) Sóng dạng vòm và dạng gai là đặc trưng của động kinh cơn nhỏ
TMP12 726
66. Tác dụng kích thích hay ức chế của một sợi giao cảm hậu hạch được xác định
bởi cấu trúc hay đặc điểm nào sau đây?
A) Chức năng của thụ thể hậu synapse mà nó gắn vào
B) Cơ quan đặc hiệu được thần kinh chi phối
C) Hạch nơi sợi hậu hạch đi ra
D) Hạch chứa sợi tiền hạch
E) Trạng thái cảm xúc của cá nhân
 A) Tác dụng kích thích hay ức chế của sợi giao cảm hậu hạch được xác định
duy nhất bởi loại thụ thể mà nó gắn vào.
TMP12 732-733
67. Sợi trục của các neuron chất đen, mà chúng đi tới nhân đuôi và bèo sẫm sử
dụng chất dẫn truyền thần kinh nào trong các chất sau?
A) Norepinephrine
B) Dopamine
C) Serotonin
D) Acetylcholine
E) GABA
 B) Các tế bào ở phần đặc của liềm đen (phần chất đen nằm ở trung não) sử dụng
chất dẫn truyền thần kinh dopamine trong sợi trục đi tới nhân đuôi và bèo sẫm
TMP12 692-693
68. Khẳng định nào trong các khẳng định sau về các neuron của tiểu não là đúng?
A) Các tế bào giỏ tạo ra các đáp ứng kích thích trong các tế bào Purkinje
B) Các tế bào hạt tạo ra các đáp ứng kích thích trong tế bào Purkinje
C) Các tế bào Golgi tạo ra các đáp ứng kích thích trong các tế bào giỏ
D) Các tế bào Purkinje tạo các đáp ứng kích thích trong các tế bào nhân tiểu não
E) Các tế bào hình sao tạo ra các đáp ứng kích thích trong các tế bào giỏ.
 B) Sợi trục của các tế bào hạt có các nhánh tạo nên hệ thống các sợi song song
trong lớp phân tử, tại đây chúng tạo ra các tín hiệu đầu vào kích thích cho sợi
nhánh của các tế bào Purkinje
TMP12 684

69. Ý nào trong các ý sau mô tả đúng mối quan hệ giữa áp lực dịch não tuỷ với áp
lực tĩnh mạch trong xoang dọc trên?
A) Cao hơn một vài mmHg
B) Thấp hơn một vài mmHg
C) Bằng nhau
D) Cao gấp hai lần
E) Bằng một nửa
 A) Dịch não tuỷ chảy dọc theo các hạt màng nhện khi áp lực dịch não tuỷ chỉ
cao hơn áp lực trong xoang dọc trên một vài mmHg
TMP12 747-748
70. Một tổn thương mạch máu gây nên tình trạng thoái triển sợi trục của dải vỏ-tuỷ
ở cầu não phần nền có thể biểu hiện ra tình trạng nào trong các tình trạng sau?
A) Liệt chủ yếu liên quan tới các cơ quanh vai đối bên và khớp háng
B) Liệt các cơ nhai
C) Mất vận động chủ động các vận động riêng lẻ của bàn tay và ngón tay đối bên
D) Mất khả năng nói rõ ràng
E) Mất khả năng chuyển trí nhớ ngắn hạn thành dài hạn
 C) Dấu hiệu đặc trưng nhất của tổn thương neuron dải vỏ-tuỷ là sự suy giảm
vận động chủ động riêng rẽ của bàn tay và ngón tay đối bên tổn thương.
TMP12 673
71. Khẳng định nào mô tả đúng nhất sự khác nhau về chức năng giữa thụ thể Golgi
ở gân của cơ và suốt cơ?
A) Tín hiệu đầu ra của thụ thể Golgi làm ức chế các cơ có liên quan tới thụ thể đó
B) Thụ thể Golgi không có chức năng trong quá trình các vận động chủ động mà
cần tới một mức tăng lực kéo căng bình thường ở các cơ có liên quan
C) Các tín hiệu từ thụ thể Golgi không tham gia vào cảm nhận có ý thức
D) Các tín hiệu từ thụ thể Golgi được liên kết kiểu synapse trực tiếp với một
neuron vận động alpha
E) Các tín hiệu từ thụ thể Golgi được dẫn truyền dọc các sợi cảm giác. Đây là các
sợi dẫn truyền với tốc độ nhanh hơn các sợi của suốt cơ.
 A) Các tín hiệu từ thụ thể Golgi làm ức chế các cơ có liên quan, trong khi hoạt
động của suốt cơ làm kích thích các cơ có liên quan với suốt cơ đó. Các thụ thể
Golgi và suốt cơ giống nhau ở chức năng trong quá trình diễn ra một động tác bình
thường. Các tín hiệu hướng tâm của thụ thể Golgi dẫn truyền chậm hơn các tín
hiệu hướng tâm của suốt cơ, và các thụ thể Golgi tạo nên các tín hiệu đầu vào cho
neuron trung gian, chứ không phải neuron vận động. Cái này cũng tương tự như ở
suốt cơ.
TMP12 661
72. Tiểu não đôi khi được mô tả như một “thiết bị tính toán thời gian”. Khẳng định
nào sau đây mô tả đúng nhất cơ sở của chức năng này?
A) Tiểu não nhận các tín hiệu đầu vào về thị giác, nó cho phép tiểu não xác định
bất kì thời điểm nào trong chu kì ngày đêm 24 giờ.
B) Tiểu não tính toán thời gian chính xác, từ đó kích thích các tế bào Purkinje liền
kề.
C) Các cung phản xạ của tiểu não chỉ xác định khoảng thời gian hoặc thời điểm kết
thúc của mỗi động tác
D) Các cung phản xạ của tiểu não làm rõ các thời điểm kích thích và ức chế trong
mỗi động tác bằng cách đưa các tín hiệu kích thích theo sau bởi các tín hiệu ức chế
có tính toán thời gian chính xác.
E) Các cung phản xạ của tiểu não chỉ xác định thời gian chính xác của các tín hiệu
kích thích và tín hiệu ức chế xuất hiện khi tiểu não dừng hoạt động chức năng.
 D) Các cung phản xạ của tiểu não làm rõ các thời điểm kích thích và ức chế
trong mỗi động tác. Điều này giúp tính toán chính xác thời gian bắt đầu và kết thúc
của mỗi động tác
TMP12 688
73. Các tín hiệu đầu ra từ thụ thể Golgi ở gân được dẫn truyền tới trung tâm cao
hơn nào sau đây?
A) Gò dưới
B) Cầu nhạt
C) Tiểu não
D) Nhân đỏ
E) Chất đen
 C) Thụ thể Golgi ở gân tạo ra các tín hiệu đầu vào cho tiểu não. Chúng không
tạo ra các tín hiệu đầu vào cho gò dưới, cầu nhạt, nhân đỏ hay chất đen.
TMP12 661
74. Loại thụ cảm thể thể cholinergic nào được tìm thấy ở synapse giữa các neuron
trước hạch và sau hạch của hệ giao cảm?
A) Muscarinic
B) Nicotinic
C) Alpha
D) Beta1

E) Beta2

 B) Thụ cảm thể cholinergic nicotinic được tìm thấy ở synapse giữa các neuron
giao cảm trước hạch và sau hạch
TMP12 733
75. Tổn thương giới hạn trong vùng vận động sơ cấp (vùng 4) được cho là gây nên
tình trạng giảm trương lực ở các cơ bị ảnh hưởng. Tuy nhiên, hầu hết các tổn
thương vỏ não, đặc biệt các tổn thương gây ra do nhồi máu, thường liên quan tới
vùng vận động sơ cấp và các cùng xung quanh hay sợi trục li tâm từ vỏ não. Loại
sau của tổn thương vỏ này gây nên tình trạng nào trong các tình trạng sau?
A) Liệt cứng
B) Liệt mềm
C) Không liệt, chỉ có các động tác co giật nhanh
D) Mất thị lực hoàn toàn mắt bên đối diện
E) Mất cảm giác ở chân bên đối diện.
 A) Các tổn thương vùng vận đông sơ cấp và các vùng vỏ não vận động xung
quanh dẫn tới liệt cứng ở các cơ bị ảnh hưởng.
TMP12 673
76. Hệ viền bao gồm vùng vỏ não trán ổ mắt, hồi dưới chai, hồi khuy và vùng nào
trong các vùng sau?
A) Vùng vận động bổ sung
B) Hồi sau trung tâm
C) Hồi lưỡi
D) Hồi cạnh hải mã
E) Tiểu thuỳ cạnh trung tâm
 D) Hồi cạnh hải mã là một phần quan trọng của hệ viền hay thuỳ viền.
TMP12 714-715

77. Tiểu não tham gia vào việc học các kĩ năng vận động và các sợiơi leo đi vào
các tế bào Purkinje được cho là đóng vai trò quan trọng nhất trong quá trình này.
Khẳng định nào sau đây liên quan tới quá trình này là đúng?
A) Trong tình trạng nghỉ ngơi, các sợiơi leo tạo ra các phức hợp điện thế dạng gai
ở tế bào Purkinje với tốc độ rất nhanh
B) Khi một động tác mới được thực hiện, động tác thực sự có thể không phù hợp
với động tác dự kiến, cái mà có xu hướng làm giảm tần suất phát xung của các điện
thế đơn giản dạng gai khởi phát bởi sợi leo.
C) Giảm tín hiệu đầu vào của sợi leo làm giảm độ nhạy cảm chung (tính kích
thích) của sợi Purkinje
D) Trong lần thử nghiệm đầu tiên của động tác mới, tế bào Purkinje nhận một mức
kích thích mới kéo dài tương đối
E) Neuron trong phức hợp nhân trám dưới và sợi trục của nó đi tới tiểu não đóng
vai trò quan trọng trong quá trình này.
 E) Sợi leo (trám dưới) đi vào tế bào Purkinje được cho là làm thay đổi tính kích
thích của tế bào Purkinje, làm nó đáp ứng phù hợp với tín hiệu đầu vào của tế bào
hạt.
TMP12 686
78. Tắc cấu trúc nào sau đây sẽ dẫn tới tình trạng não úng thuỷ thể lưu thông?
A) Cống Sylvius
B) Não thất bên
C) Lỗ Luschka
D) Lỗ Magendie
E) Hạt màng nhện
 E) Não úng thuỷ thể không lưu thông xuất hiện khi có sự tắc nghẽn dòng chảy
của dịch não tuỷ trong hệ thống não thất hay ở vị trí nối giữa hệ thống não thất và
khoang dưới nhện. Não úng thuỷ thể lưu thông xuất hiện khi có sự tắc nghẽn ở cả
khoang dưới nhện và hạt màng nhện, do đó ngăn sự thông thương giữa khoang
dưới nhện và xoang dọc trên.
TMP12 748
79. Đánh giá một bệnh nhân phát hiện các tổn thương sau: (1) giảm tính hung
hăngang và lòng ham muốn, và đáp ứng xã hội không phù hợp; (2) mất khả năng
xử lý các ý nghĩ liên tiếp nhau để giải quyết vấn đề; (3) mất khả năng xử lý lượng
lớn thông tin, cái mà sau đó sẽ được nhớ lại ngay để hoàn thiện ý nghĩ hay giải
quyết một vấn đề. Tổn thương vùng nào trong não sẽ dẫn tới các tổn thương như
trên?
A) Vùng tiền vận động
B) Vùng đỉnh chẩm ở bán cầu không ưu thế
C) Vùng Broca
D) Vùng liên kết với hệ viền
E) Vùng liên kết vùng trước trán
 E) Mất chức năng về hành vi, thay đổi nhân cách và giảm khả năng giải quyết
vấn đề là các dấu hiệu của tổn thương vùng vỏ não liên quan tới vùng trước trán.
TMP12 702-701
80. Phản xạ rút lui được khởi phát bởi sự kích thích đến từ thụ cảm thể nào trong
các thụ cảm thể sau?
A) Suốt cơ
B) Thụ thể ở bao khớp
C) Đầu tự do của các sợi thần kinhân kính ở dưới da
D) Thụ thể Golgi ở gân
E) Tiểu thể Pacini
 C) Phản xạ rút lui được khởi phát bởi các kích thích đến từ đầu tự do của các
sợi thần kinh. Suốt cơ cung cấp các tín hiệu hướng tâm cho phản xa căng cơ, thụ
thể Golgi là nguồn kích thích cho phản xạ căng cơ nghịch đảo (phản xạ dây chằng
Golgi – Golgi tendon reflex)
81. Chất nào trong các chất sau hoạt hoá thụ thể alpha và beta adrenergic cân bằng
nhau?
A) Acetylcholine
B) Norepinephrine
C) Epinephrine
D) Serotonin
E) Dopamine
 C) Epinephrine hoạt hoá thụ thể alpha và beta adrenergic cân bằng nhau.
Norepinephrine kích thích cả 2 loại thụ thể, nhưng tác dụng lớn hơn ở thụ thể alpha
TMP12 733
82. Vùng hạ đối bên và sau, cùng với vùng trước thị, có liên quan tới việc điều
khiển chức năng nào trong các chức năng sau?
A) Chức năng tim mạch liên quan tới huyết áp và nhịp tim
B) Điều hoà cảm giác khát và lượng nước nhập vào
C) Kích thích co thắt tử cung và tiết sữa
D) Báo hiệu lượng thức ăn vào đã đủ (sự no)
E) Bài tiết hormone từ thuỳ trước của tuyến yênến.
 A) Vùng hạ đồi bên và sau cùng với vùng trước thị, tạo nên một nhóm tế bào
điều khiển chức năng tim mạch như nhịp tim và huyết áp
TMP12 715-716
83. Khẳng định nào liên quan tới hệ lưới-tuỷ là đúng?
A) Neuron của dải lưới-tuỷ không nhận tín hiệu đầu vào từ vùng vận động của vỏ
não
B) Các sợi lướiươi-tuỷ hành não kích thích neuron vận động hoạt hoá các cơ duỗi.
C) Các sợi lưới-tuỷ cầu não đi trong thừng sau của tuỷ sống
D) Các sợi lưới-tuỷ hành não đi trong phần giữa của thừng trước tuỷ sống
E) Các sợi lưới-tuỷ cầu não kích thích neuron vận động tuỷ sống hoạt hoá các cơ
duỗi các chi
 E) Các sợi lưới-tuỷ cầu não kích thích neuron vận động chi phối cơ duỗi. Ngược
lại, các sợi lưới-tuỷ hành não ức chế các neuron vận động cơ duỗi.
TMP12 673-674
84. Neuron nằm ở nhân lục giải phóng chất dẫn truyền thần kinh nào ở tận cùng
synapse của chúng?
A) Norepinephrine
B) Dopamine
C) GABA
D) Acetylcholine
E) Serotonin
 A) Neuron nằm ở nhân lục giải phóng norepinephrine ở tận cùng thần kinh của
chúng.
TMP12 712-713
85. Trong phản xạ bánh chè, thành phần nào trong các thành phần sau sẽ tạo
synapse trực tiếp với neuron vận động alpha, là neuron chi phối thần kinh cho cơ bị
kéo căng?
A) Sợi cảm giác Ia
B) Sợi cgiảm giác Ib
C) Neuron trung gian kích thích
D) Neuron vận động gamma
E) Neuron trung gian ức chế
 A) Sợi cảm giác Ia tạo synapse trực tiếp với neuron vận động alpha, trong khi
sợi cảm giác Ib tạo synapse với neuron trung gian ức chế. Neuron trung gian kích
thích đóng vai trò quan trọng trong phản xạ rút lui. Neuron vận động gamma nhận
các tín hiệu đầu vào chủ yếu từ hệ trên tuỷ sống
TMP12 658-660
86. Khẳng định nào trong các khẳng định sau mô tả đúng nhất vai trò chức năng
của các sợi lưới – tuỷ cầu não (ở đây gọi tắt là hệ thống ở cầu não) so với các sợi
lưới-tuỷ hành não (ở đây gọi tắt là hệ thống ở hành não)?
A) Hệ thống ở cầu não hoạt động có sự phối hợp với hệ thống ở hành não trong
việc tạo ra các kích thích tới các cơ duỗi
B) Hệ thống ở cầu não về mặt bản chất hoạt động chức năng ngược lại so với hệ
thống ở hành não
C) Hệ thống ở cầu não tạo ra các kích thích ban đầu chậm, rồi sau đó hệ thống ởơ
hành não tạo ra các kích thích nhanh.
D) Hệ thống ở hành não tạo ra các kích thích chậm tới các neuron vận động cơ
duỗi, trong khi đó hệ thống ở cầu não tạo ra các kích thích chậm tới các neuron vận
đông của cơ gấp
E) Hệ thống ở hành não tạo ra các kích thích tới neuron vận động các cơ duỗi chi
trên, trong khi đó hệ thống cầu não tạo ra các kích thích tới neuron vận động của
cơ duỗi chi dưới.
 B) Các hệ thống lưới-tuỷ hành não và hệ thống lưới-tuỷ cầu não hoạt động chức
năng đối nghịch nhau trong quá trình tác động lên neuron vận động điều khiển các
cơ duỗi của chi và cơ trục thân.
TMP12 674
87. Phản xạ nào trong các phản xạ sau ghép cặp chính xác với cấu trúc nhận cảm
điều hoà phản xạ?
A) Phản xạ ức chế tự phát – suốt cơ
B) Phản xạ ức chế chéo – Thụ cảm thể Golgi tại gân
C) Phản xạ ức chế chéo – Tiểu thểế Pacini
D) Phản xạ căng cơ – Suốt cơ
E) Phản xạ gân – Tiểu thể Meissner.
 D) Phản xạ căng cơ được điều hoà bởi suốt cơ. Phản xạ ức chế tự phát liên quan
tới thụ cảm thể Golgi tại gân. Phản xạ ức chế chéo cũng liên quan tới suốt cơ.
TMP12 658-659
88. Thành phần nào sau đây đại diện cho cơ sở cấu trúc của hàng rào máu-dịch não
tuỷ?
A) Liên kết chặt chẽ giữa các tế bào của khoang não-tuỷ sống tạo nên thành não
thất
B) Hạt màng nhện
C) Liên kết chặt chẽ giữa các tế bào của đám rối màng mạc liền kề nhau
D) Chân của các tế bào sao
E) Liên kết chặt chẽ giữa các tế bào nội mô liền kề nhau của mao mạch máu não
 C) Liên kết chắt chẽ tạo nên giữa các tế bào biểu mô màng mạch liền kề đại
diện cho cơ sở cấu trúc của hàng não máu-dịch não tuỷ. Hàng rào máu não được
tạo thành bởi liên kết chặt chẽ giữa các tế bào nội mô liền kề nhau của mao mạch
máu não.
TMP12 748-749
89. Tổn thương vùng nào của não dẫn tới mất khả năng hiểu ngôn ngữ viết hoặc
ngôn ngữ nói?
A) Vỏ não thuỳ đảo ở bán cầu ưu thế
B) Thuỳ chẩm trước
C) Vùng nối giữa các thuỳ đỉnh, thái dương và chẩm
D) Phần giữa của hồi trước trung tâm
E) Phần trước nhất của thuỳ thái dương.
 C) Phần nối giữa các thuỳ đỉnh, thái dương và chẩm thường liên quan tới vùng
Wernicke. Vùng này của não đóng vai trò hiểu cả ngôn ngữ viết và ngôn ngữ nói.
TMP12 669
90. Ở một người bình thường khác, mất chức năng vùng não nào sẽ dẫn tới hành vi
không phù hợp trong một hoàn cảnh xã hội cho trước?
A) Các nhân bụng giữa của hạ đồi thị.
B) Thể hạnh nhân
C) Thể trai
D) Vòm não
E) Móc của hồi cạnh hải mã
 B) Thể hạnh nhân có vẻ như đóng vai trò trong nhận thức hành vi ở mức độ bán
ý thức. Thể hạnh nhân có sợi trục đi tới hệ viên nên cũng được cho là giúp nhận
thức tình trạng hiện tại của một ai đó với hoàn cảnh xảy ra xung quanh. Do đó, thể
hạnh nhận được cho là giúp tạo những hành vi phù hợp trong từng hoàn cảnh
TMP12 719-720
91. Các tuyến ở mũi, tuyến lệ, tuyến nước bọt và tiêu hoá được kích thích bởi chất
nào trong các chất sau?
A) Acetylcholine
B) Norepinephrine
C) Epinephrine
D) Serotonin
E) Dopamine
 A) Các tuyến ở mũi, tuyến lệ, tuyến nước bọt và tuyến tiêu hoá được kích thích
bởi neuron phó giao cảm hậu hạch cholinergic
TMP12 735
92. Phản xạ nào trong các phản xạ sau mô tả đúng nhất hiện tượng các tín hiệu đau
đang được tiếp nhận làm cho các nhóm cơ đối vận ở bên cùng phía của cơ thể thực
hiện động tác?
A) Phản xạ duỗi chéo
B) Phản xạ rút lui
C) Phản xạ ức chế chéo
D) Phản xạ ức chế tự phát
 A) Phản xạ duỗi chéo phụ thuộc vào các tín hiệu đau cảm nhận được phân bố ở
cả hai phía của tuỷ sống thông qua neuron trung gian kích thích
TMP12 663
93. Bó tuỷ-tiểu não liên quan tới điều khiển các động tác định sẵn. Loại động tác
này được lên kế hoạch trước toàn bộ tức là các bước khởi đầu, thực hiện và kết
thúc đều được lập trình bởi tiểu não. Khẳng định nào trong các khẳng định sau là
đúng khi đề cập tới triệu chứng quan sát được ở bệnh nhận có tổn thương tiểu não
mà có liên quan tới động tác định sẵn?
A) Các động tác khởi phát rất chậm
B) Tốc độ của động tác nhanh hơn mong đợi
C) Động tác kết thúc nhanh hơn
D) Liệt cứng xuất hiện ở các nhóm cơ bị ảnh hưởng
E) Run khi nghỉ xuất hiện ở các chi bị ảnh hưởng.
 A) Cả quá trình khởi phát và thực hiện của động tác thuộc loại định sẵn đều rất
chậm trong tổn thương liên quan tới bó tuỷ-tiểu não
TMP12 688
94. Cơn co cứng mất não xuất hiện trong hoàn cảnh nào trong các hoàn cảnh sau?
A) Tổn thương các hệ thống ở thân não điều khiển neuron vận động cơ gấp
B) Hoạt động quá mức của hệ chất lưới – tuỷ hành não dẫn tới tăng hoạt động của
các cơ duỗi ở chi
C) Mất cân bằng trong hoạt động của các hệ thống chất lưới – tuỷ và chất lưới
hành não – tuỷ cầu não. Ví dụ như kích thích các neuron vận động các cơ duỗi là
hậu quả cuối cùng.
D) Gián đoạn sợi trục của bó chất lưới – tuỷ hành não
E) Gián đoạn sợi trục của bó chất lưới– tuỷ cầu não
 C) Trong cơn co cứng mất não, sợi trục từ vỏ não bình thường kích thích hệ
chất lưới-tuỷ ức chế giờ đây bị gián đoạn. Mặc dù sự kích thích của vỏ não tới hệ
chất lưới-tuỷ cầu não kích thích cũng bị mất, nhưng hệ thống sau tiếp tục bị kích
thích bởi các đường dẫn truyền cảm giác thân thể (đường cảm giác đau) đi lên mà
không bị tổn thương. Điều này dẫn tới sự kích thích của các neuron vận động cơ
duỗi mà không có tác động ức chế của các sợi lưới-tuỷ hành não
TMP12 674
95. Phù não là một biến chứng nặng của tình trạng biến đổi dòng chảy dịch thể
trong não. Tiến triển tiếp theo của phù não có thể dẫn tới tình trạng nào trong các
tính trạng sau?
A) Giãn các cơ trơn mạch máu và giảm lưu lượng dòng máu
B) Tăng lưu lượng dòng máu dẫn tới tăng nồng độ oxy
C) Co mạch và giảm phù
D) Giãn cơ trơn mạch máu và tăng lưu lượng dòng máu
E) Co thắt mạch máu dẫn tới thiếu máu cục bộ và giãn mao mạch còn bù
 E) Phù não dẫn tới co mạch não, giảm lưu lượng máu, thiếu máu cục bộ và giãn
mạch còn bù. Điều này cuối cùng dẫn tới một tình trạng phù não nặng hơn.
TMP12 749
96. Phần nào của tiểu não có chức năng lên kế hoạch cho động tác liên tiếp nhau?
A) Nhộng tiểu não và nhân đỉnh
B) Vùng trung gian và nhân đỉnh
C) Bán cầu bên và nhân xen
D) Tiểu não vỏ (tân tiểu não) và nhân răng
E) Tiểu não tuỷ sống (tiểu não cổ) và nhân xen
 D) Tiểu não vỏ (tân tiểu não) và nhân răng có liên quan với đồi thị và vỏ não
trong việc lên kế hoạch cho các động tác phức tạp.
TMP12 682
97. Các tổn thương cả hai bên liên quan tới hạ đồi thị giữa bên sẽ dẫn tới tình trạng
nào trong các tình trạng sau?
A) Giảm ăn và uống
B) Mất nhu cầu tình dục
C) Tăng động, hoạt động ăn, giận dữ, gây gổ quá mức
D) Co bóp tử cung, to tuyến vú
E) Rối loạn ám ảnh cưỡng bức
 C) Các tổn thương có liên quan tới hạ đồi thị giữa trước dẫn tới ăn quá mức,
uống quá mức và giận dữ, gây gổ và tăng động.
TMP12 717
98. Cấu trúc nào trong các cấu trúc sau là đường dẫn truyền quan trong liên hệ giữa
hệ viền và thân não?
A) Dải vú đồi thị
B) Vòm não
C) Mép trước
D) Hồi trên trai
E) Bó giữa não trước
 E) Bó giữa não trước từ vùng vách và vùng trán ổ mắt của vỏ đại não đi xuống
quá trung tâm của hạ đồi thị tới vùng chất lưới thân não. Cấu trúc này đóng vai trò
quan trọng trong hệ thống liên hệ giữa hệ viền và thân não.
TMP12 715
99. Thuật ngữ nào sau đây mô tả đúng nhất tổn thương tiểu não trong đó có mất
chức năng thực hiện các động tác nhanh luân phiên nhau biểu hiện sự mất khả
năng tiến triển từ một phần của động tác tới phần tiếp theo?
A) Quá tầm
B) Run chủ ý
C) Rối loạn vận ngôn
D) Rung giật nhãn cầu kiểu tiểu não
E) Rối loạn liên động
 E) Rối loạn liên động là một biểu hiện của tổn thương tiểu não liên quan tới mất
khả năng tiến triển từ một phần của động tác sang phần tiếp theo. Hệ quả là động
tác mà có sự luân phiên nhanh giữa động tác gấp và duỗi sẽ bị ảnh hưởng nặng nề
nhất
TMP12 689
100. Chức năng của cơ quan hay hệ thống nào sau đây do hệ thần kinh giao cảm
chiếm ưu thế?
A) Mạch máu hệ thống
B) Tim
C) Bài tiết của các tuyến tiêu hoá
D) Tuyến nước bọt
E) Nhu động tiêu hoá
 A) Chi phối thần kinh và chức năng của mạch máu hệ thống bị ảnh hưởng chủ
yếu, nếu không muốn nói là toàn bộ, bởi hệ thần kinh giao cảm
TMP12 735
101. Cấu trúc nào trong các cấu trúc sau ở bộ máy tiền đình có vai trò phát hiện gia
tốc góc?
A) Màng thạch nhĩ
B) Vết
C) Các ống bán khuyên
D) Cầu nang (túi nhỏ)
E) Bóng ống bán khuyên
 C) Gia tốc thẳng thì nằm trên một đường thẳng; gia tốc góc thì quay xung quanh
một điểm. Các ống bán khuyên đáp ứng với các chuyển động quay của đầu và cơ
thể
TMP12 677

Unit 12: Tiêu hóa


1. Loại thức ăn nào sau đây cần động tác nhai để tiêu hoá?
A. Pho mai
B. Trứng
C. Rau
D. Thịt
 C. Nhai là một động tác quan trọng cho sự tiêu hoá các thức ăn cứng. Nó đặc
biệt quan trọng cho rau quả vì những thực phẩm này có lớp màng bọc cellulose
khó tiêu và lớp màng này phải bị phá vỡ để những thành phần bên trong có thể tiếp
xúc với các enzym tiêu hoá.
TMP12 763
2. Thông thường, loại carbohydrate nào được tiêu thụ chủ yếu trong khẩu phần
ăn của người?
A. Amylose
B. Cellulose
C. Maltose
D. Tinh bột
 D. Có 3 nguồn carbohydrate trong khẩu phần ăn bình thường của người bao
gồm sucrose chủ yếu từ cây mía và củ cải đường, lactose từ sữa, và một lượng lớn
tinh bột. Mặc dù một số khẩu phần ăn có thể chứa thêm một lượng lớn cellulose
nhưng chất này không thể tiêu hoá được ở ruột người và không được coi như một
loại thức ăn. Maltose là một sản phẩm của quá trình tiêu hoá tinh bột, nhưng không
được tiêu thụ với lượng lớn trong khẩu phần ăn của người.
TMP12 790
3. Thông thường, chất nào không được tìm thấy nhiều trong máu tĩnh mạch
cửa?
A. Các acid amin
B. Glucose
C. Các acid béo chuỗi ngắn
D. Các triglycerid
 D. Các triglyceride được tiêu hoá trong lòng ruột thành các monoglyceride và
các acid béo tự do, những chất này sau đó sẽ được hấp thụ trực tiếp qua lớp màng
của các tế bào biểu mô ruột. Sau khi đi vào các tế bào biểu mô, các acid béo và
monoglyceride được phân cắt bởi lưới nội bào không hạt của tế bào, tại đó chúng
chủ yếu được sử dụng để tạo ra các triglycerid mới và tiếp đó được giải phóng dưới
dạng chylomicron và đi qua lớp đáy của tế bào biểu mô. Các chylomicron được
hấp thụ bởi các ống bạch huyết trung tâm trong nhung mao ruột và được vận
chuyển vào hạch bạch huyết, rồi thông qua hạch vú để đi vào tuần hoàn. Do vậy,
hầu hết các triglyceride sẽ không được hấp thu qua tĩnh mạch cửa.
TMP12 792
4. Một phụ nữ 19 tuổi, vào viện vì lý do nôn, tiêu chảy, chóng mặt nhẹ và đầy
hơi. Bác sĩ cho bệnh nhân uống 50g lactose tại thời điểm ban đầu, và đo
lượng hydro trong hơi thở bệnh nhân 30 phút/lần trong 3 giờ bằng máy đo
cầm tay. Các kết quả được hiển thị như hình dưới đây. Mô tả nào đúng nhất
cho tình trạng bệnh nhân này?
A. Sự thiếu hụt enzym lactose ở diềm bàn chải
B. Sự dư thừa enzym lactose ở diềm bàn chải
C. Sự giảm enzym lactose tuỵ
D. Sự tăng enzym lactose tuỵ
E. Không có sự bất thường về tiêu hoá
 A. Các bệnh nhân thiếu hụt enzym lactase không thể tiêu hoá các sản phẩm sữa
có chứa lactose (đường sữa). Các operon (gen điều hoà sự sinh sản enzym) của vi
khuẩn ruột nhanh chóng chuyển sang dạng chuyển hoá lactose, dẫn tới quá trình
lên men mà sản phẩm tạo thành là một lượng lớn các khí (hỗn hợp khí hydro, CO2,
và CH4). Khí này có thể gây ra một loạt các triệu chứng tại bụng như đau tức
bụng, chướng và đầy hơi. Các khí này sau đó sẽ được hấp thụ qua máu (đặc biệt ở
đại tràng), và được đào thải qua 2 phổi.
TMP12 790
5. Trên những bệnh nhân có giảm tiết acid dịch vị, khả năng tiêu hoá loại thức
ăn nào sau đây bị suy giảm nhiều nhất?
A. Carbohydrate
B. Chất béo
C. Protein
 C. Giảm tiết acid dịch vị nghĩa là dạ dày mất khả năng bài tiết acid và được
chẩn đoán khi pH của dịch vị dưới 4 sau khi đã kích thích bằng pentagastrin. Khi
acid không được bài tiết, pepsin cũng thường không được bài tiết; sở dĩ sự thiếu
acid dịch vị ngăn cản sự bài tiết pepsin là do pepsin hoạt động trong môi trường
acid. Do vậy, sự tiêu hoá protein bị suy giảm.
TMP12 800
6. Tiền enzym pepsionogen được tiết ra chủ yếu ở cấu trúc nào?
A. Các tế bào của nang tuỵ
B. Các tế bào của ống tuyến tuỵ
C. Các tế bào biểu mô của ruột non
D. Các tuyến môn vị của dạ dày
 D. Pepsinogen là tiền chất của enzym pepsin. Pepsinogen được bài tiết bởi các
tế bào chính của tuyến tiêu hoá (còn gọi là tuyến sinh acid). Để chuyển từ dạng tiền
chất sang dạng hoạt động (pepsin), pepsinogen phải tiếp xúc với HCl hoặc pepsin
được tạo ra trước đó. Pepsin là một enzym phân giải protein tiêu hoá collagen và
các loại mô liên kết khác trong thịt.
TMP12 777
7. So với huyết tương, nước bọt có nồng độ ion (nồng độ tương đối) nào cao
nhất ở điều kiện thông thường?
A. HCO3-
B. Cl-
C. K+
D. Na+
 C. Trong điều kiện cơ bản, nước bọt có nồng độ ion Kali và bicarbonate cao,
nồng độ ion Natri và Clo thấp. Nước bọt đầu tiên được bài tiết bởi nang tuyết có
thành phần ion tương tự như huyết tương. Khi nước bọt chảy qua các ống tuyến,
ion Natri được tái hấp thu chủ động và ion Kali được bài tiết chủ động để đổi chỗ
cho Natri. Khi ion Natri được hấp thụ với số lượng lớn, ion Clo sẽ di chuyển theo
bậc thang điện thế gây giảm đáng kể nồng độ ion Clo trong nước bọt. Ion
bicarbonate được bài tiết nhờ quá trình vận chuyển chủ động làm tăng nồng độ ion
bicarbonate trong nước bọt. Kết quả thực tế của quá trình này là, ở điều kiện cơ
bản, nồng độ ion Natri và Clo trong nước bọt bằng khoảng 10% và 15% so với
nồng độ trong huyết tương, nồng độ bicarbonat cao gấp ba lần nồng độ trong huyết
tương và nồng độ Kali cao khoảng gấp 7 lần trong huyết tương.
TMP12 775
8. Ở điều kiện thông thường, ion nào sau đây có nồng độ cao nhất trong nước
bọt?
A. HCO3-
B. Cl-
C. K+
D. Na+
 A. Mặc dù nồng độ ion Kali trong nước bọt gấp khoảng 7 lần nồng độ trong
huyết tương, và nồng độ ion bicarbonate trong nước bọt chỉ cao gấp 3 lần trong
huyết tương, nhưng nồng độ thực của ion bicarbonate trong nước bọt là 50 đến 70
mEq/L, trong khi nồng độ ion Kali là khoảng 30 mEq/L ở điều kiện cơ bản.
TMP12 775
9. Một bệnh nhân nam 33 tuổi, vào viện vì đau ngực khi ăn, đặc biệt là khi ăn
thịt. Ông ta cũng có triệu chứng ợ hơi nhiều và ợ chua. Vợ bệnh nhân than
phiền về hơi thở hôi của chồng. Chụp X quang thấy hình ảnh thực quản giãn.
Hình nào sau đây biểu diễn sự thay đổi áp suất tại cơ thắt thực quản dưới
trước và sau khi bệnh nhân nuốt? Đường kẻ ngang tương ứng với áp suất 0
mm Hg.

 C. Co thắt thực quản là tình trạng cơ thắt thực quản dưới không thể giãn khi
nuốt. Kết quả là thức ăn được nuốt vào thực quản và không thể từ thực quản xuống
dạ dày. Đường C biểu diễn một áp suất dương, và không giảm sau khi nuốt, đây là
dấu hiệu của tình trạng co thắt thực quản. Đường A biểu diễn một đường áp suất
bình thường tại vị trí cơ thắt thực quản dưới, cụ thể thực quản có phản xạ giãn khi
tiếp xúc với lượng thức ăn lớn. Đường E tương tự đường C, nhưng áp suất dưới áp
suất khí quyển. Áp suất dưới áp suất khí quyển chỉ xảy ra trong thực quản khi nó đi
qua khoang ngực.
TMP12 799

10.Các mẫu sinh thiết được lấy ra từ niêm mạc hang vị và tá tràng của một phụ
nữ 65 tuổi. Hormone nào sau đây có thể tìm thấy trong dịch chiết mô lấy từ
2 vị trí trên?
A. Cholecystokinin (CCK)
B. Glucose - dependent insulinotropic peptide (GLIP)
C. Gastrin
D. Motilin
E. Secretin
 C. Hormone tiêu hoá được bài tiết từ các tế bào nội tiết nằm ở niêm mạc. Các tế
bào nội tiết không được tụ lại một chỗ mà nằm riêng rẽ giữa các tế bài biểu mô, do
đó không thể dùng các phương pháp phẫu thuật để loại bỏ các hormone tiêu hoá.
Gastrin là hormon duy nhất được tìm thấy ở hang vị, nhưng nó cũng được tìm thấy
trong tá tràng và một lượng nhỏ hơn trong biểu mô hỗng tràng. CCK và các tế bào
nội tiết bài tiết secretin được tìm thấy trong tá tràng, hỗng tràng và hồi tràng. Tế
bào bài tiết Motilin và GLIP được tìm thấy trong hỗng tràng và hồi tràng.
TMP12 758
11.Một bé trai 10 tuổi ăn một bánh hamburger có kèm pho mát, khoai tây chiên
và sữa lắc vị sô cô la. Bữa ăn kích thích sự bài tiết của một vài hormone tiêu
hoá. Sự có mặt của chất béo, carbohydrate hay protein trong tá tràng kích
thích biểu mô tá tràng giải phóng các hormon nào sau đây?
A. Cholecystokinin (CCK)
B. Glucose - dependent insulinotropic peptide (GLIP)
C. Gastrin
D. Motilin
E. Secretin
 B. GLIP là hormone tiêu hoá duy nhất được bài tiết bởi cả 3 loại thực phẩm
(chất béo, protein, và carbohydrate). Sự hiện diện của chất béo và protein trong
ruột non kích thích sự giải phóng CCK, nhưng carbohydrate không kích thích sự
giải phóng CCK. Sự hiện diện của protein trong hang vị dạ dày kích thích bài tiết
gastrin, nhưng chất béo và carbohydrate không kích thích sự giải phóng gastrin.
Chất béo kích thích rất ít sự bài tiết motilin và secretin, nhưng cả 2 hormone này
không được giải phóng khi có mặt protein và carbohydrate trong ống tiêu hoá.
TMP12 758
12.Hormone nào sau đây làm chậm quá trình thức ăn thoát khỏi dạ dày, và được
giải phóng khi có mặt chất béo và protein trong ruột non?
A. Cholecystokinin (CCK)
B. Glucose - dependent insulinotropic peptide (GLIP)
C. Gastrin
D. Motilin
E. Secretin
 A. CCK là hormone tiêu hoá duy nhất ức chế quá trình thức ăn thoát khỏi dạ
dày trong điều kiện sinh lý. Sự ức chế này giữ cho dạ dày chứa thức ăn trong một
thời gian dài, đó cũng là lý do vì sao bữa sáng chứa nhiều chất béo và protein sẽ
‘gắn với bạn’ lâu hơn những bữa sáng chỉ toàn carbohydrate. CCK cũng có tác
dụng trực tiếp lên trung tâm ăn ở não làm giảm việc ăn thêm. Mặc dù CCK là
hormone duy nhất ức chế quá trình thức ăn thoát khỏi dạ dày, tất cả các hormone
tiêu hoá, ngoại trừ gastrin, đều được bài tiết ở một mức độ nào đó bởi sự có mặt
của chất béo trong lòng ruột.
TMP 758
13.Một thí nghiệm lâm sàng được thực hiện trên 2 nhóm đối tượng: một nhóm
được tiêm tĩnh mạch 50g glucose và nhóm còn lại được cho uống 50g
glucose. Giải thích nào sau đây là đúng nhất cho hiện tượng lượng glucose
uống được thanh thải khỏi huyết tương nhanh hơn so với lượng glucose tiêm
tĩnh mạch? (CCK, cholecystokinin; GLIP, glucose-dependent insulinotropic
peptide; VIP, vasoactive intestinal peptide)
A. CCK gây tiết insulin
B. CCK gây tiết VIP
C. GLIP gây tiết glucagon
D. GLIP gây tiết insulin
E. VIP gây tiết GLIP
 D. GLIP được bài tiết bởi sự có mặt của chất béo, carbohydrate, hoặc protein
trong ống tiêu hoá. GLIP là một chất kích thích bài tiết insulin mạnh và điều này
giải thích cho hiện tượng lượng glucose uống bài tiết insulin nhiều hơn và được
chuyển hoá nhanh hơn so với một lượng tương đương glucose tiêm tĩnh mạch.
Glucose tiêm tĩnh mạch không kích thích sự giải phóng GLIP. Cả CCK và VIP đều
không kích thích bài tiết insulin. GLIP không kích thích bài tiết glucagon, và
glucagon có tác dụng đối ngược với insulin, điều này có nghĩa, GLIP sẽ làm giảm
tốc độ thanh thải glucose khỏi máu. VIP không kích thích bài tiết GLIP.
TMP12 758
14.Hormone nào sau đây có thể ức chế sự bài tiết acid dịch vị? (GLIP, glucose-
dependent insulinotropic peptide)
Som Secret GLI Enter Phản
atost in P ogast xạ
atin rone thần
kinh
A Khôn Không Có Khôn Có
g g
B Khôn Có Khô Khôn Khôn
g ng g g
C Khôn Có Khô Có Khôn
g ng g
D Khôn Không Khô Có Có
g ng
E Khôn Không Có Khôn Khôn
g g g
F Khôn Có Có Có Có
g
 F. Tất cả các yếu tố đó đều có thể ức chế bài tiết acid dịch vị trong điều kiện
sinh lý thông thường. Acid dịch vị kích thích sự giải phóng somatostatin (một
hormone tại chỗ), chất này có tác dụng trực tiếp lên các tế bào viền gây ức chế bài
tiết acid cũng như tác dụng gián tiếp thông qua sự ức chế bài tiết gastrin. Secretin
và GLIP ức chế bài tiết acid thông qua tác dụng trực tiếp lên tế bào viền cũng như
gián tiếp ức chế bài tiết gastrin. Enterogastrone là các chất không xác định, được
giải phóng ở tá tràng và hỗng tràng, có tác dụng ức chế trực tiếp sự bài tiết acid.
Khi acid hoặc các dung dịch ưu trương đi vào tá tràng, sẽ kéo theo sự giảm bài tiết
acid dịch vị do cơ chế điều hoà của hệ thần kinh.
15.Hormone tiêu hoá có tác dụng sinh lý ở nồng độ bình thường và tác dụng
dược lý ở nồng độ cao hơn ngưỡng bình thường. Tác dụng sinh lý trực tiếp
của các hormon lên sự bài tiết acid dịch vị là gì? (KTD: không tác dụng, KT:
kích thích, ƯC: ức chế)
Gastrin Secr Chol CLI Moti
etin ecys P lin
toki
nin
A KTD KT KT KT KT
D D
B KT ƯC KT ƯC KT
D D
C KT ƯC KT KT KT
D D D
D KT ƯC ƯC KT KT
E KT KT ƯC ƯC KT
D
B. Gastrin kích thích bài tiết acid dịch vị, trong khi secretin và GLIP ức
chế bài tiết acid dịch vị trong điều kiện sinh lý bình thường. Phân biệt tác
dụng sinh lý của các hormone tiêu hoá với các tác dụng dược lý là hết sức
quan trọng. Ví dụ, gastrin và cholecystokinin (CCK) có tác dụng giống nhau
lên chức năng tiêu hoá ở liều dược lý (liều cao), nhưng không có tác dụng
giống nhau ở nồng độ sinh lý bình thường. Mặt khác, GLIP and secretin có
nhiều tác dụng giống nhau ở liều dược lý, nhưng chỉ có một tác dụng chung
ở nồng độ sinh lý, đó là ức chế bài tiết acid dịch vị
16.Dịch vị bài tiết trong giai đoạn đầu chiếm 30% dịch vị toàn bữa ăn. Biện
pháp nào sau đây có thể ức chế toàn bộ sự bài tiết acid dịch vị ở giai đoạn
đầu?
A. Các antacid (VD: Rolaid)
B. Kháng thể kháng gastrin
C. Atropin
D. Thuốc ức chế receptor H2
E. Phẫu thuật cắt dây thần kinh phế vị
F. Phẫu thuật cắt dây thần kinh giao cảm
 E) Giai đoạn đầu của quá trình bài tiết dịch vị xảy ra trước khi thức ăn đi
vào dạ dày. Nhìn, ngửi, nhai, và nghĩ đến thức đều được nhận biết bởi não,
sau đó não sẽ truyền tín hiệu đến dạ dày để chuẩn bị cho bữa ăn. Do vậy, sự
kích thích ở giai đoạn đầu bao gồm các receptor cơ học trong miệng, các
receptor hoá học (ngửi, nếm), nghĩ đến thức ăn, và hạ đường máu. Do sự bài
tiết dịch vị trong giai đoạn đầu được điều hoà hoàn toàn bởi dây thần kinh
phế vị, nên phẫu thuật cắt dây X có thể loại bỏ sự đáp ứng. Antacids cân
bằng acid dịch vị, nhưng không ức chế bài tiết dịch vị. Một kháng thể kháng
gastrin sẽ làm giảm (nhưng không loại bỏ) sự bài tiết dịch vị ở giai đoạn đầu
do chất này không có tác dụng kích thích histamin và actetylcholine gây bài
tiết acid. Atropin sẽ làm giảm sự bài tiết dịch vị ở giai đoạn đầu bằng cách
ức chế các receptor acetylcholin trên tế bào viền; tuy nhiên, atropin không
loại bỏ kích thích acetylcholin gây bài tiết gastrin. Thuốc kháng histamin H2
sẽ làm giảm sự bài tiết dịch vị ở giai đoạn đầu, nhưng không loại bỏ nó.
17.Một bé trai mới sinh không đi ngoài phân su trong 24h đầu. Bé xuất hiện
triệu chứng chướng bụng và nôn. Các test đưa đến chẩn đoán bệnh
Hirschsprung. Hiện tượng bít tắc có thể được tìm thấy ở đoạn nào của ruột?
A. Đại tràng lên
B. Cơ thắt hồi manh tràng
C. Cơ thắt thực quản dưới
D. Môn vị
E. Đại tràng xigma
 E. Bệnh Hirschsprung được đặc trưng bởi sự mất bẩm sinh các tế bào
hạch trong đại tràng đoạn xa, dẫn tới sự tắc nghẽn thực thể tại đường tiêu
hoá. Hầu hết các ca bệnh Hirschsprung nên được nghĩ đến trên những trẻ sơ
sinh không đi ngoài phân su trong 24 đến 48 giờ đầu. Mặc dù phương pháp
chụp cản quang đại trực tràng là hữu ích cho chẩn đoán, nhưng sinh thiết
trực trạng vẫn là tiêu chuẩn chẩn đoán cơ bản. Aganglionosis (một bệnh thần
kinh) bắt đầu tại hậu môn, triệu chứng gần như luôn xuất hiện, và tiếp tục
lan rộng ra các đoạn khác nhau. Cả đám rối Auerbach và Meissner đều mất,
dẫn tới sự giảm nhu động và chức năng của ruột. Cơ chế chính xác của sự
tiến triển bệnh Hirschprung còn chưa được hiểu rõ.
18.Phức hợp vận động di chuyển xuất khoảng 90 phút/lần, giữa các bữa ăn và
được kích thích bởi hormone tiêu hoá, motilin. Sự vắng mặt của các phức
hợp vận động di chuyển sẽ làm tăng?
A. Hoạt động của tá tràng
B. Quá trình thức ăn thoát khỏi dạ dày
C. Vi khuẩn đường ruột
D. Sự co bóp khối
E. Sự nuốt
 C. Phức hợp vận động di chuyển là sự co bóp của các sóng nhu động, bắt đầu từ
dạ dày và di chuyển chậm dần theo hướng ngược lại, dọc theo toàn bộ chiều dài
ruột non xuống đại tràng. Bằng cách quét sạch những mẩu thức ăn thừa tại dạ dày
qua ruột non xuống đại tràng, phức hợp vận động di chuyển có vai trò đảm bảo ít
vi khuẩn có mặt trong đường tiêu hoá trên. Sự phát triển quá mức của vi khuẩn
trong ống tiêu hoá xảy ra khi có sự tăng sinh đáng kể vi khuẩn trong đường tiêu
hoá trên. Như vậy, rõ ràng khi mất các phức hợp vận động di chuyển sẽ làm giảm
hoạt động của tá tràng và quá trình thức ăn thoát khỏi dạ dày. Phức hợp vận động
di chuyển không có tác dụng trực tiếp lên co bóp khối và động tác nuốt.
TMP12 758–759
19.Hình ảnh nào sau đây mô tả chức năng bình thường của thực quản ở vị trị
giữa ngực trước và sau khi nuốt thức ăn (theo vị trí mũi tên chỉ). Đường kẻ
ngang tương ứng với áp suất là 0 mm Hg.
 C. Đường C biểu diễn một áp suất dưới áp suất khí quyển với một sóng dương
tính gây ra bởi sự lưu thông của lượng thức ăn lớn. Đường A không tương ứng với
bất cứ hoạt động bình thường nào trong thực quản. Đường B có thể biểu diễn hoạt
động của cơ thắt thực quản dưới trên bệnh nhân co thắt thực quản. Đường D mô tả
hoạt động bình thường của cơ thắt thực quản dưới. Đường E thể hiện đường áp lực
dương cơ bản, đường này không xảy ra khi thực quản đi qua khoang ngực.
TMP12 764
20.Quá trình thức ăn thoát khỏi dạ dày được điều hoà chặt chẽ nhằm đảm bảo
nhũ trấp được đẩy vào tá tràng với tốc độ phù hợp. Những thay đổi nào sau
đây làm tăng quá trình thức ăn thoát khỏi dạ dày dưới điều kiện sinh lý bình
thường của một người khoẻ mạnh?
Co bóp Co bóp Co thắt
nhu động phân đoạn cơ môn
vùng hang của ruột vị
non
A Giảm Giảm Giảm
B Giảm Tăng Giảm
C Tăng Giảm Giảm
D Tăng Giảm Tăng
E Tăng Tăng Tăng
 C. Quá trình thức ăn thoát khỏi dạ dày là sự phối hợp hoạt động của dạ dày,
môn vị, và ruột non. Điều kiện thuận lợi cho quá trình này bao gồm (a) tăng co bóp
của dạ dày phần gần, qua đó đẩy dịch vị xuống môn vị, (b) co bóp nhu động dạ dày
mạnh, giúp đẩy dịch vị xuống môn vị, (c) sự mở môn vị cho phép dịch vị đi vào tá
tràng, và (d) sự vắng mặt của co bóp phân đoạn trong ruột, có thể ngăn cản dịch vị
đi vào ruột.
TMP12 768
21.Kích thích hệ đối giao cảm làm tăng hoạt động của ống tiêu hoá và kích
thích giao cảm làm giảm hoạt động. Hệ thần kinh ruột (hệ thần kinh tại chỗ)
điều hoà hoạt động của ruột thông qua việc thay đổi quá trình nào sau đây?
A. Sự bài tiết gastrin
B. Tần số
C. Sự bài tiết secretin
D. Tần số sóng thấp
E. Tần số điện thế
 E. Điện thế màng của cơ trơn đường tiêu hoá có sự thay đổi nhịp nhàng, gọi là
các sóng chậm. Các sóng chậm được gây ra bởi sự thay đổi dẫn truyền Natri của
các tế bào tạo nhịp đặc biệt, gọi là tế bào kẽ của Cajal. Tần số phát nhịp của các tế
bào tạo nhịp hay nói cách khác là tần số sóng chậm là cố định (VD: Không thay
đổi) trong các đoạn khác nhau của ruột. Tần số sóng chậm trung bình khoảng 3
nhịp trên phút trong dạ dày, 12 nhịp trên phút trong tá tràng, 10 nhịp trên phút
trong hỗng tràng, và 8 nhịp trên phút trong hồi tràng. Khi một sóng chậm khử cực
hoàn toàn, nó kích thích các điện thế đỉnh (các điện thế hoạt động thực sự). Trong
ruột non, các sóng chậm không thể khởi phát sự co bóp cơ trơn khi không có các
điện thế đỉnh; tuy nhiên, bản thân các sóng chậm vẫn có thể khởi phát sự co bóp
của cơ trơn dạ dày. Số lượng các sóng đỉnh trên một sóng chậm sẽ tăng khi kích
thích đối giao cảm và giảm khi kích thích giao cảm. Do vậy, hệ thần kinh tự chủ
kiểm soát hoạt động của ruột bằng cách thay đổi tần số các điện thế đỉnh. Gastrin
và secretin đều không có ảnh hưởng đáng kể đến hoạt động của ruột ở nồng độ
sinh lý.
TMP12 755
22.Nuốt là một quá trình phức tạp bao liên quan đến sự truyền tín hiệu giữa hầu
và trung tâm nuốt ở thân não. Cấu trúc nào sau đây đóng vai trò quan trọng
quyết định một lượng thức ăn có đủ nhỏ để được nuốt?
A. Nắp thanh quản
B. Thanh quản
C. Cung khẩu cái hầu
D. Khẩu cái mềm
E. Cơ thắt thực quản trên
 C. Các nếp gấp của vòm họng ở hai bên được kéo vào giữa tạo thành một rãnh
dọc để thức ăn qua đó vào họng sau. Rãnh này có chức năng chọn lọc, cho phép
thức ăn đã được nhai kĩ đi qua nhưng ngăn cản những thức ăn có kích thước lớn
hơn đi qua. Khẩu cái mềm (họng mềm) bị kéo lên trên để đóng các lỗ mũi sau,
ngăn cản sự trào ngược thức ăn vào khoang mũi. Các dây thanh âm của thanh quản
nằm sát cạnh nhau trong quá trình nuốt, và thanh quản bị kéo lên trên và ra trước
bởi các cơ cổ. Nắp thanh quản bị đưa ra sau che kín thanh môn. Cơ thắt thực quản
trên giãn ra, cho phép thức ăn đi từ vùng hầu sau vào đoạn trên thực quản.
TMP12 764
23.Một phụ nữ 54 tuổi sau ăn một bữa ăn lành mạnh khoảng 20’ xuất hiện cảm
giác muốn đi ngoài ngay lập tức. Phản xạ nào sau đây dẫn đến cảm giác
muốn đi ngoài ngay lập tức khi dạ dày giãn ra?
A. Phản xạ tá tràng – đại tràng
B. Phản xạ dạ dày – ruột non
C. Phản xạ dạ dày – đại tràng
D. Phản xạ ruột – ruột
E. Phản xạ co trực tràng – giãn cơ thắt hậu môn
 C. Phản sự dạ dày – ruột xảy ra khi bụng chướng khí kích thích các co bóp khối
trong đại tràng. Tất cả các phản xạ ruột đều được đặt tên theo nguồn gốc giải phẫu
của chúng: tiền tố được đi kèm với tên đoạn ruột nơi xảy ra các phản xạ, ví dụ
phản xạ dạ dày – đại tràng bắt đầu từ dạ dày và kết thúc ở đại tràng. Phản xạ tá
tràng – đại tràng có chức năng tương tự phản xạ dạ dày – đại tràng. Khi tá tràng bị
giãn, tín hiệu thần kinh được truyền đến trực tràng, kích thích co bóp khối. Phản xạ
dạ dày – ruột non xảy ra khi tín hiệu xuất phát từ ruột ức chế hoạt động của dạ dày
và sự bài tiết dịch dạ dày. Phản xạ ruột – ruột xảy ra khi giãn quá mức hoặc tổn
thương ở một đoạn ruột truyền tín hiệu đến đoạn ruột khác, gây phản xạ giãn. Phản
xạ co trực tràng – giãn cơ thắt hậu môn, còn được gọi là phản xạ tống phân, được
khởi phát khi phân đi vào trực tràng và kích thích phản xạ tống phân ngay tức
khắc.
TMP12 757
24. Một phụ nữ 60 tuổi bị đứt tuỷ sống ngang mức đốt sống ngực thứ 6 (T6) trong
một tai nạn ô tô. Bà ấy có sử dụng một phương pháp làm giãn trực tràng nhằm kích
thích phản xạ tống phân. Vậy ở người phụ nữ này, việc giãn trực tràng sẽ dẫn đến
hiện tượng gì trong các hiện tượng sau:

Giãn cơ Co cơ Co thắt
thắt thắt trực
trong ngoài tràng
hậu hậu
môn môn
A Không Không Không
B Không Không Có
C Không Có Có
D Có Không Có
E Có Có Không
F Có Có Có
 D) Khi phân vào trực tràng, thành trực tràng bị căng ra, kích thích các tín hiệu
truyền vào đám rối Auerbach (myenteric plexus = Auerbach`s plexus) để khởi phát
các sóng nhu động trong đại tràng xuống, đại tràng sigma và trực tràng. Tất cả các
sóng nhu động này giúp đẩy phân về phía hậu môn. Cùng lúc đó, cơ thắt trong hậu
môn giãn cho phép phân đi qua. Ở những người bị đứt ngang tuỷ sống, các phản xạ
trên có thể dẫn đến tống phân ngoài ý muốn do bình thường cơ thắt ngoài được
điều khiển bởi ý thức qua các tín hiệu dẫn truyền trong tuỷ sống.
TMP12 771-772
25. Các hormone tiêu hoá vừa có tác dụng sinh lý ở nồng độ bình thường vừa có
tác dụng dược lý ở nồng độ cao. Vậy đâu là tác dụng sinh lý của các hormone này
lên sự thoát thức ăn khỏi dạ dày?

Gastri Secreti CC GLI Motili


n n K P n
A Giảm Giảm Giả Giả Tăng
m m
B Tăng Giảm Khô Giả Tăng
ng m
C Tăng Không Khô Tăng Tăng
ng
D Khôn Không Giả Tăng Tăng
g m
E Khôn Không Giả Khô Không
g m ng
F Khôn Không Tăn Khô Không
g g ng
 E) Cholesystokinin (CCK) là hormorn tiêu hoá duy nhất ức chế thoát thức ăn
khỏi dạ dày trong các trạng thái sinh lý. CCK ức chế thoát thức ăn khỏi dạ dày
bằng cách giãn vùng đáy vị và nửa trên thân vị (orad stomach), giúp làm tăng khả
năng co giãn (compliance) của dạ dày. Khi khả năng co giãn của dạ dày tăng, dạ
dày có thể giữ 1 lượng thể tích thức ăn lớn hơn mà không tăng quá mức áp lực
trong lòng ống. Không có hormone tiêu hoá nào làm tăng thoát thức ăn khỏi dạ dày
trong tình trạng sinh lý; tuy vậy, gastrin, secretin, và GLIP có thể ức chế thoát thức
ăn khỏi dạ dày khi dùng liều dược lý thử nghiệm.
TMP12 658
-year-old woman consumes a healthy meal. At which location are smooth muscle contractions
26. Một the có
phụ nữ 48intuổi
to have the highest frequency thói quen
diagrams ăn uống tốt cho sức khoẻ. Trong biểu đồ sau,
shown?
co cơ trơn ở vị trí nào có thể có tần số co cao nhất?

 A) Tần số của các co bóp nhu động không thay đổi ở các đoạn khác nhau của
ống tiêu hoá. Tần số co cơ trơn lớn nhất không thể vượt quá tần số co bóp nhu
ecystokinin (CCK)
động. and
Tần gastrin
số trungshare
bình multiple effectslàatkhoảng
của nhu động pharmacological
3 sóng/phútconcentrations.
ở dạ dày, 12 Which
wing effects sóng/phút
do CCK and ở tágastrin
tràng, share (or not share)
10 sóng/phút ở hỗngattràng,
physiological concentrations?
8 sóng/phút ở hồi tràng. Do đó,
trong trường hợp này, tá tràng là nơi có tần số co cơ trơn cao nhất.

ting is a complex process that requires coordination of numerous components by the vomiting
TMP12 754
27. CCK và gastrin có nhiều tác dụng giống nhau ở nồng độ dược lý. Đâu là tác
dụng giống nhau (hoặc không giống nhau) của CK và gastrin tại nồng độ sinh lý?

Kích ức chế Kích Kíc


thích thoát thích h
tiết dịch phát thíc
acid khỏi dạ triển h
dày niêm phát
mạc dạ triển
dày tuỵ
A Không Không Không Khô
ng
B Không Không Giống Khô
ng
C Không Giống Không Khô
ng
D Giống Giống Không Khô
ng
E Giống Giống Giống Giố
ng
 A) Gastrin và CCK không có bất kì tác dụng nào giống nhau lên chức năng tiêu
hoá ở tình trạng sinh lý bình thường; tuy nhiên, chúng có ảnh hưởng đồng nhất lên
chức năng tiêu hoá tại liều dược lý. Gastrin kích thích tiết acid dạ dày và phát triển
niêm mạc dạ dày và ruột non trong tình trạng sinh lý. CCK kích thích phát triển
tuyến tuỵ ngoại tiết và ức chế thoát thức ăn khỏi dạ dày trong tình trạng sinh lý.
CCK cũng kích thích co túi mật, giãn cơ vòng Oddi và kích thích tuỵ ngoại tiết bài
xuất bicarbonate và các enzyme.
TMP12 758
28. Nôn là một quá trình phức tạp cần tới sự phối hợp của rất nhiều cấu phần dưới
sự điều khiển của trung tâm nôn ở hành não. Hiện tượng nào trong các hiện tượng
sau xuất hiện khi nôn?

Cơ thắt Cơ thắt Các cơ Cơ


thực thực bụng hoành
quản quản
dưới trên
A Co Co Co Co
B Co Co Giãn Không
C Giãn Co Co Không
D Giãn Giãn Co Co
E Giãn Giãn Giãn Giãn
 D) Nôn được khởi phát bởi các sóng phản nhu động, bắt đầu từ các đoạn xa ống
tiêu hoá như hồi tràng. Giãn phần trên ống tiêu hoá (đặc biệt là tá tràng) là yếu tố
kích thích khởi phát nôn thực sự. Khi bắt đầu nôn, dạ dày và tá tràng co thắt mạnh,
kèm theo giãn một phần cơ thắt thực quản dưới. Sau đó, nôn đặc hiệu tiếp tục diễn
tiến và liên quan tới (a) thở sâu, (b) giãn cơ vòng thực quản trên, (c) đóng năp
thanh môn, và (d) co thắt mạnh các cơ bụng và cơ hoành.
TMP12 803
29. Nhiều enzyme tiêu protein được bài tiết ở dạng không hoạt động vào trong lòng
ruột. Chất nào trong các chất sau có vai trò quan trọng trong hoạt hoá một hay
nhiều enzyme tiêu protein?

Trypsin Enterokinase Pepsin


A Không Không Không
B Không Không Có
C Không Có Không
D Có Có Không
E Có Có Có
 E) Tất cả các enzyme tiêu protein thiết yếu được bài tiết ở dạng không hoạt
động, nhằm ngăn không cho chúng tự tiêu hoá cơ quan bài tiết. Enterokinase là
được gắn tự nhiên vào riềm bàn chải của các tế bào ruột nằm ở mặt trong của ruột
non. Enterokinase hoạt hoá trypsinogen thành trypsin ở trong lòng ruột. Trypsin
sau đó xúc tác ngược lại trypsinogen thành trypsin, đồng thời xúc tác cho một số
proenzyme khác (ví dụ: chymotrypsinogen, procarboxypeptidase, proelastase,…).
Pepsin đầu tiên được tiết ra dưới dạng pepsinogen, nó không có tác dụng tiêu
protein. Tuy nhiên, ngay khi nó tiếp xúc với HCl và đặc biệt tiếp xúc với dạng
pepsin kết hợp HCl, nó sẽ được hoạt hoá thành pepsin.
TMP12 778, 781
30. Co bóp khối đóng vai trò quan trọng trong động tác đại tiện. Co bóp khối gây
nên hiện tượng nào trong các hiện tượng sau?

A) Co cơ thắt hậu môn trong


B) Nhu động ruột non
C) Trào ngược dạ dày
D) Cảm giác đói
E) Giãn trực tràng.
 E) Hầu hết thời gian, trực tràng không có phân. Khi co bóp khối đẩy phân vào
trực tràng, cảm giác buồn đại tiện xuất hiện. Phản xạ co của trực tràng và giãn cơ
thắt hậu môn trong diễn ra sau đó. Nếu một người ở nơi có thể đại tiện được, cơ
thắt hậu môn ngoài sẽ giãn ra theo ý muốn và phân được bài xuất. Rõ ràng là, co
bóp khối không gây nên nhu động tá tràng, trào ngược dạ dày hay cảm giác đói
TMP12 770-771
31. Một phụ nữ 82 tuổi có tiền sử dùng thuốc NSAIDs để điều trị viêm khớp, giờ
bị đau bụng trên và có máu trong phân. Nội soi phát hiện viêm dạ dày thể mảng
(patchy gastritis). Kết quả sinh thiết âm tính với Helicobacter pylori. Tiêm tĩnh
mạch Pentagastrin sẽ dẫn tới tăng ít hơn bình thường hiện tượng nào sau đây?
A) Phát triển niêm mạc tá tràng
B) Bài tiết acid dạ dày
C) Bài tiết gastrin
D) Bài tiết enzyme tuỵ
E) Phát triển tuỵ
 B) Sự dụng thuốc chống viêm không steroid (NSAIDs) có thể dẫn tới viêm dạ
dày do NSAIDs hay loét dạ dày tá tràng. Viêm dạ dày mạn tính, theo định nghĩa, là
tổn thương thực thể trên mô bệnh học được đặc trưng bởi tình trạng viêm mạn tính
niêm mạc dạ dày. Khi viêm tác động lên thân vị, các tế bào viền bị ức chế, dẫn tới
giảm tiết acid. Mặc dù chẩn đoán viêm dạ dày mạn tính chỉ được xác định trên mô
bệnh học, nhưng việc tiêm pentagastrin có thể làm tiết acid dịch vị thấp hơn bình
thường. Pentagastrin là gastrin tổng hợp, cấu tạo bởi 4 amino acids tận cùng trong
gastrin tự nhiên và amino acid alanine. Nó có tác dụng sinh lý tương tự gastrin tự
nhiên. Mặc dù gastrin và pentagastrin đều có thể kích thích phát triển niêm mạc tá
tràng, nhưng trong các thử nghiệm lâm sàng cho thấy pentagastrin dùng đường tĩnh
mạch sẽ không gây phát triển niêm mạc thật sự. Trong bất kì trường hợp nào, việc
dùng kéo dài pentagastrin cũng không dẫn tới giảm phát triển niêm mạc tá tràng.
Pentagastrin không làm tăng tiết gastrin, không tăng tiết enzyme tuỵ hay phát triển
tuỵ.
TMP12 800
32. Trong các hiện tượng sau, hiện tượng nào có thể là hệ quả của việc cắt hồi
tràng?

A) Co thắt tâm vị
B) Viêm dạ dày teo
C) Táo bón
D) Loét dạ dày – tá tràng
E) Thiếu vitamin B12
 E) Để hấp thu được vitamin B12 cần có yếu tố nội, đây là một glycoprotein
được tế bào viền ở dạ dày bài tiết ra. Yếu tố nội gắn vào vitamin B12 là điều kiện
cần thiết cho việc gắn vào receptor đặc hiệu ở riềm bàn chải đoạn hồi tràng. Do đó,
cắt hồi tràng có thể dẫn tới thiếu hụt vitamin B12. Co thắt tâm vị là tình trạng suy
giảm thần kinh cơ chi phối cho giãn phần dưới thực quản, hậu quả là thực quản bị
giãn tiến triển, uốn khúc và rối loạn nhu động. Viêm dạ dày teo (atrophic gastritis)
là một thể của viêm dạ dày tự miễn, chủ yếu dẫn tới giảm bài tiết acid ở thân vị.
Viêm dạ dày này có tính chất lan toả và cuối cùng dẫn tới teo đét nặng. Cắt hồi
tràng có thể gây nên tiêu chảy nhưng không dẫn tới táo bón. Các tổn thương loét dạ
dày và loét tá tràng lành tính được xếp chung thành nhóm loét dạ dày – tá tràng,
mặc dù nguyên nhân của chúng có khác nhau. Trong cả hai thể loét này, acid và
pepsin là các nguyên nhân của tổn thương niêm mạc. Loét tá tràng thường phổ biến
hơn.
TMP12 778
33. Trong các yếu tố sau, yếu tố nào có vai trò sinh lý trong kích thích giải phóng
hormone hoặc kích thích phản xạ thần kinh ức chế bài tiết acid tại dạ dày?

Acid Acid Dung Dung


béo dịch dịnh
ưu đẳng
trương trương
A Không Không Có Không
B Không Không Có Có
C Có Có Không Có
D Có Có Có Có
E Có Có Có Không
 E) Sự xuất hiện của acid, acid béo và dung dịch ưu trương ở tá tràng và hỗng
tràng dẫn tới ức chế tiết acid qua nhiều cơ chế khác nhau. Acid kích thích ruột non
bài tiết secretin, là chất ức chế tế bào viền bài tiết acid. Sự acid hoá hang vị và
vùng tuyến sinh acid (oxyntic gland) của dạ dày kích thích giải phóng
somatostatin, là chất ức chế bài tiết acid bằng cách tác động trực tiếp lên tế bào
viền và tác động gián tiếp qua trung gian ức chế bài tiết gastrin. Sự có mặt của acid
béo trong ruột non kích thích giải phóng GLIP (glucose-dependent insulinotropic
peptide), cũng là chất ức chế tiết acid qua con đường trực tiếp (ức chế tế bào viền)
và gián tiếp (qua giảm tiết gastrin). Dung dịch ưu trương trong ruột non làm giải
phóng enterogastrone (chưa rõ nguồn gốc), là chất ức chế trực tiếp tế bào viền tiết
acid. Dung dịch đẳng trương không có tác dụng lên quá trình bài tiết acid.

TMP12 758, 780


34. Một sinh viên Y 23 tuổi ăn một bánh hamburger có kèm pho mát, khoai tây
chiên và sữa lắc vị sô cô la. Hormone nào sẽ gây nên tác động sinh lý tại thời điểm
một vài tiếng sau đó?

Gastrin Secretin CCK GLIP


A Không Có Có Có
B Có Không Có Có
C Có Có Không Có
D Có Có Có Có
E Có Có Có Có
 E) Tất cả các hormone tiêu hoá được giải phóng sau bữa ăn và đều có tác dụng
sinh lý/
TMP12 758
35. Một phụ nữ 68 tuổi xuất hiện nôn ra máu, ợ chua và đau dạ dày. Nội soi phát
hiện viêm vùng thân và hang vị cũng như có một tổn thương loét nhỏ. Sinh thiết
cho kết quả dương tính với Helicobacter pylori. H. pylori gây tổn thương niêm mạc
dạ dày bằng cách tăng nồng đồ chất nào trong số các chất sau?
A) Amoni
B) Muối mật
C) Gastrin
D) NSAIDs
E) Pepsin
 A) H.pylori là một vi khuẩn gặp ở 95% bệnh nhân có loét tá tràng và gần như
100% ở bệnh nhân có loét dạ dày khi đã loại trừ việc sử dụng kéo dài aspirin hoặc
các NSAIDs khác. H. pylori được đặc trưng bởi hoạt tính urease cao, giúp chuyển
hoá urea thành NH3 (ammonia). Ammonia phản ứng với H+ tạo ra NH4+. Phản
ứng này cho phép vi khuẩn chịu được trong môi trường acid của dạ dày. Việc tạo
ra NH4+ được cho là nguyên nhân chủ yếu của tổn thương gây độc tế bào do
NH4+ làm tổn thương trực tiếp tế bào biểu mô, tăng tính thấm của hàng rào niêm
mạc dạ dày. Muối mật và NSAIDs cũng có thể tổn thương hàng rào niêm mạc dạ
dày nhưng không liên quan trực tiếp với nhiễm H. pylori. Pepsin có thể làm nặng
thêm tổn thương niêm mạc gây ra bởi nhiễm H. pylori, nhưng H. pylori không làm
tăng nồng độ pepsin. Rõ ràng là gastrin không phải chất trung gian làm tổn thương
niêm mạc do H. pylori.

TMP12 801
36. Một người đàn ông 71 tuổi xuất hiện nôn ra máu, đi ngoài phân đen, và có vết
loét tiến triển ở tá tràng. Rửa trôi cục máu đông thì thấy mạch máu nổi lên ở bên
dưới. Người ta dùng đầu dò cắt đốt điện lưỡng cực để cắt đốt mạch máu thành
công. Yếu tố nào trong các yếu tố đóng vai trò chẩn đoán loét tá tràng?
Nội soi Nồng độ Tốc độ
gastrin bài tiết
huyết acid
tương
A Không Không Không
B Có Không Không
C Có Không Có
D Có Có Không
E Có Có Có
 B) Cả nồng độ gastrin huyết tương và tốc độ tiết acid dạ dày đều không đóng
vai trò chẩn đoán loét tá tràng. Tuy vậy, khi gộp chung các bệnh nhân loét tá tràng
lại, về mặt thống kê ta thấy có sự tăng đáng kể tốc độ bài tiết acid và giảm nồng độ
gastrin huyết tương. Vì sao có hiện tượng này? Giới hạn bình thường của tốc độ
bài tiết acid nền là 1-5 mEq/giờ và tốc độ bài tiết acid tối đa là 6-40mEq/giờ. Ở
bệnh nhân loét dạ dày tá tràng, tốc độ bài tiết acid nền (2-10 mEq/giờ) và tốc độ
bài tiết acid tối đa (30-80mEq/giờ) cũng trùng lặp tới gía trị trên. Sự tăng bài tiết
acid ở bệnh nhân loét tá tràng bình thường làm ức chế bài tiết gastrin từ hang vị.
Nên rõ ràng là nội soi là yếu tố chẩn đoán của loét tá tràng.
TMP12 801.
Câu 37 và 38
Biểu đồ ghi lại áp kế của một bệnh nhân trước và sau khi tập luyện cơ sàn hố chậu.
Một quả bóng được đặt vào trong trực tràng, sẽ được bơm hơi (tại vị trí mũi tên)
s 37 and 38
wing diagram shows manometric recordings from a patient before and after pelvic floor training.
n placed in the rectum was blown up (arrows) and deflated repeatedly. Tracing Z is a
rồi xả hơi,
ric recording obtained cứ thế
from the lặp đi lặpanal
external lại nhiều Đườngand
lần. before
sphincter Z biểu
afterdiễn floorđotraining.
áp lực
pelvic ở cơ thắt
ngoài hậu môn trước và sau tập luyện cơ sàn hố chậu.

37. Đâu là vị trí đo phù hợp nhất với đường biểu diễn X ở trên?
A) Trực
ich of the following tràng
best đoạn xathe origin of tracing X shown in the upper panel?
describes
Distal rectum
B) Van hồi manh tràng
Ileocecal valve
Internal anal C) Cơ thắt hậu môn trong
sphincter
Lower esophageal sphincter
D) Cơ thắt thực quản dưới
Proximal rectum
E) Trực tràng đoạn gần
ich of the following best describes the condition for which the patient received pelvic floor
 C) Cơ thắt trong hậu môn giãn khi trực tràng bị kéo căng, biểu thị bằng các
Anal fissure đường giảm
(i.e., tear or áp sau mỗi lần bơm căng bóng trực tràng. Áp lực ở đoạn xa
lực theo laceration)
superficial

Chronic diarrhea đoạn gần trực tràng sẽ tăng sau khi bơm bóng. Trong khi đó, áp lực ở cơ thắt
quản dưới
Fecal incontinence (i.e., no control overmanh
thực và van hồi tràng không bị ảnh hưởng gì.
defecation)
Hemorrhoids
Hirschsprung disease
TMP12 771-772
linical study is38.
conducted
Đâu là lýindowhich gastric
phù hợp nhấtacid secretion
khiến BN cầnisluyện
stimulated
tập cơusing pentagastrin
sàn đáy chậu? before
treatment with a histamine H2 blocker. Which of the following rates of gastric acid secretion (in
s most likely A) Lỗ dòoccurred
to have hậu mônin(ví
thisdụ, vết rách sâu hoặc bề mặt)
experiment?
B) Tiêu chảy mạn tính
C) Đại tiện không tự chủ
D) Trĩ
E) Bệnh Hirschsprung
 C) Trước khi tập luyện cơ sàn hố chậu, áp lực ở cơ thắt hậu môn ngoài không
thay đổi qua các lần bơm bóng. Chính sự suy giảm khả năng co của cơ thắt ngoài
hậu môn dẫn tới việc tống phân ra ngoài. Sau tập luyện, cơ thắt ngoài hậu môn co
được khi bơm bóng, việc này sẽ ngăn không cho tống phân tự phát.
TMP12 771-772
39. Trong một nghiên cứu lâm sàng, người ta tiến hành đo sự bài tiết acid tại dạ
dày được kích thích bằng pentagastrin trước và sau điều trị với ứng chế histamine
H2. Trong các kết quả sau, kết quả nào phù hợp nhất với nghiên cứu trên?

Pentagastrin alone Pentagastrin + H2 blocker


A) 15 15
B) 25 25
C) 25 15
D) 26 28
E) 40 45
 C) Các chất tăng bài tiết khác nhau, gồm acetylcholine, gastrin và histamine, có
tác dụng hiệp đồng lên quá trình bài tiết acid tại dạ dày. Điều này có nghĩa là,
40. A tsunami tidal wave hits the east coast of South America and the people living there are forced to
histamine
drinklàm tăngwater.
unclean tác dụng củathe
Within gastrin và acetylcholine
next several days, a largevànumber ứcpeople
thuốc of chế H2develop
cũng severe diarrhea and
làm giảm tác dụng của cả gastrin và acetylcholine. Tương tự, acetylcholine làm
about half of these people expire. Samples of drinking water are positive for Vibrio cholerae. Which of
tăng tác
thedụng của gastrin
following types ofvà
ionhistamine
channels isvàmost
atropine
likelylàm
to begiảm tác dụng
irreversibly của chúng.
opened in the epithelial cells of the
Do đó,crypts
trongofthí nghiệm được
Lieberkühn tả ở trên,
môpeople
in these withbài acid được kích thích bởi
tiết diarrhea?
severe
pentagastrin A)
sẽ Calcium do tác dụng của thuôc ức chế H2, bởi tác dụng hiệp động
bị giảm channels
tăng
của các chất B) bài tiếtchannels
Chloride như đã trình bày ở trên.
C) Magnesium channels
TMP12 D)780-781
Potassium channels
40. Một cơn E)
sóng thần channels
Sodium đánh vào bờ biển phía đông của miền Nam Hoa Kì và người
dân ở đây đã bị uống nước bẩn. Trong vài ngày sau đó, một số lượng lớn người
dân xuất41. Oneỉaofchảy
hiện the following hormones
nặng và một can stimulate
nửa trong số đó đãgrowth of thenước
chết. Mẫu intestinal
đó cómucosa
dươngand two other
hormones can stimulate pancreatic growth. Which three hormones are these?
tính với Vibrio cholera. Trên tế bào biểu mô của tuyến Lieberkuhn ở những người
bị ỉa chảy nặng này, kênh ion nào bị mở không hồi phục?
A) Kênh calci
B) Kênh clo
C) Kênh Magie
D) Kênh Kali
E) Kênh Natri
 B) Độc tố vi khuẩn tả gây nên tăng không hồi phục nồng độ cAMP trong
tế bào ruột, dẫn tới mở không hồi phục kênh clo trên bề mặt hướng vào lòng
42. Which of the following structures undergoes receptive relaxation when a bolus of food is
swallowed?
A) Orad stomach
28
45

sunami tidal wave hits the east coast of South America and the people living there are forced to
clean water. Within the next several days, a large number of people develop severe diarrhea and
lf of these people expire. Samples of drinking water are positive for Vibrio cholerae. Which of
wing types of ion channels is most likely to be irreversibly opened in the epithelial cells of the
Lieberkühn in these people with severe diarrhea?
) Calcium channels
) Chloride channels
) Magnesium channels
) Potassium channelsống tiêu hoá. Sự mất clo vào lòng ruột kéo theo natri vào theo qua con
) Sodium channels đường lách giữa các tế bào ruột. Nước bị kéo vào theo gradient nồng độ cao
ở lòng ống tiêu hoá, dẫn tới hậu quả tiêu chảy nặng.
e of the following hormones can stimulate growth of the intestinal mucosa and two other
41. Một trong số các hormones sau có thể kích thích phát triển niêm mạc ruột và
s can stimulate pancreatic growth. Which three hormones are these?
hai hormone còn lại có thể kích thích phát triển tuỵ. Đâu là ba hormone nói trên?
(đáp án C và D giống nhau)

 E) Một trong những tác dụng quan trọng nhất của hormone tiêu hoá là tác dụng
kích thích phát triển. Gastrin có thể kích thích phát triển niêm mạc toàn bộ ống tiêu
hoá cũng
hich of the following như tuyến
structures undergoes receptive
tuỵ ngoại relaxation
tiết. Nếu tất cả when a bolus
gastrin of food
nội sinh is do cắt hang vị,
bị mất
ed? thì ống tiêu hoá sẽ bị teo. Nguồn gastrin ngoại sinh sẽ giúp ngăn chặn quá trình teo
) Orad stomach ống tiêu hoá này. Cắt một phần ruột non do khối u, béo bệnh hay các nguyên nhân
khác, dẫn tới phì đại phần niêm mạc còn lại. Cơ chế của đáp ứng này tới nay chưa
rõ. Cả CCK và secretin đều kích thích phát triển tuyến tuỵ ngoại tiết. GLIP và
motilin không có tác dụng kích thích phát triển lên ống tiêu hoá.
TMP12 758
42. Cấu trúc nào tạo nên hiện tượng giãn tiếp nhận khi ta nuốt khối thức ăn?
A) Dạ dày phần gần
B) Cung khẩu cái hầu
C) Hầu
D) Thực quản đoạn ngực
E) Cơ thắt thực quản trên
 A) Trong khi nuốt, phần gần của dạ dày và cơ thắt thực quản dưới giãn ra cùng
lúc. Áp lực trong lòng ống ở cả 2 phần giảm trước khi khối thức ăn tới. Hiện tượng
này được goi là giãn tiếp nhận. Do dạ dày phần gần giãn ra mỗi khi nuốt, nên dạ
dày có thể tiếp nhận một lượng thức ăn lớn mà áp lực trong dạ dày chỉ tăng vài
mmHg. Giãn tiếp nhận được điều hoà bởi các đường dẫn truyền hướng tâm và li
tâm của dây X. Giãn tiếp nhận và khả năng giãn của dạ dày bị mất khi cắt dây X.
Cung khẩu cái hầu có vai trò quan trọng trong việc xác định liệu thức ăn có đủ nhỏ
để nuốt được hay không. Hầu và thực quản đoạn ngực tạo ra co bóp nhu động
trong quá trình nuốt, nhưng không tạo ra giãn tiếp nhận. Cơ thắt thực quản trên mở
khi nuốt nhưng không liên quản tới giãn tiếp nhận.
TMP12 765
43. Một người đàn ông 65 tuổi ăn uống tốt cho sức khoẻ. Khoảng 40 phút sau khi
cơ thắt hồi manh tràng giãn ra, chyme đi vào manh tràng. Phản xạ nào trong các
phản xạ sau khiến giãn dạ dày kéo theo giãn cơ thắt hồi manh tràng?
A) Phản xạ ruột – dạ dày
B) Phản xạ dạ dày – hồi tràng
C) Phản xạ dạ dày – đại tràng
D) Phản xạ ruột-ruột
E) Phản xạ co trực tràng – giãn cơ thắt hậu môn
 B) Sự giãn của cơ thắt hồi manh tràng xảy ra cùng lúc hoặc một thời gian ngắn
sau khi ăn. Phản xạ này có tên là phản xạ dạ dày – hồi tràng. Người ta chưa rõ liệu
phản xạ này được điều hoà bởi các hormone tiêu hoá (gastrin, CCK) hay do dây
thần kinh tự chủ từ bên ngoài tới ruột. Chú ý rằng, phản xạ dạ dày – hồi tràng được
đặt tên theo nguồn gốc của phản xạ đặt trước (gastro = dạ dày). Đây là cách đặt tên
chung cho tất cả các phản xạ của hệ tiêu hoá. Phản xạ ruột – dạ dày liên quan tới
các tín hiệu từ đại tràng và ruột non ức chế vận động và bài tiết của dạ dày. Phản
xạ dạ dày – đại tràng tác động làm đại tràng co bóp khi dạ dày dãn ra. Phản xạ ruột
ruột làm các đoạn ruột giãn ra khi nó bị kéo căng quá mức. Phản xạ co trực tràng -
giãn cơ thắt hậu môn còn được goi là phản xạ tống phân.
TMP12 769, 772
44. Một phụ nữ 21 tuổi, khoẻ mạnh, ăn một bữa ăn thịnh soạn, và sau đó bắt một
chuyến xe dài 3 giờ mà không có phòng vệ sinh. 20 phút sau ăn, cô ta cảm giác
buồn đại tiện, nhưng vẫn cố giữ không ra. Điều gì đã xảy ra ở người phụ nữ này?
Giãn cơ thắt Co cơ thắt Co trực
trong hậu ngoài hậu tràng
môn môn
A Không Không Không
B Không Có Có
B) Palatopharyngeal folds
C) Pharynx
D) Thoracic esophagus
E) Upper esophageal sphincter

43. A 65-year-old man eats a healthy meal. Approximately 40 min later the ileocecal sphincter relaxes
and chyme moves into the cecum. Gastric distention leads to relaxation of the ileocecal sphincter by way
of which reflex?
A) Enterogastric reflex
B) Gastroileal reflex
C) Gastrocolic reflex
D) Intestino-intestinal reflex
C E) Rectosphincteric
Có reflex
Không Có
44.DA healthy,
Có 21-year-old woman
Không eats a big Không
meal and then takes a 3-hr ride on a bus that does not have
a bathroom. Twenty minutes after eating, the woman feels a strong urge to defecate, but manages to hold
it. Which
E ofCó the following haveCó
occurred in thisCó
woman?

 E) Phản xạ tống phân (còn được gọi là phản xạ co trực tràng - giãn cơ thắt hậu
môn) xảy ra khi co bóp khối đẩy phân vào trực tràng. Khi trực tràng bị giãn ra, cơ
thắt hậu môn trong giãn và trực tràng co đẩy khối phân xuống hậu môn. Cơ thắt
hậu môn ngoài điều khiển theo ý muốn, có thể co lại trong hoàn cảnh không thể đại
tiện được. Do đó, khi một người cảm thấy buồn đai tiện, cơ thắt hậu môn trong
giãn ra, trực tràng co thắt và cơ thắt hậu môn ngoài có thể co hoặc giãn tuỳ theo
từng hoàn cảnh.
TMP12 771-772
45. The gastric mucosal barrier has a physiological and an anatomical basis to prevent back-leak of
45. Hàng
hydrogen ràotheniêm
ions into mạc
mucosa. dày cóarecơknown
dạ factors
Some sở sinh lý và giải
to strengthen phẫu ngăn
the integrity of thekhông cho ion
gastric mucosal
hydrogen
barrier, whereas khuếch táncan
other factors ngược
weakentrởthelạibarrier.
niêmWhich
mạc.ofMột tố đã biết
vài yếu factors
the following có tác
strengthen or dụng
tăngthecường
weaken barrier?tính toàn vẹn của hàng rào niêm mạc dạ dày, trong khi một số yếu tố
khác lại làm suy yếu. Đâu là yếu tố tăng cường hay làm yếu hàng rào?

 D) Tổn thương hàng rào niêm mạc dạ dày cho phép ion hydrogen khuếch tán
46.ngược lại niêm
A 48-year-old manmạc bằngacách
consumes healthytrao đổi
meal. At với natri.
which of thepH thấp ởlocations
following niêm mạc làm B12
is vitamin cho tế
mostbào mast
likely to berò rỉ histamine, gây tổn thương mạch máu dẫn tới tổn thương nhồi máu.
absorbed?
Nhồi máu niêm mạc lại làm nhiều ion hydrogen khuếch tán vào hơn, càng nhiều tế
bào bị tổn thương và chết, tạo thành một vòng luẩn quẩn. Các yếu tố tăng cường
hàng rào niêm mạc dạ dày bao gồm chất nhầy (ngăn cản khuếch tán ion hydrogen),
gastrin (kích thích phát triển niêm mạc), một số prostaglandins (kích thích bài tiết
chất nhầy) và các yếu tố phát triển khác có thể kích thích phát triển mạch máu,
niêm mạc dạ dày, và các mô khác. Các yếu tố làm suy yếu hàng rào niêm mạc dạ
dày gồm Helicobacter pylori (vi khuẩn gây độc qua ammonium), và aspirin,
NSAIDs, ethanol và muối mật.
TMP12 779-780
46. Một người đàn ông 48 tuổi có chế độ ăn hợp lý. Đâu là nơi vitamin B12 được
hấp thu?
 D) Vitamin B12 được hấp thu tại hồi tràng
TMP12 778
ar-old man presents with abdominal pain and hematemesis. An abdominal exam was
47. Một
gn, and abdominal người
x-rays were đàn ông 45oftuổi
suggestive a perforated đau bụng
biểu hiệnviscus. và nônrevealed
Endoscopy ra máu.a Khám bụng
chưa phát hiện bất thường, chụp XQ bụng gợi ý thủng tạng
rforated gastric ulcer, through which the liver was visible. Which of the followingrỗng.
is a Nội soi phát hiện
vết loét
gastric ulcer formation? dạ dày thủng mạn tính, qua đó nhìn thấy được gan. Đâu là yếu tố khởi phát
tạo thành
k-leak of hydrogen ions loét dạ dày?
us secretion
on pump inhibitionA) Khuêch tán ngược của ion hydrogen
ht junctions betweenB)cells
Bài tiết chất nhầy
otomy C) Ức chế bơm proton
D) Liên kết chặt chẽ giữa các tế bào
E) Cắt dây X
ar-old man is fed intravenously for several weeks following a severe automobile accident.
us feeding leadstoA)
atrophy of the gastrointestinal
Ion hydrogen mucosa
rò rì vào niêm mạcmost
khilikely
niêm because
mạc bị the thương. Vì ion
tổnblood
hydrogen tích tụ trong niêm mạc, nên các hệ đệm nội bào bị bão hoà, và pH tế bào
of the following hormones is reduced?
lecystokinin only
giảm dẫn tới tổn thương và chết tế bào. Ion hydrogen cũng tổn thương tế bào mast,
trin only
khiến chúng bài tiết histamine. Histamine làm nặng hơn tình trạng bệnh bằng cách
retin only
tổn thương vi mạch trong niêm mạc. Hậu quả là nhồi máu cục bộ, giảm ôxy và
trin and cholecystokinin
sung huyết. Tổn thương niêm mạc là yếu tố khởi phát của loét dạ dày. Bài tiết chất
trin and secretin
nhầy giúp tăng cường hàng rào niêm mạc dạ dày do chất nhầy ngăn chặn sự rò rỉ
etin and cholecystokinin
ion hydrogen vào niêm mạc. Các thuốc ức chế bơm proton được sử dụng như một
ar-old man withphương
dyspepsia andđiều
thức a history of dạ
trị loét chronic gastric
dày do chúngulcer
cóhas
thểabdominal
làm giảmpain.
bài tiết ion hydrogen
ows a large ulcer in the proximal gastric body. Biopsies were positive
(protons). Liên kết chặt chẽ giữa các tế bào trong lớp Helicobacter
for niêm mạc ngăn sự khuếch tán
of the following are used clinically for treatment of gastric ulcers of various etiologies?
ngược của ion hydrogen. Cắt dây X đã từng được sử dụng để điều trị loét dạ dày do
cắt dây X làm giảm bài tiết acid dạ dày.
TMP12 779-800
48. Một người đàn ông 19 tuổi được cho ăn qua đường tĩnh mạch trong vài tuần
này sau một tai nạn ô tô nghiêm trọng. Cho ăn qua đường tĩnh mạch dẫn tới teo
niêm mạc dạ dày. Giảm nồng độ hormone nào trong máu dẫn tới hiện tượng trên?
A) Chỉ do CCK
B) Chỉ do gastrin
C) Chỉ do secretin
D) Gastrin và CCK
E) Gastrin và secretin
F) Secretin và CCK
 B) Gastrin có vai trò quan trọng trong kích thích phát triển niêm mạc toàn bộ
ống tiêu hoá
TMP12 758
49. Một người đàn ông 62 tuổi có tiền sử loét dạ dày mạn tính, nay xuất hiện đau
bụng, khó tiêu. Nội soi phát hiện vết loét lơn ở thân vị phần gần. Sinh thiết dương
tính với Helicobacter pylori. Phương pháp nào được sử dụng trên lâm sàng để điều
trị loét dạ dày do các nguyên nhân khác nhau?
Kháng NSAI Ức Ức
sinh Ds chế chế
H2 bơm
proton
A Không Không Có Có
B Có Không Không Có
C Có Không Có Có
D Có Có Có Có
E Không Có Có Có
 C) Mục tiêu của điều trị nội khoa loét dạ dày là phục hồi sự cân bằng giữ quá
trình bài tiết acid và các yếu tố bảo vệ niêm mạc, Ức chế bơm proton là thuốc có
liên kết đồng hoá trị và ức chế không hồi phục bơm H+/K+?ATPase, ức chế hiệu
quả bài tiết acid. Liệu pháp hướng tới giải phóng histamine là thuốc ức chế H2,
như cimetidine (Tagamet), ranitidine (Zantac), famotidine (Pepcid) và nizatidine
(Axid). Các thuốc này ức chế chọn lọc receptor H2 ở tế bào viền. Liệu pháp kháng
sinh được sử dụng để diệt H. pylori. NSAIDs có thể gây tổn thương hàng rao niêm
mạc dạ dày, là yếu tố khởi phát của loét dạ dày
TMP12 801
50. Tiêu hoá thức ăn dẫn tới tăng bài tiết một lượng lón acid dạ dày, mà nó đạt
đỉnh sau khoảng 5 phút và trở về giá trị bình thường sau ăn khoảng 4 giờ. Bao lâu
sau ăn, pH của dạ dày đạt nồng độ thấp nhất (đơn vị là giờ)
A) 1.0
B) 1.5
C) 2.0
D) 2.5
E) 3.0
F) 4.0
 F) Một sự nhầm lẫn phổ biến là sau bữa ăn pH dạ dày thấp nhất (tức acid nhất)
khi sự bài tiết acid là cao nhất. Trước bữa ăn, khi dạ dày rỗng, pH của dạ dày thấp
nhất và sự bài tiết acid bị ức chế. Bài tiết acid ít do (a) acid kích thích giải phóng
somatostatin (nó tác động trực tiếp làm giảm tiết cả gastrin và acid), (b) acid tác
động trực tiếp lên tế bào viền làm giảm tiết acid. Trong bữa ăn, tác dụng đệm của
thức ăn làm pH dạ dày tăng lên, cho phép các chất tăng bài tiết kích thích tiết acid
TMP12 779.
51. Bệnh xơ hoá nang (cystic fibrosis) là rối loạn di truyền của các tuyến ngoại tiết,
ảnh hưởng trên trẻ em và người trẻ tuổi. Chất nhầy ở các tuyến ngoại tiết trở nên
dày và dích, cuối cùng làm tắc ống dẫn của các tuyến này (đặc biệt là tuỵ, phổi và
gan), tạo thành các nang. Ngừng tiên phát vận chuyển ion nào qua màng xảy ra
trong bệnh xơ hoá nang mà dẫn tới giảm bài tiết dịch?
A) Calci
B) Clo
C) Phosphate
D) Kali
E) Natri
 B) Sự vận chuyển clo ra ngoài tế bào kéo theo sự bài tiết dịch của tế bào. Bệnh
xơ hoá nang gây nên bởi sự bất thường vận chuyển ion clo qua mặt đỉnh của tế bào
biểu mô trong mô tuyến ngoại tiết. Protein điều hoà vận chuyển qua màng trong
bệnh xơ hoá nang (CFTR) hoạt động vừa như một kênh clo được điều hoà bởi
cAMP vừa như tên gọi của nó, điều hoà các kênh ion khác. Dạng phát triển đầy đủ
của CFTR được tìm thấy trong màng bào tương của tế bào biểu mô bình thường.
Sự vắng mặt của CFTR tại vị trí thích hợp thường góp phần tạo nên bệnh sinh của
bệnh xơ hoá nang. Tuy vậy, các đột biến khác trong gen của bệnh xơ hoá nang tạo
ra protein CFTR ở dạng phát triển đầy đủ tại vị trí thích hợp nhưng chỉ hoạt động
một phần chức năng hoặc không có chức năng
TMP12 775, 787, 796
52. Cặp điều hoà – kích thích nào sau đây bình thường ứng chế giải phóng gastrin?
Kích thích Điều hoà
A Acid CCK
B Acid GLIP
C Acid Somatostatin
D Acid béo Motilin
E Acid béo Somatostatin
 C) Acid tác động trực tiếp lên tế bào somatostatin kích thích giải phóng
somatostatin. Somatostatin làm giảm bài tiết acid bằng ức chế trực triếp té bào viền
bài tiết acid và gián tiếp qua ức chế bài tiết gastrin của tế bào G ở hang vị. Acid là
tác nhân kích thích yếu bài xuất CCK, nhưng CCK không ức chế (hoặc kích thích)
tiết gastrin. Acid không kích thích giải phóng GLIP. Acid béo kích thích yếu
motilin, nhưng motilin không tác động lên bài xuất gastrin. Acid béo không được
cho là yếu tố kích thích giải phóng somatostatin.
TMP12 780
53. Một trẻ sơ sinh nam xuất hiện chướng bụng, chưa ỉa phân xu trong 48 giờ đầu
và nôn liên tục. Phân tích kết quả sinh thiết trực tràng cho chẩn đoán xác định bệnh
Hirschsprung. Sự vắng mặt tế bào nào là tiêu chuẩn chẩn đoán bệnh Hirschsprung?
A) Tế bào nội mô mạch bạch huyết
B) Tế bào nội mô mạch máu
C) Tế bào hạch phó giao cảm
D) Tế bào hồng cầu
E) Tế bào cơ trơn
 C) Bệnh Hirschsprung (Bệnh giãn đại tràng bẩm sinh do thiếu tế bào hạch) là
kết quả của sự vắng mặt tế bào hạch phó giao cảm trong đám rối Auerbach và
Meissner của trực tràng và/hoặc đại tràng. Bệnh này bắt đầu ở hậu môn, là nơi luôn
biểu hiện, và tiếp tục tiến vào trong các đoạn gần của ống tiêu hoá. Vắng mặt cả
đám rối Auerbach và Meissner, dẫn tới giảm nhu động và chức năng của ruột. Cơ
chế chính xác của bệnh sinh bệnh Hirschsprung chưa rõ. Nhưng rõ ràng là sự vắng
mặt tế bào nội mô bạch huyết, tế bào nội mô mạch máu hay hồng cầu không ảnh
hưởng tới nhu động đại tràng. Tế bào cơ trơn vẫn có trong bệnh Hirschsprung.
TMP12 802
54. Co bóp khối thường được kích thích sau bữa ăn bởi sự giãn của dạ dày (phản
xạ dạ dày – đại tràng) và giãn của tá tràng (phản xạ tá tràng – đại tràng). Co bóp
khối thường dẫn tới hậu quả nào?
A) Đại tiện
B) Vận động của dạ dày
C) Túi phình đại tràng
D) Co thắt thực quản
E) Nhu động hầu họng.
 A) Co bóp khối tống phân vào trực tràng. Khi thành trực tràng dãn ra, phản xạ
tống phân được kích thích và đại tiện diễn ra khi hoàn cảnh phù hợp. Co bóp khối
không ảnh hưởng tới vận động của dạ dày. Túi phình đại tràng là các khối ở đại
tràng do sự co của các lớp cơ vòng và cơ dọc cạnh nhau. Rõ ràng là co bóp khối ở
đại tràng không ảnh hưởng tới co thắt thực quản hay nhu động của hầu họng
TMP12 770-771
55. Một người đàn ông 45 tuổi ăn rất nhiều thực phẩm giàu chất xơ và có nguồn
gốc từ lúa mỳ và cám để giảm lượng cholesterol. Ông ta đã giảm được 30 pao nhờ
chế độ ăn mới đó nhưng tác dụng không mong muốn của nó là đau bụng, đầy hơi
và tiêu chảy. Bác sĩ tiêu hoá đã chẩn đoán ông ta mắc hội chứng celiac (gluten-
enteropathy hay celiac sprue). Yếu tố nào bị giảm ở bệnh nhân này?
A) Hấp thu dinh dưỡng
B) Tiêu hoá chất béo
C) Carbohydrates trong phân
D) Chất béo trong phân
E) Nitrogen trong phân
 A) Bệnh celiac là bệnh mạn tính đường tiêu hoá liên quan tới chức năng hấp thu
dinh dưỡng từ thức ăn. Tổn thương niêm mạc thấy được trên sinh thiết đường tiêu
hoá trên chính là hậu quả của một bất thường đáp ứng miễn dịch qua trung gian tế
bào với gliadin (là một thành phần của gluten tìm thấy trong lúa mì) được xác định
bằng di truyền học. Đáp ứng tương tự xảy ra với các protein trong đại mạch và lúa
mạch đen. Gluten không có trong yến mạch, gạo và ngô. Một người bị bệnh ăn
gluten, niêm mạc của ruột non bị tổn thương do đáp ứng viêm qua trung gian miễn
dịch, dẫn tới rối loạn chức năng tiêu hoá và hấp thu của riềm bàn chải. Tiêu hoá
chất béo bình thường ở bệnh celiac do tuyến tuỵ bài tiết lipase bình thường. Do
bệnh celiac gây rối loạn hấp thu nên rõ ràng các thành phần carbohydrate, chất béo
và nitrogen tăng trong phân.
TMP12 801
56. Một người đàn ông 57 tuổi được nhập viện cấp cứu do xuất huyết tiêu hoá cao.
Nội soi phát hiện nhiều ổ loét ở tá tràng. Nồng độ gastrin huyết thanh cao gấp 8 lần
bình thường. Nghi ngờ hôi chứng Zollinger-Ellison (ZES, gastrinoma). Sử dụng
chất nào trong số các chất sau giúp khẳng định chẩn đoán?
A) Cholecystokinin (CCK)
B) Glucose-dependent insulinotropic peptide (GLIP)
C) Motilin
D) Pentagastrin
E) Secretin
 E) ZES xuất hiện do u bài tiết gastrin, nằm ở tuyến tuỵ, thành tá tràng hay hạch
lympho. Test tin cậy và đơn giản nhất xác định ZES là tiêm secretin. Secretin ức
chế gastrin bài tiết ở hang vị, nhưng kích thích bài tiết gastrin ở bệnh nhân bị ZES.
Tiêm tĩnh mạch 2 đơn vị secretin/ kg thể trọng. Nồng độ gastrin huyết thanh được
đo vào các khoảng thời gian mỗi 30 phút sau tiêm. Tăng nồng độ gastrin trên
200ng/mL được chẩn đoán là ZES. Cơ chế sinh lý của test secretin chưa rõ, tuy
nhiên, nó là một test chẩn đoán quan trọng để loại trừ các tình trạng khác cũng liên
quan tới tăng bài tiết acid. CCK, GLIP, motilin và pentagastrin có tác dụng không
đáng kể lên bài tiết gastrin và không có giá trị chẩn đoán ZES.
TMP12 758
57. Một người đàn ông 71 tuổi bị đau bụng trên và có máu trong phân, có uống
NSAIDs kèm với rượu whiskey để giảm đau. Sử dụng pentagastrin làm giảm bài
tiết acid dạ dày. Ở bênh nhân viêm dạ dày, sự bài tiết chất nào bị giảm sút?
A) yếu tố nội
B) Ptyalin
C) Rennin
D) Nước bọt
E) Trypsin
. A) Yếu tố nội là một glycoprotein được tế bào viền bài xuất. Nó cần thiết cho
sự hấp thu vitamin B12. Ở bệnh nhân viêm dạ dày mạn, khả năng bài tiết acid bị
giảm. Vì acid và yếu tố nội đều được tế bào viên bài tiết, nên sự giảm khả năng bài
A 71-year-old man with upper abdominal pain and blood in the stool takes NSAIDS for the pain and
s it down withtiết acid thường
whiskey. đi kèm
Pentagastrin với giảm khả
administration năng tiết
produced yếu
lower tố predicted
than nội. Ptyalin, một
levels of amylase do
tuyến nước bọt bài xuất, là một enzyme khởi phát quá trình tiêu hoá carbohydrate
c acid secretion. Secretion of which of the following substances is most likely to be diminished in ở
miệng.
tient with gastritis? Viêm dạ dày không ảnh hưởng tới sự bài tiết ptyalin. Rennin, còn gọi là
chymosin, là một enzyme tiêu protein được tổng hợp bởi tế bào chính ở dạ dày.
A) Intrinsic factor
B) Ptyalin Vai trò của nó trong tiêu hoá là làm đông vón sữa ở dạ dày, một quá trình vô cùng
C) Rennin quan trọng ở những động vật còn non. Rõ ràng là bài tiết nước bọt không bị ảnh
D) Saliva hưởng bởi viêm dạ dày. Trypsin là một enzyme tiêu protein được bài tiết bởi tuyến
E) Trypsin tuỵ

Gastric acid is secreted when a778


TMP12 meal is consumed. Which of the following factors have a direct
on the parietal58.
cellAcid
to stimulate
dạ dàyacid
đượcsecretion?
bải tiết khi ăn. Yếu tố nào tác động trực tiếp lên tế bào viền
kích thích bài tiết acid?

 C) Gastrin, acetylcholine và histamine có thể kích thích trực tiếp tế bào viền bài
tiết acid. Ba chất kích thích bài tiết này cũng có tác động hiệp đồng lên bài tiết
An 84-year-oldacid,
man with
ví dụ: ức chế mộtand
hematemesis melena
chất is diagnosed
sẽ làm giảm tác with
dụnga của
duodenal
2 chấtulcer.
còn Alại.patient
Acetylcholine
cũng có tác dụng gián tiếp làm tăng bài tiết acid bằng cách kích thích tế bào G tiết
osed with a duodenal ulcer is likely to exhibit which of the following?
gastrin. Somatostatin ức chế bài tiết acid
TMP12 758, 778, 780

The gastric phase of gastric secretion accounts for about 60% of the acid response to a meal. Which
following can virtually eliminate the secretion of acid during the gastric phase?
A) Antiacids (e.g., Rolaids)
B) Antigastrin antibodies
C) Atropine
D) Histamine H2 blocker
E) Proton pump inhibitor
59. Một người đàn ông 84 tuổi có nôn ra máu và đi ngoài phân đen, được chẩn
đoán là loét tá tràng. Bệnh nhân được chẩn đoán loét tá tràng thì có khả năng biểu
hiện như thế nào?
Mật độ tế bào Bài tiết Gastrin
viền acid huyết
tương
A Giảm Giảm Giảm
B Giảm Tăng Giảm
C Tăng Giảm Tăng
D Tăng Tăng Giảm
E Tăng Tăng Tăng
 D) Bệnh nhân loét tá tràng có khoảng 2 tỉ tế bào viền và có thể bài tiết khoảng
40 mEq H+/ giờ. Ở người bình thường các chỉ số trên chỉ bằng môt nửa. Nồng độ
gastrin huyết tương ngược với khả năng bài tiết acid do cơ chế feedback, nhờ đó sự
acid hoá hang vị ức chế bài xuất gastrin. Vậy, nòng độ gastrin huyết tương giảm ở
bệnh nhân loét tá tràng. Sự bài tiết acid tối đa và nồng độ gastrin huyết tương
không phải là tiêu chí chẩn đoán loét tá tràng do có sự trùng lặp lớn với những
người bình thường trong mỗi nhóm.
TMP12 800-801
60. Pha dạ dày của sự bài tiết của dạ dày chiếm khoảng 60% lượng acid tiết ra
trong bữa ăn. Yếu tố nào trong các yếu tố sau có thể cắt giảm gần như toàn bộ sự
bài tiết acid trong pha dạ dày?
A) Antiacid (ví dụ: Rolaids)
B) Kháng thể kháng gastrin
C) Atropine
D) Ức chế histamine H2
E) Ưc chế bơm proton
 E) Ức chế bơm proton như omeprazole ức chế toàn bộ bài tiết acid bằng cách ức
chế bơm H+,K+ - ATPase (bơm H+). Tế bào viềm có các receptors cho các chất
kích thích bài tiết như gastrin, acetylcholine và histamine. Do đó, kháng thể kháng
gastrin, atropine và ức chế histamine H2 có thể giảm bài tiết acid nhưng không có
chất nào có thể giảm hoàn toàn bài tiết acid. Antacids khi vào dạ dày giúp trung
hoà acid dạ dày, nhưng nó không thể ức chế bài tiết acid từ tế bào viền
TMP12 779,801
61. Một người đàn ông 53 tuổi có tiền sử bệnh loét có liên quan đến ỉa chảy tái
diễn và tiền sử gia đình bị loét tá tràng bị nghi ngờ mắc hội chứng Zollinger-
Ellison (u tế bào tiết gastrin). Secretin (2 đơn vị/kg) tiêm tĩnh mạch nhanh đã được
dùng để đánh giá u tế bào tiết gastrin. Kết quả nào dưới đây chỉ ra sự tồn tại của u Commented [LTN49]: /xác định

tế bào tiết gastrin khi sử dụng secretin?


A) Giảm gastrin huyết thanh
B) Tăng gastrin huyết thanh
C) Ức chế sự bài tiết acid dịch vị
D) Ức chế quá trình làm rỗng dạ dày
E) Kích thích tiết HCO3- của dịch tụy Commented [LTN50]: [ ]

B) Secretin ức chế giải phóng gastrin ở hang vị dạ dày nhưng nó lại kích thích
tiết gastrin tại các tế bào gastrinoma. Như vậy, bệnh nhân có tế bào gastrinoma sẽ
tăng nồng độ gastrin huyết thanh trong 30 phút sau khi dùng secretin; trong khi
secretin làm giảm nồng độ gastrin ở người bình thường. Tiêm secretin là các đơn
giản và đáng tin cậy nhất trong việc đánh giá tình trạng u tế bào tiết gastrin; tuy
nhiên cơ chế của test này còn chưa được hiểu rõ. Secretin bình thường ức chế quá
trình tiết chế acid nhưng có thể làm tăng tiết acid ở bệnh nhân bị u tế bào tiết
gastrin vì có sự tăng tiết gastrin xảy ra. Secretin có thể ức chế quá trình làm rỗng
của dạ dày nhưng nó không giúp chẩn đoán u tế bào tiết gastrin. Secretin có tác
dụng sinh lý bình thường là kích thích tụy tiết HCO3-, điều này độc lập với tình
trạng u tế bào tiết gastrin
62. Sự tổn hại của hàng rào niêm mạc dạ dày là yếu tố tiên phát dẫn tới loét dạ dày.
Tác nhân nào vừa có thể gây tổn hại cho hàng rào niêm mạc dạ dày vừa kích thích
sự bài tiết acid dịch vị?
A) Muối mật
B) Yếu tố phát triển biểu mô
C) Gastrin
D) Helicobacter pylori
E) Chất nhầy
D) Việc tìm ra H. pylori và mối liên quan của chúng với bệnh loét dạ dày, ung
thư biểu mô tuyến, u lympho dạ dày và các bệnh khác đã biến nó trở thành một
trong những phát kiến vĩ đại nhất về y học của thế kỉ. Ở Mỹ, có khoảng 26 triệu
người sẽ phải chịu đựng loét trong suốt cuộc đời còn lại của họ và hơn 90% là do
Hp. Hp là một vi khuẩn gram (-) có enzyme urease hoạt động mạnh xúc tác sự tạo
thành ammonia từ urea. NH3 chuyển thành NH4+ trong môi trường acid của dạ
dày. NH4+ gây nguy hại cho hàng rào niêm mạc dạ dày vì nó làm hại các tế bào
biểu mô. Hp cũng làm tăng cường chế tiết acid dịch vị, có thể bằng tăng số lượng
tế bào thành. Sự kết hợp này làm tăng acid cùng với tổn hại hàng rào niêm mạc dạ
dày thúc đẩy hình thành loét dạ dày. Muối mật có thể làm tổn thương hàng rào
niêm mạc nhưng chúng không gây ảnh hưởng lâm sang lớn tới sự tiết acid. Yếu tố
phát triển biểu mô, gastrin, chất nhầy làm vững bền hàng rào niêm mạc
63. Quá trình làm rỗng dạ dày được điều hòa nhằm đảm bảo nhũ chấp xuống tá
tràng theo một tỷ lệ thích hợp. Yếu tố nào thúc đẩy quá trình làm rỗng dạ dày?
A) Chứng biếng ăn
B) Nhu động hang vị
C) Chứng cuồng ăn
D) Béo phì
E) Xơ cứng bì
F) Đái tháo đường type 1
B) Nhu động hang vị đẩy vị chấp qua môn vị và góp phần làm rỗng dạ dày. Các
nhân tố khác thúc đẩy quá trình làm rỗng dạ dày bao gồm:
a. Giảm tính dãn của dạ dày dưới áp lực
b. Giãn vùng môn vị
c. Sự vắng mặt của các co bóp phân đoạn của ruột non.
Quá trình làm rỗng của dạ dày bị chậm lại trong một số rối loạn về ăn như chứng
biếng ăn, chứng cuồng ăn, béo phì. Xơ cứng bì là bệnh hệ thống ảnh hưởng tới rất
nhiều hệ cơ quan. Triệu chứng của bệnh là hệ quả của sự xơ hóa tiến triển của mô
và sự tắc nghẽn của các bõng xuất bào do tăng sản xuất và lắng đọng collagen type
1,3. Sự lắng đọng của mô xơ ở mộn vị làm giảm quá trình làm trống dạ dày. Liệt
dạ dày xảy ra ở khoảng 20% đái tháo đường type 1. Nồng độ đường cao được cho
là làm tổn hại đến dây X và qua đó làm giảm quá trình làm rỗng dạ dày
64. Bệnh xơ nang là một trong những nguyên nhân phổ biến dẫn tới viêm tụy ở trẻ
em. Điều nào sau đây giải thích hợp lý nhất cơ chế của bệnh xơ nang tụy?
A) Sự hoạt hóa enterokinase
B) Sự hoạt hóa chất ức chế trypsin
C) Sự tự tiêu của tụy
D) Sự bài tiết quá mức CCK
E) Sỏi mật bít
 C) Viêm tụy là tổn thương viêm ở tụy. Tụy tiết các enzyme tiêu hóa và lòng
ruột non, chúng có vai trò quan trọng trong tiêu hóa chất béo, protein,
carbonhydrate. Sự giảm tiết dịch của ống tụy trong bệnh xơ nang dẫn tới sự tích tụ
của các enzyme tiêu hóa trong ống. Các enzyme này sau đó được hoạt hóa trong
ống tụy và bắt đầu tiêu hóa tụy dẫn tới quá trình viêm và vô số các vấn đề khác (u
nang và chảy máu trong). Enterokinase định khu ở diềm bàn chải của tế bào ruột
non bình thường hoạt hóa Trypsin từ trypsinogen. Chất ức chế trypsin thường có ở
ống tụy nhằm ngăn sự hoạt hóa trypsin và qua đó ngăn ngừa sự tự tiêu hóa của tụy.
Khi các ống đã bị bít bởi nang xơ, lượng chất ức chế trypsin không đủ để ngăn
ngừa hoạt hóa trypsin. Sự tiết quá muwacs CCK không xảy ra trong bệnh xơ nang.
Sỏi mật bít có thể dẫn tới viêm tụy (bởi tự tiêu hóa) khi mà sự bít tắc ngăn cản dịch
tụy xuống ruột non nhưng nó không liên quan đến bệnh xơ nang.
65. Sự kích thích của dây X đóng một vai trò quan trọng trong quá trình tiết dịch vị
ở giai đoạn đầu và giai đoạn dạ dày. Sự kích thích của dây X ảnh hưởng đến sự
giải phóng gastric-releasing peptide (GRP) và somatostatin như thế nào?

GRP Somatostatin

A) ↑ ↑
B) ↓ ↓
C) ↑ ↓
D) ↓ ↑
E) ↔ ↓
C) Giai đoạn đầu trong quá trình bài tiết dịch vị được điều chỉnh thông qua dây
X: đáp ứng với loại bỏ dây X. Dây X cũng tham gia vào nhiều phần của giai đoạn
tiết dịch vị ở dạ dày. Sự kích thích dây X làm tăng tiết acid bằng cách trực tiếp
hoạt hóa tế bào thành cũng như khích thích tiết gastrin. Kích thích dây X gây
a. tăng tiết gastrin bằng kích thích trực tiếp tế bào G
b. ức chế tế bào tiết somatostatin tiết somatostatin điều này cũng ức chế tế bào G
tiết gastrin.
Gastrin releasing peptide (GRP) là chất dẫn truyền thần kinh được giải phóng để
kích thích tế bào G tiết gastrin
66. Việc kiểm soát sự bài tiết acid dịch vị trong bữa ăn bao gồm một vài sự kiện
xảy ra trong khoảng 4 tiếng đến 5 tiếng dọc theo bữa ăn. Những sự kiện này bao
gồm
(1) Giảm pH dạ dày,
(2) Tăng bài tiết acid,
(3) Giảm bài tiết acid,
(4) Tăng pH dạ dày.
Trình tự thời gian của các sự kiện xảy ra trong khoảng 4 tiếng đến 5 tiếng dọc bữa
ăn:
A) 4, 3, 2, 1
B) 3, 1, 4, 2
C) 3, 4, 1, 2
D) 2, 1, 4, 3
E) 4, 2, 1, 3
F) 1, 2, 3, 4
G) 2, 3, 1, 4
H) 1, 3, 2, 4
 E) Dọc theo bữa ăn, pH của dạ dày tăng bởi thức ăn có vai trò đệm acid của dạ
dày. Tăng pH ức chế sự giải phóng somatostatin từ tế bào delta của dạ dày (H+
kích thích giải phóng somatostatin). Vì somatostatin ức chế sự tiết của cả gastrin
và acid dịch vị nên sự giảm nồng độ somatostatin dẫn tới tăng tiết acid. Sự tăng tiết
acid dẫn tới pH trong dạ dày giảm. Khi pH trong dạ dày giảm, sự tiết acid cũng
giảm.
67. Một phụ nữ béo 43 tuổi có tiền sử sỏi mật vào khoa cấp cứu vì đau bụng dữ dội Commented [LTN51]: bệnh nhân nữ 43 tuổi béo phì

một phần tư trên bên phải. Người phụ nữ này có biểu hiện vàng da, trên X quang Commented [LTN52]: Bệnh nhân 1
có hình ảnh tắc ống mật chủ. Giá trị của bilirubin trực tiếp và gián tiếp trong huyết
tương (tính theo mg/dl) ứng với trường hợp của người phụ nữ này là:
Trực Gián tiếp
tiếp

A) 1.0 1.3

B) 2.3 2.4

C) 5.0 1.7

D) 1.8 6.4

E) 6.8 7.5
 C) Khoảng 20% những người lớn hơn 65 tuổi ở Mỹ có sỏi mật (sỏi
cholelithiasis) và 1 triêu ca mắc mới và được báo cáo mỗi năm. Sỏi mật là một
trong những nguyên nhân phổ biến gây bít tắc đường mật. Bất kể nguyên nhân của
sỏi mật, bilirubin huyết thanh (đặc biệt là loại trực tiếp hay liên hợp) thường tăng.
Bilirubin gián tiếp hay không liên hợp thường bình thường hoặc chỉ tăng nhẹ. Chỉ
có đáp án C cho thấy nồng độ cao bilirubin trực tiếp (bilirubin liên hợp) so với
nồng độ bilirubin gián tiếp (bilirubin không liên hợp).

Unit 13: Trao đổi chất

1. Giáng hóa acid béo tại ty thể tạo ra hợp chất 2 carbon nào dưới đây?
A) Acetyl coenzyme A
B) Carnitine
C) Glycerol
D) Glycerol 3-phosphate
E) Oxaloacetic acid
 A) Acid béo được giáng hóa trong ty thể qua quá trình giải phóng phân tử hai
carbon để hình thành acetyl coenzyme A. Đó được gọi là chu trình oxy hóa beta
trong quá trình giáng hóa các acid béo.
TMP12 822
2. Điểm nhiệt sinh lý điều hòa bởi vùng dưới đồi thông thường trung bình khoảng
98.6°F (37oC). Các yếu tố nào sau đây có thể làm thay đổi điểm nhiệt sinh lý do
điều hòa nhiệt trung tâm?

 E) Pyrogens (các tác nhân gây sốt) được giải phóng từ các vi khuẩn có hại hoặc
từ các mô thoái hóa trong cơ thể có thể làm tăng điểm nhiệt sinh lý được điều hòa
tự động ở vùng dưới đồi. Các thuốc hạ sốt như
aspirin, ibuprofen, and Tylenol được gọi là các thuốc chống sốt, chống nhiệt.
Những thuốc này có thể làm giảm thân nhiệt và hạ sốt. Thyroxin có thể làm tăng
tốc độ chuyển hóa và do đó làm tăng tốc độ sinh nhiệt của cơ thể, nhưng thyroxin
không làm thay đổi điểm nhiệt sinh lý của cơ thể điều hòa bởi vùng dưới đồi. Giảm
nhiệt độ da làm tăng điểm nhiệt, và điểm nhiệt sẽ giảm khi da nóng.
TMP12 875–876
3. Bệnh nhân nam 54 tuổi sau khi tiêu thụ một bữa ăn có thành phần chất béo rất
lớn. Muối mật được hấp thu nhờ quá trình vận chuyển tích cực ở vị trí nào trên ống
tiêu hóa trong hình dưới đây?
 D) Khoảng 95% muối mật được tái hấp thu ở ruột non, khoảng một nửa quá
trình này xảy ra theo cơ chế khuếch tán qua niêm mạch ở tá tràng, và phần còn lại
xảy ra theo cơ chế vậu chuyển tích cực qua niêm mạch ruột ở đoạn cuối hồi tràng.
Muối mật từ đó đi vào máu khoảng cửa và đi qua gan. Sự tái tuần hoàn muối mật
được gọi là chu trình gan ruột.
TMP12 829
4. Bệnh nhân nữ 9 tuổi có tiền sử 30 năm nghiện rượu và bệnh gan. Bệnh nhân đến
khám do chướng bụng. Nguyên nhân nào dưới đây có khả năng cao nhất gây ra cổ
trướng ở bệnh nhân?
A) Tăng áp lực động mạch gan
B) Tăng áp lực tĩnh mạch gan
C) Tăng áp suất thủy tĩnh dịch màng bụng
D) Tăng nồng độ albumin huyết tương
E) Tăng áp lực tĩnh mạch cửa
 E) Khi các tế bào nhu mô gan bị phá hủy, chúng được thay thế bằng các mô xơ
sợi, thậm chí có thể thâm nhiễm xung quanh mạch máu, do đó làm cản trở nghiêm
trọng dòng chảy của máu khoảng cửa qua gan. Tăng sức cản mạch máu dẫn tới
tăng áp lực tĩnh mạch cửa, từ đó làm tăng áp lực mao mạch nội tạng, gây ra rò rỉ số
lượng dịch lớn ra màng bụng.
TMP12 838
5. Giai đoạn đầu trong quá trình sử dụng triglycerides để sinh năng lượng là thủy
phân triglycerides ra sản phẩm nào dưới đây?
A) Acetyl coenzyme A và glycerol
B) Cholesterol và acids béo
C) Glycerol 3-phosphate và cholesterol
D) Glycerol và acid béo
E) Phospholipids và glycerol
 D) Triglycerides được thủy phân tạo glycerol và fatty acids, từ đó được oxy hóa
để cung cấp năng lượng. Hầu hết các tế bào, trừ tế bào não, có thể sử dụng acid béo
thay thế glucose để cung cấp năng lượng. TMP12 819–820
6. Hầu hết lượng nhiệt mất ở người không mặc quần áo xảy ra theo cơ chế nào
dưới đây?
A) Dẫn nhiệt ra không khí
B) Dẫn nhiệt tới các đồ vật khác
C) Đối lưu
D) Bay hơi
E) Bức xạ nhiệt
 E) Khoảng 60% nhiệt cơ thể bị mất qua quá trình bức xạ. Mất nhiệt qua bức xạ
có nghĩa là mất qua dạng sóng nhiệt hồng ngoại, một dạng của sóng điện từ. Tất cả
các vật đều bức xạ nhiệt, do đó sóng nhiệt được bức xạ từ tường trong phòng hay
các vật dụng khác hướng về cơ thể và cơ thể bức xạ song nhiệt tới các vật xung
quanh. Nếu thân nhiệt lớn hơn nhiệt độ xung quanh các vật, có nhiều hơn sóng
nhiệt bức xạ từ cơ thể ra các vật xung quanh hơn là chiều ngược lại.
TMP12 868–869
7. Tăng bilirubin huyết tương ở bệnh nhân xơ gan rượu chủ yếu do nguyên nhân
nào dưới đây?
A) Giảm bài tiết bilirubin vào mật
B) Tăng hấp thu bilirubin ở các tế bào gan
C) Tăng kết hợp của bilirubin
D) Tan máu quá
 A) Phá hủy tế bài gan trong bệnh lý xơ gan dẫn tới vàng da do tắc nghẽn. Tốc
độ hình thành bilirubin bình thường, và bilirubin tự do vào các tế bào gan và được
liên hợp bình thường. Tuy nhiên bilirubin liên hợp từ gan vào máu có thể bị gián
đoạn do tắc nghẽn đường mất và trực tiếp đổ vào bạch huyết rời khỏi gan.
TMP12 841–842
8. Bệnh nhân nam 65 tuổi có tiền sử 25 năm nghiện rượu và bệnh gan. Bệnh nhân
đến khám do phù chân. Giảm thành phần nào dưới đây có thể góp phần làm phát
triển hiện tượng phù chân trên bệnh nhân?
A) Áp suất thủy tĩnh mao mạch
B) Áp suất tĩnh mạch đùi
C) Áp suất thủy tĩnh khoảng kẽ
D) Lưu lượng bạch huyết gan
E) Nồng độ albumin huyết tương
 E) Hầu hết albumin trong huyết tương được hình thành tại gan. Một trong
những biến chứng của xơ gan là suy tế bào gan làm giảm sản xuất albumin, dẫn
đến giảm áp suất keo thẩm thấu trong huyết tương và gây phù. Trong tình trạng
bình thường, khoảng 75% áp suất keo huyết tương được hình thành do albumin sản
xuất tại gan.
TMP12 833–834
9. Bệnh nhân nam 45 tuổi người Mỹ-Phi vào khoa cấp cuuws do đau vùng hạ sườn
phải. Bệnh nhân có tiền sử mắc bệnh hồng cầu hình liềm. Các kết quả xét nghiệm
cho thấy nồng độ bilirubin huyết tương tăng gấp 3 lần so với bình thường. Siêu âm
cho thấy sỏi mật. Thành phần sỏi mật là gì ở bệnh nhân này?
A) Sắc tố mật
B) Calcium
C) Cholesterol
D) Acid béo
E) Đường
 A) Các bệnh tan máu, như bệnh hồng cầu hình liền gây ra sự phá hủy trước khi
trưởng thành của hồng cầu. Lượng hemoglobin quá mức được giải phóng từ hồng
cầu dẫm tới tạo ra quá mức bilirubin từ đại thực bào. Tăng sản xuất bilirubin dẫn
tới tăng sỏi sắc tố trong túi mật được tạo nên chủ yếu từ bilirubin.
TMP12 840–842
Câu hỏi 10 và 11
Cho hình dưới đây, trả lời câu hỏi 10 và 11.

10. Lượng dư thừa adenosine triphosphate (ATP) trong tế bào chất có ở bước nào
trong quá trình phân giải glucose(đường phân)?
A) Chuyển glucose thành glucose-6-phosphate
B) Chuyển fructose-6-phosphate thành fructose-1,6-diphosphate
C) Chuyển 1,3-diphosphoglyceric acid thành 3-phosphoglyceric acid
D) Chuyển phosphoenolpyruvic acid thành pyruvic acid
 B) Giải phóng năng lượng liên tục từ glucose khi tế bào không cần năng lượng
là một chu trình cực kì lãng phí. Cả ATP và adenosine diphosphate (ADP) kiểm
soát tốc độ các phản ứng hóa học trong chu trình chuyển hóa năng lượng. Khi ATP
dư thừa trong tế bào, nó giảm chuyển hóa năng lượng bằng cách ức chế chuyển
fructose-6-phosphate thành fructose-1,6-diphosphate. Nó cũng ức chế hoạt động
enzyme phosphofructokinase.
TMP12 815
11. Lượng dư thừa adenosine diphosphate (ADP) hoặc adenosine monophosphate
(AMP) kích thích bước nào sau đây trong quá trình đường phân?
A) Chuyển glucose thành glucose-6-phosphate
B) Chuyển fructose-6-phosphate thành fructose-1,6-diphosphate
C) Chuyển 1,3-diphosphoglyceric acid thành 3-phosphoglyceric acid
D) Chuyển phosphoenolpyruvic acid thành pyruvic acid
 B) Cả ADP và AMP làm tăng hoạt động của enzyme phosphofructokinase và
tăng chuyển fructose-6-phosphate thành fructose-1,6-diphosphate.
TMP12 812
12. Vận chuyển glucose qua màng tế bào mô xảy ra nhờ quá trình nào?
A) Khuếch tán thuận hóa
B) Vận chuyển chủ động tiên phát
C) Đồng bận chủ động thứ phát
D) Đồng vận chuyển ngược
E) Khuếch tán đơn thuần
 A) Vận chuyển glucose qua màng tế nào ở hầu hết các tế nào khác so với vận
chuyển qua màng tiêu hóa hoặc qua biểu mô ống thận. Ở cả hai vị trí trên, glucose
được vận chuyển theo cơ chế đồng vận chuyển tích cực thứ phát, trong đó vận
chuyển tích cực Natri cung cấp năng lượng cho hấp thụ glucose chống lại bậc
thang nồng độ. Cơ chế đồng vận chuyển với Natri này có tác dụng chỉ trong những
tế bào biểu mô nhất định được đặc trưng thích hợp cho hấp thu tích cực glucose. Ở
các màng tế bào khác, glucose được vận chuyển từ nơi có nồng độ cao đến nơi có
nồng độ thấp theo cơ chế khuếch tán thuận hóa thông qua các đặc tính gắn của
protein xuyên màng mang glucose.
TMP12 810–811
13. Cơ chế nào dưới đây gây mất nhiệt ở người bình thường khi nhiệt độ môi
trường là 106°F (41o C) và độ ẩm tương đối ít hơn10%?
A) Dẫn nhiệt
B) Đối lưu
C) Bay hơi
D) Bức xạ
 C) Bốc hơi là cơ chế duy nhất gây mất nhiệt khi nhiệt độ không khí lớn hơn
nhiệt độ cơ thể. Mỗi gam nước bốc hơi từ bề mặt cơ thể làm mất 0.58 kilocalorie từ
cơ thể. Kể cả khi một người không đổ mồ hôi, nước vẫn bốc hơi từ da và phổi ở
tốc độ 450 đến 600 mL/ngày, tức là mất nhiệt khoảng 12 đến 16 kilocalories mỗi
giờ
TMP12 869
14. Khoảng 75% máu chảy qua gan từ tĩnh mạch cửa, và phần còn lại qua động
mạch gan trong suốt quá trình nghỉ ngơi. Tình trạng nào dưới đây mô tả tuần hoàn
gan về sức cản, áp lực và lưu lượng chảy?

 D) Gan có lưu lượng máu chảy cao, sức cản mạch thấp và áp lực dòng máu
thấp. Trong điều kiện nghỉ ngơi, khoảng 27% cung lượng tim qua gan, lúc này áp
lực tĩnh mạch của tới gan trung bình chỉ 9 mm Hg. Lưu lượng cao và áp lực thấp
cho thấy sức cản dòng máu và các xoang tĩnh mạch gan thường là rất thấp.
TMP12 838
15. Một thợ lặn có bình khí khám phá ra dòng dung nham dưới đáy biển có nhiệt
độ nước xung quanh là 102°F(39o C). Cơ chế mất nhiệt nào xảy ra ở bệnh nhân
này?
 B) Không có cơ chế mất nhiệt hiệu quả ở người trong môi trường nước có nhiệt
độ nước lớn hơn nhiệt độ cơ thể. Thay vòa đó, cơ thể tiếp tục lấy nhiệt cho đến khi
nhiệt độ cơ thể cân bằng với nhiệt độ nước. TMP12 868–869
16. Bệnh nhân nữ 57 tuổi béo phì đang điều trị bằng liệu pháp thay thế hormone
phát hiện có sỏi thận. Thành phần cấu tạo nên sỏi thận ở bệnh nhân này là gì?
A) Muối mật
B) Bilirubin
C) Muối calci
D) Cholesterol
E) Acid béo tự do
D) Trong điều kiện bình thường, cholesterol xuất hiện trong mật có thể bị kết
tủa, gây ra hình thành sỏi cholesterol, chiếm 80% trong số các loại sỏi mật. Các
yếu tố thuận lợi gây ra sỏi túi mật bao gồm béo phì, quá mức estrogen ở người có
thai hoặc sử dụng liệu pháp thay thế hormone, giới tính. Phụ nữ trong độ tuổi từ 20
đến 60 có nguy cơ sỏi mật cao gấp 2 lần nam giới cùng độ tuổi.
TMP12 No discussion
17. Khử amin có nghĩa là loại bỏ nhóm amino ra khỏi phân tử aminoacids. Hợp
chất nào sau đây được tao ra khi khử amin bằng phản ứng chuyển nhóm amino?
A) Acetyl coenzyme A
B) NH3
C) Citrulline
D) Ornithine
E) α-ketoglutaric acid
 B) Giáng hóa amino acids xảy ra hầu hết toàn bộ ở gan, bắt đầu bằng quá trình
khử amin, xảy ra chủ yếu theo lược đồ chuyển amin sau: Nhóm amin từ các amino
acids được chuyển sang α-ketoglutaric acid, tạo ra acid glutamic. Sau đó acid
glutamic chuyển nhóm amino tới các hợp chất khác hoặc giải phóng ra dưới dạng
NH3. Trong quá trình chuyển nhóm amino, acid glutamic lại trở về dạng α-
ketoglutaric acid, do đó, chu trình được lặp lại liên tục.
TMP12 834
18. Hầu hết năng lượng được giải phóng từ phân tử glucose xảy ra trong quá trình
nào sau đây?
A) Chu trình acid citric (chu trình Krebs)
B) Tổng hợp glycogen
C) Phân giải glycogen
D) Thủy phân glucose
E) Phosphoryl hóa oxy hóa
 E) Khoảng 90% tổng adenosine triphosphate (ATP) sản xuất trong chu trình
chuyển hóa glucose được hình thành trong quá trình oxi hóa các nguyên tử H
(được giải phóng trong đầu chu trình giáng hóa glucose). Chu trình này được gọi là
phosphoryl hóa oxi hóa. Chỉ 2 phân tử ATP được hình thành qua quá trình đường
phân, và 2 phân tử khác được hình thành trong chu trình Krebs. ATP không được
tạo ra trong tân tạo hay phân giải glycogen.
TMP12 814
19. Bệnh nhân nam 30 tuổi chạy quanh cối xay guồng dốc inclined treadmill cho
đến khi gần kiệt sức trong thí nghiệm kiểm tra phản ứng của con người với bài tập
yếm khí. Các khẳng định dưới đây liên quan đến chuyển hóa acid pyruvic thành
lactic trong điều kiện yếm khí là đúng, TRỪ:
A) Các sản phẩm chuyển hóa trong chu trình đường phân có thể triệt tiêu nhờ quá
trình chuyển hóa acid pyruvic thành lactic
B) Acid pyruvic kết hợp với NAD+ để tạo ra acid lactic và NADH
C) NAD+ được tạo ra nhờ quá trình chuyển hóa acid pyruvic thành lactic có thể
kết hợp với hai nguyên tử H và cho phép chu trình đường phân diễn ra
D) Trong các điều kiện trên, tim có thể sử dụng acid lactic như một nguồn năng
lượng.
 B) Hai sản phẩm cuối cùng của quá trình đường phân— acid pyruvic và các
nguyên tử H—kết hợp với NAD+ để tạo ra NADH và H+. Sự tích lũy một hoặc cả
2 sản phẩm trên sẽ dừng quá trình đường phân và ức chế hình thành ATP. Trong
điều kiện yếm khí, chủ yếu acid pyruvic được chuyển thành acid lactic. Do đó, acid
lactic có thể triệt tiêu các sản phẩm của quá trình đường phân.
TMP12 816
20. Bệnh nhân nam 45 tuổi vào khoa cấp cứu sau khi được phát hiện nằm trên
đường trong tình trạng say rượu. Khám thấy bệnh nhân nhợt nhạt rõ kèm theo da
và củng mạc mắt, bụng chướng và gõ đục vùng thấp cho thấy có cổ trướng. Gan to
khoảng 5 cm dưới bờ sườn và mật độ mềm. Lách không sờ thấy. Có phù 2 cẳng
chân và bàn chân. Giá trị nào của bilirubin trực tiếp và gián tiếp trong máu (tính
theo mg/dL) phù hợp với tình trạng của bệnh nhân?
Trực tiếp Gián tiếp
A) 1.1 1.2
B) 1.7 5.4
C) 2.4 2.5
D) 5.2 1.8
E) 5.8 7.2
 D) Bệnh nhân có xơ gan, trong trường hợp này, tốc độ tạo ra bilirubin bình
thường và bilirubin tự do vẫn đi vào các tế bào gan và liên hợp bình thường.
Bilirubin liên hợp (trực tiếp) hầu hết được đưa trở lại máu do tắc đường mật, do đó
chỉ một số lượng nhỏ đi vào trong túi mật. Kết quả là làm tăng nồng độ bilirubin
trực tiếp (liên hợp) trong máu, nồng độ bilirubin không liên hợp (gián tiếp) bình
thường hoặc gần bình thường.
TMP12 840–841
Câu hỏi 21–23
Sử dụng biểu đồ dưới đây để trả lời câu hỏi từ 21 tới 23. Biểu đồ cho thấy ảnh
hưởng của sự thay đổi điểm nhiệt sinh lý điều hòa bởi vùng dưới đồi. Đường màu
đỏ biểu thị nhiệt độ cơ thể và đường màu xanh biểu thị điểm nhiệt sinh lý vùng
dưới đồi.
21. Các thay đổi nào dưới đây xảy ra ở điểm W, so sánh với điểm V?

 E) Khi điểm nhiệt sinh lý vùng dưới đồi cao hơn thân nhiệt, cơ thể sẽ cảm thấy
lạnh và đưa ra các đáp ứng làm tăng thân nhiệt. Những đáp ứng này bao gồm run
và co mạch cũng như sởn da gà và tiết epinephrine. Tăng run cơ để sinh nhiệt.
Tăng tiết epinephrine có thể ngay lập tức làm tăng tốc độ chuyển hóa tế bào, được
gọi là sinh nhiệt hóa học. Co mạch máu dưới da làm giảm lượng nhiệt mất qua da.
TMP12 876
22. Những thay đổi nào dưới đây xảy ra ở điểm Y, so sánh với điểm V?
 B) Khi điểm nhiệt sinh lý vùng dưới đồi thấp hơn thân nhiệt, cơ thể cảm thấy
nóng và sẽ sinh ra các đáp ứng làm hạ thân nhiệt. Các đáp ứng bao gồm tiết mồ hôi
và giãn mạch. Tiết mồ hôi làm tăng mất nhiệt qua bốc hơi. Giãn mạch máu dưới da
hỗ trợ mất nhiệt thông qua tăng tuần hoàn máu tới da.
TMP12 876
23. Các thay đổi nào dưới đây xảy ra ở điểm X, so sánh với điểm V?

 A) Khi điểm nhiệt sinh lý vùng dưới đồi bằng thân nhiệt, cơ thể không có các
cơ chế sinh nhiệt hay thải nhiệt, kể cả khi thân nhiệt cao hơn nhiều so với bình
thường. Do đó, bệnh nhân không cảm thấy nóng kể cả khi thân nhiệt rất cao. Do đó
bệnh nhân không cảm thấy nóng khi thân nhiệt là
104°F (40cC).
TMP12 876
24. Phosphate giàu năng lượng nào dưới đây dư thừa nhiều nhất được dự trữ trong
tế bào?
A) Adenosine triphosphate (ATP)
B) Phosphocreatine
C) Adenosine diphosphate (ADP)
D) Creatine
E) Creatinine
 B) Phosphocreatine chứa các phân tử phosphate cao năng lượng và có lượng
ATP hay ADP cao gấp 3 đến 8 lần trong tế bào. Creatinine không chứa phân tử
phosphate cao năng lượng. Creatinine là sản phẩm phân hủy creatine phosphate
trong cơ. TMP12 860
25. Bệnh nhân nam 54 tuổi vào khoa cấp cứu sau khi được tìm thấy nằm trên sân
cạnh máy cắt cỏ đang chạy trong thời tiết mùa hè. Nhiệt độ cơ thể là 106°F (41oC),
huyết áp bình thường, nhịp tim là 160l/p. Những thay đổi nào dưới đây có thể xảy
ra trên bệnh nhân này?

 B) Bệnh nhân này mắc chứng say nắng (nóng). Bệnh nhân say nắng thường có
thở nhanh và tăng thông khí gây ra do kích thích trực tiếp hệ thần kinh trung ương,
toan hóa, thiếu oxi mô. Mạch máu dưới da giãn ra, da nóng. Ngừng tiết mồ hôi ở
bệnh nhân này với say nắng thực sự chủ yếu do nhiệt độ cao gây tổn thương vùng
dưới đồi trước-preoptic area. Dây thần kinh phát xung từ vùng này được truyền
theo đường hệ thần kinh tự chủ tới tủy sống và qua đường đi hệ giao cảm tới da
gây tiết mồ hôi.
TMP12 869, 876
26. Năng lượng tiêu tốn hằng ngày lớn nhất ở người nghỉ ngơi tĩnh tại được sử
dụng cho quá trình nào?
A) Chuyển hóa năng lượng cơ bản
B) Duy trì tư thế cơ thể
C) Sinh nhiệt không run Nonshivering thermogenesis
D) Tác động nhiệt của thức ăn
 A) Tốc độ chuyển hóa cơ bản chiếm khoảng 50% tới 70% năng lượng tiêu tốn
hằng ngày ở người nghỉ ngơi tĩnh tại. Không hoạt động thể lực, như cựa quậy nhúc
nhích hoặc duy trì tư thế tiêu tốn khoảng 7% năng lượng hằng ngày, và tác động
nhiệt lên thức ăn chiếm khoảng 8%. Quá trình sinh nhiệt không run có thể xảy ra
để đáp ứng với tình trạng lạnh, nhưng đáp ứng tối đa ở người trường thành ít hơn
15% tổng tốc độ chuyển hóa.
TMP12 863
27. Hầu hết năng lượng cho hoạt động thể lực kéo dài hơn 5 tới 10s nhưng ít hơn 1
đến 2 phút được cung cấp từ nguồn nào?
A) Adenosine triphosphate (ATP)
B) Anaerobic glycolysis
C) Oxi hóa carbohydrates
D) Oxi hóa of lactic acid
E) Chuyển acid lactic thành pyruvic acid
 B) Hầu hết năng lượng tối đa cho hoạt động thể lực kéo dài hơn 5 đến 10s và ít
hơn 1 đến 2 phút được lấy từ quá trình đường phân yếm khí. Giải phóng năng
lượng từ quá trình đường phân xảy ra nhanh hơn nhiều so với oxi hóa giải phóng
năng lượng (chậm hơn rất nhiều để có thể cung cấp cho nhu cầu của cơ trong vài
phút đầu hoạt động). ATP và phosphocreatine có sẵn trong tế bào cơ mất trong
vòng ít hơn 5 đến 10s. Sau khi co cơ kết thúc, chuyển hóa oxi hóa được sử dụng để
biến đổi lại đa số acid lactic tích tụ thành glucose, phần còn lại thành acid pyruvic
được giáng hóa và oxi hóa trong chu trình Krebs
TMP12 860–861
28. Amoni NH3 giải phóng trong quá trình khử amin của các amino acids được
loại bỏ khỏi máu hầu như toàn bộ bằng cách chuyển hóa thành hợp chất nào sau
đây?
A) Ammonium
B) Carbon dioxide
C) Citrulline
D) Ornithine
E) Urea
 E) Hai phân tử NH3 và một phân tử CO2 kết hợp hình thành một phân tử urea
và một phân tử nước. Quan trọng là tất cả urea hình thành trong cơ thể đều được
tổng hợp tại gan, Trong trường hợp mất gan hoặc bệnh gan nặng, NH3 tích lũy
trong máu. NH3 gây độc cho não có thể dẫn đến hôn mê gan.
TMP12 835
29. Hồng cầu thường xuyên chết đi và được thay thế. Heme từ hemoglobin giáng
hóa được chuyển hóa thành hợp chất nào dưới đây trước khi thải trừ khỏi cơ thể?
A) Bilirubin
B) Cholesterol
C) Acid cholic
D) Globin
E) Verdigarbin
 A) Hemoglobin được chuyển hóa nhờ các đại thực bào mô (còn được gọi là hệ
liên võng nội mô). Hemoglobin ban đầu phá vỡ tạo globin và heme, sau đó vòng
heme mở ra giải phóng sắt tự do và một chuỗi thẳng gồm 4 nhân pyrrole, từ đó
hình thành bilirubin. Bilirubin tự do được tế bào gan bắt giữ và hầu hết liên hợp
với acid glucuronic, bilirubin liên hợp đi vào trong đường mật và vào ruột.
TMP12 840
30. Amino acids tạo thành glucose qua chu trình nào dưới đây?
A) Tân tạo glucose
B) Tổng hợp glycogen
C) Phân giải glycogen
D) Đường phân
E) Thủy phân
 A) Khi dự trữ carbohydrates trong cơ thể giảm xuống dưới mức bình thường,
một số lượng glucose được tạo ra từ các amino acid và phần glycerol của chất béo.
Quá trình này được gọi là Tân tạo đường. Tổng hợp glycogen là hình thành
glycogen. Phân giải glycogen là phá vỡ phân tử glycogen dự trữ để tạo ra glucose
trong tế bào. Đường phân là phân giải glucose thành hai phân tử acid
pyruvic. Thủy phân là quá trình mà trong đó một phân tử bị phân cắt thành hai
thành phần với sự có mặt của phân tử nước.
TMP12 817
31. Bệnh nhân nữ 32 tuổi mang thai vào quý 3 của thai kì vào khoa cấp cứu do đau
nhiều vùng hạ sườn phải sau bữa ăn gà chiên. Huyết áp bệnh nhân là 130/84 mm
Hg, nhịp tim là 105 lần/phút, nhịp thở là 30 l/phút. BMI trước khi mang thai là 45
kg/m2. Khám thấy phản ứng thành bụng và toát mồ hôi. Nồng độ bilirubin huyết
thanh và WBC bình thường. Bệnh nhân được chẩn đoán bệnh nào dưới đây?
A) Sỏi túi mật
B) Táo bón
C) Viêm gan
D) Viêm tụy
E) Viêm phúc mạc
 A) Sỏi mật là sự có mặt của sỏi trong túi mật hoặc đường mật. Bệnh nhân này
cho thấy các triệu chứng điển hình của sỏi mật. TMP12 No discussion
32. Một thiết bị có chứa tế bào gan được thử nghiệm để hỗ trợ cho các bệnh nhân
chưa có chỉ định ghép gan hay phẫu thuật phục hồi gan liver regeneration. Khả
năng sống sót của tế bào gan được chứng minh khi chỉ số nào dưới đây tăng?
A) Hấp thu lactate dehydrogenase (LDH)
B) Lượng ethanol
C) Lượng albumin
D) Hấp thu Glucuronic acid
E) Lượng Oxygen
F) Hấp thu Carbon dioxide
 C) Tất cả albumin thông thường được tạo ra tại gan. Các tế bào gan sống sử
dụng O2 và tạo ra CO2. Acid glucuronic tạo ra từ tế bào gan được sử dụng để hình
thành bilirubin glucuronide. Lactate dehydrogenase là enzyme chuyển hóa acid
pyruvic thành acid lactic trong tình trạng yếm khí.
TMP12 833
33. Tốc độ chuyển hóa của một người thông thường thể hiện qua tốc độ giải phóng
nhiệt gây ra cho các phản ứng hóa học trong cơ thể. Tốc độ chuyển hóa có thể
được đánh giá khá chính xác qua tổng tiêu thụ oxy trong cơ thể. Yếu tố nào sau
đây sẽ làm tăng hoặc làm giảm tốc độ chuyển hóa?
 E) Hormone tăng trưởng GH có thể làm tăng tốc độ chuyển hóa lên 15% tới
20% do kích thích trực tiếp chuyển hóa tế bào. Sốt do bất kì nguyên nhân nào làm
tăng các phản ứng hóa học trong ở thể, trung bình khoảng 120% khi tăng 10°C.
Tốc độ chuyển hóa giảm 10% đến 15% khi ngủ. Thiếu dinh dưỡng kéo dài có thể
làm giảm tốc độ chuyển hóa 20% đến 30% do thiếu chất dinh dưỡng cung cấp cho
tế bào.
TMP12 863–864
34. Bài tiết nitrogen qua nước tiểu đo được trên bệnh nhân là 16.0 g trong 24 giờ.
Lượng protein bị phân giải trong 24h khoảng bao nhiêu?
A) 16.0
B) 17.6
C) 100
D) 110
E) 120
 D) Tốc độ chuyển hóa protein được đánh giá bằng cách đo nitrogen trong nước
tiểu, rồi cộng 10% (do khoảng 90% nitrogen trong protein được bài tiết qua nước
tiếu) rồi nhân với 6.25
(100/16) do trung bình protein chứa khoảng 16% nitrogen.
TMP12 845
35. Thiếu hụt protein nào dưới đây có thể gây ra them ăn?
A) Hormone MCH (hormone tăng nồng độ melanin)
B) NPY
C) Ghrelin
D) Orexin A
E) Leptin
F) Agouti-related peptide
 E) Leptin là một hormone gây chán ăn. Do đó, thiếu hụt leptin cây ra ăn uống
quá mức. Tất cả các hormone và chất dẫn truyền thần kinh khác kích thích ăn uống
và thiếu hụt chúng gây chán ăn. TMP12 846–849, Table 71-2
36. Ở bệnh nhân ĐTĐ typ 1 không được điều trị bằng insulin và có đường máu bất
kỳ là 400mg/100 mL, hệ số hô hấp RQ 2h sau khi ăn bữa ăn nhẹ chứa 60%
carbohydrates, 20% protein, and 20% chất béo ở bệnh nhân là bao nhiêu?
A) 0.5
B) 0.7
C) 0.9
D) 1.0
E) 1.2
 B) ĐTĐ typ 1 đặc trưng bởi thiếu hụt insulin. Khi thiếu insulin, carbohydrate
hầu như không được tế bào sử dụng, và hệ số hô hấp giữ gần với hệ số cho chuyển
hóa chất béo (0.70).
TMP12 845
37. Một đột biến gen gây ra hình thành protein nào sau đây có thể gây ra béo phì
độ 3 (béo bệnh)?
A) NPY
B) Ghrelin
C) Hormone MCH (Hormon tăng nồng độ melanin)
D) Melanocortin 4 receptor
E) Agouti-related peptide (AgRP)
F) Orexin A
 D) Hoạt hóa melanocortin 4 receptor ức chế đưa thức ăn vào cơ thể, khi thiếu
hhiutjreceptor này gây ăn uống quá mức và béo bệnh. Tất cả các chất dẫn truyền
thần kinh và hormone khác có tác dụng ngược lại, và khi thiếu chúng gây ra chán
ăn.
TMP12 846—847
38. Thiếu hụt chất nào dưới đây có thể gây ra hiện tượng quáng gà ở người?
A) Vitamin A
B) Vitamin B1
C) Vitamin B6
D) Vitamin B12
E) Vitamin C
F) Niacin
 A) Một trong những chức năng chủ yếu của Vitamin A là hình thành sắc tố
võng mạc và ngăn chặn quáng gà.
TMP12 853
39. Thay đổi nào dưới đây có thể kích thích cảm giác đói ở bệnh nhân không ăn gì
trong vòng 24h?
A) Tăng neuropeptide Y ở vùng dưới đồi
B) Tăng tiết leptin
C) Tăng tiết peptide YY
D) Giảm tiết ghrelin
E) Hoạt hóa các noron pro-opiomelanocortin (POMC) vùng dưới đồi
F) Tăng tiết cholescystokinin (CCK)
 A) Neuropeptide Y là một chất dẫn truyền thần kinh kích thích sự ngon miệng,
nó tăng lên khi thiếu hụt thức ăn. Leptin, peptide YY, cholecystokinin, và hoạt hóa
POMC neurons đều giảm khi đói.
Ghrelin tăng, không giảm khi đói.
TMP12 846–849, Table 71-2
40. Thiếu chất nào sau đây gây ra bệnh pellagra ở người hay “lưỡi đen”ở chó?
A) Vitamin A
B) Vitamin B1
C) Vitamin B12
D) Vitamin C
E) Niacin
 F) Niacin, còn được gọi là acid nicotinic, có chức năng như một coenzyme và
kết hợp với các nguyên tử H khi chúng bị tách khỏi thức ăn qua các quá trình khử
hydro. Khi thiếu hụt niacin, tốc độ khử hydro giảm, quá trình oxi hóa và vận
chuyển năng lượng từ thức ăn không thể hoạt động bình thường, từ đó gây ra viêm
da, nhiễm trùng niêm mạc, rối loạn tâm thần, cũng như rối loạn thực thể được gọi
là bệnh pellagra ở người và bệnh lưỡi đen ở chó.
TMP12 853–854
41. Thiếu vitamin nào dưới đây là nguyên nhân chính gây ra bệnh beriberi?
A) Vitamin A
B) Thiamine (vitamin B1)
C) Riboflavin (vitamin B2)
D) Vitamin B12
E) Pyridoxine (vitamin B6)
 B) Thiamine cần cho giai đoạn chuyển hóa cuối cùng của carbohydrates và
amino acids. Giảm sử dụng các chất này do thiếu thiamine gây ra các biểu hiện của
bệnh beriberi, bao gồm giãn mạch ngoại vi và phù, tổn thương hệ thống mạch
ngoại vi và trung tâm cũng như rối loạn tiêu hóa.
TMP12 875, 876
Unit 14: Nội tiết, sinh dục
Câu 6-8
Đường màu đỏ của biểu đồ trên mô tả mối quan hệ bình thường giữa nồng độ
insulin huyết tương với glucose sản xuất tại gan và giữa nồng độ insulin huyết Commented [LTN53]: sự sản xuất glucose

tương với glucose sử dụng ở cơ Commented [LTN54]: sự tiêu thụ glucose

6. Trên biểu đồ, những đường nào mô tả mối quan hệ trên ở bệnh nhân đái tháo
đường type 2?
A) A và C
B) A và D
C) B và C
D) B và D
 C) Đái tháo đường type 2 đặc trưng bởi sự giảm nhạy cảm của chuyển hóa mô
đích với insulin gọi là sự kháng insulin. Dẫn đến, lượng glucose vào gan bị suy yếu Commented [LTN55]: sự giảm nhạy cảm của mô đích với
tác động chuyển hóa của insulin, còn gọi là sự kháng insulin
và glucose giải phóng được tăng cường. Trong cơ, lượng glucose vào cơ bị suy yếu
Commented [LTN56]: có sự giảm hấp thụ glucose
7. Trên biểu đồ, những đường nào mô tả mối quan hệ trên ở người bị bệnh to cực
chi?
A) A và C
B) A và D
C) B và C
D) B và D
 C) Trong bệnh to cực chi, nồng độ cao của GH dẫn đến kháng insulin. Hậu quả, Commented [LTN57]: trong huyết tương

làm tăng sự tổng hợp glucose ở gan và suy yếu khả năng xâm nhập của glucose
vào mô ngoại vi.
8. Trên biểu đồ, Đường D mô tả ảnh hưởng của:
A) Tập thể dục
B) Béo phì
C) GH
D) Cortisol
E) Glucagon
 A) Trong khi tập thể dục, sự tiêu thu glucose ở cơ tăng, điều này phần lớn độc
lập với insulin Commented [LTN58]: hầu như không liên quan tới

9. Một cách điều trị cho rối loạn cương dương là tiêm một chất vào vật hang của
dương vật. Tiêm chất nào sau đây có thể dẫn đến cương dương?
A) Norepinephrine
B) Chất ức chế quá trình hình thành NO
C) Thromboxane A2, một prostaglandin co mạch
D) Angiotensin II
E) Tất cả đều sai
 E) Sự cương dương yêu cầu sự giãn của các cơ trơn mạch máu của các kháng
mạch dẫn tới vật hang. Tất cả các chất kể trên là chất co mạch và sẽ ngăn sự cương
dương
10. Một đứa trẻ sinh ra có dương vật, bìu nhưng không có tinh hoàn, không âm
đạo, mang NST XX. Hướng đến một trường hợp lưỡng tính. Đâu là nguyên nhân
của bất thường này?
A) Sản xuất HCG cao bất thường của tế bào lá nuôi phôi
B) Sự hiện diện của u chế tiết testosterone ở tuyến thượng thận phải của mẹ
C) Nồng độ LH cao bất thường trong máu mẹ
D) Nồng độ testosterone thấp bất thường trong máu mẹ
E) Tỷ lệ sản xuất estrogen của nhau thai thấp bất thường
 B) Nồng độ rất cao của testosterone ở thai nữ sẽ thúc đẩy sự hình thành cơ quan
sinh dục nam. U tuyến thượng thận ở mẹ làm tạo ra nhiều testosterone, lượng
khổng kiểm soát này có thể dẫn đến sự nam hóa. Commented [LTN59]: Việc khối u tuyến thượng thận ở
người mẹ đang tổng hợp một lượng cao không được kiểm
11. Một người phụ nữ trẻ hàng ngày đều tiêm một chất nào đó kể từ ngày thứ 16 soát hormon testosterone

của chu kì kinh nguyệt bình thường và liên tục trong 3 tuần. Trong quá trình tiêm
chất đó, cô ấy không có kinh. Chất được sử dụng có thể là?
A) Testosterone
B) FSH
C) Chất ức chế hoạt động của progesterone
D) Chất ức chế prostaglandin E2
E) HCG
 E) HCG có tác dụng đồng kích thích như LH ở hoàng thể. Dùng HCG có thể
dẫn đến việc hoàng thể tiếp tục tiết estrogen và progesterone, ngăn cản sự thoái hóa
của nội mạc tử cung và khởi đầu chu kì kinh nguyệt
12. Điều gì có thể ức chế sự khởi đầu của quá trình lâm bồn Commented [LTN60]: chuyển dạ

A) Sử dụng chất đối kháng với tác dụng của progesterone


B) Sử dụng hormone LH
C) Sử dụng chất đối kháng với ảnh hưởng của prostaglandin E2
D) Giản nở và kích thích khung chậu một cách cơ học
E) Sử dụng oxytocin
C) Chất đối kháng tác dụng của progesterone, sự giãn cổ tử cung và oxytocin
đều làm tăng tính hưng phấn của cơ trơn tử cung và tạo điều kiện cho các cơn co
thắt để cuộc đẻ bắt đầu. LH không có ảnh hưởng. PGE2 kích thích mạnh sự co cơ Commented [LTN61]: hình thành các cơ co và bắt đầu
quá trình chuyển dạ
trơn tử cung và được hình thành với mức độ ngày càng tăng bởi nhau thai trong
Commented [LTN62]: tác dụng này
gian đoạn muộn của thai kì
13) Tiếp xúc với tia UV trực tiếp tạo điều kiện cho quá trình:
A) Chuyển cholesterol thành 25-hydroxycholicalciferol
B) Chuyển 25-hydroxycholicalciferol thành 1,25-dihydroxycholicalciferol
C) Vận chuyển calcium vào dịch ngoại bào
D) Tạo ra protein gắn calcium
E) Dự trữ vitamin D3 ở gan
 A) Tia cực tím được hấp thu bởi da trực tiếp tạo điều kiện cho sự chuyển hóa
cholesterol thành 25-hydroxycholesterol
14. Sau khi sinh, áp lực trong động mạnh phổi giảm rất nhiều. Đâu là nguyên nhân
của hiện tượng này ?
A) Tăng áp lực của động mạch hệ thống
B) Đóng ống động mạch
C) Tăng áp lực thất trái
D) Tăng sức cản của hệ mạch phổi
D) Sức cản của hệ mạch phổi giảm đáng kể do sự nở phổi. Ở phổi của những
đứa trẻ không nở ra được, mạch máu bị nén bởi thể tích phổi nhỏ. Sự giãn nở ngay Commented [LTN63]: Trên những thai nhi có phổi không
nở được
lập tức sẽ làm các mạch máu không bị nén ép nữa và sức cản của dòng máu giảm
đi vài lần
15. Nếu một phản ứng miễn dịch phóng xạ được tiến hành đúng cách và lượng
hormone có đánh dấu phóng xạ gắn với kháng thể ở mức thấp, thì nó chỉ ra rằng:
A) Lượng hormone nội sinh trong huyết tương cao Commented [LTN64]: /Mức
B) Lượng hormone nội sinh trong huyết tương thấp
C) Cần nhiều kháng thể hơn
D) Cần ít hormone có đánh dấu phóng xạ hơn
 A) Trong miễn dịch phóng xạ, có quá ít kháng thể gắn hoàn toàn vào hormone
đánh dấu phóng xạ và hormone ở trong huyết tương để phân tích. Như vậy, có sự
bổ sung giữa nhãn và hormone nội sinh ở vị thí gắn của kháng thể. Hậu quả, nếu
lượng hormone phóng xạ gắn với kháng thể thấp, nó sẽ đáng giá nồng độ hormone Commented [LTN65]: chỉ ra rằng

nội sinh huyết tương ở mức cao.


16. Quá trình sản sinh tinh trùng được điều hòa bởi cơ chế điều hòa ngược âm tính
mà trong đó FSH kích thích các bước của sự hình thành tế bào tinh trùng. Tín hiệu
điều hòa ngược âm tính liên quan đến sự sản xuất tinh trùng nào làm ức chế sự
tổng hợp FSH của tuyến yên?
A) Testosterone
B) Inhibin
C) Estrogen
D) LH
B) Tế bào Sertoli trong ống sinh tinh tiết ra inhibin với tỉ lệ tương xứng với mức
độ sản xuất tinh trùng. Inhibin có tác dụng ức chế trực tiếp ảnh hưởng đến sự bài
tiết FSH ở thùy trước tuyến yên. FSH gắn với thụ thể đặc biệt ở trên tế bào Sertoli
dẫn tới các tế bào này phát triển và tiết các chất kích thích sản xuất tinh trùng. Sự
bài tiết inhibin sau đó gây ra tín hiệu điều hào ngược âm tính từ ống sinh tinh lên
tuyến yên
17. Trong thời gian 12 tiếng trước khi rụng trứng, điều gì sau đây là đúng?
A) Nồng độ estrogen hyết tương đang tăng
B) Sự tăng lên đột ngột của LH được bài tiết bởi tuyến yên Commented [LTN66]: Tuyến yên đột ngột tăng tiết LH

C) Sự tăng tiết LH đột ngột xảy ra ngay lập tức sau sự hình thành hoàng thể
D) Sự tăng tiết LH đột ngột theo sau sự giảm nồng độ progesterone huyết tương
ngay lập tức
E) Số lượng trứng đang phát triển tăng lên
 B) Sự rụng trứng không xảy ra nếu không có đỉnh LH xảy ra trước. Ngay trước Commented [LTN67]: chỉ xảy ra sau khi LH đạt đỉnh

khi rụng trứng, số lượng nang trứng giảm do sự thoái hóa bình thường của tất cả
trừ 1 nang và dẫn tới giảm sự tổng hợp estrogen của buồng trứng. Sự tổng hợp
progesterone được kích thích bởi đỉnh LH
18. Khi nào thì nồng độ progesterone đạt cao nhất trong chu kì kinh nguyệt của Commented [LTN68]: Thời điểm

phụ nữ
A) Giữa thời kì rụng trứng và bắt đầu hành kinh Commented [LTN69]: đầu kỳ kinh

B) Ngay sau khi rụng trứng Commented [LTN70]: trước

C) Khi nồng độ LH trong máu đạt đỉnh


D) Khi 12 nang trứng đang phát triển đến giai đoạn tạo hang Commented [LTN71]: làm tổ

A) Hoàng thể là nguồn sản xuất progesterone duy nhất, trừ một số được tiết ra
bởi nang trứng trước khi rụng trứng. Hoàng thể có chức năng ở giữa giai đoạn rụng
trúng và bắt đầu chu kì kinh nguyệt, trong thời gian đó nồng độ LH bị át chế thấp Commented [LTN72]: đầu kỳ kinh

dưới mức có thể đạt được khi LH đạt đỉnh trước khi rụng trứng
19. Một số tế bào tiết các chất hóa học vào dịch ngoại bào tác động vào các tế bào
trên cùng mô đó. Điều hòa như trên thuộc cơ chế nào? Commented [LTN73]: Mô tả trên thuộc cơ chế điều hòa
nào dưới đây?
A) Thần kinh
B) Nội tiết
C) Thần kinh nội tiết
D) Chất cận tiết
E) Chất tự tiết
 D) Sự thông tin qua chất cận tiết tổ ám chỉ sự bài tiết khuếch tán vào dịch ngoại
bào và ảnh hưởng tới các tế bào lân cận
20. Cặp chất nào là ví dụ cho loại điều hòa được nói tới ở câu 19?
A) Somatostatin—Sự tiết GH
B) Somatostatin—Sự tiết insulin
C) Dopamine—Sự tiết prolactin
D) Norepinephrine—Sự tiết CRH
E) CRH—Sự tiết hormone vỏ thượng thận
B) Tế bào delta của tụy tiết ra somatostatin, chất này ức chế sự bài tiết của
insulin và glucagon từ tế bào beta và alpha tương ứng của tụy. D là ví dụ của sự Commented [LTN74]: [Đáp án]

thông tin qua thần kinh và các lựa chọn còn lại là ví dụ của sự thông tin bằng cơ
chế thần kinh nội tiết
21. Chức năng sinh sản bình thường của nam cần đến estrogen. Đâu là nơi sản xuất
estrogen chủ yếu ở nam giới?
A) Tế bào Leydig
B) Tạo cốt bào
C) Tế bào gan
D) Tế bào tuyến tiền liệt
 C) Lượng lớn estrogen được tạo từ testosterone và androstanediol ở gan, chiếm
hơn 80% tổng lượng estrogen được sản xuất ở nam
22. Một vận động viên nữ chuyên nghiệp ở giữa độ tuổi 20 mà không có kinh trong
5 năm qua, mặc dù vậy đo mật độ xương cho thấy sự khoáng hóa xương vẫn bình
thường. Trong quá trình khai thác bệnh sử, điều gì có thể gợi mở cho việc giải
thích những điều quan sát được ở trên?
A) Cô ấy ăn một chế độ giàu carbonhydrate
B) Bà cô ấy bị gãy cổ xương đùi ở tuổi 79
D) Nồng độ estrogen trong huyết tương của cô ấy rất thấp
E) Cô ấy đã sử dụng steroid hóa hợp bổ sung trong 5 năm qua
E) Anabolic steroids gắn vào thụ thể của testosterone ở vùng dưới đồi gây ra sự Commented [LTN75]: Các steroid đồng hóa

ức chế ngược vào chu kì bình thường của buồng trứng và ngăn cản chu kì hành
kinh, cũng như kích thích hoạt động của tạo cốt bào ở xương
23. Trong hệ tuần hoàn bào thai, yếu tố nào trước sinh sẽ lớn hơn sau khi sinh?
A) Po2 động mạch
B) Áp lực nhĩ phải
C) Áp lực động mạch chủ
D) Áp lực thất trái
 B) Áp lực nhĩ phải hạ xuống đáng kể sau khi bắt đầu hít thở dẫn đến sự giảm
sức cản mạch phổi, áp lực động mạch phổi và áp lực thất phải
Cấu 24-26
Hãy nối mỗi bệnh nhân được miêu tả ở câu 24 đến 26 với giá trị chính xác nồng độ
một số chất được liệt kê ở bảng dưới. Giá trị bình thường: nồng độ aldostrerone
huyết tương, 10 ng/dL; nồng độ cortisol huyết tương, 10ng/dL; nồng độ kali huyết
tương, 4.5 mEq/L.
24. Bệnh nhân bị bệnh Addison
 Sự bài tiết hormone vỏ thượng thận giảm ở bệnh nhân bị bệnh Addison. Kết quả
là có thể thấy nồng độ thấp của aldosterone và cortisol trong huyết tương. Với kết
quả của nồng độ thấp aldosterone, nồng độ kali huyết tương sẽ tăng.
25. Bệnh nhân bị hội chứng Conn
 E) Bệnh nhân bị hội chứng Conn có u ở lớp cầu sẽ tiết ra lượng lớn aldosterone.
Hậu quả, nồng độ aldosterone tăng lên dẫn tới hạ kali. Sự bài tiết của cortisol ở lớp
bó vẫn bình thường
26. Bệnh nhân ăn chế độ ít natri
Nồng độ Nồng Nồng
Aldosteron độ độ kali
e Cortisol
A) 10.0 2.0 4.5
B) 2.0 2.0 6.0
C) 40.0 30.0 2.0
D) 40.0 10.0 4.5
E) 40.0 10.0 2.0
D) Sự bài tiết aldosterone bị tăng khi ăn chế độ ít natri, nhưng sự bài tiết cortisol
vẫn bình thường. Mặc dù aldosterone tăng tỉ lệ bài tiết kali bởi các tế bào cơ sở ở
ống góp, điều này ảnh hưởng đến giảm dòng chảy ở ống lượn xa. Hậu quả, có rất ít Commented [LTN76]: song chỉ một dòng chảy nhỏ ở ống
lượn xa cũng làm ngừng sự tăng tiết này.
sự thay đổi trong bài xuất kali hay nồng độ kali huyết tương
Commented [LTN77]: Dẫn đến
27. Trong biểu đồ sau, những đường nào phản ánh sự đáp ứng của bệnh nhân với
đái tháo nhạt tại thận?
A) A và C
B) A và D
C) B và C
D) B và D

 B) Bệnh nhân bị đái tháo nhạt tại thận, thận không đáp ứng một cách thích đáng
với ADH, và khả năng hình thành nước tiểu cô đặc bị làm suy yếu. Đối lập với đó,
sự bài tiết ADH bình thường đáp ứng với sự thay đổi áp lực thẩm thấu Commented [LTN78]: ADH vẫn bài tiết bình thường để
Commented [LTN79]: huyết tương
28. Yếu tố nào sau sinh lớn hơn trước sinh?
A) Dòng máu qua lỗ bầu dục
B) Áp lực nhĩ phải
C) Dòng máu qua ống động mạch
D) Áp lực động mạch chủ
 D) Do sự mất dòng máu qua nhau thai, sức cản mạch hệ thống tăng gấp đôi khi
sinh. Gây tăng áp lực mạch chủ cũng như áp lực trong thất trái và nhĩ trái Commented [LTN80]: từ đó
29. Hormone tuyến cận giáp trực tiếp:
A) Điều hòa tỷ lệ 25-hydroxycholicalciferol được tạo thành
B) Điều hòa sự vận chuyển calci qua nhu mô ruột non Commented [LTN81]: niêm mạc

C) Điều hòa tỷ lệ tổng hợp protein gắn protein Commented [LTN82]: calci

D) Điều hòa tỷ lệ hình thành 1,25- dihydroxycholicalciferol


E) Kích thích sự tái hấp thu phosphate ở ống thận
 D) Hormone cận giáp tác động lên vỏ thận để kích thích phản ứng tạo 1, 25-
dihydroxycholicalciferol từ 25-hydroxycholicalciferol. Nó không có ảnh hương lên
các phản ứng khác
30. Trong quá trình bài tiết insulin, chất nào được sản xuất tăng lên nhiều nhất? Commented [LTN83]: được tăng sản xuất

A) Amino acids
B) Amino acids và glucose
C) Amino acids và somatostatin
D) Glucose và somatostatin
 B) Cả amino acids và glucose đều kích thích tiết insulin. Hơn nữa, amino acid
làm cho sự kích thích tiết insulin của glucose trở nên mạnh mẽ hơn. Somatostatin
ức chế sự tiết insulin
31. Để sự biệt hóa giới tính nam xảy ra trong quá trình phát triển phôi thai,
testosterone phải được tinh hoàn tiết ra. Cái gì kích thích sự bài tiết testosterone Commented [LTN84]: Trong quá trình phát triển phôi
thai, tinh hoàn tiết testosterone để biệt hóa giới tính nam
trong quá trình phát triển phôi thai?
Commented [LTN85]: Chất nào
A) LH từ tuyến yên của mẹ
B) HCG
C) Inhibin tiết ra từ hoàng thể
D) GnRH từ vùng dưới đồi của phôi
B) HCG cũng gắn vào thụ thể của LH trên tế bào kẽ của tinh hoàn của thai nhi
nam dẫn đến sự sản xuất testosterone ở bào thai nam cho tới lúc sinh. Lượng bài
tiết nhỏ này của testosterone là nguyên nhân dẫn đến thai phát triển cơ quan sinh
dục nam thay vì cơ quan sinh dục nữ
32. Bệnh nhân bị tăng nồng độ T4 huyết tương, nồng độ TSH huyết tương thấp và
tuyến giáp nhỏ hơn bình thường. Đâu là lý do tốt nhất giải thích cho những điều Commented [LTN86]: giải thích đúng nhất

trên?
A) Bệnh nhân có một khối ở thùy trước tuyến yên ngăn cản sự tiết TSH
B) Bệnh nhân đang dùng propylthiouracil
C) Bệnh nhân đang dùng chiết xuất tuyến giáp
D) Bệnh nhân đang tiêu thụ một lượng lớn iod
E) Bệnh nhân bị bệnh Graves
 C) Nếu đối tượng được dùng một lượng lớn chiết xuất tuyến giáp ngoại sinh để
nâng nồng độ T4 lên mức bình thường, điều hòa ngược sẽ dẫn tới giảm sự tiết
TSH. Nồng độ thấp TSH huyết tương dẫn tới teo tuyến giáp. Trong bệnh Graves,
sự thảy đổi đồng thời của T4 và TSH sẽ xảy ra, nhưng tuyến giáp không bị teo.
Thực tế rằng bướu cổ thường gặp ở bệnh nhân bị bệnh Graves. Sự tổn thương vùng Commented [LTN87]: cho thấy

trước tuyến yên ngăn cản sự tiết TSH hoặc sử dụng propylthiouracil hay một lượng
lớn iod có thể liên quan đến nồng độ thấp T4 huyết tương.
33. Hoạt động của ion calci ngoài tế bào sẽ giảm trong vòng 1 phút sau khi: Commented [LTN88]: bởi

A) Hoạt động của ion phosphate ngoại bào tăng


B) pH ngoại bào tăng
C) Pco2 ngoại bào giảm
D) Tất cả các phương án trên
E) Không phương án nào đúng
 D) Từ A đến C, tất cả sẽ thay đổi cán cân hoạt động lớn theo chiều hướng ủng Commented [LTN89]: Các hiện tượng trong đáp án A đến
C đều dẫn tới
hộ sự liên hợp của ion calci với các chất phosphate hay các chất anionic, dẫn đến
Commented [LTN90]: tăng
giảm nồng độ ion calci tự do
Commented [LTN91]: gây
34. Khi hành kinh xong nồng độ estrogen trong máu tăng lên nhanh chóng. Nguồn
tiết estrogen là:
A) Hoàng thể
B) Nang trứng đang phát triển
C) Nội mạc tử cung
D) Tế bào mô đệm của buồng trứng
E) Thùy trước tuyến yên
 B) Ở phụ nữ không mang thai, nguồn tiết estrogen một cách đáng kể là các nang
trứng hoặc hoàng thể. Chu kì kinh nguyệt bắt đầu khi hoàng thể thoái hóa. Chu kì
kinh nguyệt kết thúc khi nang trứng đang phát triển tiết ra lượng đủ estrogen nâng
lượng tuần hoàn lên mức kích thích tăng sinh trở lại của nội mạc tử cung
35. Hormone nào được tiết ra ở thùy trước tuyến yên đóng vai trò chính trong việc
điều hòa tuyến đích không phải tuyến nội tiết?
A) ACTH
B) TSH
C) Prolactin
D) FSH
E) LH
C) Mô đích chính của prolactin là ngực nơi mà nó sẽ kích thích tiết sữa. Các
hormone khác của tuyến yên (adrenocorticotropic hormone, thyroid-stimulating
hormone, folliclestimulating hormone, và luteinizing hormone) kích thích hormone Commented [LTN92]: ADH, TSH, FSH, LH

ở các tuyến nội tiết khác


36. Một thí nghiệm được tiến hành trong đó ADH được sử dụng ở giờ thứ 3 cho 4
đối tượng (A đến D). Trong biểu đồ dưới đây, các đường nào phản ánh sự đáp ứng
với việc sử dụng ADH ở người bình thường và ở bệnh nhân bị đái tháo nhạt thể
trung ương?

Bình Đái tháo nhạt thể trung


thường ương
A) B A
B) B D
C) D A
D) D B
D) Ở bệnh nhân đái tháo nhạt thể trung tâm, có sự bài tiết thấp không phù hợp
của ADH trong đáp ứng với thay đổi áp lực thẩm thấu nhưng không có sự suy yếu
trong đáp ứng của thận với ADH. Vì nồng độ huyết tương của ADH bị giảm, có sự
suy yếu khả năng cô đặc nước tiểu và một thể tích lớn nước tiểu pha loãng bị bài
xuất. Mất nước dẫn tới tăng áp lực thẩm thấu gây kích thích trung tâm khát. Điều
này dẫn tới mức độ rất cao nước nhập vào
37. Một nữ vận động viên sử dụng steroids giống testosterone trong vài tháng bị
dừng chu kì kinh nguyệt bình thường. Điều gì giải thích tốt nhất cho hiện tượng
trên?
A) Testosterone kích thích sự sản xuất inhibin từ hoàng thể
B) Testosterone gắn vào các thụ thể trên nội mạc tử cung kết quả là ngừng phát
triển nội mạc tử cung trong chu kì bình thường
C) Testosterone gắn vào các thụ thể ở thùy trước tuyến yên kích thích sự bài tiết
của FSH và LH
D) Testosterone ức chế vùng dưới đồi tiết ra GnRH, ức chế tuyến yên tiết LH và
FSH
D) Tế bào thùy trước tuyến yên tiết ra LH và FSH, và tế bào vùng dưới đồi tiết
ra GnRH đều bị ức chế bởi cả estrogen và testosterone. Steroid được dung bởi
người phụ nữ này gây ra sự ức chế vừa đủ dẫn tới đình chỉ chu kì kinh nguyệt hàng
tháng
38. Điều nào có thể dự đoán ở một bệnh nhân bị suy giảm gen trong 11β-
hydroxysteroid dehydrogenase type 2?
A) Tăng hoạt động của corticoid khoáng
B) Tăng hoạt động của corticoid đường
C) Tăng kali máu
D) Giảm áp lực máu
E) Hạ đường huyết
 A) Enzyme 11β-hydroxysteroid dehydrogenase type 2 xuất hiện trong tế bào
ống thận và chuyển cortisol thành cortisone, chất này không gắn được vào thụ thể
của mineralocorticoid. Khi thiếu enzyme này, cortisol sẽ hiện hữu trong huyết
tương với nồng độ cao đáng kể so với aldosterone và gắn với thụ thể của
mineralocorticoid và gây ra tác dụng của mineralocorticoid. Tăng kali và giảm áp
lực máu phản ánh sự thiếu mineralocorticoid
39. Điều gì làm giảm sức cản trong động mạch đi tới các xoang của dương vật?
A) Sự kích thích của thần kinh giao cảm phân bố trong động mạch
B) NO
C) Sự ức chế hoạt động của thần kinh phó giao cảm tới động mạch
D) Tất cả các điều trên
 B) NO là chất giãn mạch được giải phóng một cách bình thường dẫn đến giãn
động mạch
40. Bệnh nhân có một bướu cổ liên quan đến nồng độ hormon TRH và TSH cao
trong huyết tương. Nhịp tim của cô ấy bị tăng lên. Chẩn đoán phù hợp nhất cho
bệnh nhân này là gì?
A) Bướu cổ địa phương
B) U vùng dưới đồi tiết ra lượng lớn TRH
C) U tuyến yên tiết ra lượng lớn TSH
D) Bệnh Graves
B) U vùng dưới đồi tiết ra một lượng lớn TRH sẽ kích thích tuyến yên tăng tiết
TSH. Dẫn đến sự bài tiết hormone tuyến giáp tăng và dẫn tới tăng nhịp tim. Ngược
lại, bệnh nhân bệnh nhân bị u tuyến yên chế tiết một lượng lớn TSH hoặc bị bệnh
Graves sẽ có nồng độ TRH thấp do cơ chế điều hòa ngược. Cả nồng độ TRH và
TSH đều tăng trong bướu cổ địa phương nhưng nhịp tim giảm vì sự bài tiết T4 ở
mức thấp
41. Một phụ nữ 40 tuổi đến phòng cấp cứu vì gãy cổ xương đùi. X quang cho thấy
sự khử chất khoáng tổng quát ở xương tại vị trí này. Nồng độ ion calci huyết tương
lớn hơn một cách rõ rệt so với bình thường: 12.2 mg/dL. Tình trạng nào sau đây
phù hợp với những điều trình bày ở trên?
A) Loãng xương
B) Bệnh Rickets
C) Cường tuyến cận giáp
D) Suy thận
C) Sự khử khoáng của xương có thể là nguyên nhân của bất cứ lựa chọn nào
nhưng chỉ tăng nồng độ hormone cận giáp mới vừa dẫn đến sự khử khoáng và
tăng nồng độ calci huyết tương. Nồng độ hormone cận giáp tăng dẫn tới kích thích
quá mức hủy cốt bào, mất calci ở xương, kích thích sự hấp thu calci ở dịch ống
thận và ức chế bài xuất calci, kích thích tạo 1,25-dihydroxycholecalciferol gây tăng
tỉ lệ hấp thu calci từ hệ tiêu hóa
42. Một người đàn ông ăn một bữa ăn ít carbohydrate nhưng giàu protein bao gồm
các acid amin có thể kích thích tiết insulin. Đáp ứng nào giải thích cho việc bệnh
nhân này không bị hạ đường huyết?
A) Sự ức chế của GH
B) Sự ức chế bài tiết somatomedin C
C) Sự kích thích bài tiết cortisol
D) Sự kích thích bài tiết glucagon
E) Sự kích thích bài tiết epinephrine
 D) Sự tiêu thụ amino acid kích thích sự bài tiết của cả GH và glucagon. Tăng
bài tiết glucagon dẫn tới tăng nồng độ glucose máu và do đó đối lập với tác dụng
của insulin dẫn tới hạ đường huyết
43. Một người đàn ông 46 tuổi bị “sưng húp” da và đang hôn mê. Nồng độ TSH
huyết tương giảm thấp và tăng một cách đáng kể khi được bổ sung TRH. Chẩn
đoán phù hợp nhất là?
A) Cường giáp do u tuyến giáp
B) Cường giáp do bất thường vùng dưới đồi
C) Suy giáp do bất thường tuyến giáp
D) Suy giáp do bất thường vùng dưới đồi
E) Suy giáp do bất thường tuyến yên
D) Hôn mê và phù niêm là dấu hiệu của suy giáp. Nồng độ thấp của hormone
TSH huyết tương chir điểm sự bất thường của cả vùng dưới đồi và tuyến yên. Vì
tuyến yên đáp ứng với việc dùng TRH, điều này gợi ý rằng chức năng tuyến yên
bình thường và vùng dưới đồi đang sản xuất thiếu TRH
44. Hormone nào vừa được tổng hợp vừa được tích trữ ở tuyến yên?
A) GH
B) GHRH
C) ADH
D) Somatostatin
E) Somatomedin
.A) GH và ADH được dự trữ tương ứng ở thùy trước và sau của tuyến yên. Tuy
nhiên, mặc dù GH thì được tổng hợp ở thùy trước tuyến yên còn ADH thì không
như thế. ADH, GHRH và các hormone ức chể ở vùng dưới đồi (somatostatin) được
tổng hợp ở vùng dưới đồi. Các somatomedin là yếu tố tăng trưởng (protein nhỏ)
kích thích sự phát triển của xương và mô ngoại vi. Một trong những somatomedin
quan trọng là somatomedin C, nó được sản xuất ở gan trong đáp ứng với GH.
Somatomedin C kích thích tất cả các phương diện của sự phát triển xương
45. Một người đàn ông đang sử dụng một số thuốc, một trong số đó khi dùng gây
trở ngại cho giai đoạn xuất tinh khi quan hệ tình dục. Thuốc nào là nguyên nhân
của vấn đề này?
A) Thuốc kéo dài thời gian hoạt động của NO
B) Thuốc ức chế receptor của NO trên cơ trơn
C) Thuốc làm tăng giải phóng NO
D) Hợp chất androgen giống testosterone
E) Chất ức chế receptor beta-adrenergic của hệ thần kinh
 E) Sự xuất tinh xảy ra bởi phản xạ trung gian qua hệ beta-adrenergic. Chất đối
kháng beta-adrenergic ảnh hưởng đến phản xạ này. Các đáp án khác không tham
gia vào quá trình này
46. Trong điều hòa bài tiết aldosterone, angiotensin II tác động lên cấu trúc nào?
A) Lớp cầu
B) Lớp bó
C) Lớp lưới
D) Tủy tuyến thượng thận
 A) Tế bào của lớp cầu tiết ra phần lớn aldosterone. Những tế bào này có thụ thể
với angiotensin II, chất kiểm soát chính sự bài tiết aldosterone.
47. Phụ nữ có thai dùng prostaglandin E2 (PGE2) có thể dẫn tới xảy thai. Đâu là sự
giải thích tốt nhất cho tác dụng này?
A) PGE2 kích thích mạnh sự co thắt của tử cung
B) PGE2 dẫn tới co động mạch cấp máu cho nhau thai
C) PGE2 kích thích sự giải phóng của oxytocin ở thùy sau tuyến yên
D) PGE2 làm tăng sự bài tiết của progesterone từ hoàng thể
 A) Phần thuộc về thai nhi của nhau thai giải phóng ra nồng độ cao
prostaglandins trong thời kì lâm bồn. Sự giải phóng này liên quan đến sự thoái hóa
của nhau thai. Prostaglandin, đặc biệt là PGE2, kích thích mạnh mẽ sự co thắt cơ tử
cung.
48. Trong vòng vài năm sau mãn kinh, nồng độ FSH bình thường vẫn cực kì cao.
Một phụ nữ 56 tuổi hoàn toàn mãn kinh 3 năm trước. Tuy nhiên, nồng độ FSH
trong máu của bà ấy lại thấp. Sự giải thích hợp lý nhất cho điều này là?
A) Cô ấy có sử dụng liệu pháp hormone thay thế gồm estrogen và progesterone kể
từ khi mãn kinh hoàn toàn
B) Tuyến thượng thận tiếp tục sản xuất estrogen
C) Buồng trứng tiếp tục tiết ra estrogen
D) Cô ấy sử dụng thuốc tránh thai 20 năm trước khi mãn kinh
 A) Sau khi mãn kinh, sự thiếu vắng cơ chế ức chế ngược của estrogen và
progesterone dẫn tới tỷ lệ bài tiết cực lớn của FSH. Phụ nữ dùng estrogen như một
phần của liệu pháp hormone thay thế để giảm các triệu chứng liên quan đến tiền
mãn kinh có sự đàn áp nồng độ FSH gây ra bởi sự ức chế của estrogen.
49. Cặp hormone và hành động tương ứng nào đúng?
A) Glucagon-tăng quá trình đường phân trong gan
B) Glucagon-tăng quá trình đường phân trong cơ vân
C) Glucagon-tăng tổng hợp glucose
D) Cortisol-tăng tổng hợp glucose
E) Cortisol-giảm sự hấp thu glucose vào cơ
 B) Glucagon kích thích giáng hóa glycogen ở gan, nhưng nó không có ảnh
hưởng sinh lý đến cơ. Cả glucagon và cortisol tăng tổng hợp đường, và cortisol
làm giảm tiêu thụ glucose ở cơ.
50. Bệnh nhân được cho một liều lớn Insulin tiêm tĩnh mạch. Các hormone sau sẽ
thay đổi như thế nào trong huyết tương để đáp ứng với việc tiêm insulin?

 C) Tiêm insulin dẫn tới giảm đường huyết. Hạ đường huyết kích thích tiết GH,
glucagon, và epinephrine, tất cả các chất này góp phần điều hòa làm tăng nồng độ
đường trong máu.
51. Chậm thở lúc mới sinh là một tình trạng nguy hiểm mà trẻ sơ sinh hay gặp.
Nguyên nhân thường gặp dẫn tới chậm thở?
A) Thai nhi thiếu oxi trong quá trình sinh đẻ
B) Mẹ thiếu oxi trong quá trình đẻ
C) Tình trạng tăng CO2 của thai nhi
D) Tình trạng tăng CO2 của mẹ
A) Sự thiếu oxi kéo dài của thai nhi trong quá trình sinh đẻ có thể dẫn tới tình
trạng suy giảm trầm trọng hoạt động của trung tâm hô hấp. Thiếu oxi có thể xảy ra
trong cuộc đẻ vì sự chèn ép của dây rốn, sự bong sớm của nhau thai, sự co bóp quá
mức của tử cung hoặc gây tê quá mức cho mẹ
52. Hormone nào sau đây phần lớn không gắn với protein huyết tương?
A) Cortisol
B) Thyroxine (T4)
C) ADH
D) Estradiol
E) Progesterone
 C) Nhìn chung, peptide hormones tan trong nước và không bám nhiều vào
protein huyết tương. ADH, một peptide hormone của thùy sau tuyến yên, không
gắn với protein huyết tương. Đối lập với chúng, steroid và hormone tuyến giáp có
khả năng gắn cao với protein huyết tương.
53. Một đứa trẻ 3 tuần tuổi đến phòng cấp cứu trong tình trạng hôn mê. Tiền sử
được cho ăn thức ăn công thức ở dạng đặc không pha loãng trong 5 ngày (Dạng
bào chế của thức ăn công thức cho trẻ sơ sinh thường được bán ở dạng đặc và phải
được pha loãng với nước trước khi cho trẻ ăn). Áp lực thẩm thấu huyết tương của
trẻ là 352 mOsm/L (bình thường là 280 đến 300 mOsm/L), và áp lực thẩm thấu
nước tiểu là 497 mOsm/L. Đâu là lời giải thích chính xác cho tình trạng tăng áp lực
thẩm thấu huyết tương?
A) Trẻ bị rối loạn điều hoà hormon ADH
B) Trẻ có tình trạng bài tiết quá mức aldosterone
C) Trẻ không có khả năng cô đặc nước tiểu một cách hiệu quả để bài tiết lượng
chất hòa tan từ thức ăn công thức mà không làm mất nhiều nước hơn việc duy trì
áp lực thẩm thấu huyết tương bình thường
D) Trẻ có ống góp bất thường ngăn cản sự hình thành nước tiểu cô đặc
 C) Sự phát triển chức năng của thận chưa hoàn thiện cho đến khi kết thúc tháng
thứ nhất của cuộc đời. Thận của trẻ sơ sinh có thể cô đặc nước tiểu khoảng 1.5 lần
áp lực thẩm thấu huyết tương (so với 3 đến 4 lần áp lực thẩm thấu huyết tương ở
người lớn). Nếu thức ăn công thức ở dạng đặc có nồng độ thẩm thấu rất cao, trẻ sẽ
không thể bài xuất các chất hòa tan để tạo nước tiểu cô đặc một cách hiệu quả
nhằm ngăn cản tình trạng tăng áp lực thẩm thấu huyết tương.
54. Tại sao sữa chỉ được sản xuất sau đẻ?
A) Nồng độ LH và FSH khá thấp trong thai kì không đủ hỗ trợ quá trình sản xuất
sữa
B) Nồng độ cao của progesterone và estrogen ức chế quá trình sản xuất sữa
C) Các tế bào tuyến của vú không thể trưởng thành cho đến sau khi đẻ
D) Nồng độ cao oxytocin cần để khởi động quá trình sản xuất sữa và oxytocin
không được tiết ra trừ khi đứa trẻ kích thích đầu vú
B) Mặc dù estrogen và progesterone là cần thiết cho sự phát triển sinh lý của vú
trong thời kì mang thai, nhưng hiệu quả đặc biệt của cả hai hormone này là ức chế
sự tiết sữa. Mặc dù nồng độ prolactin tăng 10 đến 20 lần ở cuối thời kì mang thai,
tác dụng ức chế của estrogen và progesterone ngăn sản xuất sữa cho đến khi đứa
trẻ sinh ra. Ngay sau khi đứa trẻ sinh ra, hiện tượng giảm đột ngột sự bài tiết của cả
estrogen và progesterone ở nhau thai dẫn tới sự tổng hợp sữa dưới tác dụng của
prolactin để thúc đẩy quá trình sản xuất sữa
55. Trong một thí nghiệm, bệnh nhân trong nhóm 1 được cho chất X, bệnh nhân
của nhóm 2 được cho chất Y. Sau 1 tuần, bệnh nhân nhóm 1 có mức chuyển hóa
thấp hơn và tuyến giáp to hơn các bệnh nhân trong nhóm 2. Xác định chất X, Y là
chất nào (T4, thyroxine; TRH, thyrotropin-releasing hormone;
TSH, thyroid-stimulating hormone)
Chất X Chất Y
A) TSH Giả dược
B) T4 Giả dược
C) Giả dược TSH
D) Giả dược T4
E) Giả dược TRH
 D) Sử dụng T4 để tăng nồng độ của hormone này trong huyết tương lên trên
mức bình thường có thể làm tăng mức chuyển hóa và giảm sự bài tiết TSH. Giảm
nồng độ TSH huyết tương dẫn tới teo tuyến giáp. Do đó, nhóm bệnh nhân 1 sẽ có
mức chuyển hóa thấp hơn và tuyến giáp to hơn nhóm bệnh nhân 2, nhóm được cho
dùng T4
56. Điều gì sau đây làm tăng tỉ lệ bài xuất calci ở thận?
A) Giảm nồng độ calcitonin huyết tương
B) Tăng nồng độ ion phosphate huyết tương
C) Giảm nồng độ hormone tuyến cận giáp trong huyết tương
D) Nhiễm kiềm chuyển hóa
 C) Nồng độ của PTH điều hòa mạnh mẽ sự hấp thu ion calci ở ống thận. Sự
giảm nồng độ hormone làm giảm tái hấp thu calci và tăng sự bài xuất calci trong
nước tiểu. Các lựa chọn khác ít gây ảnh hưởng hoặc làm giảm bài xuất calci.
57. Một bệnh nhân bị cường giáp do u tuyến yên. Những thay đổi sinh lý nào có
thể mong đợi ở bệnh nhân này?
Tổng hợp Nhịp Lồi mắt
thyroglobuli tim
n
A) ↑ ↑ +
B) ↑ ↑ -
C) ↑ ↓ +
D) ↓ ↓ +
E) ↓ ↓ -
F) ↓ ↑ -
 B) U chế tiết của tuyến yên tiết ra một lượng lớn TSH được cho là sẽ kích thích
tuyến giáp tiết ra một lượng lớn hormone tuyến giáp. TSH kích thích một vài bước
của quá trình tổng hợp hormone tuyến giáp, bao gồm việc tổng hợp thyroglobulin.
Tăng nhịp tim là một trong số các đáp ứng sinh lý với nồng độ cao hormone tuyến
giáp. Tuy nhiên, nồng độ hormone tuyến giáp huyết tương cao không dẫn tới lồi
mắt. Globulin miễn dịch dẫn tới lồi mắt trong bệnh Graves, một trong những dạng
thường gặp của cường giáp
58. Một người đàn ông 25 tuổi bị chấn thương nặng sau khi bị đâm bởi một
phương tiện chạy tốc độ cao và bị mất 20% khối lượng máu. Những thay đổi sinh
lý nào có thể mong đợi xảy ra nhằm đáp ứng với sự chảy máu ở bệnh nhân này?
(ADH, hormone bài niệu)
Độ hoạt Độ hoạt Sự
động thụ động thụ bài
thể với sự thể áp lực tiết
căng của của động
tâm nhĩ mạnh ADH
A) ↓ ↓ ↑
B) ↓ ↓ ↓
C) ↔ ↑ ↑
D) ↑ ↑ ↑
E) ↑ ↑ ↓
 A) Xuất huyết gây giảm hoạt hóa thụ thể sức căng ở tâm nhĩ và thụ thể áp lực ở
động mạch. Giảm hoạt hóa của những thụ thể này dẫn tới tăng tiết ADH
59. Một bệnh nhân có chức năng tuyến giáp bình thường bị kê nhầm thuốc. Những
thay đổi nào có thế xuất hiện nếu bệnh nhân này dùng propylthiouracil một vài
tuần? (T4,
thyroxine; TSH, thyroid-stimulating hormone)
Kích Nồng độ Nồng
thước T4 huyết độ
tuyến giáp tương TSH
huyết
tương
A) ↓ ↓ ↓
B) ↓ ↓ ↑
C) ↑ ↓ ↓
D) ↑ ↓ ↑
E) ↑ ↑ ↑
D) Propylthiouracil ngăn cản một số bước của quá trình tổng hợp hormone tuyến
giáp nhưng không ngăn cản sự tạo thành của thyroglobulin trong tế bào nang. Do
đó, nếu propylthiouracil được dùng ở bệnh nhân bình thường, nồng độ T4 huyết
tương sẽ giảm, và làm giảm cơ chế ức chế ngược dẫn tới sự tăng bài tiết TSH.
Nồng độ cao TSH huyết tương dẫn tới phì đại tuyến giáp mặc dù sự sản xuất
hormone tuyến giáp bị giảm
60. Nếu một phụ nữ có một khối u chế tiết một lượng lớn estrogen từ tuyến thượng
thận thì điều gì sẽ xảy ra?
A) Nồng độ progesterone trong máu sẽ rất thấp
B) Tỷ lệ bài tiết hormon LH sẽ hoàn toàn bị ức chế
C) Cô ấy sẽ không có chu kì kinh nguyệt bình thường
D) Xương của cô ấy sẽ cốt hóa bình thường
E) Tất cả các điều trên
E) Chọn A và D: Sự bài tiết LH sẽ ức chế (B) bởi sự điều hòa ngược âm tính của
estrogen từ khối u; do đó, cô gái này sẽ không có kinh nguyệt (C); khi không có
chu kì bình thường, hoàng thể không phát triển và sẽ không tạo thành progesterone
(A). Nồng độ cao estrogen được sản xuất bởi khối u sẽ kích thích hoạt động của tạo
cốt bào nhằm duy trì hoạt động bình thường của xương
61. Ở phụ nữ trẻ tuổi bình thường, áp lực lớn nhất có thể đạt được lên vật hang
trong quan hệ tình dục là bao nhiêu?
A) 20 đến 40 mm Hg
B) 60 đến 80 mm Hg
C) 150 đến 250 mm Hg
D) 400 đến 600 mm Hg
 C) Sức cản động mạch dẫn vào thể hang giảm nhiều trong quá trình cương
dương, trong khi sức cản tĩnh mạch dẫn máu ra khỏi xoang tăng lên do sự chèn ép
của các tĩnh mạch chống lại tổ chức xơ bao quanh xoang. Do đó, áp lực trong
xoang khi cương lớn hơn hoặc bằng áp lực động mạch thì tâm thu. Tuy nhiên, sự
co của các cơ vân ở tầng sinh môn che phủ một phần của vật hang có thể ép vào
các khoang, làm áp lực trong xoang tăng đến mức cao hơn đáng kể so với huyết áp
tâm thu.
Câu 62 đến 63
62. Khi so sánh với giai đoạn sau khi tiêu hóa (hấp thụ hoàn toàn sau bữa ăn),
những thay đổi chuyển hóa nào xảy ra trong giai đoạn vừa ăn xong?
 D) Sau khi ăn, sự bài tiết insulin tăng. Dẫn đến tăng tỉ lệ của glucose vào cả gan
và cơ. Insulin đồng thời ức chế lipase nhạy cảm với hormone, chất làm giảm thủy
phân triglyceride trong tế bào mỡ.
63. Khi so sánh với trạng thái nghỉ ngơi, những thay đổi chuyển hóa nào xảy ra
trong quá trình tập thể dục?
Lượng Lượng Độ hoạt
glucose glucose động của
vào gan vào cơ lipase
nhạy cảm
với
hormone
A) ↑ ↑ ↑
B) ↑ ↓ ↑
C) ↓ ↑ ↓
D) ↑ ↑ ↓
E) ↓ ↑ ↑
 E) Khi tập thể dục, glucose vào cơ được tăng cường dẫn tới giảm đường huyết.
Sự bài tiết insulin bị giảm xuống trong quá trình tập thể dục, trong khi sự bài tiết
glucagon và epinephrine thì tăng lên. Do đó, glucose vào gan bị hạn chế và hoạt
động của lipase nhạy cảm hormone tăng lên.
64. Từ rất sớm trong giai đoạn phát triển phôi thai, testosterone được tạo thành
trong bào thai nam. Chức năng của hormone ở giai đoạn này của sự phát triển là?
A) Kích thích xương phát triển
B) Kích thích phát triển cơ quan sinh dục nam
C) Kích thích sự phát triển hệ cơ vân
D) Ức chế sự bài tiết LH
 B) Chức năng cơ bản của testosterone đối với sự phát triển phôi thai nam là kích
thích hình thành cơ quan sinh dục nam.
65. Thay đổi nào có thể xảy ra khi tăng sự gắn của hormone với protein huyết
tương?
A) Tăng độ thanh thải huyết tương của hormone
B) Giảm thời gian bán hủy của hormone
C) Tăng hoạt động của hormone
D) Tăng mức độ điều hòa ngược âm tính gây ra bởi hormone
E) Tăng dự trữ huyết tương cho sự bổ sung nhanh chóng của hormone tự do
 E) Hormone gắn với protein không có tác dụng sinh học và không bị chuyển
hóa. Như vậy, tăng protein gắn sẽ dẫn tới giảm hoạt động của hormone và độ thanh
thải huyết tương và tăng nửa đời sống của hormone. Hormone tự do có vai trò ức
chế ngược âm tính sự bài tiết hormone. Do đó, sự tăng đột ngột hormone gắn với
protein huyết tương làm giảm điều hòa ngược âm tính. Tuy nhiên protein gắn
hormone cung cấp kho dự trữ cho sự thay thế nhanh chóng hormone tự do.
66. Bệnh nhân đến phòng cấp cứu với tình trạng sốc tim do nhồi máu cơ tim nặng.
Mẫu máu động mạch đầu tiên cho thấy nồng độ các ion như sau:
Natri 137 mmol/L
Bicarbonate 14 mmol/L
Calcium tự do 2.8 mmol/L
Kali 4.8 mmol/L
pH 7.16
Để điều trị tình trạng nhiễm toan, bác sĩ bắt đầu truyền natri bicarbonate và sau 1
giờ lấy máu đi xét nghiệm cho kết quả:
Natri 138 mmol/L
Bicarbonate 22 mmol/L
Calcium tự do 2.3 mmol/L
Kali 4.5 mmol/L
pH 7.34
Đâu là nguyên nhân của tình trạng giảm nồng độ ion calcium?
A) Tăng pH động mạch do truyền natri bicarbonate đã ức chế tuyến cận giáp tiết
hormone
B) Tăng pH đã dẫn đến kích thích tạo cốt bào lấy calci trong hệ tuần hoàn
C) Tăng pH đã dẫn đến sự tăng của nồng độ ion HPO4-, làm chuyển dịch phương
trình giữa HPO4- và Ca++ tạo CaHPO4
D) Sự tăng pH động mạch kích thích sự tạo thành 1,25-dihydroxycholecalciferol
và dẫn đến tăng tỷ lệ hấp thu calcium ở hệ tiêu hóa
 C) Sự giảm ion H+ biểu thị bởi sự tăng pH làm tăng nồng độ ion phosphate
mang điện tích âm sẵn sàng để kết hợp với ion calci. Hậu quả, nồng độ ion calci tự
do bị giảm.
67. Một phụ nữ 30 tuổi nuôi con bằng sữa mẹ. Trong quá trình cho bú, đáp ứng với
hormone nào xảy ra?
A) Tăng tiết ADH từ nhân trên thị
B) Tăng tiết ADH từ nhân cạnh não thất
C) Tăng tiết oxytocin từ nhân cạnh não thất
D) Giảm tiết protein vận chuyển ADH và oxytocin xuống tuyến yên
E) Tăng nồng độ của cả ADH và oxytocin
 C) Khi trẻ bú, sự kích thích các thụ thể ở đầu vú tăng tín hiệu thần kinh hướng
tâm đền cả nhân trên thị và nhân cạnh não thất. Sự hoạt hóa của các nhân này dẫn
tới sự giải phóng oxytocin và neurophysin từ các nang tiết ở thùy trước tuyến yên.
Trẻ bú không làm kích thích sự bài tiết ADH lượng đáng kể.
68. Một người đàn ông 30 tuổi bị hội chứng Conn. Những thay đổi sinh lý nào xảy
ra ở bệnh nhân so với người bình thường?
Huyết áp Thể tích Bài
động dịch xuất
mạch ngoại natri
bào
A) ↔ ↔ ↔
B) ↑ ↔ ↔
C) ↑ ↑ ↔
D) ↔ ↑ ↓
E) ↑ ↑ ↓
.C) Trong hội chứng Conn, lượng lớn aldosterone được bài tiết. Vì aldosterone
dẫn tới giữ natri, nên tăng huyết áp thường thấy ở bệnh nhân mắc hội chứng này.
Tuy nhiên, mức độ giữ natri laf khiêm tốn so với hệ quả làm tăng thể tích dịch
ngoại bào. Nó xảy ra vì sự tăng áp lực động mạch bù lại ảnh hưởng giữ natri của
aldosterone, giới hạn sự giữ natri và cho phép cân bằng natri trong ngày được giữ
vững.
69. Tại sao cho trẻ ăn mỗi vài tiếng lại quan trọng?
A) Khả năng dự trữ của gan và sự tổng hợp glycogen và glucose không đủ để duy
trì nồng độ glucose huyết tương ở giới hạn bình thường một vài giờ sau khi ăn
B) Nếu lượng dịch thích hợp không được bổ sung thường xuyên, nồng độ protein
huyết tương sẽ tăng lớn hơn bình thường trong vòng vài giờ
C) Chức năng của hệ tiêu hóa còn phát triển kém và có thể cải thiện bằng tiếp tục
thêm thức ăn vào dạ dày mọi thời điểm
D) Chức năng của gan trong việc hình thành protein huyết tương còn hạn chế và
yêu cầu một lượng sẵn có ổn định amino acid từ thức ăn để tránh phù do giảm
protein
 A) Vì chức năng gan chưa hoàn thiện trong tuần đầu của sự sống, nồng độ
glucose trong máu không ổn định và giảm xuống ngưỡng rất thấp vài giờ sau khi
cho ăn.
70. Điều gì sau đây liên quan đến việc thay đổi đồng thời sự bài tiết của
aldosterone và cortisol?
A) Bệnh Addison
B) Bệnh Cushing
C) Hội chứng Cushing (u tuyến thượng thận)
D) Chế độ ăn ít natri
E) Sử dụng thuốc ức chế enzyme chuyển
.A) Trong bệnh Addison, có sự giảm tiết của cả corticoid đường (cortisol) và
corticoid khoáng (aldosterone). Trong bệnh Cushing và hội chứng Cushing, sự bài
tiết cortisol tăng nhưng sự bài tiết aldosterone lại bình thường. Chế độ ăn nghèo
natri liên quan tới mức bài tiết aldosterone cao nhưng mức độ bài tiết cortisol bình
thường. Bằng việc ức chế các dòng ANG II và như vậy ảnh hưởng từ sự kích thích
của ANG II lên lớp cầu, việc cho dùng chất ức chế enzyme chuyển sẽ làm giảm tiết
aldosterone mà không làm thay đổi sự bài tiết của cortisol.
71. RU486 dẫn đến xảy thai nếu được dùng trước hoặc sau khi cấy hợp tử vào tử
cung. Hiệu quả đặc biệt của RU486 là?
A) Nó gắn với thụ thể của LH kích thích tiết progesterone ở hoàng thể
B) Nó ức chế thụ thể của progesterone làm cho progesterone không còn tác dụng
với cơ thể
C) Nó ức chế sư bài tiết của FSH ở tuyến yên
D) Nó ức chế tác dụng của thụ thể với oxytocin ở cơ tử cung
 B) Progesterone cần để duy trì các tế bào màng rụng ở nội mạc tử cung. Nếu
nồng độ progesterone hạ thấp thường xảy ra vào ngày cuối của chu kì kinh bình
thường ở phụ nữ không mang thai, thì chu kì kinh nguyệt sẽ xuất hiện sau vài ngày
gây xảy thai. Bổ sung các chất chặn thụ thể progesterone trong một vài ngày sau
khi thụ thai sẽ làm đình chỉ thai nghén.
72. Tiếp sau quá trình cương dương, dòng máu động mạch chảy vào vật hang giảm
về mức bình thường ở mức khi nghỉ ngơi dẫn đến trạng thái xìu. Lý giải hợp lý cho
sự giảm của dòng máu?
A) Áp lực động mạch hệ thống giảm do thiếu kích thích tình dục
B) Mức độ kích thích thần kinh giao cảm tới các tiểu động mạch cấp máu cho vật
hang giảm
C) Sức cản của tiểu động mạch cấp máu cho vật hang tăng
D) Sự hình thành NO trong tế bào nội mạc của tiểu động mạch cấp máu cho vật
hang được kích thích bởi sự tăng hoạt động của các sợi phó giao cảm
E) Sức cản của các tiểu tĩnh mạch thoát máu khỏi xoang của vật hang tăng
 C) Đáp án đúng duy nhất là tăng sức cản động mạch nhỏ trong hệ mạch cấp
máu cho vật cương (C). Các đáp án khác nhằm duy trì sự cương (B,D,E). (A), giảm
áp lực máu động mạch có ảnh hưởng không đáng kể.
73. Cái gì làm giảm tỷ lệ bài tiết ion calci của thận?
A) Tăng nồng độ calcitonin huyết tương
B) Giảm nồng độ ion phosphate huyết tương
C) Tăng nồng độ hormone tuyến cận giáp
D) Toan chuyển hóa
E) Tăng hoạt động của ion calci trong huyết tương
 C) Hormone cận giáp kích thích mạnh mẽ sự hấp thu ion calci từ dịch ống thận,
giảm bài xuất calci. Các lựa chọn khác không có ảnh hưởng hoặc làm tăng bài xuất
calci.
74. Một người đàn ông 55 tuổi đang tiến triển hội chứng liên quan đến sự tiết ADH
một cách không hợp lý do ung thư biểu mô ở phổi. Đáp ứng sinh lý nào có thể
mong đợi ở bệnh nhân này?
A) Tăng áp lực thẩm thấu huyết tương
B) Áp lực thẩm thấu nước tiểu thấp một cách không hợp lý (liên quan đến áp lực
thẩm thấu huyết tương)
C) Tăng cảm giác khát
D) Giảm tiết ADH ở tuyến yên
 D) Mức độ bài tiết cao không phù hợp của ADH từ phổi thúc đẩy sự tái hấp thu
nước quá mức, dẫn đến giảm thể tích nước tiểu và giảm áp lực thẩm thấu huyết
tương. Áp lực thẩm thấu huyết tương thấp làm giảm cảm khát và ức chế tiết ADH
từ tuyến yên.
75. Trong thời kì mang thai, cơ trơn tử cung bị ngừng hoạt động. Khoảng tháng thứ
9 của thai kì cơ tử cung trở nên dần dần bị kích động hơn. Yếu tố nào góp phần vào
sự tăng kích động đó?
A) Sự tổng hợp Estrogen của nhau thai tăng ở mức độ cao
B) Progesterone tổng hợp ở nhau thai giảm
C) Dòng máu đến tử cung đạt mức độ cao nhất
D) Prostaglandin E2 tổng hợp bởi nhau thai giảm
E) Hoạt động của thai nhi giảm xuống mức thấp
 B) Nồng độ rất cao của progesterone duy trì thạng thái tĩnh của cơ tử cung trong
quá trình mang thai. Trong tháng cuối của thai kì, nồng độ progesterone bắt đầu hạ,
gây tăng tính hưng phấn của cơ tử cung.
76. Một người đàn ông 40 tuổi bị hạ natri được cho dùng chất ức chế enzyme
chuyển angiotensin (ACE) trong 2 tuần. Những thay đổi sinh lý nào xảy ra sau khi
bệnh nhân dùng chất ức chế ACE trong 2 tuần?

 E) Khi natri cạn kiệt, hệ renin-angiotensin được hoạt hóa, nồng độ cao
angiotensin II tuần hoàn kích thích tuyến thượng thận tăng tiết aldosterone. Ngược
lại, angiotensin II không có tác dụng trong việc bài tiết cortisol, và sự cạn kiệt natri
liên quan với nồng độ bình thường cortisol huyết tương. Hậu quả, giảm nồng độ
angiotensin II huyết tương do dùng chất ức chế ACE sẽ làm giảm aldosterone
huyết tương xuống mức bình thường nhưng không ảnh hưởng tới nồng độ cortisol
huyết tương. Vì nồng độ cao aldosterone gây giữ natri, giảm aldosterone bằng việc
dùng chất ức chế ACE có xu hướng dẫn tới đái natri và giảm huyết áp động mạch.
Cùng lúc đó, tác dụng đối kháng của giảm huyết áp và giảm bài tiết aldosterone
trong sự bài xuất natri sẽ bù trừ cho nhau, và cân bằng natri sẽ luôn đạt được ở
mức huyết áp thấp.
77. Một phụ nữ 20 tuổi không có kinh nguyệt. Nồng độ progesterone huyết tương
được tìm thấy ở mức tối thiểu. Đâu là lời giải thích phù hợp cho việc nồng độ
progesterone thấp như vậy?
A) Mức bài tiết LH tăng
B) Mức bài tiết LH bị ức chế
C) Mức bài tiết FSH bị ức chế
D) Không có hoàng thể
E) Nồng độ inhibin cao trong huyết tương đã ức chế sự tổng hợp progesterone
 D) Hoàng thể là nguồn tiết progesterone duy nhất, và nếu cố ấy không có kinh
thì sẽ không có hoàng thể.
78. Trước khi LH đạt đỉnh trong thời kì rụng trứng, tế bào lớp hạt của nang trứng
tiết ra chất gì?
A) Testosterone
B) Progesterone
C) Estrogen
D) Inhibin
 C) FSH kích thích tế bào nang của nang trứng tiết estrogen.
Câu 79 – 80
79. Dựa vào biểu đồ sau, nhóm đường cong nào phản ánh đáp ứng của một cá thể
khỏe mạnh và một bệnh nhân bị đái tháo đường type 1 và type 2?
 E) Trong đáp ứng với tăng đường huyết, nồng độ insulin huyết tương thường
tăng trong khoảng 60 phút sau khi nhập glucose qua đường miệng. Đái tháo đường
type 1, sự bài tiết insulin bị hạ xuống. Ngược lại, đái tháo đường type 2, sự kháng
insulin là thường thấy và ở giai đoạn sớm của bệnh, mức bài tiết insulin cao bất
thường.

80. Dựa vào đồ thị trên, nhóm đường cong nào phản ánh đáp ứng của một cá thể
khỏe mạnh và một bệnh nhân bị hội chứng Cushing giai đoạn sớm?
Khỏe Hội chứng
mạnh Cushing
A) 3 2
B) 1 2
C) 1 3
D) 2 1
E) 2 3
 D) Trong hội chứng Cushing, nồng độ cao cortisol huyết tương làm giảm sự thu
nhập glucose ở mô ngoại vi dẫn tới tăng đường huyết. Dẫn tới, insulin đáp ứng khi
nhập glucose bằng đường miệng được tăng cường.
81. Trẻ sơ sinh được được đặt trong lồng ấp 100% Oxi trong vài ngày trở nên mù
lòa khi chúng được đưa ra ngoài, điều này được cho là do bệnh võng mạc do sinh
non. Sự giải thích đúng cho việc mất thị lực?
A) Nồng độ cao oxi kích thích sự phát triển mô xơ ở võng mạc
B) Nồng độ cao oxi dẫn tới gián đoạn mạch máu trong võng mạc dẫn tới sự xâm
nhập sợi vào dịch thủy tinh
C) Nồng độ cao oxi làm chậm phát triển mạch máu trong võng mạc nhưng khi
dừng liệu pháp oxi, sự giảm nồng độ oxi máu làm kích thích sự phát triển quá mức
của mạch máu võng mạc và dịch thủy tinh, sau đó ngưng kết sợi và ngăn cản ánh
sáng vào đồng tử
D) Nồng độ cao oxi phá hủy tế bào thần kinh võng mạc
C) Quá nhiều oxi trong lồng ấp làm ngừng sự phát triển mạch máu mới trong
võng mạc. Khi dừng liệu pháp oxi, sự phát triển quá mức của võng mạc xảy ra với
một lượng lớn mạch phát triển qua dịch kính. Sau đó các mạch máu bị thay thế bởi Commented [LTN93]: vào buồng

tổ chức xơ dẫn tới mù vĩnh viễn.


82. Hormone nào hoạt hóa thụ thể liên kết enzyme?
A) ADH
B) Insulin
C) ACTH
D) PTH
E) Aldosterone
 B) Nhìn chung, hormone có bản chất protein gây ra tác dụng sinh lý nhờ gắn
vào thụ thể màng tế bào. Tuy nhiên, trong 4 protein hormone trên chỉ có insulin
hoạt hóa thụ thể liên kết với enzyme. Aldosterone là steroid hormone và thâm nhập
vào tế bào chất trước khi găn vào thụ thể của nó.
83. Chất gì được sản xuất bởi tế bào lá nuôi trong 3 tuần đầu của thai kì?
A) Estrogen
B) LH
C) Oxytocin
D) HCG
E) Không chất nào ở trên
 D) HCG được tiết ra từ tế bào nuôi ngay sau khi phôi bào gắn vào nội mạc tử
cung.
84. Điều gì có thể thấy ở một bệnh nhân bị phù niêm? Commented [LTN94]: Triệu chứng nào sau đây thường
gặp trên
A) Tình trạng mơ màng Commented [LTN95]: Ngủ gà

B) Đánh trống ngực


C) Tăng tần số thở
D) Tăng cung lượng tim
E) Giảm cân
 A) Tình trạng mơ màng là một đặc điểm hay gặp trong suy tuyến giáp. Đánh Commented [LTN96]: Ngủ gà là triệu chứng

trống ngực, tăng tần số thở, tăng cung lượng tim, và giảm cân đều liên quan tới
cường giáp.
85. Trong khi sinh, một trẻ to được nuôi dưỡng tốt được thấy là có nồng độ glucose Commented [LTN97]: [ ]

huyết tương là 17 mg/dL (bình thường 80 đến 100 mg/dL) và nồng độ insulin
huyết tương lớn hơn 2 lần bình thường. Lời giải thích cho những điều trên là? Commented [LTN98]: gấp

A) Trẻ sơ sinh bị suy dinh dưỡng trong tử cung


B) Mẹ thiếu nuôi dưỡng trong quá trình mang thai Commented [LTN99]: suy dinh dưỡng

C) Mẹ bị tiểu đường, kiểm soát đường huyết kém


D) Mẹ bị thừa cân
 C) Trẻ sinh ra ở một bà mẹ bị đái tháo đường không điều trị sẽ có thể bị phì đại, Commented [LTN100]: Con của

quá sản chức năng một cách đáng kể tiểu đảo Langerhans ở tụy. Dẫn đến hậu quả,
đường huyết của trẻ có thể hạ thấp dưới 20mg/dL ngay sau khi sinh.
86. Điều gì kích thích tiết PTH?
A) Tăng hoạt động của ion calci ngoài tế bào trên giá trị bình thường
B) Tăng nồng độ calcitonin
C) Toan hô hấp
D) Tăng tiết hormone giải phóng PTH ở vùng dưới đồi
E) Tất cả đều sai
 E) A đến D không kích thích bài tiết PTH: (A) (tăng nồng độ Ca) ức chế tiết
PTH; (B) (calcitonin) không ảnh hưởng đến quá trình bài tiết PTH; (C) (acidosis)
tăng Ca tự do trong dịch ngoại bào do đó ức chế bài tiết PTH; không như (D)
(hormone kích thích giải phóng PTH).
87. Một phụ nữ 40 tuổi ăn một chế độ giàu kali trong vài tuần. Thay đổi thuộc
hormone có thể xảy ra là?
A) Tăng tiết dehydroepiandrosterone
B) Tăng tiết cortisol
C) Tăng tiết aldosterone
D) Tăng tiết ACTH
E) Giảm tiết CRH
.C) Kali là tác nhân kích thích tiềm năng cho sự bài tiết aldosterone giống như
angiotensin II. Do đó, bệnh nhân ăn chế độ giàu kali sẽ thể hiện nồng độ Commented [LTN101]: có
aldosterone cao trong tuần hoàn. Commented [LTN102]: máu
Commented [LTN103]: [ ]
88.Những thay đổi sinh lý nào xảy ra trên bệnh nhân bị bệnh to cực chi
giai đoạn sớm?
 C) Sự bài tiết quả mức hormone GH ở tuổi trưởng thành gọi là bệnh to
cực chi và thường liên quan đến u tuyến yên. Tăng nồng độ GH huyết tương
kích thích gan và các mô khác sản xuất somatomedin C. Dưới hệ quả của
điều hòa ngược, tăng nồng độ somatomedin C dẫn tới vùng dưới đồi tăng
tiết hormone ức chế GH, somatostatin. Tăng nồng độ GH huyết tương cũng
dẫn tới tăng nồng độ glucose huyết tương dẫn tới tăng bài tiết insulin.
89. Nếu một phụ nữ nghe thấy con cô ấy khóc, cô ấy có thể trải qua sự tiết sữa ở
đầu vú ngay cả trước khi đứa trẻ được bú. Lời giải thích cho điều này là?
A) Tiếng khóc của đứa trẻ đang đói gây ra sự bài tiết oxytocin từ thùy sau tuyến
yên, nó đến vú và dẫn đến sự co của các tế bào cơ biểu mô Commented [LTN104]: [ ]

B) Tiếng khóc của đứa trẻ đang đói dẫn đến phản xạ giãn các tế bào cơ biểu mô
cho phép sữa chảy ra
C) Tiếng khóc của đứa trẻ đang đói gây ra sự tiết nhiều prolactin từ thùy trước
tuyến yên, nó kích thích bài tiết sữa từ vú một cách nhanh chóng
D) Tiếng khóc của đứa trẻ đang đói gây ra sự phóng xung của hệ thần kinh giao
cảm dẫn tới sự co của các tế bào cơ biểu mô
 A) Tín hiệu thần kinh ở vùng cao hơn của não đến vùng dưới đồi có thể kích
thích sự bài tiết oxytocin vào máu từ thùy trước tuyến yên. Khi đến vú, oxytocin
kích thích co các tế bào cơ biểu mô, dồn sữa từ các ống ra đầu vú.
90. Những hormone nào không được lưu trữ ở tuyến nội tiết sản xuất nó?
A) T4
B) PTH
C) Aldosterone
D) ACTH
E) Insulin
 C) Hormone steroid không được dự trữ ở một cách đáng kể ở tuyến nội tiết sinh Commented [LTN105]: [ ]

ra nó. Điều này đúng cho aldosterone, chất được sản xuất ở vỏ tuyến thượng thận.
Đối lập với nó, có sự dự trữ một cách đáng kể hormone tuyến giáp và các peptide
hormone tại nơi sản xuất ra nó.
91. Một phụ nữ trẻ đến phòng cấp cứu với một gãy lún cột sống. X quang cột sống
cho thấy sự giảm khoáng hóa chung. Cô ấy ăn chay, không hút thuốc, nồng độ kali Commented [LTN106]: toàn bộ

huyết tương bình thường 5.4 mEq/L, nồng độ natri 136 mEq/L, nồng độ calci
huyết tương 7.0 mg/dL. Giá trị vitamin D3 trong một vài thời điểm lớn hơn bình
thường mặc dù nồng độ 1,25-dihydroxycholecalciferol ở mức giới hạn thấp có thể
tìm thấy. Cô ấy bị suy thận 5 năm và lọc máu 3 lần mỗi tuần. Đâu là nguyên nhân Commented [LTN107]: [ ]

của nồng độ 1,25-dihydroxycholecalciferol thấp ở cô gái này? Commented [LTN108]: nay

A) Toan chuyển hóa


B) Kiềm chuyển hóa
C) Cô ấy không thể tạo thành 1,25-dihydroxycholecalciferol bởi bệnh thận lan tỏa
D) Cô ấy đang lọc máu với dịch lọc không có calci
E) Cô ấy không được bổ sung calci
 C) 1,25-dihydroxycholecalciferol chỉ được tạo thành tại vỏ thận. Đa phần các
bệnh thận làm giảm số lượng các tế bào vỏ, làm mất nguồn điều hòa tích cực calci,
1,25-dihydroxycholecalciferol.
92. Một trẻ sơ sinh tiến triển vàng da với nồng độ bilirubin 10 mg/dL trong ngày
thứ 2 ( bình thường 3 mg/dL trong ngày thứ 2). Bác sĩ nhũ nhi có thể tự tin cho
rằng tình trạng này là tan máu sơ sinh nếu điều nào sau đây đúng?
A) Nồng độ bilirubin không tăng lên nữa
B) Hematocrit chỉ giảm nhẹ
C) Bố, mẹ, và đứa trẻ đều Rh- Commented [LTN109]: (có nhóm máu)

D) Mẹ không có tiền sử suy gan Commented [LTN110]: rối loạn chức năng gan

 C) Để cho quá trình tan máu sơ sinh xảy ra, đứa trẻ phải được nhận tê bào máu
có Rh+ từ cha. Nếu mẹ có Rh-, sẽ xuất hiện miễn dịch chống lại kháng nguyên
Rh+ trong máu con, kháng thể mẹ sẽ hủy hoại tế bào hồng cầu của con, giải phóng Commented [LTN111]: của

một lượng lớn bilirubin vào huyết tương thai nhi.


93. Điều gì có thể được phàn nàn bởi một bệnh nhân ăn thiếu iod? Commented [LTN112]: Triệu chứng có thể gặp ở

A) Giảm cân
B) Căng thẳng
C) Tăng tiết mồ hôi
D) Tăng tổng hợp thyroglubulin
E) Nhịp tim nhanh
 D) Vì iod cần cho sự tổng hợp hormone tuyến giáp, sự sản xuất hormone tuyến
giáp sẽ bị hạn chế nếu thiếu iod. Theo cơ chế điều hòa ngược, nồng độ hormone
kích thích tuyến giáp sẽ tăng và kích thích tế bào nang tăng tiết thyroglobulin. Điều
này dẫn tới bướu cổ. Tăng chuyển hóa, đổ mồ hôi, lo lắng, tim đập nhanh là những
biểu hiện thường gặp của cường giáp, không thấy trong suy giáp do thiếu iod.
94. Trước khi giao hợp, một người phụ nữ rửa âm đạo của cô ấy với dung dịch làm
giảm pH của dich âm đạo xuống 4.5. Điều này sẽ ảnh hưởng đến tinh trùng trong
âm đạo như thế nào?
A) Mức chuyển hóa sẽ tăng
B) Mức độ chuyển động sẽ giảm
C) Sự tạo thành prostaglandin E2 sẽ tăng
D) Mức tiêu thụ oxy sẽ tăng
 B) Khả năng di động của tinh trùng giảm khi pH giảm dưới 6.8. Ở pH 4.5, khả
năng di động của tinh trùng giảm một cách đáng kể. Tuy nhiên, tác dụng đệm của
natri bicarbonate trong dịch tuyến tiền liệt tăng pH phần nào cho phép tinh trùng
lấy lại khả năng di động. Commented [LTN113]: một phần

95. Đáp ứng thuộc hormone nào có thể xảy ra sau bữa ăn giàu protein?

.B) Bữa ăn có protein kích thích cả 3 hormone nêu trên.


96. Đàn ông dùng một lượng lớn testosterone-like androgenic steroids trong một Commented [LTN114]: steroid androgen dạng
testosteron
thời gian dài bị vô sinh. Đâu là giải thích cho hiện tượng này?
A) Nồng độ cao androgen gắn vào thụ thể của testosterone trên tế bào Sertoli dẫn
tới kích thích quá mức sự tạo thành inhibin
B) Kích thích quá mức sản sinh tế bào tinh trùng dẫn tới sự hình thành tinh trùng
khiếm khuyết
C) Nồng độ cao androgen ức chế sự tiết GnRH của vùng dưới đồi dẫn tới ức chế
LH và FSH giải phóng ở thùy trước tuyến yên
D).Nồng độ cao androgen sinh ra phì đại mất chức năng tuyến tiền liệt
 C) Testosterone tiết ra bởi tinh hoàn bởi đáp ứng với LH ức chế tiết GnRH ở
vùng dưới đồi, do đó ức chế thùy trước tuyến yên tiết LH và FSH. Dùng liều lớn
steroid giống testosterone cũng làm suy yếu sự bài tiết GnRH và hormone hướng
sinh dục của tuyến yên dẫn đến việc không có tinh trùng.
97. Một phụ nữ 30 tuổi dùng cortisone để điều trị bệnh tự miễn. Điều gì có thể xảy
ra?
A) Tăng tiết ACTH
B) Tăng tiết cortisone
C) Tăng tiết insulin
D) Tăng khối cơ
E) Hạ đường huyết giữa các bữa ăn
 C) Steroid với hoạt động mạnh của glucocorticoid có xu hướng làm tăng đường
huyết. Dẫn tới, sự bài tiết insulin được kích thích. Tăng hoạt động của
glucocorticoid cũng làm giảm protein cơ. Do điều hòa ngược, sự bổ sung cortisone
dẫn tới giảm sự bài tiết ACTH và do đó giảm nồng độ cortisol huyết tương.
98. Chức năng của cái gì sẽ tăng khi tăng nồng độ hormone tuyến cận giáp? Commented [LTN115]: nào sau đây

A) Hủy cốt bào


B) Sự tạo thành 25-hydroxycholecalciferol ở gan
C) Con đường tái hấp thu phosphate ở ống thận
D) Tất cả các điều trên
 A) Tăng nồng độ của hormone cận giáp dẫn tới kích thích sự tồn tại của hủy cốt
bào, qua thời gian làm tăng lượng hủy cốt bào ở xương.
99. Câu nào về hormone có nguồn gốc peptide và protein thường đúng?
A) Chúng có thời gian bán hủy lâu hơn hormone steroid
B) Chúng có thụ thể trên màng tế bào
C) Chúng bắt đầu có tác dụng chậm hơn so với hormone steroid và hormone tuyến
giáp
D) Chúng không được dự trữ tại các tuyến chế tiết
 B) Nhìn chung, peptide hormone gây ra tác dụng sinh học nhờ việc gắn với thụ Commented [LTN116]: các hormon peptid

thể trên màng tế bào. Peptide hormone được lưu trữ trong nang chế tiết tại tế bào Commented [LTN117]: nt
tuyến sản xuất ra nó và có thời gian bán hủy tương đối ngắn bởi chúng không có
khả năng gắn cao với protein huyết tương. Protein hormone thường tác dụng nhanh Commented [LTN118]: Các hormon protein

bởi vì không như steroid hormone và hormone tuyến giáp, sự tổng hợp protein Commented [LTN119]: hormon steroid
thường không sinh ra chất có tác dụng sinh học ngay.
100. Những thay đổi sinh lý nào xảy ra ở bệnh nhân to cực chi?

 D) U tuyến yên chế tiết GH xuất hiện làm tăng kích thước tuyến yên. Ảnh
hưởng của việc tiết quá nhiều GH dẫn tới tăng kích thước của nội tạng trong đó có
thận. Vì to cực chi là một dạng của chế tiết quá mức hormone GH sau khi đầu
xương đã lại gần nhau, tăng kích thước xương đùi không thể xảy ra.
101. Cortisol và GH có tác dụng không tương đồng trong quá trình chuyển hóa
nào?
A) Tổng hợp protein ở cơ
B) Sự vận chuyển glucose vào mô ngoại vi
C) Nồng độ glucose huyết tương
D) Huy động triglycerides
 A) GH va cortisol có tác dụng đối lập nhau trong quá trình tổng hợp protein của
cơ. GH đồng hóa và thúc đẩy tổng hợp protein ở phần lớn tế bào của cơ thể, trong
khi cortisol giảm tổng hợp protein tại các tế bào ngoài gan trong đó có cơ. Cả hai
hormone hạn chế glucose vào mô ngoại vi và do đó tăng đường huyết. Cả hai cũng
có tác dục huy động triglyceride từ nguồn lưu trữ chất béo.
102. Đứa trẻ của những bà mẹ có chế độ ăn cân bằng trong quá trình thai kì không
cần bổ sung sắt hoặc chế độ ăn giàu sắt trong khoảng 3 tháng đầu. Tawij sao lại
thế? Commented [LTN120]: vì?

A) Sự phát triển của đứa trẻ không yêu cầu đến sắt cho đến khi qua 3 tháng tuổi Commented [LTN121]: tăng trưởng
Commented [LTN122]: tới sau
B) Gan của trẻ dự trữ đủ sắt cho nhu cầu của đứa trẻ cho đến khi được 3 tháng tuổi
C) Sự tổng hợp hồng cầu mới bắt đầu sau 3 tháng tuổi
D) Tế bào cơ phát triển trước tháng thứ 3 không chứa myoglobin
 B) Nếu người mẹ có chế độ ăn với lượng sắt cân bằng, gan của trẻ thường có đủ
nguồn dự trữ sắt cho việc hình thành tế bào máu trong 4 đến 6 tháng sau sinh. Tuy
nhiên, nếu mẹ không đủ sắt, đứa trẻ sẽ thiếu máu trầm trọng sau 3 tháng của cuộc
đời.
103. Cái gì ít liên quan đến nhiễm độc giáp? Commented [LTN123]: Triệu chứng nào sau đây (ít nghĩ
tới)
A) Nhịp tim nhanh
B) Tăng cảm giác thèm ăn
C) Tình trạng lơ mơ Commented [LTN124]: ngủ gà

D) Tăng tiết mồ hôi


E) Run cơ
C) Nhiễm độc giáp biệu hiện ảnh hưởng của tăng quá mức hormone tuyến giáp.
Hormone tuyến giáp làm hưng phấn synap. Đối lập với nó, tình trạng lơ mơ đặc
trưng cho suy giáp. Tim đập nhanh, tăng cảm giác thèm ăn, tăng tiết mồ hôi và run
cơ đều là dấu hiệu của cường giáp
104. Nơi nào thường diễn ra sự thụ tinh?
A) Tử cung
B) Cổ tử cung
C) Buồng trứng
D) Bóng của ống Fallopian
 D) Trứng được thụ tinh một cách bình thường tại bóng của một ống dẫn trứng.
105. Một bệnh nhân đến phòng cấp cứu và được phát hiện có một sự giảm nhẹ
dưới mức bình thường nồng độ calci trong máu ( ion calci hoạt động = 0.9
mmol/L), nồng độ phosphate xấp xỉ 50% dưới mức bình thường (HPO4− = 0.5
mmol/L), và không định lượng được lượng ion calci trong nước tiểu. Điều gì mong
chờ tìm được ở bệnh nhân?
A) Nồng độ calcitonin trong máu trên mức bình thường
B) Nồng độ hormone tuyến cận giáp trong máu lớn hơn bình thường
C) Ức chế hoạt động của hủy cốt bào ở xương
D) pH máu dưới bình thường
 B) Nồng độ calci thấp dưới mức bình thường ở bệnh nhân này sẽ được mong
đợi là kích thích mạnh mẽ sự bài tiết hormone cận giáp, đến lượt nó sẽ được trông
mong làm tăng tỷ lệ bài xuất ion phosphate ở thận và giảm tỷ lệ bài xuất ion calci
trong nước tiểu. Do đó, tất cả những thứ khai thác được góp phần cho việc tăng Commented [LTN125]: mọi dấu hiệu/triệu chứng này
đều
quá mức nồng độ hormone cận giáp.
Commented [LTN126]: /làm
106. Điều gì xảy ra ở bệnh nhân đái tháo đường type 1 không kiểm soát? Commented [LTN127]: cao hơn bình thường

A) Giảm áp lực thẩm thấu


B) Tăng thể tích huyết tương
C) Tăng pH huyết tương
D) Tăng giải phóng glucose ở gan
E) Giảm phân hủy lipid
 D) Vì sự bài tiết insulin thiếu ở đái tháo đường type 1, có sự tăng (không phải
giảm) giải phóng glucose từ gan. Nồng độ insulin huyết tương thấp cũng dẫn tới tỷ
lệ thoái hóa lipid cao; tăng áp lực thẩm thấu, giảm thể tích, và nhiễm toan là triệu
chứng của đái tháo đường type 1 không kiểm soát.
107. Sự bài tiết GH bị đàn áp bởi tình trạng nào sau đây? Commented [LTN128]: ức chế

A) Bệnh to cực chi


B) Bệnh khổng lồ
C) Ngủ sâu
D) Tập thể dục
E) Tăng đường huyết cấp
 E) Dưới tình trạng cấp, tăng đường máu sẽ giảm bài tiết GH. Sự bài tiết GH Commented [LTN129]: Tăng đường huyết cấp tính

tăng một cách đặc trưng trong trạng thái sinh lý bệnh mạn tính của bệnh to cực chi
và bệnh khổng lồ. Ngủ sâu và tập thể dục là tác nhân kích thích tăng tiết GH.
108. 2 ngày trước khi bắt đầu chu kì kinh nguyệt, sự bài tiết FSH và LH chạm đến
mức thấp nhất. Đâu là nguyên nhân của sự bài tiết thấp này? Commented [LTN130]: /hiện tượng

A) Thùy trước tuyến yên trở nên không đáp ứng với ảnh hưởng kích thích của
GnRH
B) Estrogen từ nang trứng đang phát triển dẫn tới ức chế ngược vùng dưới đồi
C) Tăng than nhiệt ức chế vùng dưới đồi tiết GnRH
D) Sự tiết estrogen, progesterone và inhibin bởi hoàng thể ức chế vừng dưới đồi
tiết GnRH và tuyến yên tiết FSH
 D) Estrogen, và ở mức độ thấp hơn, progesterone tiết bởi hoàng thể trong giai
đoạn hoàng thể có tác dụng điều hòa ngược mạnh mẽ thùy trước tuyến yên để duy
trì mức bài tiết thấp của LH và FSH. Thêm vào đó, hoàng thể tiết inhibin, ức chế
tiết FSH.
109. Điều gì xảy ra trong giai đoạn sớm nhất của đái tháo đường type 2?
A) Tăng độ nhạy cảm của insulin
B) Nồng độ C-peptide tuần hoàn cao
C) Giảm glucose gan đầu ra
D) Toan chuyển hóa
E) Giảm thể tích tuần hoàn
 B) Giai đoạn sớm của type 2 liên quan đến sự thiếu nhạy cảm của mô đích với
tác dụng chuyển hóa của insulin, đó là tình trạng kháng insulin. Giảm độ nhạy cảm
insulin có xu hướng làm tăng đường huyết, một phần thúc đẩy gan giải phóng
glucose. Tăng đường huyết dẫn tới tăng bù sự bài tiết insulin và peptide-C, một
mảnh của tiền insulin. Toan chuyển hóa và giảm thế tích diễn ra ở đái tháo đường
type 1 nhưng không xảy ra ở giai đoạn sơm của đái tháo đường type 2.
110. Một đứa trẻ sinh ra với dương vật, bìu nhưng không có tinh hoàn, không có
âm đạo, NST XX. Những điều trên hướng tới chẩn đoán lưỡng giới. Đâu là nguyên
nhân của bất thường này?
A) Sản xuất cao bất thường HCG của tế bào lá nuôi phôi
B) Tỷ lệ sản xuất estrogen của nhau thai thấp bất thường
C) Nồng độ cao bất thường LH trong máu mẹ
D) Nồng độ thấp bất thường testosterone trong máu mẹ
 D) Nếu phụ nữ mang thai một trẻ nữ có nồng độ cao androgenic hormone trong Commented [LTN131]: người mẹ

giai đoạn sớm của thai kì, đứa trẻ sinh ra sẽ có cơ quan sinh dục nam dẫn tới một
dạng của lưỡng giới.
111. Cái gì góp phần tạo nên “thoát natri” trong hội chứng Conn? Commented [LTN132]: /Điều
Commented [LTN133]: /gây
A) Giảm nồng độ peptide bài natri của tâm nhĩ
B) Tăng nồng độ angiotensin II huyết tương
C) Giảm tái hấp thu natri ở ống góp
D) Tăng áp lức động mạch
 D) Dưới điều kiện mạn tính, ảnh hưởng của nồng độ cao aldosterone trong việc Commented [LTN134]: Ở một số tình trạng bệnh mạn
tính
thúc đẩy tái hấp thu natri ở ống góp được duy trì. Tuy nhiên, sự ứ đọng natri liên
tục không xảy ra vì các thay đổi đồng thời thúc đẩy bài xuất natri. Chúng bao gồm
tăng huyết áp động mạch và tăng nồng độ peptide bài natri ở nhĩ (ANP) và giảm
angiotensin 2 huyết tương.
112. Một nhà khoa học nghiên cứu về sinh lý phát triển tâm thần tiến hành một thí
nghiệm trong đó một chất được cung cấp cho những con chuột mang thai đẻ ra các
con chuột con có NST XY nhưng có cơ quan sinh sản nữ. Chất chuột được dùng
là?
A) Kháng thể ức chế hiệu quả của HCG trong phôi và thai Commented [LTN135]: tác dụng

B) Lượng lớn chất giống estrogen


C) FSH
D) Testosterone
 A) Để thai nam có thể phát triển cơ quan sinh dục nam, testosterone phải xuất
hiện trong bào thai. Bình thường, HCG tiết bởi tế bào nuôi kích thích tiết
testosterone từ tinh hoàn. Cho kháng thể chặn HCG sẽ ngăn tiết testosterone dẫn
tới sự phát triển cơ quan sinh dục nữ ở thai nhi nam.
113. Một thí nghiệm được tiến hành trong đó bệnh nhân nhóm 1 được cho chất X,
bệnh nhân nhóm 2 được cho chất Y. Sau 3 tuần, nghiên cứu chỉ ra bệnh nhân nhóm Commented [LTN136]: dùng

1 có tỷ lệ bài tiết ACTH cao hơn và nồng độ glucose máu thấp hơn nhóm 2. Chất
X và Y là?
Chất X Chất Y
A) Cortisone Giả dược
B) Cortisol Giả dược
C) Giả dược Cortisol
D) ACTH Giả dược
E) Giả dược ACTH
 C) Tăng nồng độ cortisol huyết tương làm tăng đường huyết và ức chế bài tiết
ACTH. Do đó, nếu cortisol được cung cấp cho bệnh nhân nhóm 2, bệnh nhân
nhóm 1 sẽ có đường huyết thấp hơn và lượng huyết tương cao hơn.
114. Một phụ nữ 30 tuổi đến với bác sĩ lâm sàng để khám sức khỏe thường kì. Commented [LTN137]: [ ]

Khám cho thấy cô ấy có thai. Nồng độ TSH cao nhưng nồng độ T4 (gắn với Commented [LTN138]: Kết quả thăm khám
protein và tự do) bình thường. Cái gì phản ánh tốt nhất lâm sàng của bệnh nhân? Commented [LTN139]: Biểu hiện lâm sàng của bệnh
nhân giống nhất với
A) Bệnh Graves
B) Bệnh Hashimoto
C) U tuyến yên chế tiết TSH
D) U vùng dưới đồi chế tiết TRH
E) Bệnh nhân dùng chiết xuất tuyến giáp
 B) Lượng tuần hoàn của T4 tự do gây tác dụng sinh học và được điều hòa bởi
cơ chế ức chế ngược sự bài tiết TSH từ thùy trước tuyến yên. T4 gắn với protein là
chất không có hoạt tính sinh học. T4 tuần hoàn gắn mạnh với protein huyết tương,
đặc biệt với globulin gắn hormone tuyến giáp (TGB), chất tăng lên trong thời kì
mang thai. Tăng TBG làm giảm T4 tự do, dẫn tới tăng bài tiết TSH, làm tuyến giáp
tăng tiết hormone. Tăng tiết hormone tuyến giáp duy trì đến khi T4 tự do quay về
mức bình thường, tại thời điểm đó không còn sự kích thích tiết TSH. Do đó, trong
tình trạng ổn định mạn tính liên quan đến tăng TBG, nồng độ T4 tổng huyết tương
cao (gắn và tự do) và mức độ TSH huyết tương bình thường được mong chờ. Ở
phụ nức có thai, nồng độ T4 tổng bình thường theo cùng nồng độ cao TS huyết
tương chỉ ra sự thấp bất thường T4 huyết tương tự do. Thiếu bài tiết hormone
tuyến giáp ở bệnh nhân này sẽ phù hợp với bệnh Hashimoto, dạng thường gặp của
suy giáp.
115. Chất chuyển hóa nào được chuyển hóa một cách ưu tiên bởi GH?
A) Chất béo
B) Protein
C) Glycogen
D) Glucose
 A) Chất béo sẵn sàng bị oxi hóa bởi GH. Ngược lại, GH làm giảm sử dụng
carbonhydrate và thúc đẩy sự hợp nhất của các aminoacid thành protein.
116. Một người đàn ông bị bệnh dẫn tới hủy hoại tế bào thần kinh vận động ở đĩa
tủy bên dưới vùng ngực. Chức năng sinh dục nào không thực hiện được?
A) Hưng tính
B) Sự cương dương
C) Sự bôi trơn
D) Sự xuất tinh
 D) Nơ ron vận động của đốt tủy thắt lưng và ngực là nơi kích thích các cơ của
đáy chậu có liên quan đến xuất tinh.
117. Một chương trình nâng tạ nặng dài ngày không gián đoạn sẽ làm tăng khối
xương. Đâu là cơ chế của ảnh hưởng này từ việc nâng tạ?
A) Tăng hoạt động chuyển hóa kích thích sự bài tiết hormone cận giáp
B) Stress cơ học lên xương tăng hoạt động của tạo cốt bào
C) Tăng hoạt động chuyển hóa dẫn đến sự tăng lượng calci chế độ ăn vào cơ thể
D) Tăng hoạt động chuyển hóa dẫn đến sự kích thích tiết calcitonin
 B) Xương được lắng đọng tương ứng với lượng sức ép mà xương đó phải chịu.
Áp lực cơ học nặng kích thích tạo cốt bào lắng đọng và vôi hóa xương.
118. Mức transcortin bị tăng ở phụ nữ có thai. Xét nghiệm nào có thể mong chờ
thấy ở bệnh nhân? Commented [LTN140]: Có thể thu được kết quả xét
nghiệm nào
A) Tăng nồng độ tổng cortisol huyết tương (gắn protein và tự do)
B) Tăng nồng độ cortisol tự do huyết tương (không gắn protein)
C) Giảm nồng độ tổng cortisol huyết tương
D) Giảm nồng độ cortisol tự do huyết tương
E) Thay đổi nhẹ hoặc không thay đổi nồng độ tổng cortisol huyết tương
 A) Cortisol có khả năng gắn cao với protein huyết tương, đặc biệt là transcortin.
Tăng nồng độ transcortin huyết tương, như trong quá trình mang thai, gây giảm
nồng độ cortisol tự do, nhưng hậu quả của điều hòa ngược trong tăng bài tiết
ACTH, sẽ kích thích bài tiết cortisol cho đến khi nồng độ steroid tự do huyết tương
chở về mức bình thường. Do đó, trong tình trạng ổn định, nồng độ cortisol huyết
tương tổng (gắn cộng tự do) bị tăng, nhưng nồng độ cortisol tự do vẫn bình
thường.
119. Thuốc tránh thai bao gồm sự kết hợp của estrogen và progesterone tổng hợp Commented [LTN141]: chứa

được dùng vào 21 ngày của chu kì kinh nguyệt có hiệu quả trong việc ngăn ngừa
mang thai. Giái thích cho hiệu quả này là:
A) Ngăn ngừa sự đạt đỉnh của LH tiết ra bởi tuyến yên trước khi trứng rụng
B) Ngăn chặn sự phát triển của nang trứng
C) Át chế chức năng của hoàng thể ngay sau khi nó được tạo thành
D) Ngăn ngừa sự phát triển bình thường của nội mạc tử cung
 A) Bổ sung estrogen hoặc progesterone ở một lượng thích hợp trong nửa đầu
chu kì kinh nguyệt có thể ngăn cản sự rụng trứng do ngăn chặn sự đạt đỉnh của LH
ở thùy trước tuyến yên giai đoạn trước rụng trứng, điều này cần cho sự rụng trứng.
120. Đáp ứng sinh lý nào với triiodothyronine (T3) mạnh hơn thyroxine (T4)?
A) Mức độ bài tiết ở tuyến giáp
B) Nồng độ huyết tương
C) Thời gian nửa đời sống ở huyết tương
D) Khả năng gắn với thụ thể trong nhân ở mô đích
E) Giai đoạn tiềm ẩn cho việc bắt đầu hoạt động trong mô đích
 D) Ở mô đích, thụ thể ở nhân của hormone tuyến giáp có ái lực cao với T3 hơn
là T4. Mức bài tiết, nửa thời gian sống, và khởi phát tác dụng đều lớn hơn ở T4 so
với T3.
121. Chất “tránh thai” cho đàn ông đã được tìm kiếm hàng thập kỉ. Chất nào cho
hiệu quả vô tinh trùng?
A) Chất bắt chước tác dụng của LH
B) Chất ức chế hoạt động của inhibin
C) Chất ức chế hoạt động của FSH
D) Chất bắt chước hoạt động của GnRH
 C) Ngăn chặn hoạt động của FSH trong tế bào Sertoli của ống sinh tinh làm
gián đoạn sự sản xuất tinh trùng. Chon C là lựa chọn duy nhất dẫn tới sự không sản Commented [LTN142]: [ ]

xuất tinh trùng. Commented [LTN143]: [ ]

122. Để cho sữa chảy ra từ đầu vú của mẹ vào miệng của đứa trẻ mới đẻ, điều gì
phải xảy ra?
A) Tế bào cơ biểu mô phải thư giãn
B) Nồng độ prolactin phải hạ xuống
C) Sự bài tiết oxytocin ở thùy sau tuyến yên phải xảy ra
D) Mồm đứa trẻ phải tạo đủ áp lực âm khỏe quanh đầu vú
E) Tất cả các điều trên
 C) Oxytocin được tiết từ thùy sau tuyến yên và được đưa vào máu đến tuyến vú,
tại đây nó tác động tới các tế bào ở thành ngoài của hệ thống nang và ống gây co.
Sự co của các tế bào này dẫn tới tăng áp lực thủy tĩnh của sữa trong ống lên 10 đến
20 mmHG. Hệ quả, dòng sữa chảy ra khỏi núm vú đến miệng đứa trẻ.
123. Thất bại trong việc đóng ống động mạch là một khiếm khuyết phát triển
thường gặp. Điều gì sẽ gặp ở đứa trẻ 12 tháng tuổi còn ống động mạch mà không
tắc lại?
A) PO2 động mạch dưới mức bình thường
B) PCO2 động mạch dưới mức bình thường
C) Áp lực máu động mạch lớn hơn bình thường
D) Áp lực động mạch phổi thấp hơn bình thường
 A) Nếu ống động mạch không tắc, máu ít oxi từ động mạch phổi chảy vào động Commented [LTN144]: /đóng

mạch chủ, nhận lượng oxi trong máu động mạch thấp hơn bình thường.
124. Những thay đổi sinh lý nào có thể mong chờ ở một bệnh nhân không có tình
trạng tiểu đường bị bệnh Cushing?

 F) Trong bệnh Cushing, có sự bài tiết ở mức độ cao cortisol, nhưng sự bài tiết
aldosterone bình thường. Nồng độ cortisol huyết tương cao gây tăng đường huyết
do hạn chế glucose vào tế bào ngoại vi và bởi thúc đẩy tân tạo đường. Tuy nhiên,
cuối giai đoạn sớm của bệnh Cushing, xu hướng tăng đáng kể đường huyết bị
chống lại bởi việc tăng tiết insulin.
125. Nhau thai:
A) Phát triển từ các tế bào hạt
B) Bài tiết LH
C) Bài tiết estrogen
D) Cho phép trực tiếp trộn máu mẹ và máu con
E) Không đáp án nào ở trên đúng
 C) Rau thai tiết cả estrogen và progesterone từ tế bào nuôi.
126. Tại sao loãng xương thường gặp ở phụ nữ già hơn đàn ông lớn tuổi? Commented [LTN145]: lớn tuổi

A) Đàn ông tiếp tục sản xuất testosterone trong suốt cuộc đời mình trong khi phụ
nữ ngưng sản xuất estrogen sau khi mãn kinh
B) Phụ nữ tiêu thụ ít calci trong chế độ ăn hơn đàn ông
C) Hệ tiêu hóa hấp thu calci hiệu quả hơn ở đàn ông so với phụ nữ
D) Xương phụ nữ chứa ít calci hơn đàn ông kể cả trước khi mãn kinh
 A) Testosterone kích thích chức năng của tế bào xương dẫn tới sự tạo xương.
Sự bài tiết testosterone ở các tế bào kẽ giảm theo tuổi, nhưng vẫn tiếp tục ở mức độ
đủ để kích thích tạo xương trong suốt cuộc đời của nam giới. Trái lại, sự sản xuất
estrogen ở phụ nữ giảm bằng 0 sau khi mãn kinh, dẫn đến xương không được kích
thích dưới tác dụng của estrogen. Hệ quả, loãng xương thường thấy ở phụ nữ mãn
kinh.
127. Khi so sánh với giá trị vào buổi tối muộn thường quan sát được ở đối tượng
bình thường, nồng độ huyết tương của cả ACTH và cortisol sẽ lớn hơn so với cá
thể nào sau đây?
A) Đối tượng bình thường sau khi đi bộ vào buổi sang
B) Đối tượng bình thường dùng dexamethasone
C) Bệnh nhân bị hội chứng Cushing (u tuyến thượng thận)
D) Bệnh nhân bị bệnh Addison
E) Bệnh nhân bị hội chứng Conn
 A) Ở bệnh nhân khỏe mạnh, mức độ bài tiết ACTH và cortisol thấp vào buổi tối
nhưng cao vào sáng sớm. Ở bệnh nhân bị bệnh Cushing (u biểu mô tuyến thượng
thận) hoặc bệnh nhân dùng dexamethasone, ACTH huyết tương rất thấp và tất
nhiên không cao hơn mức bình thường vào sáng sớm. Ở bệnh nhân bị bệnh
Addison, ACTH huyết tương tăng do sự giảm bài tiết cortisol của tuyến thượng
thận. Sự bài tiết ACTH và cortisol thường ở mức độ bình thường ở hội chứng
Conn.
128. Điều kiện hoặc hormone nào tăng sự bài tiết GH?
A) Tăng đường huyết
B) Tập thể dục
C) Somatomedin
D) Somatostatin
E) Tuổi tác
 B) Tập thể dục kích thích tiết GH. Tăng đường huyết, somatomedin, và
hormone ức chế vùng dưới đồi somatostatin đều ức chế tiết GH. Sự bài tiết GH
cũng giảm theo tuổi.
129. Mạch máu trong thai có Po2 cao nhất là?
A) Ống động mạch
B) Tĩnh mạch rốn
C) Động mạch chủ lên
D) Nhĩ trái
 B) Máu quay về từ nhau thai qua tĩnh mạch rốn qua ống tĩnh mạch. Máu từ
nhau thai có nồng độ oxi cao nhất ở thai nhi.
130. Một người phụ nữ 59 tuổi bị loãng xương, cao huyết áp, rậm lông và tăng sắc
tố. MRI cho thấy tuyến yên không to. Điều kiện nào dưới đây phù hợp với những
gì tìm thấy? Commented [LTN146]: Bệnh nào sau đây phù hợp nhất
với các triệu chứng trên
A) U chế tiết ACTH tuyến yên
B) U chế tiết ACTH ngoài tử cung
C) Mức bài tiết cao không phù hợp của CRH
D) U tuyến thượng thận
E) Bệnh Addison
 B) Loãng xương, cao huyết áp, rậm long và tăng săc tố là các triệu chứng của
hội chứng Cushing liên quan đến nồng độ cao ACTH. Nếu nồng độ ACTH huyết
tương cao là kết quả của một u biểu mô tuyến yên hoặc mức độ bài tiết cao bất
thường CRH từ vùng dưới đồi, bệnh nhân sẽ bị tăng kích thước tuyến yên. Ngược
lại, tuyến yên sẽ không to lên nếu một khối u ngoài tử cung tiết lượng lớn ACTH.
131. Hormone tuyến yên nào đáp ứng không phù hợp với hormone vùng dưới đồi
được liệt kê? (ACTH, adrenocorticotropic hormone; CRH, corticotropin-releasing
hormone; GH,
growth hormone; GnRH, gonadotropin-releasing hormone; LH, luteinizing
hormone; TRH, thyrotropinreleasing
hormone; TSH, thyroid-stimulating hormone)
Sự bài tiết Hormone
hormone vùng tuyến yên
dưới đồi
A) Somatostatin ↓GH
B) Dopamine ↑Prolactin
C) GnRH ↑LH
D) TRH ↑TSH
E) CRH ↑ACTH
 B) Sự bài tiết prolactin bị ức chế, không được kích thích bởi dopamine của vùng
dưới đồi qua cuống tuyến yên. GH bị ức chế bởi hormone ức chế ở vùng dưới đồi
somatostatin. Sự bài tiết LH, TSH và ACTH đều dưới sự kiểm soát của các
hormone giải phóng tường ứng.
132. Nồng độ calci ngoại bào còn lại chỉ giảm nhẹ so với bình thường trong vài
tháng khi calci từ thức ăn ở mức tối thiểu. Cái gì giải thích cho khả năng duy trì
nồng độ calci dịch ngoại bào?
A) Chỉ giảm nhẹ nồng độ calci huyết tương kích thích sự tăng nhiều chịu đựng
được bài tiết hormone tuyến cận giáp
B) Hủy cốt bào bị kích thích bởi nồng độ cao hormone cận giáp lấy calci từ nguồn
dự trữ lớn trong xương do đó duy trì nồng độ calci ngoại bào gần mức bình thường
C) Sự bài xuất calci ở thận giảm nhiều dưới sự ảnh hưởng của nồng độ cao
hormone tuyến cận giáp
D) Tất cả các điều trên
 D) Trong thiếu calci lâu dài, nồng độ calci huyết tương bắt đầu hạ xuống. Tuy
nhiên, khi giảm xuống, sự bài tiết hormone tuyến cận giáp tăng một cách mạnh mẽ, Commented [LTN147]: calci

do đó kích thích hủy cốt bào giáng hóa xương và giải phóng calci ra dịch ngoại
bào. Ngay thời điểm đó, nồng độ hormone cận giáp tăng mạnh kích thích tái hấp
thu calci ở ống góp của thận và giảm bài xuất calci xuống mức rất thấp.
133. Một bệnh nhân sử dụng lượng đủ thyroxine (T4) để tăng nồng độ hormone
huyêt tương lên vài lần. Những thay đổi nào ở bệnh nhân xuất hiện sau vài tuần
dùng T4?
 B) Tăng nhịp tim, tăng tần số thở và giảm nồng độ cholesterol là những đáp ứng
với tăng quá mức hormone tuyến giáp.
134. Trong giai đoạn cuối của thai kì, rất nhiều phụ nữ trải qua tình trạng mọc lông
cơ thể trong tình trạng mẫu nam tính. Lí giải cho hiện tượng này? Commented [LTN148]: giống nam

A) Buồng chứng tiết ra một lượng testosterone đi kèm với một lượng lớn estrogen
được sản xuất muộn trong thai kì
B) Buồng trứng và tinh hoàn thai nhi tiết ra androgenic steroids
C) Tuyến thượng thận của mẹ và thai nhi tiết ra lượng lớn androgenic steroids
được dùng để tạo estrogen ở nhau thai
D) Nhau thai tiết một lượng lớn estrogen, một số được chuyển hóa thành
testosterone
 C) Estrogen tiết bởi nhau thai không được tổng hợp từ các chất cơ bản của nhau
thai. Thay vào đó, nó được tạo từ các chất androgenic steroid được tạo từ tuyến
thượng thận của mẹ và thai nhi. Những chất androgenic được chuyển vào máu đến
nhau thai và chuyển hóa bởi tế bào nuôi thành estrogen. Nồng độ của chúng trong
máu mẹ có thể kích thích mọc long trên cơ thể.
135. Nguyên nhân của mãn kinh?
A) Giảm lượng hormone hướng sinh dục được tiết ra ở thùy trước tuyến yên
B) Giảm sự đáp ứng của buồng trứng với ảnh hưởng kích thích của hormone
hướng sinh dục
C) Giảm tỷ lệ bài tiết của progesterone của hoàng thể
D) Giảm số lượng nang trứng khả dĩ trong buồng trứng cho sự kích thích của
hormone hướng sinh dục
 D) Ở tuổi 45, chỉ có một số năng nguyên thủy còn lại trong buồng trứng để
được kích thích bởi hormone hướng sinh dục và sự sản xuất estrogen giảm khi số
nang trứng về không. Khi nồng độ estrogen giảm thấp dưới ngưỡng, nó không còn
khả năng ức chế sự sản xuất hormone hướng sinh dục của tuyến yên. FSH và LH
được sản xuất một lượng lớn nhưng các nang còn lại tịt, sự sản xuất bởi buồng
trứng bằng không.
136. Giải phóng hormone nào là ví dụ của sự bài tiết thần kinh nội tiết?
A) GH
B) Cortisol
C) Oxytocin
D) Prolactin
E) ACTH
 C) Sự bài tiết các chất truyền tin hóa học (hormone thần kinh) từ các nơron vào
máu được xem như sự bài tiết thần kinh nội tiết. Như vậy, đối lập với hoạt động tại
chỗ của chất dẫn truyền thần kinh ở đầu tận cùng thần kinh, hormone thần kinh
tuần hoàn trong máu trước khi gây tác dụng sinh học ở mô đích. Oxytocin được
tổng hợp từ tế bào nơron magnocellular, than của chúng ở nhân trên thị và nhân
cạnh não thất và đầu tận ở thùy sau tuyến yên. Mô đích của oxytocin tuần hoàn là
tuyến vú và tử cung, nơi mà hormone đóng vai trò tiết sữa và sinh đẻ.
137. Trong vòng 1 tuần sau sự rụng trứng, niêm mạc tử cung tăng độ dày lên 5 đến Commented [LTN149]: [ ]

6mm. Cái gì kích thích sự tăng chiều dày này? Commented [LTN150]: Chất nào

A) LH
B) Estrogen từ hoàng thể
C) Progesterone từ hoàng thể
D) FSH
 C) Progesterone tiết lượng lớn từ hoàng thể dẫn tới sự xung huyết đáng kể và
phát triển tuyến tiết ở nội mạc tử cung
138. Đáp ứng chuyển hóa nào mong đợi trong giai đoạn sau hấp thu so với giai Commented [LTN151]: sẽ xảy ra

đoạn vừa ăn xong?


 A) Vài giờ sau ăn (giai đoạn sau hấp thu), đường huyết có xu hướng giảm. Hậu
quả, sự bài tiết insulin giảm. Vì insulin ức chế lipase nhạy cảm hormone ở mô mỡ,
hoạt động của enzyme này tăng trong đáp ứng với giảm insulin huyết tương. Điều
này dẫn tới tăng thủy phân triglyceride. Sự giảm nồng độ insulin cũng giảm nhập
glucose vào tế bào mỡ, dẫn tới giảm sản xuất α-glycerol phosphate, chất cần thiết
trong việc tạo thành triglycerides từ acid béo. Tất cả những đáp ứng phụ thuộc
insulin ở tế bào mỡ thúc đẩy giải phóng acid béo vào tuần hoàn. Khi giai đoạn sau
ăn vẫn duy trì, sự giảm đường huyết kích thích bài tiết glucagon, dẫn tới xu hướng
bảo toàn đường huyết bằng kích thích phân hủy glycogen và tạo đường mới.
139. Sự ức chế bơm iod là nguyên nhân của thay đổi nào?
A) Tăng tổng hợp thyroxine (T4)
B) Tăng tổng hợp thyroglobulin
C) Tăng mức độ chuyển hóa
D) Giảm bài tiết TSH
E) Căng thẳng cực độ
 B) Sự ức chế bơm iod giảm sự tổng hợp hormone tuyến giáp nhưng không hạn
chế sản xuất thyroglobulin bới tế bào nang. Giảm nồng độ hormone tuyến giáp
huyết tương dẫn tới giảm chuyển hóa và dẫn tới tăng bài tiết TSH. Tăng nồng độ
TSH kích thích tế bào nang tổng hợp nhiều thyroglobulin. Lo lắng là triệu chứng
của cường giáp và không xảy ra khi thiếu hormone tuyến giáp.
140. Trước khi thụ thai, phôi nang lấy chất dinh dưỡng từ nội mạc tử cung tiết ra.
Làm thế nào phôi nang lấy được chất dinh dưỡng trong 1 tuần sau khi thụ thai?
A) Nó tiếp tục lấy chất dinh dưỡng từ nội mạc tử cung tiết ra
B) Các tế bào của phôi nang chứa chất dinh dưỡng dự trữ được chuyển hóa để hỗ
trợ dinh dưỡng
C) Nhau thaicung cấp dinh dưỡng lấy từ máu mẹ
D) Tế bào lá nuôi phôi tiêu hóa tế bào nội mạc tử cung giàu dinh dưỡng và hấp thu
chúng để sử dụng cho phôi nang
 D) Khi phôi bào bám thành công, tế bào nuôi xâm lấn niêm mạc tử cung, tiêu
hóa và hấp thu chúng. Dinh dưỡng dự trữ ở tế bào niêm mạc tử cung được dùng
bởi phôi thai cho sự lớn lên và phát triển của phôi. Trong một tuần sau thụ thai, đây
là nguồn duy nhất mà phôi có thể lấy dinh dưỡng. Thai tiếp tục lấy dinh dưỡng
theo cách này trong 8 tuần tiếp theo, mặc dù nhau thai đã hình thành để cung cấp
dinh dưỡng ngày thứ 16 sau thụ thai (ít hơn 1 tuần sau khi thụ thai)
141. Cái gì tăng tỷ lệ lắng đọng và giảm tỷ lệ hấp thu của xương? Commented [LTN152]: Điều gì làm

A) Sự tăng nồng độ hormone cận giáp


B) Sự tăng nồng độ estrogen
C) Sự tăng nồng độ ion H+ ngoại bào
D) Sự giảm stress cơ học lên xương
141.B) Sự tăng nồng độ estrogen kích thích hoạt động của tạo cốt bào và giảm
mức độ giáng hóa của xương bởi hủy cốt bào.
142. Hormone tuyến yên nào có cấu trúc hóa học tương tự như của ADH?
A) Oxytocin
B) ACTH
C) TSH
D) FSH
E) Prolactin
.A) Cả ADH và oxytocin đều là peptide chứa 9 aminoacid. Cấu trúc hóa học chỉ
khác nhau ở hai aminoacid.
143. Đâu là nguyên nhân thường gặp nhất dẫn tới hội chứng suy hô hấp cấp ở trẻ
đẻ non 7 tháng tuổi?
A) Phù phổi do tăng áp động mạch phổi
B) Sự tạo thành màng hyaline quang bề mặt phế nang
C) Thất bại trong tạo lượng đủ surfactant của niêm mạc phế nang
D) Tính thấm của màng phế nang với nước quá lớn
 C) Một trong các đặc điểm tìm được trong hội chứng suy hô hấp là giảm khả
năng tiết surfactant của niêm mạc đường hô hấp ở phế nang. Surfactant giảm sức
căng bề mặt của dịch phế nang, cho phép các phế nang nở dễ trong quá trình hít
vào. Nếu không đủ surfactant, phế nang bị co lại, và gây xu hướng hình thành phù
phổi.
144. Steroid hormones không được tổng hợp ở bất cứ mức độ nào lớp bó? Commented [LTN153]: Các hormone steroid

A) Aldosterone
B) Cortisol
C) Corticosterone
D) Dehydroepiandrosterone
E) Deoxycorticosterone
 A) Enzyme tổng hợp aldosterone không tìm thấy trong lớp bó. Hệ quả,
aldosterone không được tổng hợp ở những tế bào này ở vỏ thượng thận.
145. Một phụ nữ 45 tuổi có một khối ở hố tuyến yên tạo nên sự đè ép vào mạch Commented [LTN154]: gây

cửa, gián đoạn sự tiếp cận tuyến yên khỏi sự bài tiết của vùng dưới đồi. Mức bài
tiết của hormone nào tăng ở bệnh nhân?
A) ACTH
B) GH
C) Prolactin
D) LH
E) TSH
 C) Yếu tố điều hòa cơ sở sự bài tiết của ACTH, GH, LH, TSH từ thùy trước
tuyến yên là các hormone giải phóng của vùng dưới đồi. Chúng tiết ra qua cuống
tuyến yên và tiếp theo chảy vào hệ mạch cửa dưới đồi tuyến yên trước khi đến các
tế bào thùy trước tuyến yên. Ngược lại, sự bài tiết prolactin từ tuyến yên bị ảnh
huwowhr cơ bản bởi chất ức chế hormone của vùng dưới đồi dopamine. Hậu quả,
tắc mạch hệ thống cửa dẫn tới giảm bài tiết ACTH, GH, LH, TSH nhưng tăng tiết
prolactin.
146. Một người đàn ông bị phơi nhiễm với mức độ cao tia phóng xạ gamma không
có khả năng sản sinh tinh trùng do có sự phá hủy các tế bào biểu mầm của ông sinh
tinh, mặc dù anh ấy có nồng độ testosterone bình thường. Điều gì có thể được tìm
ra ở bệnh nhân này?
A) Sự bài tiết bình thường của GnRH
B) Nồng độ inhibin bình thường
C) Nồng độ FSH bị đàn áp
D) Thiếu tế bào Leydig
 A) Tia gamma hủy hoại các tế bào trải qua quá trình nguyên phân và giảm phân
mức độ cao, như tế bào mầm ở tinh hoàn. Người đàn ông được miêu tả trên có mức
testosterone bình thường, gợi ý rằng sự bài tiết GnRH và LH bình thường, và rằng
tế bào kẽ vẫn còn chức năng. Vì anh ấy không sinh được tinh trùng, lược inhibin
được tiết ra bởi Sertolin sẽ bị áp chế tối đa, và nồng độ FSH sẽ tăng lên nhanh
147. Bệnh nhân bị suy giáp do bất thường nguyên phát của tuyến giáp. Yếu tố nào
của huyết tương tăng?
A) Cholesterol
B) Globulin gắn Thyroxine
C) Reverse triiodothyronine (RT3)
D) Diiodotyrosine
E) Iodide
 A) Tăng nồng độ cholesterol huyết tương thường thấy trong suy giáp.
Cấu 148 và 149
Một thí nghiệm được tiến hành trong đó chuột được tiêm 1 trong 2 hormone hoặc
nước muối (nhóm chứng) trong 2 tuần. Khám nghiệm tử thi được tiến hành và cân
nặng của các cơ quan được đo (bằng miligrams)
148. Hormone 1 là?
A) TRH
B) TSH
C) T4
D) ACTH
E) Cortisol
 B) Trong thí nghiệm này, kích thước của tuyến giáp tăng vì TSH dẫn tới phì đại
và quá sản tuyến đích và tăng sự bài tiết hormone tuyến giáp. Tăng nồng độ huyết
tương tuyến giáp ức chế tiết TRH, gây giảm sự kích thích của các tế bào thyrotrope
của tuyến yên, dẫn tới giảm kích thước tuyến yên. Nồng độ cao của hormone tuyến
giáp cũng gây giảm chuyển hóa và giảm cân.
149. Hormone 2 là?
A) TSH
B) T4
C) CRH
D) ACTH
E) Cortisol
C) Trong thí nghiệm này, kích thước của tuyến yên và tuyến thượng thận tăng vì
CRH kích thích tế bào corticotrope tuyến yên tiết ACTH, đến lượt nó kích thích
tuyến thượng thận tiết corticosterone và cortisol. Nồng độ cortisol huyết tương cao
hơn làm tăng giáng hóa protein và thoái hóa mỡ do đó làm giảm cân

Unit 15: Thể dục thể thao


1. Hệ nào sau đây có thể sản xuất ra lượng ATP lớn nhất trong 1 phút trong khoảng
thời gian ngắn?
A) Hệ ái khí
B) Hệ phosphagen
C) Hệ Glycogen - acid lactic (axit glycogen-lactic)
D) Hệ phosphocreatine
E) ATP dự trữ
 B) Trong một khoảng thời gian ngắn hẹ phosphagen có thể sản xuất ra 4 mol
ATP/phút. Hệ phosphagen bao gồm hệ ATP và hệ phosphocreatine kết hợp lại.
Tuy nhiên, khi người chạy một quãng đường dài, ví dụ như chạy đua 10km, hệ
phosphagen chỉ có thể cung cấp nhiên liệu trong 8 đến 10 giây. Hệ glycogen- acid
lactic cung cấp năng lượng có thể sản xuất 2,5 mol ATP/phút. Vì vậy, hệ ái khí,
bao gồm chuyển hoá đường, chất béo và amino acid, có thể sản xuất 1 mol
ATP/phút.
TMP12 1033
2. Điều gì làm cho lượng cơ ở nam giới trung bình nhiều hơn so với ở nữ giới?
A) Tăng tiết testosterone ở nam giới
B) Tăng tiết estrogen ở nữ giới
C) Cường độ tập luyện ở nam giới cao hơn
D) Sự lắng đọng glycogen nhiều hơn ở nam
 A) Lượng cơ nhiều hơn ở nam giới xảy ra do testosterone, chất được tiết bởi 2
tinh hoàn. Chất này có tác dụng đồng hoá rất mạnh, gây ra sự tăng đáng kể sự lắng
đọng protein ở mọi nơi của cơ thể, nhưng đặc biệt là trong cơ. Estrogen ở nữ gây ra
tăng lắng đọng chất béo chứ không phải protein.
TMP12 1031
3. Định nghĩa của sức cơ là gì?
A) Khối lượng nặng nhất cơ có thể nâng
B) Lực cần thiết để co giãn sau khi đã co lại
C) Công mà cơ thực hiện trong một khoảng thời gian
D) Thời gian cơ có thể nâng một khối lượng nhất định
 A) Sức cơ được quyết định bới khối lượng lớn nhất mà cơ đó có thể nhấc lên.
Sức mạnh cơ là công mà cơ đó có thể thực hiện trong 1 khoảng thời gian. Điều này
không chỉ được quyết định bởi sức co thắt của cơ mà còn bởi khoảng cách co thắt
và thời gian cơ co thắt trong mỗi phút. Sức mạnh cơ thường được đo bằng đơn vị
kilogram-mét/phút. Đơn vị này tương đương với 1 cơ nhấc 1 kg lên cao 1m trong 1
phút.
TMP12 1032
4. Mệnh đề nào sau đây chính xác nhất khi so sánh sợi cơ co rút chậm và sợi cơ co
rút nhanh?
A) Sợi cơ nhanh ít phụ thuộc vào hệ phosphagen và hệ glycogen- acid lactic
hơn
B) Sợi cơ chậm có nhiều ty thể ở quanh chúng hơn
C) Sợi cơ chậm có ít myoglobin hơn
D) Số lượng hệ mao mạch xung quanh sợi cơ chậm ít hơn
E) Sợi cơ nhanh có đường kính nhỏ hơn
 B) Điểm khác nhau cơ bản giữa sợi cơ nhanh và sợi cơ chậm là những điểm
sau: Sợi cơ nhanh phụ thuộc nhiều hơn vào chuyển hoá yếm khí còn sợi cơ chậm
nhiều vào chuyển hoá ái khí. Trong sợi cơ nhanh, sự phụ thuộc vào hệ phosphagen
và hệ glycogen- acid lactic lớn hơn so với sợ cơ nhanh. Sợi cơ chậm được sắp xếp
hợp lí hơn cho sự chịu đựng và phụ thuộc nhiều vào chuyển hoá ái khí; vì vậy,
chúng có nhiều ty thể và myoglobin hơn, các cơ quan này kết hợp với oxy trong
sợi cơ. Số lượng của hệ mao mạch cung cấp oxy ở vùng kế cận sợi cơ chậm nhiều
hơn nhiều so với vùng kế cận của sợi cơ nhanh.
TMP12 1036
5. Vận động viên Olypic thi đấu marathon hoặc trượt tuyết xuyên quốc gia có cung
lượng tim cao nhất lớn hơn rất nhiều so với người không phải vận động viên. Mệnh
đề nào sau đây là chính xác nhất khi nói về tim của những vận động viên này so
với người không phải vận động viên?
A) Thể tích nhát bóp của vận động viên Olympic lớn hơn khoảng 5% lúc nghỉ
B) Phần trăm tăng lên của nhịp tim khi luyện tập nặng nhất lớn hơn rất nhiều ở
vận động viên Olympic
C) Cung lượng tim lớn nhất của vận động viên Olympic chỉ lớn hơn khoảng 3-
4%
D) Nhịp tim lúc nghỉ của vận động viên Olympic cao hơn đáng kể
 B) Khi so sánh vận động viên Olympic với người không phải vận động viên, ta
thấy rằng có nhiều điểm khác nhau trong đáp ứng của tim. Thể tích nhát bóp khi
nghỉ ngơi của vận động viên Olympic cao hơn và nhịp tim thì thấp hơn nhiều. Nhịp
tim có thể tăng lên xấp xỉ 270% ở vận động viên Olympic khi luyện tập tối đa, là
số phần trăm lớn hơn rất nhiều so với người không phải vận dộng viên. Thêm vào
đó, sự tăng tối đa của cung lượng tim lớn hơn xấp xỉ 30% ở vận động viên
Olympic.
TMP12 1039
6. Mệnh đề nào sau đây chính xác nhất về hô hấp trong luyện tập?
A) Lượng tiêu thụ oxy lớn nhất ở nam giới chạy marathon ít hơn so với nam
giới trung bình không luyện tập
B) Lượng tiêu thụ oxygen lớn nhất có thể tăng lên 100% bởi luyệ tập
C) Khả năng khuếch tán oxy lớn nhất ở nam giới chạy marathon cao hơn rất
nhiều so với nam giới trung bình không luyện tập
D) Nồng độ oxy và CO2 trong máu bất thường trong khi luyện tập
.C) Khi luyện tập, lượng tiêu thụ oxy tối đa ở nam giới chạy marathon lớn hơn
rất nhiều so với nam giới trung bình không luyện tập. Tuy nhiên, vận động viên
được luyện tập tăng lượng tiêu thụ oxy tối đa chỉ 10%. Vì vậy, lượng tiêu thụ oxy
tối đa ở người chạy marathon có thể được quyết định một phần bởi di truyền.
Những người này cũng tăng nhiều trong khả năng khuếch tán oxy tối da, và nồng
độ oxy và CO2 trong máu duy trì ở mức bình thường trong quá trình luyện tập.
TMP12 1036-1037
7. Vận động viên nào sau đây có khả năng luyện tập lâu nhất trước khi kiệt sức xảy
ra?
A) Có chế độ ăn giàu chất béo
B) Có chế độ ăn giàu carbonhydrate
C) Có chế độ ăn hỗn hợp chất béo - carbonhydrate
D) Có chế độ ăn giàu protein
E) Có chế độ ăn hỗn hợp protein-chất béo
 B) Vận đông viên có chế độ ăn giàu carbonhydrate sẽ dự trữ lượng glycongen
trong cơ gấp đôi so với vận động viên có chế độ ăn hỗn hợp carbonhydrate và chất
béo. Glycongen này sẽ được chuyển thành acid lacti và cung cấp 4 mol ATP phân
tử cho mỗi mol glucose. Nó cung hình thành ATP nhanh hơn gấp 2,5 lần chuyển
hoá oxi hoá trong ty thể. Nguồn năng lượng dư thừa này từ glycogen làm tăng
đáng kể thời gian vận động viên có thể luyện tập.
TMP12 1035
8. Nếu sức cơ tăng lên với bài tập kháng trở, tình trạng nào sau đây sẽ có khả năng
xảy ra nhất?
A) Giảm số lượng sợi myofibrin
B) Tăng enzyme ty thể
C) Giảm thành phần của hệ thống năng lượng phosphagen
D) Giảm dự trữ triglyceride
 B) Trong luyện tập có kháng trở, cơ phải co thắt với ít nhất 50% lực co tối đa
trong ít nhất 3 lần một tuần để có được sự tăng tối ưu của sức cơ. Điều này gây ra
phì đại cơ và một số thay đổi khác diễn ra. Sẽ có sự tăng số lượng myofibril và sự
tăng lên tới 120% của enzyme ty thể. Có thể xảy ra sự tăng các thành phần của hệ
năng lượng phosphagen từ 60% đến 80%, và sự tăng đến 50% trong lượng
glycogen dự trữ. Đồng thời, sự tăng lên tới 75% đến 100% của triglyceride dự trữ
cũng có thể xảy ra.
TMP12 1035-1036
9. Hút thuốc lá gây ra hậu quả nào sau đây trên hệ hô hấp?
A) Giãn các tiểu phế quản tận (đích)
B) Giảm sức cản đưởng thở
C) Giảm tiết dịch ở cây phế quản
D) Tê liệt lông mao các tế bào biểu mô đường hô hấp
 D) Hút thuốc là làm giảm khả năng thông khí phổi của vận động viên. Điều này
đúng bới 1 số lí do sau. Thứ nhất, nicotine gây co các tiểu phế quản tận của phổi,
điều này làm tăng sức cản đường thở để khí đi vào và đi ra khỏi phổi. Thứ hai, hút
thuốc có hậu quả kích thích các cây phế quản làm tăng tiết dich. Thứ ba, nicotine
làm tê liệt các lông mao trên bề mặt của tế bào biểu mô đường hô hấp, các lông
mao này thường di động liên tục để loại vỏ các dịch dư thừa và các chất ngoại lai
xâm nhập.
TMP12 1037-1038
10. Ở vận động viên sử dụng androgen để tăng khả năng thi đấu, điều nào sau đây
dễ dàng xảy ra nhất?
A) Giảm HDL máu
B) Giảm LDL máu
C) Tăng chức năng tinh hoàn
D) Giảm tỷ lệ tăng huyết áp
 A) Sử dụng hormone sinh dục nam (các androgens) hoặc một steroid yếm khi
khác để làm tăng sức cơ sẽ làm tăng khả năng thi đấu của vận động viên trong một
số trường hợp nhưng có thể có một số tác dụng phụ với cơ thể. Các steroid yếm khi
tăng nguy cơ của biến cố hê tim mạch, bởi chúng làm tăng tỷ lệ tăng huyết áp,
giảm HDL máu và tăng LDL máu. Những yếu tố này cũng kích thích nhưng cơn
đột quỵ và đau tim. Các chất thuộc họ androgenic cũng làm giảm chức năng của
tinh hoàn, làm giảm sự hình thành tinh trùng và sự sản xuất testosteron tự nhiên
của cơ thể.
TMP12 1040

You might also like